You are on page 1of 324

BÀI TẬP TỰ LUYỆN

CHINH PHỤC PHÁT ÂM - NGUYÊN ÂM


PEN-C TIẾNG ANH - CÔ HƯƠNG FIONA

1. Pick out the word whose underlined part is pronounced differently from that of the other words.
A. performed B. finished
C. interviewed D. delivered
2. Pick out the word whose underlined part is pronounced differently from that of the other words.
A. theatre B. therefore
C. throughout D. thunder
3. Pick out the word whose underlined part is pronounced differently from that of the other words.
A. worker B. teacher
C. better D. prefer
4. Pick out the word whose underlined part is pronounced differently from that of the other words.
A. measure B. leisure
C. ensure D. pleasure
5. Pick out the word whose underlined part is pronounced differently from that of the other words.
A. government B. glory
C. worry D. wonder
6. Pick out the word whose underlined part is pronounced differently from that of the other words.
A. promise B. devise
C. surprise D. realize

7. Pick out the word whose underlined part is pronounced differently from that of the other words.
A. weight B. height
C. eight D. vein

8. Pick out the word whose underlined part is pronounced differently from that of the other words.
A. hood B. hook
C. stood D. tool

9. Pick out the word whose underlined part is pronounced differently from that of the other words.
A. natural B. nature
C. native D. nation
10. Pick out the word whose underlined part is pronounced differently from that of the other words.
A. terrace B. territory
C. terrific D. terror
11. Pick out the word whose underlined part is pronounced differently from that of the other words.
A. kite B. blind
C. right D. rigid
12. Pick out the word whose underlined part is pronounced differently from that of the other words.
A. fond B. off
C. follow D. honey
13. Pick out the word whose underlined part is pronounced differently from that of the other words.
A. pretty B. level
C. lesson D. length
14. Pick out the word whose underlined part is pronounced differently from that of the other words.
A. monk B. boss
C. shop D. got
15. Pick out the word whose underlined part is pronounced differently from that of the other words.
A. eight B. weight
C. heighten D. freight

16. Pick out the word whose underlined part is pronounced differently from that of the other words.

Trang 1/3
1
A. tribe B. distribute
C. triangle D. trial
17. Pick out the word whose underlined part is pronounced differently from that of the other words.
A. clown B. cow
C. plough D. tough
18. Pick out the word whose underlined part is pronounced differently from that of the other words.
A. below B. beverage
C. begin D. beneath
19. Pick out the word whose underlined part is pronounced differently from that of the other words.
A. south B. brown
C. soup D. house

20. Pick out the word whose underlined part is pronounced differently from that of the other words.
A. comb B. come
C. dome D. home

21. Pick out the word whose underlined part is pronounced differently from that of the other words.
A. that B. hat
C. park D. taxi
22. Pick out the word whose underlined part is pronounced differently from that of the other words.
A. surgical B. surround
C. surrender D. survival
23. Pick out the word whose underlined part is pronounced differently from that of the other words.
A. opportunity B. menu
C. aptitude D. muscular
24. Pick out the word whose underlined part is pronounced differently from that of the other words.
A. karate B. hazard
C. glad D. blanket
25. Pick out the word whose underlined part is pronounced differently from that of the other words.
A. any B. ask
C. aspect D. castle
26. Pick out the word whose underlined part is pronounced differently from that of the other words.
A. adult B. husband
C. mutual D. culture
27. Pick out the word whose underlined part is pronounced differently from that of the other words.
A. bound B. ground
C. bounce D. cough

28. Pick out the word whose underlined part is pronounced differently from that of the other words.
A. addition B. adventure
C. advertise D. advantage

29. Pick out the word whose underlined part is pronounced differently from that of the other words.
A. dealt B. dreamt
C. steal D. jealous

30. Pick out the word whose underlined part is pronounced differently from that of the other words.
A. fame B. change
C. champagne D. blame
31. Pick out the word whose underlined part is pronounced differently from that of the other words.
A. nourish B. enough
C. courage D. encounter
32. Pick out the word whose underlined part is pronounced differently from that of the other words.
A. bad B. fan
C. catch D. wash

Trang 2/3
2
33. Pick out the word whose underlined part is pronounced differently from that of the other words.
A. use B. umbrella
C. union D. unit
34. Pick out the word whose underlined part is pronounced differently from that of the other words.
A. whole B. when
C. which D. while
35. Pick out the word whose underlined part is pronounced differently from that of the other words.
A. measure B. decision
C. pleasure D. permission
36. Pick out the word whose underlined part is pronounced differently from that of the other words.
A. while B. which
C. who D. white
37. Pick out the word whose underlined part is pronounced differently from that of the other words.
A. chorus B. cherish
C. chaos D. scholarship
38. Pick out the word whose underlined part is pronounced differently from that of the other words.
A. honourable B. honesty
C. historic D. heir
39. Pick out the word whose underlined part is pronounced differently from that of the other words.
A. research B. resent
C. resemble D. resist

40. Pick out the word whose underlined part is pronounced differently from that of the other words.
A. decision B. occasion
C. expansion D. erosion

Trang 3/3
3
BÀI TẬP TỰ LUYỆN
CHINH PHỤC PHÁT ÂM - PHỤ ÂM
PEN-C TIẾNG ANH - CÔ HƯƠNG FIONA

1. Mark the letter A, B, C, or D on your answer sheet to indicate the word whose underlined part differs from the other three in pronunciation
in each of the following questions.
A. sufferings B. advantages
C. species D. disasters
2. Mark the letter A, B, C, or D on your answer sheet to indicate the word whose underlined part differs from the other three in pronunciation
in each of the following questions.
A. roofs B. caps
C. players D. posts
3. Mark the letter A, B, C, or D on your answer sheet to indicate the word whose underlined part differs from the other three in pronunciation
in each of the following questions.
A. opponent B. go
C. color D. note
4. Mark the letter A, B, C, or D on your answer sheet to indicate the word whose underlined part differs from the other three in pronunciation
in each of the following questions.
A. level B. medal
C. legal D. effort

5. Mark the letter A, B, C, or D on your answer sheet to indicate the word whose underlined part differs from the other three in pronunciation
in each of the following questions.
A. mutual B. question
C. initiate D. picture
6. Mark the letter A, B, C, or D on your answer sheet to indicate the word whose underlined part differs from the other three in pronunciation
in each of the following questions.
A. considered B. wounded
C. believed D. combined
7. Mark the letter A, B, C, or D on your answer sheet to indicate the word whose underlined part differs from the other three in pronunciation
in each of the following questions.
A. wounded B. dedicated
C. appealed D. adopted

8. Mark the letter A, B, C, or D on your answer sheet to indicate the word whose underlined part differs from the other three in pronunciation
in each of the following questions.
A. damaged B. increased
C. proposed D. destroyed
9. Mark the letter A, B, C, or D on your answer sheet to indicate the word whose underlined part differs from the other three in pronunciation
in each of the following questions.
A. parrot B. swallow
C. panda D. advocate
10. Mark the letter A, B, C, or D on your answer sheet to indicate the word whose underlined part differs from the other three in pronunciation
in each of the following questions.
A. forged B. noticed
C. struggled D. composed

11. Mark the letter A, B, C, or D on your answer sheet to indicate the word whose underlined part differs from the other three in pronunciation
in each of the following questions.
A. beliefs B. rights
C. politics D. pioneers
12. Mark the letter A, B, C, or D on your answer sheet to indicate the word whose underlined part differs from the other three in pronunciation
in each of the following questions.
A. scoreboard B. fascinating
C. accompany D. discriminate
13. Mark the letter A, B, C, or D on your answer sheet to indicate the word whose underlined part differs from the other three in pronunciation
in each of the following questions.

Trang 1/4
4
A. pollution B. ordinary
C. alcohol D. doctor
14. Mark the letter A, B, C, or D on your answer sheet to indicate the word whose underlined part differs from the other three in pronunciation
in each of the following questions.
A. protected B. balanced
C. developed D. established
15. Mark the letter A, B, C, or D on your answer sheet to indicate the word whose underlined part differs from the other three in pronunciation
in each of the following questions.
A. increase B. measure
C. weapon D. spread

16. Mark the letter A, B, C, or D on your answer sheet to indicate the word whose underlined part differs from the other three in pronunciation
in each of the following questions.
A. character B. psychology
C. technical D. chores
17. Mark the letter A, B, C, or D on your answer sheet to indicate the word whose underlined part differs from the other three in pronunciation
in each of the following questions.
A. involved B. impressed
C. organized D. carried
18. Mark the letter A, B, C, or D on your answer sheet to indicate the word whose underlined part differs from the other three in pronunciation
in each of the following questions.
A. celebrate B. break
C. straight D. pleasure

19. Mark the letter A, B, C, or D on your answer sheet to indicate the word whose underlined part differs from the other three in pronunciation
in each of the following questions.
A. leaves B. misses
C. potatoes D. goes
20. Mark the letter A, B, C, or D on your answer sheet to indicate the word whose underlined part differs from the other three in pronunciation
in each of the following questions.
A. approach B. challenge
C. achieve D. chemical
21. Mark the letter A, B, C, or D on your answer sheet to indicate the word whose underlined part differs from the other three in pronunciation
in each of the following questions.
A. leaders B. supports
C. victims D. colonies

22. Mark the letter A, B, C, or D on your answer sheet to indicate the word whose underlined part differs from the other three in pronunciation
in each of the following questions.
A. decades B. actions
C. poachers D. elephants
23. Mark the letter A, B, C, or D on your answer sheet to indicate the word whose underlined part differs from the other three in pronunciation
in each of the following questions.
A. secure B. future
C. contribute D. shuttle
24. Mark the letter A, B, C, or D on your answer sheet to indicate the word whose underlined part differs from the other three in pronunciation
in each of the following questions.
A. tends B. roars
C. sweeps D. grows
25. Mark the letter A, B, C, or D on your answer sheet to indicate the word whose underlined part differs from the other three in pronunciation
in each of the following questions.
A. spirit B. describe
C. final D. title

26. Mark the letter A, B, C, or D on your answer sheet to indicate the word whose underlined part differs from the other three in pronunciation
in each of the following questions.
A. threatened B. endangered
C. advanced D. recovered

Trang 2/4
5
27. Mark the letter A, B, C, or D on your answer sheet to indicate the word whose underlined part differs from the other three in pronunciation
in each of the following questions.
A. typical B. penalty
C. carry D. identify
28. Mark the letter A, B, C, or D on your answer sheet to indicate the word whose underlined part differs from the other three in pronunciation
in each of the following questions.
A. hour B. honest
C. humour D. honour
29. Mark the letter A, B, C, or D on your answer sheet to indicate the word whose underlined part differs from the other three in pronunciation
in each of the following questions.
A. bride B. oblige
C. confide D. determine
30. Mark the letter A, B, C, or D on your answer sheet to indicate the word whose underlined part differs from the other three in pronunciation
in each of the following questions.
A. pool B. look
C. book D. good
31. Mark the letter A, B, C, or D on your answer sheet to indicate the word whose underlined part differs from the other three in pronunciation
in each of the following questions.
A. cough B. laugh
C. plough D. rough
32. Mark the letter A, B, C, or D on your answer sheet to indicate the word whose underlined part differs from the other three in pronunciation
in each of the following questions.
A. ethnic B. although
C. there D. gather
33. Mark the letter A, B, C, or D on your answer sheet to indicate the word whose underlined part differs from the other three in pronunciation
in each of the following questions.
A. enthusiasm B. pressure
C. resume D. future

34. Mark the letter A, B, C, or D on your answer sheet to indicate the word whose underlined part differs from the other three in pronunciation
in each of the following questions.
A. duty B. early
C. university D. apply
35. Mark the letter A, B, C, or D on your answer sheet to indicate the word whose underlined part differs from the other three in pronunciation
in each of the following questions.
A. enthusiast B. athlete
C. healthy D. southern
36. Mark the letter A, B, C, or D on your answer sheet to indicate the word whose underlined part differs from the other three in pronunciation
in each of the following questions.
A. indicate B. grade
C. average D. application

37. Mark the letter A, B, C, or D on your answer sheet to indicate the word whose underlined part differs from the other three in pronunciation
in each of the following questions.
A. close-knit B. certificate
C. compulsory D. campus
38. Mark the letter A, B, C, or D on your answer sheet to indicate the word whose underlined part differs from the other three in pronunciation
in each of the following questions.
A. habitat B. exhibit
C. honor D. hour
39. Mark the letter A, B, C, or D on your answer sheet to indicate the word whose underlined part differs from the other three in pronunciation
in each of the following questions.
A. chemical B. technical
C. charming D. character

40. Mark the letter A, B, C, or D on your answer sheet to indicate the word whose underlined part differs from the other three in pronunciation
in each of the following questions.

Trang 3/4
6
A. earthquakes B. epidemics
C. delegates D. countries

Trang 4/4
7
BÀI TẬP TỰ LUYỆN
TRỌNG ÂM (1)
PEN-C TIẾNG ANH - CÔ HƯƠNG FIONA

1. Choose the word that has the stress differently from that of the other words
A. constructive B. subordinate
C. several D. particular
2. Choose the word that has the stress differently from that of the other words
A. nightmare B. tunnel
C. retain D. thesis
3. Choose the word that has the stress differently from that of the other words
A. relationship B. engineering
C. enthusiast D. priority
4. Choose the word that has the stress differently from that of the other words
A. maintaining B. colony
C. credible D. maintenance
5. Choose the word that has the stress differently from that of the other words
A. recommendation B. punctuality
C. undergraduate D. solidarity
6. Choose the word that has the stress differently from that of the other words
A. astonish B. opponent
C. develop D. document

7. Choose the word that has the stress differently from that of the other words
A. judgment B. intelligent
C. prediction D. compulsion

8. Choose the word that has the stress differently from that of the other words
A. parameter B. conscious
C. speedometer D. thermometer

9. Choose the word that has the stress differently from that of the other words
A. rainfall B. machine
C. inquire D. confide
10. Choose the word that has the stress differently from that of the other words
A. sample B. personal
C. personnel D. entry
11. Choose the word that has the stress differently from that of the other words
A. production B. politician
C. definition D. situation
12. Choose the word that has the stress differently from that of the other words
A. appearance B. imagine
C. confidence D. discussion
13. Choose the word that has the stress differently from that of the other words
A. imply B. address
C. common D. include
14. Choose the word that has the stress differently from that of the other words
A. operate B. terminate
C. machine D. cultivate
15. Choose the word that has the stress differently from that of the other words
A. sufficient B. efficient
C. deficient D. ancient

16. Choose the word that has the stress differently from that of the other words

Trang 1/3
8
A. special B. wonder
C. relax D. problem
17. Choose the word that has the stress differently from that of the other words
A. industry B. memory
C. advertise D. ambiguous
18. Choose the word that has the stress differently from that of the other words
A. remember B. industry
C. division D. reflection
19. Choose the word that has the stress differently from that of the other words
A. simultaneous B. different
C. feedback D. errors
20. Choose the word that has the stress differently from that of the other words
A. humidity B. incredible
C. environment D. definition

21. Choose the word that has the stress differently from that of the other words
A. modernization B. urbanization
C. unexpectedly D. communication
22. Choose the word that has the stress differently from that of the other words
A. appropriate B. emotional
C. situation D. pronounce
23. Choose the word that has the stress differently from that of the other words
A. expensive B. promise
C. constructive D. original
24. Choose the word that has the stress differently from that of the other words
A. comparison B. reasonable
C. responsible D. original
25. Choose the word that has the stress differently from that of the other words
A. countless B. imply
C. arrive D. before
26. Choose the word that has the stress differently from that of the other words
A. understand B. recommend
C. negative D. pioneer
27. Choose the word that has the stress differently from that of the other words
A. delicious B. edible
C. visual D. ancient

28. Choose the word that has the stress differently from that of the other words
A. singing B. burning
C. concerning D. forests

29. Choose the word that has the stress differently from that of the other words
A. exact B. facial
C. sleepy D. trophy

30. Choose the word that has the stress differently from that of the other words
A. necessity B. authority
C. commercially D. academic
31. Choose the word that has the stress differently from that of the other words
A. superman B. synthetic
C. professor D. conversion
32. Choose the word that has the stress differently from that of the other words
A. Canadian B. photography
C. occasion D. qualification

Trang 2/3
9
33. Choose the word that has the stress differently from that of the other words
A. decorator B. temperature
C. calculator D. individual
34. Choose the word that has the stress differently from that of the other words
A. applicable B. emotional
C. simultaneous D. astrology
35. Choose the word that has the stress differently from that of the other words
A. dinosaur B. history
C. eternal D. calendar
36. Choose the word that has the stress differently from that of the other words
A. kingdom B. mountain
C. passage D. begin
37. Choose the word that has the stress differently from that of the other words
A. margarine B. internal
C. involvement D. committee
38. Choose the word that has the stress differently from that of the other words
A. computation B. satisfaction
C. desirable D. unemployment

39. Choose the word that has the stress differently from that of the other words
A. substantial B. contribute
C. tableland D. artistic

40. Choose the word that has the stress differently from that of the other words
A. variety B. associate
C. accountancy D. influential

Trang 3/3
10
BÀI TẬP TỰ LUYỆN
TRỌNG ÂM (2)
PEN-C TIẾNG ANH - CÔ HƯƠNG FIONA

1. Mark A, B, C, or D on your answer sheet to indicate the word that differs from the other three in the position of primary stress in each of
the following questions.
A. nonsense B. struggle
C. rubbish D. development
2. Mark A, B, C, or D on your answer sheet to indicate the word that differs from the other three in the position of primary stress in each of
the following questions.
A. interfere B. develop
C. industrial D. activity
3. Mark A, B, C, or D on your answer sheet to indicate the word that differs from the other three in the position of primary stress in each of
the following questions.
A. extremely B. necessity
C. necessary D. co-operate
4. Mark A, B, C, or D on your answer sheet to indicate the word that differs from the other three in the position of primary stress in each of
the following questions.
A. picture B. water
C. advice D. number

5. Mark A, B, C, or D on your answer sheet to indicate the word that differs from the other three in the position of primary stress in each of
the following questions.
A. understand B. recommend
C. entertain D. develop

6. Mark A, B, C, or D on your answer sheet to indicate the word that differs from the other three in the position of primary stress in each of
the following questions.
A. comfortable B. involvement
C. tsunami D. precision
7. Mark A, B, C, or D on your answer sheet to indicate the word that differs from the other three in the position of primary stress in each of
the following questions.
A. exercise B. disaster
C. difficult D. prisoner

8. Mark A, B, C, or D on your answer sheet to indicate the word that differs from the other three in the position of primary stress in each of
the following questions.
A. nursery B. intelligence
C. curriculum D. certificate

9. Mark A, B, C, or D on your answer sheet to indicate the word that differs from the other three in the position of primary stress in each of
the following questions.
A. fascinate B. accept
C. support D. believe
10. Mark A, B, C, or D on your answer sheet to indicate the word that differs from the other three in the position of primary stress in each of
the following questions.
A. pleasure B. however
C. product D. satisfy

11. Mark A, B, C, or D on your answer sheet to indicate the word that differs from the other three in the position of primary stress in each of
the following questions.
A. expectation B. temperament
C. influential D. opportunity

12. Mark A, B, C, or D on your answer sheet to indicate the word that differs from the other three in the position of primary stress in each of
the following questions.
A. windsurfing B. leftovers
C. enthusiast D. scoreboard
13. Mark A, B, C, or D on your answer sheet to indicate the word that differs from the other three in the position of primary stress in each of
the following questions.

Trang 1/4
11
A. engrave B. memorialize
C. discover D. categorize
14. Mark A, B, C, or D on your answer sheet to indicate the word that differs from the other three in the position of primary stress in each of
the following questions.
A. police B. banquet
C. culture D. spirit
15. Mark A, B, C, or D on your answer sheet to indicate the word that differs from the other three in the position of primary stress in each of
the following questions.
A. inversion B. difficulty
C. miserable D. pleasant

16. Mark A, B, C, or D on your answer sheet to indicate the word that differs from the other three in the position of primary stress in each of
the following questions.
A. determine B. sacrifice
C. maintain D. apologize
17. Mark A, B, C, or D on your answer sheet to indicate the word that differs from the other three in the position of primary stress in each of
the following questions.
A. category B. maximum
C. applicant D. inflation
18. Mark A, B, C, or D on your answer sheet to indicate the word that differs from the other three in the position of primary stress in each of
the following questions.
A. communication B. expedition
C. intervention D. renovation

19. Mark A, B, C, or D on your answer sheet to indicate the word that differs from the other three in the position of primary stress in each of
the following questions.
A. offer B. listen
C. precede D. follow

20. Mark A, B, C, or D on your answer sheet to indicate the word that differs from the other three in the position of primary stress in each of
the following questions.
A. employee B. referee
C. refugee D. committee
21. Mark A, B, C, or D on your answer sheet to indicate the word that differs from the other three in the position of primary stress in each of
the following questions.
A. marriage B. value
C. nature D. belief

22. Mark A, B, C, or D on your answer sheet to indicate the word that differs from the other three in the position of primary stress in each of
the following questions.
A. biologist B. counterpart
C. kindergarten D. compliment

23. Mark A, B, C, or D on your answer sheet to indicate the word that differs from the other three in the position of primary stress in each of
the following questions.
A. develop B. difference
C. determine D. appearance

24. Mark A, B, C, or D on your answer sheet to indicate the word that differs from the other three in the position of primary stress in each of
the following questions.
A. wildlife B. bamboo
C. rainfall D. different
25. Mark A, B, C, or D on your answer sheet to indicate the word that differs from the other three in the position of primary stress in each of
the following questions.
A. interest B. improvement
C. occasion D. believe

26. Mark A, B, C, or D on your answer sheet to indicate the word that differs from the other three in the position of primary stress in each of
the following questions.
A. embarrass B. discourage
C. organize D. surrounding

Trang 2/4
12
27. Mark A, B, C, or D on your answer sheet to indicate the word that differs from the other three in the position of primary stress in each of
the following questions.
A. rewrite B. eject
C. oblige D. happiness
28. Mark A, B, C, or D on your answer sheet to indicate the word that differs from the other three in the position of primary stress in each of
the following questions.
A. romantic B. following
C. physical D. summary
29. Mark A, B, C, or D on your answer sheet to indicate the word that differs from the other three in the position of primary stress in each of
the following questions.
A. finance B. service
C. order D. company
30. Mark A, B, C, or D on your answer sheet to indicate the word that differs from the other three in the position of primary stress in each of
the following questions.
A. maintain B. announce
C. stimulate D. subtract

31. Mark A, B, C, or D on your answer sheet to indicate the word that differs from the other three in the position of primary stress in each of
the following questions.
A. inform B. attempt
C. hesitate D. appeal
32. Mark A, B, C, or D on your answer sheet to indicate the word that differs from the other three in the position of primary stress in each of
the following questions.
A. company B. compose
C. consist D. compare
33. Mark A, B, C, or D on your answer sheet to indicate the word that differs from the other three in the position of primary stress in each of
the following questions.
A. advise B. defeat
C. compete D. integrate

34. Mark A, B, C, or D on your answer sheet to indicate the word that differs from the other three in the position of primary stress in each of
the following questions.
A. interview B. appropriate
C. describe D. employ
35. Mark A, B, C, or D on your answer sheet to indicate the word that differs from the other three in the position of primary stress in each of
the following questions.
A. swallow B. survive
C. finish D. option
36. Mark A, B, C, or D on your answer sheet to indicate the word that differs from the other three in the position of primary stress in each of
the following questions.
A. title B. podium
C. pioneer D. notice
37. Mark A, B, C, or D on your answer sheet to indicate the word that differs from the other three in the position of primary stress in each of
the following questions.
A. discriminate B. argue
C. recognize D. manage
38. Mark A, B, C, or D on your answer sheet to indicate the word that differs from the other three in the position of primary stress in each of
the following questions.
A. historic B. electric
C. specific D. politics
39. Mark A, B, C, or D on your answer sheet to indicate the word that differs from the other three in the position of primary stress in each of
the following questions.
A. addition B. interview
C. customer D. agency
40. Mark A, B, C, or D on your answer sheet to indicate the word that differs from the other three in the position of primary stress in each of
the following questions.

Trang 3/4
13
A. gorilla B. rhinoceros
C. opponent D. personality

Trang 4/4
14
BÀI TẬP TỰ LUYỆN
VOCABULARY - EDUCATION
PEN-C TIẾNG ANH - CÔ HƯƠNG FIONA

1. Mark the letter A, B, C, or D on your answer sheet to indicate the correct answer to each of the following questions.

In the UK, _______ schools refer to government-funded schools which provide education free of charge to pupils
A. secondary B. state
C. primary D. independent
2. Mark the letter A, B, C, or D on your answer sheet to indicate the correct answer to each of the following questions.

Schools in which all students can attend without paying tuition _______ are public schools
A. charge B. fees
C. bill D. payment
3. Mark the letter A, B, C, or D on your answer sheet to indicate the correct answer to each of the following questions.

If you want to learn a new language , you must _______ foreign language classes
A. assist B. attend
C. present D. follow

4. Mark the letter A, B, C, or D on your answer sheet to indicate the correct answer to each of the following questions.

In Vietnam a school year lasts for nine months and is divided _______ 2 terms
A. on B. from
C. into D. to
5. Read the following passage and type the letter A, B, C or D to indicate the correct answer to each of the questions.

There is a strange paradox to the success of the Asian education model. On the one hand, class sizes are huge by Western standards with
between 30 and 40 students per class, in countries like Japan and Korea. On the other hand, school children in developed Asian economies
rank among the highest in the world for academic achievement in the areas of science and mathematics, especially on standardised tests.
Meanwhile, British secondary school students fail to shine in conditions most educational researchers would say are far more likely to help
them succeed.

Classroom management seems to be easier in places like Korea, and perhaps lessons are more effective as a direct consequence. After all,
we are only too aware of the decline in discipline standards in our own school: belligerent and disrespectful students appear to be the norm
these days. Teachers in Britain seem powerless to control what happens anymore. Surely this situation cannot create a very effective
learning environment, so perhaps the number of students is far less relevant than is the manner in which they conduct themselves.

But there are other factors to consider, too. There is the home environment. The traditional family unit still remains relatively intact in
Korea. Few children come from broken homes, so there is a sense of security, safety and trust both at home and at school. In Britain
meanwhile, one in every two marriages fails and divorce rates are sky high. Perhaps children struggle to cope with unstable family
conditions and their only way to express their frustration is by misbehaving at school.

But while the Japanese, Korean and Asian models generally do seem to produce excellent results, the statistics don’t tell the whole truth.
You see, behind those great maths and science scores, there is a quite remarkable work ethic. Asian students tend to put their education
before literally everything else. They do very few extracurricular activities and devote far more time to their studies than their British
peers.

Trang 1/6
15
There has been a lot of attention and praise given to these Asian models and their “impressive” statistics of late. And without question,
some of this praise is justified, but it seems to be a case of two extremes in operation here. At one end, there is the discipline and
unbelievably hard work ethic of the Asian students – success in education before all else. At the other end, British students at times appear
careless and extremely undisciplined by comparison, but at least they Do have the free time to enjoy their youth and explore their interests.
Is either system better outright? Or is it perhaps about time we stopped comparing and started trying to combine the best bits of both, so
that we can finally offer our students a balanced, worthwhile education.

Question 1: The word “They” in paragraph 4 refers to


A. British students
B. Asian students
C. Korean students
D. Japanese students
Question 2: British secondary school students
A. have larger class sizes
B. fail at school more than they succeed
C. do better on stadardised tests
D. enjoy better classroom conditions
Question 3: What can be implied from the writer’s opinion of the two educational systems discussed?
A. The Asian system is clearly better.
B. The British system is too strict.
C. Neither system is perfect.
D. Both systems are quite satisfactory for different reasons.
Question 4: The traditional family unit
A. is more common in Korean than in Britain
B. is disappearing in Korean due to high divorce rates
C. is bad for children that come from broken homes
D. is unstable in Korean due to conditions in the home
Question 5: What does the writer mean when he says there is a “paradox” in the Asian education model?
A. There are too many students in each class.
B. You would expect larger classes to get poorer results but they do not.
C. Class sizes are much smaller in other parts of the world.
D. Asian students outperform their peers in other countries.
Question 6: What does the writer suggest might make lessons in Korean schools more successful than in Britain?

A. Better teacher
B. Better school Boards of Management
C. More effective lesson planning
D. Better discipline
Question 7: The word “unstable ” in paragraph 3 can be best replaced by
A. unsteady
B. unchangeable
C. unpredictable
D. unimportant
Question 8: According to the writer, Asian students
A. focus too much on recreational activities
B. don’t have as good a work ethic as British ones
C. don’t allow themselves much time to relax and have fun
D. make a big deal of their good results
6. Read the following passage and type the letter A, B, C or D to indicate the correct answer to each of the questions from 1 to 8.

Learning means acquiring knowledge or developing the ability to perform new behaviors. It is common to think of learning as something
that takes place in school, but much of human learning occurs outside the classroom, and people continue to learn throughout their lives.

Trang 2/6
16
Even before they enter school, young children learn to walk, to talk, and to use their hands to manipulate toys, food, and other objects.
They use all of their senses to learn about the sights, sounds, tastes, and smells in their environments. They learn how to interact with their
parents, siblings, friends, and other people important to their world. When they enter school, children learn basic academic subjects such as
reading, writing, and mathematics. They also continue to learn a great deal outside the classroom. They learn which behaviors are likely to
be rewarded and which are likely to be punished. They learn social skills for interacting with other children. After they finish school,
people must learn to adapt to the many major changes that affect their lives, such as getting married, raising children, and finding and
keeping a job.

Because learning continues throughout our lives and affects almost everything we do, the study of learning is important in many different
fields. Teachers need to understand the best ways to educate children. Psychologists, social workers, criminologists, and other human-
service workers need to understand how certain experiences change people’s behaviors. Employers, politicians, and advertisers make use
of the principles of learning to influence the behavior of workers, voters, and consumers.

Learning is closely related to memory, which is the storage of information in the brain. Psychologists who study memory are interested in
how the brain stores knowledge, where this storage takes place, and how the brain later retrieves knowledge when we need it. In contrast,
psychologists who study learning are more interested in behavior and how behavior changes as a result of a person’s experiences.

There arc many forms of learning, ranging from simple to complex. Simple forms of learning involve a single stimulus. A stimulus is
anything perceptible to the senses, such as a sight, sound, smell, touch, or taste. In a form of learning known as classical conditioning,
people learn to associate two stimuli that occur in sequence, such as lightning followed by thunder. In operant conditioning, people learn
by forming an association between a behavior and its consequences (reward or punishment). People and animals can also learn by
observation - that is, by watching others perform behaviors. More complex forms of learning include learning languages, concepts, and
motor skills.

Question 1: According to the passage, which of the following is learning in broad view comprised of?
A. Knowledge acquisition outside the classroom
B. Knowledge acquisition and ability development
C. Acquisition of academic knowledge
D. Acquisition of social and behavioral skills
Question 2: According to the passage, what are children NOT usually taught outside the classroom?
A. literacy and calculation
B. life skills
C. interpersonal communication
D. right from wrong
Question 3: Getting married, raising children, and finding and keeping a job are mentioned in paragrapli 2 as examples of

A. the changes to which people have to orient themselves


B. the areas of learning which affect people’s lives
C. the situations in which people cannot teach themselves
D. the ways people’s lives are influenced by education
Question 4 Which of the following can be inferred about the learning process from the passage?
A. It becomes less challenging and complicated when people grow older.
B. It takes place more frequently in real life than in academic institutions.
C. It is more interesting and effective in school than that in life.
D. It plays a crucial part in improving the learner’s motivation in school.
Question 5: According to the passage, the study of learning is important in many fields due to
A. the great influence of the on-going learning process
B. the need for certain experiences in various areas
C. the influence of various behaviors in the learning process
D. the exploration of the best teaching methods
Question 6: It can be inferred from the passage that social workers, employers, and politicians concern themselves with the study
of learning because they need to
A. change the behaviors of the objects of their interest towards learning
B. make the objects of their interest more aware of the importance of learning
C. thoroughly understand the behaviors of the objects of their interest
D. understand how a stimulus relates to the senses of the objects of their interest
Question 7: Which of the following statements is NOT true according to the passage?
A. Psychologists are all interested in memory as much as behaviors.
B. Psychologists studying memory are concerned with how the stored knowledge is used.
C. Psychologists studying learning are interested in human behaviors.
D. Psychologists studying memory are concerned with the brain’s storage of knowledge.
Question 8: The passage mainly discusses
A. practical examples of learning inside the classroom
B. application of learning principles to formal education
C. general principles of learning
D. simple forms of learning
7. Mark the letter A, B, C, or D on your answer sheet to indicate the correct answer to each of the following questions.

My favorite _______ at school is history Trang 3/6


17
A. class B. topic
C. theme D. subject
8. Mark the letter A, B, C, or D on your answer sheet to indicate the correct answer to each of the following questions.

Education has been developed in _______ with modern industry and the mass media
A. selected B. parallel
C. compulsory D. following
9. Mark the letter A, B, C, or D on your answer sheet to indicate the correct answer to each of the following questions.

We want the kids to have the best _______ education


A. probably B. possibly
C. probable D. possible
10. Mark the letter A, B, C, or D on your answer sheet to indicate the correct answer to each of the following questions.

Many students now have been paid for _______ during terms
A. employ B. unemployment
C. employees D. employed
11. Mark the letter A, B, C, or D on your answer sheet to indicate the correct answer to each of the following questions.

In Vietnamese schools, English, mathematics, and literature are three _______ subjects, which are compulsory in many important national
examinations.
A. part B. center
C. middle D. core
12. Mark the letter A, B, C, or D on your answer sheet to indicate the correct answer to each of the following questions.

_______ the end of the school year, students take an exam in each subject.
A. To B. On
C. In D. At
13. Mark the letter A, B, C, or D on your answer sheet to indicate the correct answer to each of the following questions.

School uniform is compulsory in most of Vietnamese schools


A. required B. depended
C. divided D. paid

14. Mark the letter A, B, C, or D on your answer sheet to indicate the correct answer to each of the following questions.

We have to _______ an exam at the end of the course


A. do B. write
C. take D. make
15. Read the following passage and type the letter A, B, C, or D into the blanks to indicate the correct word or phrase that best fits
each of the numbered blanks from 1 to 5.

Schools in the United States have not always had a large number of libraries. As (1) as 1958 about half of the public
schools in the United States had no libraries at all. The (2) of public school libraries increased dramatically when the
federal government passed the Elementary and Secondary Education Act of 1965, (3) provided funds for school
districts to improve their education programs and facilities, including their libraries. (4) , many educators claim that
since the legislation was passed federal spending has not increased sufficiently to meet the rising (5) of new library
technologies such as computer databases and Internet access.

Question 1: A. frequentlyB. recently C. freshly D. newly


Question 2: A. digit B. amount C. number D. numeral
Question 3: A. that B. who C. which D. this
Question 4: A. OtherwiseB. Nevertheless C. Therefore D. Consequently
Question 5: A. fine B. fee C. cost D. sum

16. Mark the letter A, B, C, or D on your answer sheet to indicate the correct answer to each of the following questions.

In England schooling is compulsory _______ all children from the age of 5 to 16

Trang 4/6
18
A. for B. to
C. with D. over

17. Mark the letter A, B, C, or D on your answer sheet to indicate the correct answer to each of the following questions.

Most children in the UK remain in _______ education until they are at least 16 years
A. full time B. full
C. part time D. full – period
18. Mark the letter A, B, C, or D on your answer sheet to indicate the correct answer to each of the following questions.

Children receive its early ____ at home so parents should buy some _______ magazines to put them in the bookcase
A. educational / educate B. education / educational
C. education / educated D. educational /education
19. Mark the letter A, B, C, or D on your answer sheet to indicate the correct answer to each of the following questions.

For the government, education is now at the top of _______.


A. outline B. plan
C. order D. agenda
20. Mark the letter A, B, C, or D on your answer sheet to indicate the correct answer to each of the following questions.

School boards are usually made _______ of people who live in the area, often parents of children in the school
A. of B. into
C. over D. up

21. Mark the letter A, B, C, or D on your answer sheet to indicate the correct answer to each of the following questions.

The academic year in Vietnam is over _______ the end _______ May
A. from / in B. for / on
C. at / of D. on / in
22. Mark the letter A, B, C, or D on your answer sheet to indicate the correct answer to each of the following questions.

His school report last term was very _______.


A. fulfilling B. satisfied
C. satisfactory D. full
23. Mark the letter A, B, C, or D on your answer sheet to indicate the correct answer to each of the following questions.

Students usually _______ their GCSE at the age of sixteen


A. pass B. make
C. take D. follow
24. Mark the letter A, B, C, or D on your answer sheet to indicate the correct answer to each of the following questions.

Many high school students in Vietnam have to work very hard to _______ at a university
A. lose touch with B. get a cold
C. win a place D. keep pace with

25. Mark the letter A, B, C, or D on your answer sheet to indicate the correct answer to each of the following questions.

Tommy left high school _______ the age _______ seventeen


A. in / for B. on / with
C. at / of D. of / in
26. Mark the letter A, B, C, or D on your answer sheet to indicate the correct answer to each of the following questions.

Watching TV can be very _______.


A. educated B. educate
C. education D. educational
27. Mark the letter A, B, C, or D on your answer sheet to indicate the correct answer to each of the following questions.

The national _______ is a program of study in all the main subjects that children aged 5 to 16 in state schools must follow
A. syllabus B. plan
C. curriculum D. course
Trang 5/6
19
28. Mark the letter A, B, C, or D on your answer sheet to indicate the correct answer to each of the following questions.

The expansion of _______ education at college and university should be a powerful force for change
A. higher B. further
C. extended D. extra

Trang 6/6
20
BÀI TẬP TỰ LUYỆN
CÁC THÌ ĐƠN TRONG TIẾNG ANH
PEN-C TIẾNG ANH - CÔ HƯƠNG FIONA

1. Mark the letter A, B, C or D on your answer sheet to indicate the correct answer to each of the following questions.

While her husband was in the army, Janet ________ to him twice a week
A. had written B. wrote
C. was written D. was writing
2. Mark the letter A, B, C or D on your answer sheet to indicate the correct answer to each of the following questions.

I will come and see you before I _______ for America


A. will leave B. leave
C. shall leave D. have left
3. Mark the letter A, B, C or D on your answer sheet to indicate the correct answer to each of the following questions.

Bad driving often ________ many accidents


A. has cause B. causes
C. caused D. had caused

4. Mark the letter A, B, C or D on your answer sheet to indicate the correct answer to each of the following questions.

I don’t understand this sentence. What ___________?


A. does mean this word B. means this word
C. does this word mean D. have this word mean
5. Mark the letter A, B, C or D on your answer sheet to indicate the correct answer to each of the following questions.

I think the weather _______ nice later


A. will be B. be
C. has been D. had
6. Mark the letter A, B, C or D on your answer sheet to indicate the correct answer to each of the following questions.

They ________ me about it last week.


A. was telling B. had told
C. told D. would tell
7. Mark the letter A, B, C or D on your answer sheet to indicate the correct answer to each of the following questions.

Columbus ________ America more than 400 years ago


A. had discovered B. he has gone
C. has discovered D. discovered

8. Mark the letter A, B, C or D on your answer sheet to indicate the correct answer to each of the following questions.

My grandfather ________ many years ago


A. died B. had died
C. was dying D. has died
9. Mark the letter A, B, C or D on your answer sheet to indicate the correct answer to each of the following questions.

I __________ here at the end of the month


A. is leaving B. would leave
C. would have left D. will leave

10. Mark the letter A, B, C or D on your answer sheet to indicate the correct answer to each of the following questions.

It is a nice day. I ________ we go out for a walk


A. is suggesting B. suggested
C. are suggesting D. suggest
11. Mark the letter A, B, C or D on your answer sheet to indicate the correct answer to each of the following questions.

He will take the dog out for a walk as soon as he _______ dinner

Trang 1/4
21
A. finish B. will finish
C. finishes D. shall have finished
12. Mark the letter A, B, C or D on your answer sheet to indicate the correct answer to each of the following questions.

John _________ tennis once or twice a week


A. usually play B. usually plays
C. have usually played D. is usually playing
13. Mark the letter A, B, C or D on your answer sheet to indicate the correct answer to each of the following questions.

The sun _______ in the East


A. is rising B. rises
C. rose D. has risen

14. Mark the letter A, B, C or D on your answer sheet to indicate the correct answer to each of the following questions.

She ________ very angry when she knows this


A. will be B. shall be
C. has been D. will have been
15. Mark the letter A, B, C or D on your answer sheet to indicate the correct answer to each of the following questions.

He said he _________ return later


A. would B. can
C. will D. would be

16. Mark the letter A, B, C or D on your answer sheet to indicate the correct answer to each of the following questions.

The Olympic Games _______ every four years


A. takes place B. take place
C. is taking place D. took place

17. Mark the letter A, B, C or D on your answer sheet to indicate the correct answer to each of the following questions.

I usually _________ away at weekends


A. have gone B. goes
C. go D. am going
18. Mark the letter A, B, C or D on your answer sheet to indicate the correct answer to each of the following questions.

Tomorrow I _______ my grandparents


A. will have visited B. visit
C. have visited D. am going to visit
19. Mark the letter A, B, C or D on your answer sheet to indicate the correct answer to each of the following questions.

I ________ my son the money for that last week


A. have given B. gave
C. was given D. given
20. Mark the letter A, B, C or D on your answer sheet to indicate the correct answer to each of the following questions.

The man got out the car, ________ round to the back and opened the boot
A. walk B. walks
C. walked D. walking
21. Mark the letter A, B, C or D on your answer sheet to indicate the correct answer to each of the following questions.

The train __________ half an hour ago


A. had left B. left
C. has been leaving D. has left
22. Mark the letter A, B, C or D on your answer sheet to indicate the correct answer to each of the following questions.

Henry _________ into the restaurant last night


A. went B. was going
C. has gone D. did go
Trang 2/4
22
23. Mark the letter A, B, C or D on your answer sheet to indicate the correct answer to each of the following questions.

__________ you go to the dentist?


A. how often do B. how often does
C. how are D. how often are
24. Mark the letter A, B, C or D on your answer sheet to indicate the correct answer to each of the following questions.

I come from Canada. Where ______ you ______ from?


A. are coming B. have/come
C. do/come D. did/ come
25. Mark the letter A, B, C or D on your answer sheet to indicate the correct answer to each of the following questions.

The earth ______ round the sun


A. has gone B. went
C. go D. goes

26. Mark the letter A, B, C or D on your answer sheet to indicate the correct answer to each of the following questions.

When I was young, I _________ to be a singer


A. was wanting B. want
C. wanted D. had wanted
27. Mark the letter A, B, C or D on your answer sheet to indicate the correct answer to each of the following questions.

Ann ________ tea very often


A. don’t drink B. didn’t drink
C. hasn’t drunken D. doesn’t drink
28. Mark the letter A, B, C or D on your answer sheet to indicate the correct answer to each of the following questions.

The dancing club ______ north of the city


A. located B. lain
C. lays D. lies
29. Mark the letter A, B, C or D on your answer sheet to indicate the correct answer to each of the following questions.

What time _____ the banks close in Britain?


A. did B. do
C. can D. does

30. Mark the letter A, B, C or D on your answer sheet to indicate the correct answer to each of the following questions.

I _______ at 8 o’clock every morning


A. was getting up B. get up
C. is getting up D. got up
31. Mark the letter A, B, C or D on your answer sheet to indicate the correct answer to each of the following questions.

Oil ______ if you pour it on water


A. will be floated B. floats
C. float D. floated

32. Mark the letter A, B, C or D on your answer sheet to indicate the correct answer to each of the following questions.

Last year, they __________ 22 million TV sets


A. has sold B. sold
C. had sold D. were selling
33. Mark the letter A, B, C or D on your answer sheet to indicate the correct answer to each of the following questions.

Since ______,I have heard nothing from him


A. he had left B. he has left
C. he left D. he was left

34. Mark the letter A, B, C or D on your answer sheet to indicate the correct answer to each of the following questions.

Trang 3/4
23
There was a time when watching TV _______ really family entertainment
A. had been B. was
C. is D. were

Trang 4/4
24
BÀI TẬP TỰ LUYỆN
CÁC THÌ TIẾP DIỄN TRONG TIẾNG ANH
PEN-C TIẾNG ANH - CÔ HƯƠNG FIONA

1. Mark the letter A, B, C or D on your answer sheet to indicate the correct answer to each of the following questions.

Listen! Can you hear those people next door? They ____________.
A. yell B. yells
C. is yelling D. are yelling
2. Mark the letter A, B, C or D on your answer sheet to indicate the correct answer to each of the following questions.

Look! The policeman ____________after a pick pocket


A. running B. runs
C. are running D. is running
3. Mark the letter A, B, C or D on your answer sheet to indicate the correct answer to each of the following questions.

Why ________ at me like that? What’s the matter?


A. did you look B. do you look
C. have you looked D. are you looking

4. Mark the letter A, B, C or D on your answer sheet to indicate the correct answer to each of the following questions.

You looked very busy when I (see) ________ you last night. What you (do) _________?
A. see/ did you do B. saw/ were you doing
C. saw/ are you doing D. see/ was you doing
5. Mark the letter A, B, C or D on your answer sheet to indicate the correct answer to each of the following questions.

It _______ dark. Shall I turn on the light?


A. has got B. get
C. got D. is getting
6. Mark the letter A, B, C or D on your answer sheet to indicate the correct answer to each of the following questions.

“Are you ready, Ann?” - “Yes, I ______”


A. have came B. come
C. am coming D. came
7. Mark the letter A, B, C or D on your answer sheet to indicate the correct answer to each of the following questions.

He fell down when he _______ towards the church


A. run B. was running
C. had run D. runs

8. Mark the letter A, B, C or D on your answer sheet to indicate the correct answer to each of the following questions.

I ______ Texas State University now


A. attended B. am attending
C. was attending D. attend
9. Mark the letter A, B, C or D on your answer sheet to indicate the correct answer to each of the following questions.

What ____________ yesterday at 10.20 pm?


A. are you doing B. did you do
C. was you doing D. were you doing

10. Mark the letter A, B, C or D on your answer sheet to indicate the correct answer to each of the following questions.

I _________ the dishwasher on when heard the shot


A. is turning B. was turning
C. am turning D. turned
11. Mark the letter A, B, C or D on your answer sheet to indicate the correct answer to each of the following questions.

They ______ table tennis when their father comes back home

Trang 1/4
25
A. play B. would play
C. will play D. will be playing
12. Mark the letter A, B, C or D on your answer sheet to indicate the correct answer to each of the following questions.

Her brother ______ in Canada at present


A. works B. work
C. working D. is working
13. Mark the letter A, B, C or D on your answer sheet to indicate the correct answer to each of the following questions.

They __________ ping pong when their father comes back home
A. will be playing B. will play
C. play D. would play

14. Mark the letter A, B, C or D on your answer sheet to indicate the correct answer to each of the following questions.

He _______ us to go out in the boat yesterday as a strong wind _________.


A. don’t allow/ were blowing B. doesn’t allow/ was blowing
C. didn’t allow/ was blowing D. not allow/ blow
15. Mark the letter A, B, C or D on your answer sheet to indicate the correct answer to each of the following questions.

I ______ for Christine. Do you know where she is?


A. looked B. looks
C. am looking D. look

16. Mark the letter A, B, C or D on your answer sheet to indicate the correct answer to each of the following questions.

While I was going to school yesterday, I____________ him


A. met B. will meet
C. am meeting D. meet

17. Mark the letter A, B, C or D on your answer sheet to indicate the correct answer to each of the following questions.

John ________ a book when I saw him


A. read B. is reading
C. reading D. was reading
18. Mark the letter A, B, C or D on your answer sheet to indicate the correct answer to each of the following questions.

It began to rain when they ________________basket ball at 4.00


A. is playing B. are playing
C. were playing D. play
19. Mark the letter A, B, C or D on your answer sheet to indicate the correct answer to each of the following questions.

Excuse me! I ____________ for a phone book. Is there one here?


A. am looking B. are looking
C. looking D. is looking
20. Mark the letter A, B, C or D on your answer sheet to indicate the correct answer to each of the following questions.

I ___________ Louisiana state University


A. attend B. attended
C. am attending D. was attending
21. Mark the letter A, B, C or D on your answer sheet to indicate the correct answer to each of the following questions.

Where is Peter? He ____________ a bath


A. talking B. taking
C. is talking D. is taking
22. Mark the letter A, B, C or D on your answer sheet to indicate the correct answer to each of the following questions.

I ________ along the street when I suddenly heard footsteps behind me


A. was walking B. walked
C. am walking D. walk
Trang 2/4
26
23. Mark the letter A, B, C or D on your answer sheet to indicate the correct answer to each of the following questions.

At 5 o’clock yesterday evening, I ________ my clothes


A. have ironed B. was ironing
C. am ironing D. ironed
24. Mark the letter A, B, C or D on your answer sheet to indicate the correct answer to each of the following questions.

When I last saw him, he ________ in London.


A. has been living B. has lived
C. was living D. is living
25. Mark the letter A, B, C or D on your answer sheet to indicate the correct answer to each of the following questions.

Why ____________ your coat today? It’s very warm today


A. you are wearing B. do you wear
C. are you wearing D. are you wear

26. Mark the letter A, B, C or D on your answer sheet to indicate the correct answer to each of the following questions.

Look! That man _________ to open the door of your car


A. try B. tried
C. has tried D. is trying
27. Mark the letter A, B, C or D on your answer sheet to indicate the correct answer to each of the following questions.

We _______ there when our father died


A. lived still B. were still living
C. still lived D. was still living
28. Mark the letter A, B, C or D on your answer sheet to indicate the correct answer to each of the following questions.

I’m going on holiday on Saturday. This time next week I _________ on a beach in the sea
A. should be lying B. am lying
C. will lie D. will be lying
29. Mark the letter A, B, C or D on your answer sheet to indicate the correct answer to each of the following questions.

When I last _______ Jane, she __________ to find a job


A. have seen/tried B. see/was trying
C. saw/tried D. saw/was trying

30. Mark the letter A, B, C or D on your answer sheet to indicate the correct answer to each of the following questions.

He fell down when he ______ towards the church


A. run B. was running
C. had run D. runs
31. Mark the letter A, B, C or D on your answer sheet to indicate the correct answer to each of the following questions.

The boys broke a window when they____________ football


A. were playing B. are playing
C. had played D. played

32. Mark the letter A, B, C or D on your answer sheet to indicate the correct answer to each of the following questions.

When I last saw him, he _____ in London


A. was living B. has lived
C. is living D. has been living
33. Mark the letter A, B, C or D on your answer sheet to indicate the correct answer to each of the following questions.

John ______ a book when I saw him


A. is reading B. was reading
C. read D. reading

34. Mark the letter A, B, C or D on your answer sheet to indicate the correct answer to each of the following questions.

Trang 3/4
27
It was noisy next door. Our neighbors _______ a party
A. have had B. were having
C. had D. had had

Trang 4/4
28
BÀI TẬP TỰ LUYỆN
CÁC THÌ HOÀN THÀNH VÀ HOÀN THÀNH TIẾP DIỄN
PEN-C TIẾNG ANH - CÔ HƯƠNG FIONA

1. Mark the letter A, B, C or D on your answer sheet to indicate the correct answer to each of the following questions.

Before you asked, the letter ________

A. had written B. had been written


C. has been written D. was written

2. Mark the letter A, B, C or D on your answer sheet to indicate the correct answer to each of the following questions.

Since _______, I have heard nothing from him

A. he was left B. he has left


C. he left D. he had left
3. Mark the letter A, B, C or D on your answer sheet to indicate the correct answer to each of the following questions.

When the first child was born, they ________ for three years

A. had been married B. have been married


C. will been married D. will have been married
4. Mark the letter A, B, C or D on your answer sheet to indicate the correct answer to each of the following questions.

Tim was tired. He ________ hard all day

A. has been studying B. studied


C. had been studying D. studies
5. Mark the letter A, B, C or D on your answer sheet to indicate the correct answer to each of the following questions.

Almost everyone ________ for home by the time we arrived

A. had left B. leave


C. leaves D. left
6. Mark the letter A, B, C or D on your answer sheet to indicate the correct answer to each of the following questions.

Ted and Amy ________ for 24 years

A. married B. has been married


C. have been married D. were married
7. Mark the letter A, B, C or D on your answer sheet to indicate the correct answer to each of the following questions.

I have never played badminton before. This is the first time I __________ .to play

A. have tried B. try


C. tried D. am trying
8. Mark the letter A, B, C or D on your answer sheet to indicate the correct answer to each of the following questions.

He has been selling motorbikes ________.


A. for ten years B. ten years ago
C. since ten years D. for ten years ago
9. Mark the letter A, B, C or D on your answer sheet to indicate the correct answer to each of the following questions.

I couldn’t cut the grass because the lawn mower ______ a few days previously
A. broke down B. had broken down
C. breaks down D. has been broken

10. Mark the letter A, B, C or D on your answer sheet to indicate the correct answer to each of the following questions.

Trang 1/5
29
We _______ Doris since last Sunday

A. haven’t seen B. didn’t see


C. hadn’t seen D. don’t see
11. Mark the letter A, B, C or D on your answer sheet to indicate the correct answer to each of the following questions.

Jack ______ the door

A. will have painted B. has just painted


C. painting D. paint
12. Mark the letter A, B, C or D on your answer sheet to indicate the correct answer to each of the following questions.

She ________ German for two years

A. has learnt B. learn


C. is learning D. learns
13. Mark the letter A, B, C or D on your answer sheet to indicate the correct answer to each of the following questions.

My teacher arrived after I ____________ for him for ten minutes


A. have waited B. Was waiting
C. had been waiting D. waited
14. Mark the letter A, B, C or D on your answer sheet to indicate the correct answer to each of the following questions.

Jack ______ the door for a few seconds

A. painting B. will have painted


C. has just painted D. paint
15. Mark the letter A, B, C or D on your answer sheet to indicate the correct answer to each of the following questions.

For several years his ambition ________ to be a pilot

A. is B. has been
C. had been D. was
16. Mark the letter A, B, C or D on your answer sheet to indicate the correct answer to each of the following questions.

I ______ to New York three times this year

A. have been B. had been


C. were D. was

17. Mark the letter A, B, C or D on your answer sheet to indicate the correct answer to each of the following questions.

Ask her to come and see me when she _________ her work

A. finish B. has finished


C. finished D. finishing

18. Mark the letter A, B, C or D on your answer sheet to indicate the correct answer to each of the following questions.

By the age of 25, he ______ two famous novels

A. had written B. writes


C. has written D. wrote
19. Mark the letter A, B, C or D on your answer sheet to indicate the correct answer to each of the following questions.

I _________ to New York three times this year

A. was B. have been


C. were D. had been
20. Mark the letter A, B, C or D on your answer sheet to indicate the correct answer to each of the following questions.

Trang 2/5
30
I’m feeling sick. I ________ so much chocolate last night
A. mustn’t eat B. shouldn’t have eaten
C. did not eat D. needn’t to eat
21. Mark the letter A, B, C or D on your answer sheet to indicate the correct answer to each of the following questions.

Daniel ______ the bus. It doesn’t run on Sundays


A. had not caught B. couldn’t have caught
C. did not catch D. have not caught
22. Mark the letter A, B, C or D on your answer sheet to indicate the correct answer to each of the following questions.

He has been selling motorcycles ________

A. ten years ago B. for ten years ago


C. for ten years D. since ten years
23. Mark the letter A, B, C or D on your answer sheet to indicate the correct answer to each of the following questions.

I ______ to the same barber since last year.

A. am going B. go
C. had gone D. have been going
24. Mark the letter A, B, C or D on your answer sheet to indicate the correct answer to each of the following questions.

The little girl asked what ________ to her friend.

A. would have been happened B. happened


C. had happened D. has happened
25. Mark the letter A, B, C or D on your answer sheet to indicate the correct answer to each of the following questions.

After I ________ lunch, I looked for my bag

A. had had B. had


C. have had D. have has

26. Mark the letter A, B, C or D on your answer sheet to indicate the correct answer to each of the following questions.

Their football team___________ a championship until last season

A. had never been winning B. is never winning


C. have never won D. had never won
27. Mark the letter A, B, C or D on your answer sheet to indicate the correct answer to each of the following questions.

I have been waiting for you ________

A. since 9 am B. for two hours


C. all are correct D. since early morning
28. Mark the letter A, B, C or D on your answer sheet to indicate the correct answer to each of the following questions.

By the end of next year, George _______ English for two years

A. will learn B. has learned


C. would learn D. will have learned
29. Mark the letter A, B, C or D on your answer sheet to indicate the correct answer to each of the following questions.

Almost everyone _______ for home by the time we arrived

A. leaves B. leave
C. left D. had left

30. Mark the letter A, B, C or D on your answer sheet to indicate the correct answer to each of the following questions.

The little girl asked what _______ to her friend


Trang 3/5
31
A. had happened B. has happened
C. would have been happened D. happened

31. Mark the letter A, B, C or D on your answer sheet to indicate the correct answer to each of the following questions.

We _______ there since our father died


A. were still living B. lived still
C. was still lived D. have lived

32. Mark the letter A, B, C or D on your answer sheet to indicate the correct answer to each of the following questions.

By the end of next year, Mike _________ English for 2 years

A. would learn B. will learn


C. has learned D. will have learned
33. Mark the letter A, B, C or D on your answer sheet to indicate the correct answer to each of the following questions.

By Christmas, I _______ for Mr. Smith for six years

A. shall work B. shall be working


C. have been working D. shall have been working
34. Mark the letter A, B, C or D on your answer sheet to indicate the correct answer to each of the following questions.

After I _______ lunch, I looked for my bag

A. had B. have has


C. have had D. had had
35. Mark the letter A, B, C or D on your answer sheet to indicate the correct answer to each of the following questions.

We _______ Dorothy since last Saturday

A. haven’t seen B. don’t see


C. hadn’t seen D. didn’t see
36. Mark the letter A, B, C or D on your answer sheet to indicate the correct answer to each of the following questions.

Yesterday I passed by Peter’s house, but the front door was closed. He _______ out

A. had had B. has gone


C. went D. must have gone
37. Mark the letter A, B, C or D on your answer sheet to indicate the correct answer to each of the following questions.

My sister ________ for you since yesterday

A. was looking B. looked


C. is looking D. has been looking

38. Mark the letter A, B, C or D on your answer sheet to indicate the correct answer to each of the following questions.

Jack _______ the door

A. open B. will have opened


C. opening D. has just opened

39. Mark the letter A, B, C or D on your answer sheet to indicate the correct answer to each of the following questions.

At last the bus came. We _________ for half an hour.

A. waited B. had been waiting


C. have waited D. was waiting
40. Mark the letter A, B, C or D on your answer sheet to indicate the correct answer to each of the following questions.

According to this newspaper, John is said ________ a new record for the long jump
Trang 4/5
32
A. to have established B. establishing
C. established D. to establish

Trang 5/5
33
BÀI TẬP TỰ LUYỆN
SỰ KẾT HỢP GIỮA CÁC THÌ
PEN-C TIẾNG ANH - CÔ HƯƠNG FIONA

1. Mark the letter A, B, C or D on your answer sheet to indicate the correct answer to each of the following questions

Your suitcase is so big. I ________ it for you


A. have taken B. will take
C. am going to take D. take
2. Mark the letter A, B, C or D on your answer sheet to indicate the correct answer to each of the following questions

Look! The man in black _______ to break the car.


A. tried B. is trying
C. has tried D. tries
3. Mark the letter A, B, C or D on your answer sheet to indicate the correct answer to each of the following questions

Don’t call him at 2:30 p.m. He _________ job applicants at that time.
A. will interview B. has interviewed
C. will be interviewing D. will have interviewed

4. Mark the letter A, B, C or D on your answer sheet to indicate the correct answer to each of the following questions

__________ your work by 9 tonight?

A. Will you finish B. Have you finished


C. Do you finish D. Will you have finished
5. Mark the letter A, B, C or D on your answer sheet to indicate the correct answer to each of the following questions

The bell ______ while Tom ______ a bath


A. was ringing / was taking B. rung / was taking
C. rang / was taking D. was ringing/ tooks

6. Mark the letter A, B, C or D on your answer sheet to indicate the correct answer to each of the following questions

When you come tomorrow, I ___________ at my desk


A. will work B. am working
C. will have worked D. will be working
7. Mark the letter A, B, C or D on your answer sheet to indicate the correct answer to each of the following questions

Where _____________ this morning? It’s nearly mid day


A. were you B. have you been
C. are D. will you be

8. Mark the letter A, B, C or D on your answer sheet to indicate the correct answer to each of the following questions

The last time I _________ him was three years ago


A. see B. have seen
C. saw D. seen
9. Mark the letter A, B, C or D on your answer sheet to indicate the correct answer to each of the following questions

She ______ to go for a walk at the moment.

A. is wanting B. wanted
C. wants D. has wanted
10. Mark the letter A, B, C or D on your answer sheet to indicate the correct answer to each of the following questions

What ____________ about before I arrived?

A. were you just talking B. did you just talk


C. are you talking D. have you talked

Trang 1/3
34
11. Mark the letter A, B, C or D on your answer sheet to indicate the correct answer to each of the following questions

If I see him, I _______ him about that


A. remind B. am going to remind
C. will remind D. have reminded
12. Mark the letter A, B, C or D on your answer sheet to indicate the correct answer to each of the following questions

We ________ English classes every other day


A. are having B. had
C. have had D. have
13. Mark the letter A, B, C or D on your answer sheet to indicate the correct answer to each of the following questions

I __________ you someday.


A. see B. saw
C. will see D. am seeing
14. Mark the letter A, B, C or D on your answer sheet to indicate the correct answer to each of the following questions

When I got up this morning, I ___________ that I ________ to prepare breakfast


A. realized / am assigning B. was realized / assigned
C. was realizing / was assigned D. realized / was assigned

15. Mark the letter A, B, C or D on your answer sheet to indicate the correct answer to each of the following questions

I usually _______ coffee; however, I __________ tea today instead.

A. drinks / am drinking B. drink / drink


C. am drinking / am drinking D. drink / am drinking

16. Mark the letter A, B, C or D on your answer sheet to indicate the correct answer to each of the following questions

This was a beautiful day. The sun ________ and the birds _______ in the sky
A. shone / sang B. shined / sang
C. was shining / was singing D. was shining / were singing
17. Mark the letter A, B, C or D on your answer sheet to indicate the correct answer to each of the following questions

Over the last decade, the pollution problem ____________ worse.


A. has been getting B. got
C. gets D. will get
18. Mark the letter A, B, C or D on your answer sheet to indicate the correct answer to each of the following questions

Hurry up or we ________ late for school


A. are B. are going to be
C. will be D. will have been
19. Mark the letter A, B, C or D on your answer sheet to indicate the correct answer to each of the following questions

We ________ for you when you ________ back tomorrow.


A. have been waiting / come B. will wait / will come
C. will be waiting / come D. will be waiting / will come

20. Mark the letter A, B, C or D on your answer sheet to indicate the correct answer to each of the following questions

My grandfather ________ for this company for over thirty years then he retired
A. works B. worked
C. is working D. has worked
21. Mark the letter A, B, C or D on your answer sheet to indicate the correct answer to each of the following questions

I want to get married, but I ____________ the right person yet


A. haven’t meet B. didn’t met
C. haven’t met D. didn’t meet
22. Mark the letter A, B, C or D on your answer sheet to indicate the correct answer to each of the following questions

Trang 2/3
35
By the time your brother ___________ here next year, the city will have changed a lot
A. will have returned B. will return
C. returns D. return
23. Mark the letter A, B, C or D on your answer sheet to indicate the correct answer to each of the following questions

What _________ when you saw her?

A. did she read B. has she read


C. was she reading D. had she read
24. Mark the letter A, B, C or D on your answer sheet to indicate the correct answer to each of the following questions

Five kilometres _________ a long way for a little boy to walk


A. is B. are
C. have been D. has been

25. Mark the letter A, B, C or D on your answer sheet to indicate the correct answer to each of the following questions

Mr. Green _______ English in this school since he _______ from the university in 1986
A. teaches / graduates B. taught / graduated
C. has taught / has graduated D. has taught / graduated
26. Mark the letter A, B, C or D on your answer sheet to indicate the correct answer to each of the following questions

The train from Hanoi to Ho Chi Minh city _______ at 10 o’clock tomorrow morning
A. will depart B. departs
C. depart D. will be departing
27. Mark the letter A, B, C or D on your answer sheet to indicate the correct answer to each of the following questions

Where _____________ all day? I ___________ you everywhere


A. have you been / am looking for B. were you / looked for
C. are you / have looked for D. have you been / have been looking for
28. Mark the letter A, B, C or D on your answer sheet to indicate the correct answer to each of the following questions

As we _______ the street, we _________ an accident


A. crossed / saw B. were crossing / were seeing
C. crossed / were seeing D. were crossing /saw
29. Mark the letter A, B, C or D on your answer sheet to indicate the correct answer to each of the following questions

By next month, I _______ my first novel


A. will finish B. have finished
C. will be finishing D. will have finished
30. Mark the letter A, B, C or D on your answer sheet to indicate the correct answer to each of the following questions

The light ______ out while we ______ dinner


A. was going / were having B. went / had
C. was going / had D. went / were having

Trang 3/3
36
BÀI TẬP TỰ LUYỆN
VOCABULARY - POWER
PEN-C TIẾNG ANH - CÔ HƯƠNG FIONA

1. Read the following passage and type the letter A, B, C or D into the blanks to indicate the correct answer to each of the following
questions from 1 to 5.

In the world today, particularly in the two most industrialized areas, North America and Europe, recycling is big news. People are talking
about it, practicing it, and discovering new ways to be sensitive to the environment. Recycling means finding was to use products a second
time. The motto of the recycling movement is "Reduce, Reuse, Recycle".

The first step is to reduce garbage. In stores, a shopper has to buy products in blister packs, boxes and expensive plastic wrappings. A
hamburger from a fast food restaurant comes in lots of packaging: usually paper, a box, and a bag. All that packaging is wasted resources.
People should try to buy things that are wrapped simply, and to reuse cups and utensils. Another way to reduce waste is to buy high-quality
products. When low quality appliances break, many customers throw them away and buy new ones - a loss of more resources and more
energy. For example, if a customer buys a high-quality appliance that can be easily repaired, the manufacturer receives an important
message. In the same way, if a customer chooses a product with less packaging, that customer sends an important message to the
manufacturers. To reduce garbage, the throwaway must stop.

The second step is to reuse. It is better to buy juices and soft drinks in returnable bottles. After customers empty the bottles, they return
them to the store. The manufacturers of the drinks collect the bottles, wash them, and then fill them again. The energy that is necessary to
make new bottles is saved. In some parts of the world, returning bottles for money is a common practice. In those places, the garbage
dumps have relatively little glass and plastic from throwaway bottles.

The third step is being environmentally sensitive is to recycle. Spent motor oil can be cleaned and used again. Aluminum cans are
expensive to make. It takes the same amount of energy to make one aluminum can as it does to run a color TV set for three hours. When
people collect and recycle aluminum (for new cans), they help save one of the world's precious resources.

Question 1. What is the main topic of the passage?


A. How to reduce garbage disposal
B. What people often understand about the term 'recycle'
C. What is involved in the recycling movement
D. How to live sensitively to the environment
Question 2. People can do the following to reduce waste EXCEPT
A. buy high-quality product
B. buy simply-wrapped things
C. reuse cups
D. buy fewer hamburgers
Question 3. Why is it a waste when customers buy low-quality products?
A. Because people will soon throw them away.
B. Because they have to be repaired many times.
C. Because customers change their ideas all the time.
D. Because they produce less energy.
Question 4. What best describes the process of reuse?
A. The bottles are collected, washed, returned and filled again.
B. The bottles are filled again after being returned, collected and washed.
C. The bottles are washed, retuned, filled again and collected.
D. The bottles are collected, returned filled again and washed.
Question 5. What are the two things mentioned as examples of recycling?
A. Aluminum cans and plastic wrappings.
B. Hamburger wrappings and spent motor oil.
C. Aluminum cans and spent motor oil.
D. TV sets and aluminum cans.
2. Read the following passage and type the letter A, B, C, or D into the blanks to indicate the best option for each of the blanks.

If you're an environmentalist, plastic is a word you tend to say with a sneer or a snarl. It has become a symbol of our wasteful, throw-away
society. But there seems little doubt it is here to stay, and the truth is, of course, that plastic has brought enormous (1)
, even environmental ones. It’s not really the plastics themselves that are the evil ─ it's the way society chooses to
use and (2) them.
Almost all the 50 or so different kinds of modern plastic are made from oil, gas or coal ─ non-renewable natural (3) .
We (4) well over three million tones of the stuff in Britain each year and, sooner or later, most of it is thrown away.
A high (5) of our annual consumption is in the form of packaging, and this (6) about seven
percent by weight of our domestic refuse.

Trang 1/2
37
Question 1. A. savings B. pleasures C. benefits D. profits
Question 2. A. abuse B. endanger C. store D. dispose
Question 3. A. processesB. resources C. products D. fuels
Question 4. A. import B. consign C. remove D. consume
Question 5. A. amount B. proportion C. portion D. rate
Question 6. A. makes B. carries C. takes D. constitutes

3. Read the following passage and type the letter A, B, C, or D into the blanks to indicate the correct word or phrase that best fits
each of the numbered blanks from 1 to 5.

NOISE POLLUTION

Noise is more than a mere nuisance. At certain levels and durations of exposure, it can cause physical damage to the eardrum, and (1)
in temporary or permanent hearing loss. In addition to causing hearing loss, (2) noise
exposure can also (3) blood pressure and pulse rate, cause irritability, anxiety, and mental fatigue, and interfere with
sleep, recreation, and personal communication. Noise pollution is, therefore, (4) importance in the workplace and in
the community.
Noise effects can be (5) by a number of techniques, for example, increasing the distance or blocking the path
between the noise source and the recipient, reducing noise level at the source, and protecting recipients with earplugs or earmuffs.

Question 1: A. bring B. result C. create D. affect


Question 2: A. exceedingly B. excess C. excessively D. excessive
Question 3: A. enlarge B. maximize C. raise D. rise
Question 4: A. for B. of C. at D. within
Question 5: A. minimized B. created C. brought D. done

Trang 2/2
38
BÀI TẬP TỰ LUYỆN
SỰ HÒA HỢP CHỦ VỊ
PEN-C TIẾNG ANH - CÔ HƯƠNG FIONA

1. Mark the letter A, B, C or D on your answer sheet to indicate the correct answer to each of the following questions

The number of unemployed people __________ recently


A. increase B. have increased
C. has increased D. is increasing
2. Mark the letter A, B, C or D on your answer sheet to indicate the correct answer to each of the following questions

Both Tuan and Thanh has been to Paris before


A. before B. has
C. and D. Both
3. Mark the letter A, B, C or D on your answer sheet to indicate the correct answer to each of the following questions

The poet and the writer _____ from London


A. comes B. come
C. is coming D. has came

4. Mark the letter A, B, C or D on your answer sheet to indicate the correct answer to each of the following questions

Every man and woman _____the same right


A. are B. have
C. has D. were
5. Mark the letter A, B, C or D on your answer sheet to indicate the correct answer to each of the following questions

The existence of many stars in the sky _____ us to suspect that there may be life on another planet.
A. have led B. leading
C. leads D. lead
6. Mark the letter A, B, C or D on your answer sheet to indicate the correct answer to each of the following questions

Everyone in both cars ________ injured in the accident last night, ________?
A. were/ weren't they B. was/ wasn't he
C. was/ weren't they D. were/ were they
7. Mark the letter A, B, C or D on your answer sheet to indicate the correct answer to each of the following questions

It was announced that neither the passengers nor the driver _____ in the crash
A. are injured B. have been injured
C. was injured D. were injured

8. Mark the letter A, B, C or D on your answer sheet to indicate the correct answer to each of the following questions

A number of students ____ coming to the seminar


A. has been B. are
C. is D. was
9. Mark the letter A, B, C or D on your answer sheet to indicate the correct answer to each of the following questions

Only one of our gifted students _____ to participate in the final competition
A. have been chosen B. has been chosen
C. were choosing D. chosen
10. Mark the letter A, B, C or D on your answer sheet to indicate the correct answer to each of the following questions

This orange juice ___ awful. I bet you ___ it


A. has smelled/ liked B. smelling/ will not like
C. smells/ will not like D. is smelling/ like
11. Mark the letter A, B, C or D on your answer sheet to indicate the correct answer to each of the following questions

Psychologists have found that the number of social contacts we have ______ only reason for loneliness

Trang 1/4
39
A. are not the B. is not an
C. is not the D. are not an
12. Mark the letter A, B, C or D on your answer sheet to indicate the correct answer to each of the following questions

Air pollution, together with littering, are causing many problems in your large, industrial cities today.
A. industrial cities B. are causing
C. many problems D. with littering
13. Mark the letter A, B, C or D on your answer sheet to indicate the correct answer to each of the following questions

Not only the athlete but also his wife are going to the party tonight
A. not only B. are
C. the D. but

14. Mark the letter A, B, C or D on your answer sheet to indicate the correct answer to each of the following questions

The amounts of oxygen and nitrogen in the air almost always remain stable, but the amount of water vapor vary considerably
A. stable B. almost always
C. The amounts of D. vary
15. Mark the letter A, B, C or D on your answer sheet to indicate the correct answer to each of the following questions

My dog ________ my cats eats twice a day


A. nor B. or
C. both D. as well as
16. Mark the letter A, B, C or D on your answer sheet to indicate the correct answer to each of the following questions

A large number of reporters ________ at the meeting yesterday


A. is B. were
C. are D. being
17. Mark the letter A, B, C or D on your answer sheet to indicate the correct answer to each of the following questions

Neither Tom nor his brothers _____ willing to help their mother with the housework
A. has been B. was
C. are D. is
18. Mark the letter A, B, C or D on your answer sheet to indicate the correct answer to each of the following questions

The shocking news in newspapers are what people are talking about this morning
A. are B. in
C. shocking D. what

19. Mark the letter A, B, C or D on your answer sheet to indicate the correct answer to each of the following questions

The Prime Minister, accompanied by the Ministers, ______ going to the meeting
A. is B. are
C. were D. have
20. Mark the letter A, B, C or D on your answer sheet to indicate the correct answer to each of the following questions

What we know about certain diseases are still not sufficient to prevent them from spreading easily among the population
A. are B. among
C. What we know about D. spreading

21. Mark the letter A, B, C or D on your answer sheet to indicate the correct answer to each of the following questions

Neither the students nor their lecturer___________English in the classroom


A. use B. have used
C. are using D. uses
22. Mark the letter A, B, C or D on your answer sheet to indicate the correct answer to each of the following questions

Since the flood the number of homeless people _______ dramatically


A. has increased B. had increased
C. increase D. are increasing
Trang 2/4
40
23. Mark the letter A, B, C or D on your answer sheet to indicate the correct answer to each of the following questions

The superstar, accompanied by the other members of the band, _____ to visit our school next week.
A. are B. has had
C. are going D. is going
24. Mark the letter A, B, C or D on your answer sheet to indicate the correct answer to each of the following questions

The number of working women _______ from 49% in 1980 to nearly 65% in China today
A. has been increased B. is increasing
C. has increased D. have increased
25. Mark the letter A, B, C or D on your answer sheet to indicate the correct answer to each of the following questions

They was told to plan everything very carefully before the journey
A. was B. carefully
C. journey D. everything

26. Mark the letter A, B, C or D on your answer sheet to indicate the correct answer to each of the following questions

________ have made communication faster and easier through the use of email and the Internet is widely recognized
A. That computers B. Computers that
C. That it’s computers D. It is that computers
27. Mark the letter A, B, C or D on your answer sheet to indicate the correct answer to each of the following questions

Neither you nor I _____ responsible for the bad result.


A. be B. are
C. am D. is
28. Mark the letter A, B, C or D on your answer sheet to indicate the correct answer to each of the following questions

The number of cars on the roads _____ increasing, so we need to build more roads
A. was B. were
C. is D. are
29. Mark the letter A, B, C or D on your answer sheet to indicate the correct answer to each of the following questions

If either of you take a vacation now, we won’t be able to finish this work
A. able to finish B. take
C. won’t be D. either

30. Mark the letter A, B, C or D on your answer sheet to indicate the correct answer to each of the following questions

Three years ____ too long for me to school


A. were B. have
C. is D. are
31. Mark the letter A, B, C or D on your answer sheet to indicate the correct answer to each of the following questions

Mrs. Stevens, along with her cousins from New Mexico, are planning to attend the festivies.
A. are B. her cousins
C. along with D. to attend
32. Mark the letter A, B, C or D on your answer sheet to indicate the correct answer to each of the following questions

The world is becoming more industrialized and the number of animal species that have become extinct have increased
A. extinct B. industrialized
C. have D. species

33. Mark the letter A, B, C or D on your answer sheet to indicate the correct answer to each of the following questions

The students, along with the teacher, _____ coming to the meeting
A. has been B. was
C. is D. are

34. Mark the letter A, B, C or D on your answer sheet to indicate the correct answer to each of the following questions

Trang 3/4
41
To meet the customers’ demand _____ our first priority
A. is B. to be
C. were D. are

35. Mark the letter A, B, C or D on your answer sheet to indicate the correct answer to each of the following questions

The only excuse that he gave for his actions ____ that he wanted to protect himself
A. were B. are
C. is D. was

36. Mark the letter A, B, C or D on your answer sheet to indicate the correct answer to each of the following questions

The assumption that smoking has bad effects on our health have been proved
A. have been proved B. smoking
C. on D. effects

37. Mark the letter A, B, C or D on your answer sheet to indicate the correct answer to each of the following questions

Today the number of people who enjoys winter sports is almost double that of twenty
A. that of B. almost double
C. who enjoys D. number

38. Mark the letter A, B, C or D on your answer sheet to indicate the correct answer to each of the following questions

Buying clothes are often a very time-consuming practice because those clothes that a person likes are rarely the ones that fit him or her
A. are B. a very time-consuming
C. are rarely the ones D. because those

39. Mark the letter A, B, C or D on your answer sheet to indicate the correct answer to each of the following questions

Each of the members of the group were made to write a report every week
A. to write B. were
C. week D. members

40. Mark the letter A, B, C or D on your answer sheet to indicate the correct answer to each of the following questions

On the floor of the Pacific Ocean is hundreds of flat-topped mountains more than a mile beneath sea level.
A. is B. On
C. hundreds of D. more than

Trang 4/4
42
BÀI TẬP TỰ LUYỆN
DANH ĐỘNG TỪ
PEN-C TIẾNG ANH - CÔ HƯƠNG FIONA

1. Mark the letter A, B, C or D on your answer sheet to indicate the correct answer to each of the following questions

Suddenly he stopped the car in order ______.


A. to smoke B. smoke
C. has smoked D. smoking
2. Mark the letter A, B, C or D on your answer sheet to indicate the correct answer to each of the following questions

I’ve put off _____ the letters so many times. I really must do it today
A. writing B. write
C. to write D. written
3. Mark the letter A, B, C or D on your answer sheet to indicate the correct answer to each of the following questions

You can’t stop me from _____ what I want.


A. do B. to do
C. doing D. that I do

4. Mark the letter A, B, C or D on your answer sheet to indicate the correct answer to each of the following questions

That old man tried to stop _____ because of his bad health
A. smoked B. smoking
C. smoke D. smokes
5. Mark the letter A, B, C or D on your answer sheet to indicate the correct answer to each of the following questions

What a stupid thing to do! Can you imagine anybody _____so silly.
A. be B. to be
C. being D. were
6. Mark the letter A, B, C or D on your answer sheet to indicate the correct answer to each of the following questions

Can you manage_____ ______ the report in the morning?


A. finishing / writing B. finishing / to write
C. to finish / to write D. to finish / writing
7. Mark the letter A, B, C or D on your answer sheet to indicate the correct answer to each of the following questions

Rex hotel was the best place for us _____.


A. stay B. to stay
C. stays D. staying

8. Mark the letter A, B, C or D on your answer sheet to indicate the correct answer to each of the following questions

Have you considered _____ to live in another country?


A. to go B. goes
C. gone D. going
9. Mark the letter A, B, C or D on your answer sheet to indicate the correct answer to each of the following questions

Jim is 65, but he isn’t going to retired yet. He wants to carry on _____.
A. to work B. work
C. working D. works

10. Mark the letter A, B, C or D on your answer sheet to indicate the correct answer to each of the following questions

Have you finished ______ your hair yet?


A. washed B. to wash
C. washing D. wash
11. Mark the letter A, B, C or D on your answer sheet to indicate the correct answer to each of the following questions

Would you mind _____ the door?

Trang 1/4
43
A. open B. opened
C. opening D. to open
12. Mark the letter A, B, C or D on your answer sheet to indicate the correct answer to each of the following questions

If you walk into the road without looking, you risk _____ knocked down
A. be B. to be
C. being D. been
13. Mark the letter A, B, C or D on your answer sheet to indicate the correct answer to each of the following questions

Cathy suggested _____ to the cinema


A. not to go B. went
C. not go D. going

14. Mark the letter A, B, C or D on your answer sheet to indicate the correct answer to each of the following questions

The machine needs_____.


A. repairing B. to be repaired
C. to repair D. "repairing" and "to be repaired" are correct
15. Mark the letter A, B, C or D on your answer sheet to indicate the correct answer to each of the following questions

I like _____ the kitchen as often as possible

A. not cleaned B. clean


C. that I clean D. to clean

16. Mark the letter A, B, C or D on your answer sheet to indicate the correct answer to each of the following questions

Do you want ______ with you or do you want to go alone?


A. that I will come B. me coming
C. me to come D. that I come
17. Mark the letter A, B, C or D on your answer sheet to indicate the correct answer to each of the following questions

I must go now. I promise _______ late.


A. not being B. to not be
C. not to be D. I won’t be
18. Mark the letter A, B, C or D on your answer sheet to indicate the correct answer to each of the following questions

I enjoy ______ to music


A. to listen B. listen
C. listening D. listened

19. Mark the letter A, B, C or D on your answer sheet to indicate the correct answer to each of the following questions

Remember _____ John a present. Today is his birthday


A. sent B. to send
C. sending D. send
20. Mark the letter A, B, C or D on your answer sheet to indicate the correct answer to each of the following questions

Sarah gave up ______ to find a job in this country and decided to go abroad
A. apply B. applying
C. to apply D. application

21. Mark the letter A, B, C or D on your answer sheet to indicate the correct answer to each of the following questions

I suggest _____ some more mathematical puzzles


A. doing B. done
C. to do D. do
22. Mark the letter A, B, C or D on your answer sheet to indicate the correct answer to each of the following questions

Mr. Thomas does not allow people_____ in his office

Trang 2/4
44
A. smoked B. smoking
C. to smoke D. smoke
23. Mark the letter A, B, C or D on your answer sheet to indicate the correct answer to each of the following questions

Could you please stop _______ so much noise?


A. make B. to have make
C. to make D. making

24. Mark the letter A, B, C or D on your answer sheet to indicate the correct answer to each of the following questions

I remember _____ but he said I did not


A. buy him a book B. buying him a book
C. to buy him a book D. to have bought him a book
25. Mark the letter A, B, C or D on your answer sheet to indicate the correct answer to each of the following questions

Hello! Fancy _____ you here! What a surprise!


A. to see B. see
C. seeing D. seen
26. Mark the letter A, B, C or D on your answer sheet to indicate the correct answer to each of the following questions

You should give up _____ or you will die of cancer


A. smoke B. smoked
C. to smoke D. smoking
27. Mark the letter A, B, C or D on your answer sheet to indicate the correct answer to each of the following questions

I couldn’t help _____ when I heard the story


A. laughed B. to laugh
C. laughing D. laugh
28. Mark the letter A, B, C or D on your answer sheet to indicate the correct answer to each of the following questions

Peter usually helps his mother ______.


A. to do housework B. all are correct
C. do housework D. with housework
29. Mark the letter A, B, C or D on your answer sheet to indicate the correct answer to each of the following questions

My mother was afraid to let the boy _____ the tree


A. to risk clim B. risk climbing
C. to risk to climb D. risk to climb

30. Mark the letter A, B, C or D on your answer sheet to indicate the correct answer to each of the following questions

I can’t bear ______this dirty room


A. to see B. seen
C. seeing D. saw
31. Mark the letter A, B, C or D on your answer sheet to indicate the correct answer to each of the following questions

We are looking forward _____ you again soon


A. to see B. saw
C. have seen D. to seeing

32. Mark the letter A, B, C or D on your answer sheet to indicate the correct answer to each of the following questions

I’m sure I locked the door. I clearly remember _____ it.


A. to lock B. to be locked
C. locking D. to have locked
33. Mark the letter A, B, C or D on your answer sheet to indicate the correct answer to each of the following questions

There is nothing prevent us from______ the mystery of that palace


A. discovering B. discoveries
C. discover D. discovery
Trang 3/4
45
34. Mark the letter A, B, C or D on your answer sheet to indicate the correct answer to each of the following questions

We are not allowed _____ in that restricted area.


A. entering B. to entered
C. to enter D. enter

35. Mark the letter A, B, C or D on your answer sheet to indicate the correct answer to each of the following questions

They caused her _____ a lot.


A. cried B. cry
C. crying D. to cry
36. Mark the letter A, B, C or D on your answer sheet to indicate the correct answer to each of the following questions

She used that money _____ a new car


A. buying B. buy
C. for buying D. to buy

37. Mark the letter A, B, C or D on your answer sheet to indicate the correct answer to each of the following questions

You had better _____ at home until you feel better


A. stay B. to stay
C. stayed D. staying
38. Mark the letter A, B, C or D on your answer sheet to indicate the correct answer to each of the following questions

He tried to avoid ______ my question


A. how to answer B. to answer
C. answer D. answering
39. Mark the letter A, B, C or D on your answer sheet to indicate the correct answer to each of the following questions

The driver stopped _____ a coffee because he felt sleepy.


A. having B. have
C. to have D. had
40. Mark the letter A, B, C or D on your answer sheet to indicate the correct answer to each of the following questions

She tried to be serious, but she couldn’t help ______.


A. that she laughed B. to laugh
C. laughing D. laugh

Trang 4/4
46
BÀI TẬP TỰ LUYỆN
BÀI. ENVIRONMENT
PEN-C TIẾNG ANH - CÔ HƯƠNG FIONA

1. Read the following passage and mark the letter A, B, C or D on your answer sheet to indicate the correct word or phrase that best
fits each of the numbered blanks from 1 to 5.

The warming of the Earth is caused by exhaust gas from automobile engines, fatories and power (1) . Carbon
dioxide goes up into the atmosphere, and it form a kind of screen that keeps or allows the sunshine in but stop the Earth heart (2)
getting out. It works like a greenhouse, that’s why we call (3) the Green House effect.
Because of this effect, the Earth is getting warmer all the time. This (4) in temperature will cause big changes to the
world’s climate. The sea level will increase as the ice (5) the poles will melt.

Question 1:
A. companies
B. factories
C. sites
D. Stations
Question 2:
A. from
B. up
C. against
D. Away
Question 3:
A. is
B. be
C. it
D. Them
Question 4:
A. raise
B. rise
C. drop
D. fall
Question 5:
A. covering
B. covers
C. covered
D. Cover
2. Read the following passage and mark the letter A, B, C, or D on your answer sheet to indicate the correct answer to each of the
questions.

For many people who live in cities, parks are an important part of the landscape. They provide a place for people to relax and play sports,
as well as a refuge from the often harsh environment of a city. What people often overlook is that parks also provide considerable
environmental benefits.
One benefit of parks is that plants absorb carbon dioxide—a key pollutant—and emit oxygen, which humans need to breathe. According to
one study, an acre of trees can absorb the same amount of carbon dioxide that a typical car emits in 11,000 miles of driving. Parks also
make cities cooler. Scientists have long noted what is called the Urban Heat Island Effect: building materials such as metal, concrete, and
asphalt absorb much more of the sun’s heat and release it much more quickly than organic surfaces like trees and grass. Because city
landscapes contain so much of these building materials, cities are usually warmer than surrounding rural areas. Parks and other green
spaces help to mitigate the Urban Heat Island Effect.
Unfortunately, many cities cannot easily create more parks because most land is already being used for buildings, roads, parking lots, and
other essential parts of the urban environment. However, cities could benefit from many of the positive effects of parks by encouraging
citizens to create another type of green space: rooftop gardens. While most people would not think of starting a garden on their roof,
human beings have been planting gardens on rooftops for thousands of years. Some rooftop gardens are very complex and require
complicated engineering, but others are simple container gardens that anyone can create with the investment of a few hundred dollars and a
few hours of work.
Rooftop gardens provide many of the same benefits as other urban park and garden spaces, but without taking up the much-needed land.
Like parks, rooftop gardens help to replace carbon dioxide in the air with nourishing oxygen. They also help to lessen the Urban Heat
Island Effect, which can save people money. In the summer, rooftop gardens prevent buildings from absorbing heat from the sun, which
can significantly reduce cooling bills. In the winter, gardens help hold in the heat that materials like brick and concrete radiate so quickly,
leading to savings on heating bills. Rooftop vegetable and herb gardens can also provide fresh food for city dwellers, saving them money
and making their diets healthier. Rooftop gardens are not only something everyone can enjoy, they are also a smart environmental
investment.

Question 1: Based on its use in paragraph 2, it can be inferred that mitigate belongs to which of the following word groups
?
A. exacerbate, aggravate, intensify
B. obliterate, destroy, annihilate Trang 1/4
47
C. allay, alleviate, reduce
D. absorb, intake, consume
Question 2: Using the information in paragraph 2 as a guide, it can be inferred that .
A. cities with rooftop gardens are cooler than those without rooftop gardens
B. some plants are not suitable for growth in rooftop gardens
C. most people prefer parks to rooftop gardens
D. most people prefer life in the country over life in the city
Question 3: Based on the information in paragraph 3, which of the following best describes the main difference between parks and
rooftop gardens?
A. Parks are expensive to create while rooftop gardens are not.
B. Parks are public while rooftop gardens are private.
C. Parks absorb heat while rooftop gardens do not.
D. Parks require much space while rooftop gardens do not.
Question 4: The author claims all of the following to be the benefits of rooftop gardens except .
A. increased space for private relaxation
B. savings on heating and cooling costs
C. better food for city dwellers
D. improved air quality
Question 5: According to the author, one advantage that rooftop gardens have over parks is that they .
A. decrease the Urban Heat Island Effect
B. replenish the air with nourishing oxygen
C. do not require the use of valuable urban land
D. are less expensive than traditional park spaces
Question 6: The author’s tone in the passage is best described as
A. descriptive
B. passionate
C. informative
D. argumentative
Question 7: It can be inferred from the passage that the author would most likely endorse a program that .
A. permitted the construction of buildings in city park land provided they have rooftop gardens
B. extended discounts on plants to customers who use them to create rooftop gardens
C. offered free admission to schools willing to take their students on field trips to the city park
D. promised vacation getaways to cooler destinations for those trapped in the city at the peak of summer
3. Read the following passage and mark the letter A, B, C, or D on your answer sheet to indicate the correct answer to each of the
questions.

If the salinity of ocean waters is analyzed, it is found to vary only slightly from place to place. Nevertheless, some of these small changes
are important. There are three basic processes that cause a change in oceanic salinity. One of these is the subtraction of water from the
ocean by means of evaporation-conversion of liquid water to water vapor. In this manner, the salinity is increased, since the salts stay
behind. If this is carried to the extreme, of course, white crystals of salt would be left behind: this, by the way, is how much of the table salt
we use is actually obtained.
The opposite of evaporation is precipitation. such as rain, by which water is added to the ocean. Here the ocean is being diluted so that the
salinity is decreased. This may occur in areas of high rainfall or in coastal regions where rivers flow into the ocean. Thus salinity may be
increased by the subtraction of water by evaporation, or decreased by the addition of fresh water by precipitation or runoff.
Normally in tropical regions where the Sun is very strong, the ocean salinity is somewhat higher than it is in other parts of the world where
there is not as much evaporation. Similarly, in coastal regions where rivers dilute the sea salinity is somewhat lower than in other oceanic
areas.
A third process by which salinity may be altered is associated with the formation and melting of sea ice. When seawater is frozen, the
dissolved materials are left behind. In this manner, seawater directly beneath freshly formed sea ice has a higher salinity than it did before
the ice appeared. Of course, when this ice melts, it will tend to decrease the salinity of the surrounding water.
In the Weddell Sea, off Antarctica, the densest water in the oceans is formed as a result of this freezing process, which increases the
salinity of cold water. This heavy water sinks and is found in the deeper portions of the oceans of the world.

Question 1: What does the passage mainly discuss?


A. The bodies of water of the world
B. The elements of salt
C. The many forms of ocean life
D. The salinity of ocean water
Question 2: According to the passage, the ocean generally has more salt in
A. coastal areas
B. tropical areas
C. rainy areas
D. turbulent areas
Question 3: All of the following are processes that decrease salinity EXCEPT
A. evaporation
B. precipitation
C. runoff
Trang 2/4
48
D. melting
Question 4: Which of the following statements about the salinity of a body of water can best be inferred from the passage?

A. The temperature of the water is the most important factor.


B. How quickly the water moves is directly related to the amount of alt.
C. Ocean salinity has little effect on sea life.
D. Various factors combine to cause variations in the salt content of water.
Question 5: The word "it" in line 16 refers to which of the following?
A. Sea ice
B. Salinity
C. Seawater
D. Manner
Question 6: Why does the author mention the Weddell Sea?
A. To show that this body of water has salinity variations
B. To compare Antarctic waters with Arctic waters
C. To give an example of cold-water salinity
D. To point out the location of deep waters
Question 7: Which of the following is NOT a result of the formation of ocean ice?
A. The salt remains in the water
B. The surrounding water sinks
C. Water salinity decreases
D. The water becomes denser
Question 8: What can be inferred about the water near the bottom of oceans?
A. It is relatively warm.
B. Its salinity is relatively high.
C. It does not move.
D. It is formed by melting sea ice.
4. Read the following passage and mark the letter A, B, C, or D on your answer sheet to indicate the correct answer to each of the
questions.

Noise is unwanted sound and is among the most pervasive pollutants today. Noise from road traffic, jet planes, jet skis, garbage trucks,
construction equipment, manufacturing processes, lawn mowers, leaf blowers, and boom boxes, to name a few, are among the unwanted
sounds that are routinely broadcast into the air.
The problem with noise is not only that it is unwanted, but also that it negatively affects human health and well-being. Problems related to
noise include hearing loss, stress, high blood pressure, sleep loss, distraction and lost productivity, and a general reduction in the quality of
life and opportunities for tranquility.
We experience noise in a number of ways. On some occasions, we can be both the cause and the victim of noise, such as when we are
operating noisy appliances or equipment. There are also instances when we experience noise generated by others just as people experience
second-hand smoke. While in both instances, noises are equally damaging, second-hand noise is more troubling because it has negative
impacts on us but is put into the environment by others, without our consent.
The air into which second-hand noise is emitted and on which it travels is a “commons”, a public good. It belongs to no one person or
group, but to everyone. People, businesses, and organizations, therefore, do not have unlimited rights to broadcast noise as they please, as
if the effects of noise were limited only to their private property. On the contrary, they have an obligation to use the commons in ways that
are compatible with or do not detract from other uses.
People, businesses, and organizations that disregard the obligation to not interfere with others' use and enjoyment of the commons by
producing noise pollution are, in many ways, acting like a bully in a school yar Although perhaps unknowingly, they nevertheless disregard
the rights of others and claim for themselves rights that are not theirs.
We have organized to raise awareness of noise pollution and help communities take back the commons from those acting like bullies. Our
efforts include building a library of resources and tools concerning noise pollution, establishing links to other groups that have similar
collections, establishing networks among local noise activists, assisting communities and activists who are working to reduce noise
pollution, and monitoring and advocating for stronger noise controls.

Question 1: It is stated in the passage that all of the following are things the noise comes from EXCEPT
A. Television, radio
B. Lawn mower, leaf blowers
C. Road traffic, garbage trucks
D. Jet planes, watercraft
Question 2: It can be inferred from the passage that the effects of noise pollution on the human body are that
A. people get accustomed to ear and heart diseases
B. people focus mainly on hearing
C. people have difficulty falling asleep
D. people are sensitive about everything around them
Question 3: Which of the following is supposed to be way to reduce noise pollution?
A. cooperation with other noise activists
B. fining people who break laws in noise regulation
C. construction of large projects along roads
D. prohibiting public transport during peak hours

Trang 3/4
49
Question 4: The word “it” in paragraph 4 refers to
A. the air
B. a public good
C. effect of noise
D. second-hand noise
Question 5: It is mentioned in the passage that when we are operating noisy appliances or we experience noise generated by others

A. The noise is so troublesome if we make it or not.


B. We feel more comfortable if we don’t experience second-hand noise.
C. The noise have a strong influence on us, but not on others.
D. We just ignore the serious detriment if the noise is put into the environment by others.
Question 6: The word “disregard” in paragraph 5 could be best replaced by which of the following?
A. ignore
B. obey
C. invade
D. disturb
Question 7: Which of the following is NOT true according to the passage?
A. Noise that is experienced by people who did not produce it is called second-hand noise.
B. People’s enjoyment of the commons by producing noise pollution is regarded as a bully.
C. Noise is one of the most common contaminants nowadays.
D. People intentionally make noise when they work
Question 8: Which of the following best describes the author’s tone in the passage?
A. Incentive
B. Explanatory
C. Cynical
D. Provocative

Trang 4/4
50
BÀI TẬP TỰ LUYỆN
DANH TỪ VÀ ĐẠI TỪ
PEN-C TIẾNG ANH - CÔ HƯƠNG FIONA

1. Choose the answer (A, B, C, D) which best fits the space

Their _______ is to report back by march on how the new tax will change employment
A. mandate B. mandates
C. mandated D. mandatory
2. Choose the answer (A, B, C, D) which best fits the space

Of course I did. I gave _________ back on Tuesday


A. their B. they
C. it D. its
3. Choose the answer (A, B, C, D) which best fits the space

Divers use external air supplies to enable _________ to breathe under water
A. yourselves B. himself
C. ourselves D. themselves

4. Choose the answer (A, B, C, D) which best fits the space

The brilliant student fulfilled teacher’s __________ that he would be a successful doctor
A. prophesy B. prophetical
C. prophecy D. prophet
5. Choose the answer (A, B, C, D) which best fits the space

In that case, just look at the table and tell _______ what you see
A. me B. I
C. yourself D. my
6. Choose the answer (A, B, C, D) which best fits the space

She accepted his _______ at once


A. proposal B. proposed
C. proposer D. proposes
7. Choose the answer (A, B, C, D) which best fits the space

He doesn’t trust __________?


A. me B. him
C. I D. it

8. Choose the answer (A, B, C, D) which best fits the space

Deep-sea divers wear wetsuits to protect ________ under the water


A. ourselves B. itself
C. yourselves D. themselves
9. Choose the answer (A, B, C, D) which best fits the space

Dillon studied _______at Manchester University


A. economy B. economize
C. economist D. economics

10. Choose the answer (A, B, C, D) which best fits the space

Satellite photos show the smallest details with great ________


A. reliable B. reliability
C. rely D. reliance
11. Choose the answer (A, B, C, D) which best fits the space

Our hotel, ________ is converted 17th century manor house, is very comfortable

Trang 1/5
51
A. which B. where
C. that D. when
12. Choose the answer (A, B, C, D) which best fits the space

Well you and your wife have borrowed things sometimes and not given ________ back.
A. they B. theirs
C. them D. their
13. Choose the answer (A, B, C, D) which best fits the space

The countries have kept their own distinct political and cultural ________
A. identities B. identification
C. identifies D. identify

14. Choose the answer (A, B, C, D) which best fits the space

Decompression sickness manifests _________ with joint pain and can be fatal
A. himself B. myself
C. itself D. herself
15. Choose the answer (A, B, C, D) which best fits the space

Despite being famous, he does not give_______to the press


A. interview B. interviewers
C. interviews D. interviewer

16. Choose the answer (A, B, C, D) which best fits the space

The university welcomes _____ from mature students


A. applicants B. application
C. applicant D. applications

17. Choose the answer (A, B, C, D) which best fits the space

All ________ must be received by 10 April


A. submissions B. submission
C. submit D. submitted
18. Choose the answer (A, B, C, D) which best fits the space

Yesterday a ________ of the UN visited the area


A. representative B. represented
C. representing D. representation
19. Choose the answer (A, B, C, D) which best fits the space

I have a _____ that he will want to participate in the investigation


A. suspicion B. suspected
C. suspect D. suspects
20. Choose the answer (A, B, C, D) which best fits the space

Well, he says that he doesn’t trust _______


A. yours B. she
C. you D. them
21. Choose the answer (A, B, C, D) which best fits the space

If you were a diver, you would buy _______ a JIM suit and a helmet
A. yourself B. myself
C. himself D. herself
22. Choose the answer (A, B, C, D) which best fits the space

The bends occur when divers rise __________ to the surface too quickly
A. yourselves B. itself
C. themselves D. ourselves
Trang 2/5
52
23. Choose the answer (A, B, C, D) which best fits the space

That’s exactly what he said about _______


A. you B. I
C. yours D. mine
24. Choose the answer (A, B, C, D) which best fits the space

He obtained a master degree in ______ and politics at the University of Sydney


A. sociologist B. sociological
C. sociology D. sociologically
25. Choose the answer (A, B, C, D) which best fits the space

One person who made _________ a career based on the sea was the authoress Rachel Carson.
A. himself B. herself
C. itself D. myself

26. Choose the answer (A, B, C, D) which best fits the space

He bought all the books _________ are needed for the English course
A. what B. whose
C. those D. that
27. Choose the answer (A, B, C, D) which best fits the space

My brother enjoys studying physics. He wants to become a ________


A. physics B. physicist
C. physical D. physicians
28. Choose the answer (A, B, C, D) which best fits the space

The comprehensive ________ of the bank revealed no illegal activity


A. investigator B. investigative
C. investigate D. investigation
29. Choose the answer (A, B, C, D) which best fits the space

It was a show with a lot of audience ________


A. participated B. participant
C. participates D. participation

30. Choose the answer (A, B, C, D) which best fits the space

A diver must supply _______ with a mixture of oxygen and different gasses
A. itself B. himself
C. myself D. yourself
31. Choose the answer (A, B, C, D) which best fits the space

Their _______ of the story generated some interesting discussion


A. interpretation B. interpret
C. interpreter D. interpretive

32. Choose the answer (A, B, C, D) which best fits the space

Her action was an indication of her good _______


A. intentionally B. intentional
C. intended D. intentions
33. Choose the answer (A, B, C, D) which best fits the space

So did you give the book to _______?


A. he B. she
C. his D. him

34. Choose the answer (A, B, C, D) which best fits the space

Trang 3/5
53
Successful _____ will be notified by telephone.
A. applied B. applies
C. applicants D. applying

35. Choose the answer (A, B, C, D) which best fits the space

Rachel’s Carson’s life was marked with conservation efforts that we ________ can learn from.
A. herself B. themselves
C. ourselves D. yourselves

36. Choose the answer (A, B, C, D) which best fits the space

Oh dear, I can see _______. That must be the book I haven’t given back
A. them B. his
C. it D. him
37. Choose the answer (A, B, C, D) which best fits the space

Rachel Carson’s first book, Under the Sea-Wind, described struggle sea creatures find _________facing.
A. yourselves B. themselves
C. itself D. ourselves
38. Choose the answer (A, B, C, D) which best fits the space

Sugar is the _________ of healthy teeth


A. destroying B. destructively
C. destruction D. destroyer

39. Choose the answer (A, B, C, D) which best fits the space

The interests of ________ and consumers may be in conflict


A. production B. produce
C. producers D. products
40. Choose the answer (A, B, C, D) which best fits the space

Rachel Carson put on a diver’s helmet and explored the reefs of the Florida Keys for __________
A. ourselves B. myself
C. himself D. herself
41. Choose the answer (A, B, C, D) which best fits the space

What percentage of the city’s ______ live in poverty


A. household chores B. houses
C. households D. household
42. Choose the answer (A, B, C, D) which best fits the space

Do you know the boy _________ at the party last week?


A. we talked about B. about him we talked
C. who we talked about him D. we talk about him
43. Choose the answer (A, B, C, D) which best fits the space

I have bought a present for my brother, and now I need some _________
A. wrapping paper B. paper wrapper
C. wrapped paper D. wrap paper
44. Choose the answer (A, B, C, D) which best fits the space

Well I think you’re being very unfair to ________


A. us B. our
C. you D. ours

45. Choose the answer (A, B, C, D) which best fits the space

We are talking about the writer _________ latest book is one of the best-sellers this year.

Trang 4/5
54
A. whose B. whom
C. who D. which

46. Choose the answer (A, B, C, D) which best fits the space

The children, __________ parents work late, are taken home by bus
A. that B. whom
C. their D. whose
47. Choose the answer (A, B, C, D) which best fits the space

Smoke detectors must be installed to comply with fire_______.


A. regular B. regulators
C. regulates D. regulations
48. Choose the answer (A, B, C, D) which best fits the space

They are the best ________ on the police force


A. detect B. detected
C. detectives D. detection
49. Choose the answer (A, B, C, D) which best fits the space

Would all _________ please make their way to the start?


A. competition B. competitors
C. competed D. competitive

50. Choose the answer (A, B, C, D) which best fits the space

I find that difficult to believe. Do you trust _______?


A. his B. my
C. they D. me

Trang 5/5
55
BÀI TẬP TỰ LUYỆN
WORD FORMATION
PEN-C TIẾNG ANH - CÔ HƯƠNG FIONA

1. Mark the letter A, B, C or D on your answer sheet to indicate the correct answer to each of the following questions

Tom is getting ever keener on doing research on _____


A. biological B. biology
C. biologically D. biologist
2. Mark the letter A, B, C or D on your answer sheet to indicate the correct answer to each of the following questions

This __________ dictionary includes a few animations


A. electrical B. electronic
C. electric D. electricity
3. Mark the letter A, B, C or D on your answer sheet to indicate the correct answer to each of the following questions

Most _________ understand that disciplinary actions do not always work with students.
A. educate B. education
C. educators D. educating

4. Mark the letter A, B, C or D on your answer sheet to indicate the correct answer to each of the following questions

Children who are isolated and lonely seem to have poor language and ____________.
A. Communication B. Communicate
C. Communicator D. Communicative
5. Mark the letter A, B, C or D on your answer sheet to indicate the correct answer to each of the following questions

The information in this article is ________ inaccurate


A. historical B. historically
C. history D. historic
6. Mark the letter A, B, C or D on your answer sheet to indicate the correct answer to each of the following questions

Before the interview, you have to send a letter of _____ and your resume to the company
A. application B. apply
C. applicant D. applying
7. Mark the letter A, B, C or D on your answer sheet to indicate the correct answer to each of the following questions

The _____ dressed woman in the advertisement has a pose smile on her face
A. stylishly B. stylish
C. stylistic D. stylistical

8. Mark the letter A, B, C or D on your answer sheet to indicate the correct answer to each of the following questions

The rhinoceros, whose numbers have dropped alarmingly recently, has been declared a/an _______species
A. endanger B. endangered
C. danger D. dangerous
9. Mark the letter A, B, C or D on your answer sheet to indicate the correct answer to each of the following questions

The top tourist ________in Vietnam, Ha Long bay features thousands of islands, each topped with thick jungle vegetation, forming a
spectacular seascape of limestone pillars.
A. attractiveness B. attract
C. attraction D. attractive

10. Mark the letter A, B, C or D on your answer sheet to indicate the correct answer to each of the following questions

Someone who is _______ is hopeful about the future or the success of something in particular.
A. optimistic B. pessimist
C. pessimistic D. optimist
11. Mark the letter A, B, C or D on your answer sheet to indicate the correct answer to each of the following questions

Trang 1/4
56
The marriage pattern of parents plays an extremely important role in __________ marriage roles of children
A. socialism B. socialist
C. social D. socializing

12. Mark the letter A, B, C or D on your answer sheet to indicate the correct answer to each of the following questions

The mobile phone is an effective means of ____ in the world nowadays


A. communicate B. communicative
C. communication D. communicated
13. Mark the letter A, B, C or D on your answer sheet to indicate the correct answer to each of the following questions

She has been very interested in doing research on _______.


A. biological B. biologist
C. biologically D. biology
14. Mark the letter A, B, C or D on your answer sheet to indicate the correct answer to each of the following questions

As a child, I had an___________friend called Polly


A. image B. imaginative
C. Imaginary D. imagine

15. Mark the letter A, B, C or D on your answer sheet to indicate the correct answer to each of the following questions

The polar bear’s________depends on its ability to catch fish


A. surviving B. survival
C. survive D. survivor
16. Mark the letter A, B, C or D on your answer sheet to indicate the correct answer to each of the following questions

It is not ____________ to be drunk in the street


A. respectably B. respecting
C. respectable D. respectful
17. Mark the letter A, B, C or D on your answer sheet to indicate the correct answer to each of the following questions

He became interested in ________ when he started taking pictures for the local newspaper.
A. photography B. photographic
C. photograph D. photographer
18. Mark the letter A, B, C or D on your answer sheet to indicate the correct answer to each of the following questions

In 1973, when the tigers appeared to be facing ______ the World Wide Fund for _______ and the Indian Government agreed to set up
“Operation Tiger”.
A. extinct/ Nature B. extinction/ Nature
C. extinction/ Natural D. extinct/ Naturalists
19. Mark the letter A, B, C or D on your answer sheet to indicate the correct answer to each of the following questions

Most of us would agree that physical______does not play a major partinhowwe react to the people wemeet.
A. attractive B. attract
C. attractiveness D. attractively
20. Mark the letter A, B, C or D on your answer sheet to indicate the correct answer to each of the following questions

Water volume in rivers in central Vietnam will decline by 30 to 50 percent, potentially leading to a _______ between April and June
A. scarcely B. scarce
C. scarcity D. scarify

21. Mark the letter A, B, C or D on your answer sheet to indicate the correct answer to each of the following questions

Travelling to _______ countries in the world enables me to learn many interesting things
A. diffently B. difference
C. differ D. different
22. Mark the letter A, B, C or D on your answer sheet to indicate the correct answer to each of the following questions

Don’t worry. He’ll do the job as_________as possible

Trang 2/4
57
A. economically B. economic
C. economical D. economizing
23. Mark the letter A, B, C or D on your answer sheet to indicate the correct answer to each of the following questions

He became one of the most __________ actors in Vietnam


A. Success B. Successfully
C. Successful D. Successive

24. Mark the letter A, B, C or D on your answer sheet to indicate the correct answer to each of the following questions

He was turned down for the job because he is __________


A. unqualified B. qualified
C. qualification D. qualifying
25. Mark the letter A, B, C or D on your answer sheet to indicate the correct answer to each of the following questions

They are always ____ of what their children do.


A. supportive B. supporting
C. support D. supportively
26. Mark the letter A, B, C or D on your answer sheet to indicate the correct answer to each of the following questions

The success of the company in such a ____ market is remarkable


A. competitive B. Compete
C. competitively D. competition
27. Mark the letter A, B, C or D on your answer sheet to indicate the correct answer to each of the following questions

Although David was ________ , he still tried to help his wife with household chores
A. exhausted B. exhausting
C. Exhaustion D. exhaustive
28. Mark the letter A, B, C or D on your answer sheet to indicate the correct answer to each of the following questions

Many young men prefer scuba-diving because it is _______.


A. adventure B. adventurously
C. adventurist D. adventurous
29. Mark the letter A, B, C or D on your answer sheet to indicate the correct answer to each of the following questions

The government has succeeded in reducing the level of _________ to only 7%.
A. unemployment B. employer
C. unemployed D. employment

30. Mark the letter A, B, C or D on your answer sheet to indicate the correct answer to each of the following questions

Many countries’ cutural ____________is a result of talking in immigrants from all over the world.
A. diversity B. diverse
C. diversify D. diversified
31. Mark the letter A, B, C or D on your answer sheet to indicate the correct answer to each of the following questions

She has always shown her great self- _____ in not becoming angry.
A. controlled B. controller
C. controll D. controlling

32. Mark the letter A, B, C or D on your answer sheet to indicate the correct answer to each of the following questions

Eugenie Clark has a wide __________ about cultures of many countries in the world
A. known B. knowing
C. know D. knowledge
33. Mark the letter A, B, C or D on your answer sheet to indicate the correct answer to each of the following questions

Students normally enter university from the onwards and study for an _______ degree
A. academically B. academial
C. academy D. academic
Trang 3/4
58
34. Mark the letter A, B, C or D on your answer sheet to indicate the correct answer to each of the following questions

He was offered the job thanks to his ________performance during his job interview.
A. impressive B. impress
C. impressively D. impression

35. Mark the letter A, B, C or D on your answer sheet to indicate the correct answer to each of the following questions

Thanks to the women's liberation, women can take part in ________ activities
A. socially B. social
C. socialize D. society
36. Mark the letter A, B, C or D on your answer sheet to indicate the correct answer to each of the following questions

It is not always easy to make a good ______ at the last minute


A. decide B. decisively
C. decision D. decisive

37. Mark the letter A, B, C or D on your answer sheet to indicate the correct answer to each of the following questions

The more _______ you look, the better impression you will make on the interviewer
A. confide B. confident
C. confidently D. confidence
38. Mark the letter A, B, C or D on your answer sheet to indicate the correct answer to each of the following questions

You should concentrate on what the__________is saying and try to answer all the questions thoroughly
A. interviewer B. interview
C. interviewed D. interviewee
39. Mark the letter A, B, C or D on your answer sheet to indicate the correct answer to each of the following questions

In Vietnam, children begin their primary ____ at the age of six.


A. educationally B. educate
C. educational D. education
40. Mark the letter A, B, C or D on your answer sheet to indicate the correct answer to each of the following questions

Lessons from the ___ developed countries are worth learning to save our time.
A. economize B. economic
C. economical D. economically

Trang 4/4
59
BÀI TẬP TỰ LUYỆN
BÀI. HEALTH
PEN-C TIẾNG ANH - CÔ HƯƠNG FIONA

1. Read the following passage and mark the letter A, B, C, or D on your answer sheet to indicate the correct answer to each of the
questions.

As heart disease continues to be the number-one killer in the United States, researchers have become increasingly interested in identifying
the potential risk factors that trigger heart attacks. High-fat diets and “life in the fast lane” have long been known to contribute to 5 the
high incidence of heart failure. But according to new studies, the list of risk factors may be significantly longer and quite surprising.
Heart failure, for example, appears to have seasonal and temporal 10 patterns. A higher percentage of heart attacks occur in cold weather,
and more people experience heart failure on Monday than on any other day of the week. In addition, people are more susceptible to heart
attacks in the first few hours after waking. Cardiologists first observed this morning phenomenon in the mid-1980, and have since 15
discovered a number of possible causes. An early-morning rise in blood pressure, heart rate, and concentration of heart stimulating
hormones, plus a reduction of blood flow to the heart, may all contribute to the higher incidence of heart attacks between the hours of 8:00
A.M. and 10:00 A.M.
In other studies, both birthdays and bachelorhood have been implicated as risk factors. Statistics reveal that heart attack rates increase
significantly for both females and males in the few days immediately preceding and following their birthdays. And unmarried men are
more 25 at risk for heart attacks than their married counterparts. Though stress is thought to be linked in some way to all of the
aforementioned risk factors, intense research continues in the hope of further comprehending why and how heart failure is triggered.

Question 1: What does the passage mainly discuss?


A. risk factors in heart attacks
B. seasonal and temporal patterns of heart attacks
C. cardiology in the 1980s
D. diet and stress as factors in heart attacks
Question 2: In paragraph 1, the word “potential” could best be re-placed by which of the following?
A. harmful
B. primary
C. unknown
D. Possible
Question 3: The word “trigger” as used in paragraph 1 is closest in meaning to which of the following?
A. involve
B. affect
C. cause
D. Encounter
Question 4: The phrase “susceptible to” in paragraph 2 could best be replaced by
A. aware of
B. affected by
C. accustomed
D. prone to
Question 5: According to the passage, which of the following is NOT a possible cause of many heart attacks?
A. decreased blood flow to the heart
B. increased blood pressure
C. lower heart rate
D. increase in hormones
Question 6: Which of the following is NOT cited as a possible risk factor?
A. Having a birthday
B. Getting married
C. Rating fatty foods
D. Being under stress
Question 7: Which of the following does the passage infer?
A. We now fully understand how risk factors trigger heart attacks.
B. We recently began to study how risk factors trigger heart attacks.
C. We have not identified many risk factors associated with heart attacks.
D. We do not fully understand how risk factors trigger heart attacks.
2. Read the following passage and mark the letter A, B, C, or D on your answer sheet to indicate the correct answer to each of the
questions.

Aging is the process of growing old. It occurs eventually in every living thing provided, of course, that an illness or accident does not kill it
prematurely. The most familiar outward signs of aging may be seen in old people, such as the graying of the hair and the wrinkling of the
skin. Signs of aging in a pet dog or cat include loss of playfulness and energy, a decline in hearing and eyesight, or even a slight graying of
the coat. Plants age too, but the signs are much harder to detect.
Most body parts grow bigger and stronger, and function more efficiently during childhood. They reach their peak at the time of maturity, or
early adulthood. After that, they begin to decline. Bones, for example, gradually become lighter and more brittle. In the aged, the joints
between the bones also become rigid and more inflexible. This can make moving very painful.
Trang 1/3
60
All the major organs of the body show signs of aging. The brain, for example, works less efficiently, and even gets smaller in size.
Thinking processes of all sorts are slowed down. Old people often have trouble in remembering recent events.
One of the most serious changes of old age occurs in the arteries, the blood vessels that lead from the heart. They become thickened and
constricted, allowing less blood to flow to the rest of body. This condition accounts, directly or indirectly, for many of the diseases of the
aged. It may, for example, result in heart attack.
Aging is not a uniform process. Different parts of the body wear out at different rates. There are great differences among people in their
rate of aging. Even the cells of the body differ in the way they age. The majority of cells are capable of reproducing themselves many times
during the course of a lifetime. Nerve cells and muscle fibers can never be replaced once they wear out.
Gerontologists - scientists who study the process of aging-believe this wearing out of the body is controlled by a built-in biological time-
clock. They are trying to discover how this clock works so that they can slow down the process. This could give man a longer life and a
great number of productive years.

Question 1: What is the main idea of the first paragraph?


A. Signs of aging are easier to detect in animals than in plants.
B. Aging occurs in every living thing after it has reached maturity
C. Not all signs of aging are visible.
D. The outward signs of aging may be seen in old people.
Question 2: What does the word “it” in paragraph 1 refer to?
A. aging
B. a living thing
C. an illness
D. an accident
Question 3: When does the human body begin to lose vigor and the ability to function efficiently?
A. Soon after reaching adulthood
B. During childhood
C. Early adulthood
D. In old age
Question 4: What happens to memorization when the brain begins to age?
A. It works less.
B. It becomes forgetful.
C. It declines.
D. It slows down
Question 5: The word “brittle” in paragraph 2 means
A. soft and easily bent
B. hard and endurable
C. hard but easily broken
D. rigid and inflexible
Question 6: According to the passage, what condition is responsible for many of the diseases of the old?
A. The arteries have become thickened and constricted.
B. The blood vessels lead from the heart.
C. The brain gets smaller in size.
D. Bones become lighter and brittle.
Question 7: According to the passage, which of the following is NOT true?
A. All living things grow old.
B. Aging is unavoidable in any living things.
C. Plants show less signs of aging than any other living things.
D. Most body parts wear out during the course of a lifetime.
Question 8: What is the main idea of the last paragraph?
A. Gerontologists have controlled the process of aging.
B. Gerontologists are working hard to help people live longer and more healthily.
C. Gerontologists are trying to give people an eternal life.
D. Gerontologists are now able to slow down the process of aging.
3. Read the following passage and mark the letter A, B, C, or D on your answer sheet to indicate the correct answer to each of the
question

The principle of use and disuse states that those parts of organisms' bodies that are used grown larger. Those parts that are not tend to
wither away. It is an observed fact that when you excercise particular muscles they grow. Those that are never used dimish. By examining
a man’s body, we can tell which muscles he uses and. which he doesn't, we may even be able to guess his profession or his reaction.
Enthusiasts of the "body- building"cult make use of the principle of use and disuse to "build" their bodies, almost like a piece of sculpture
into whatever unnatural shape is demanded by fashion in this peculiar minority culture. Muscles are not the only parts of the body that
respond to use in this kind of way. Walk barefoot and you acquire harder skin on your soles. It is easy to tell a farmer from a bank teller by
looking at their hands alone. The farmer’s hands are horny, hardenedby long exposure to rough work. The teller's hands are relatively soft.
The principle of use and disuse enables animals to become better at the job of surviving in their world progressively better during their
lifetime as a result of living in that world. Humans, through direct exposure to sunlight, or lack of It, develop a skin color which equips
them better to survive in the particular local conditions

Trang 2/3
61
Too much sunlight is dangerous. Enthusiastic sunbathers with very fair skins are susceptible to skin cancer. Too little sunlight, on the other
hand, leads to vitamin-D deficiency and rickets. The brown pigment melanin which is synthesized under the influence of sunlight, makes a
screen to protect the underlying tissues from the harmful effects of further sunlight. If a suntanned person moves to a less sunny climate,
the melanin disappears and the body is able to benefit from what little sun there is. This can be represented as an instance of the principle
of use and disuse: skin goes brown when it is "used", and fades to white when it is not.

Question 1: What does the pasage mainly discuss?


A.How the principles of use and disuse change people's concepts of themselves.
B.The changes that occur according to the principle of use and disuse
C.The way in which people change themselves to conform to fashion.
D.The effects of the sun on the principle of use and disuse.
Question 2: The phrase "wither away" in bold is closest in meaning to
A. split
B. rot
C. perish
D. shrink
Question 3: The word "Those" in bold refers to
A. organisms
B. bodies
C. parts
D. muscles
Question 4: According to the passage, men who body build
A. appear like sculptures
B. change their appearance
C. belong to strange cults
D. are very fashionable
Question 5: From the passage, it can be inferred that author views body building
A. with enthusiasm
B. as an artistic from
C. with scientific interest
D. of doubtful benefic
Question 6: It can be inferred from the passage that the principle of use and dissure enables organisms to
A.change their existence
B.automatically benefit
C. survive in any condition
D.improve their lifetime
Question 7: The author suggests that melanin
A.is necessary for the production of vitamin-D
B. is beneficial in sunless climates
C. helps protect fair-skinned people
D. is a synthetic product
Question 8: In the second paragraph, the author mentions suntanning as an example of
A.humans improving their local condition
B. humans surviving in adverse conditions
C. humans using the priciple of use and disuse
D. humans running the risk of skin cancer

Trang 3/3
62
BÀI TẬP TỰ LUYỆN
TÍNH TỪ VÀ TRẠNG TỪ
PEN-C TIẾNG ANH - CÔ HƯƠNG FIONA

1. Mark the letter A, B, C or D on your answer sheet to indicate the correct answer to each of the following questions.

The river has been seriously ________


A. unpolluted B. pollution
C. polluted D. pollutant
2. Mark the letter A, B, C or D on your answer sheet to indicate the correct answer to each of the following questions.

The child swam ________.far out into the bay that she did not have enough strength left to return to shore
A. very B. so
C. such D. too
3. Mark the letter A, B, C or D on your answer sheet to indicate the correct answer to each of the following questions.

If we don’t meet those deadlines, we’re going to be in _______ trouble


A. heavy B. big
C. large D. great

4. Mark the letter A, B, C or D on your answer sheet to indicate the correct answer to each of the following questions.

The boy needs _________ balls for this activity


A. ten small cotton B. ten cotton small
5. Mark the letter A, B, C or D on your answer sheet to indicate the correct answer to each of the following questions.

I hope the company has enough money to pay for these ________ renovations
A. likely B. needy
C. costly D. late
6. Mark the letter A, B, C or D on your answer sheet to indicate the correct answer to each of the following questions.

There are many_______ species that need human protection


A. dangerously B. danger
C. dangerous D. endangered
7. Mark the letter A, B, C or D on your answer sheet to indicate the correct answer to each of the following questions.

The disease _________spreads to the rest of the country


A. subsequent B. sequences
C. subsequently D. sequence
8. Mark the letter A, B, C or D on your answer sheet to indicate the correct answer to each of the following questions.

The old wreck was_________ located southwest of Florida


A. presumably B. presumable
C. presume D. presumption

9. Mark the letter A, B, C or D on your answer sheet to indicate the correct answer to each of the following questions.

All of us were________ at her success


A. surprising B. surprised
C. surprisingly D. surprise
10. Mark the letter A, B, C or D on your answer sheet to indicate the correct answer to each of the following questions.

________, our homepage is no longer listed on the site

A. regrettably B. regret
C. regretful D. regrettable

11. Mark the letter A, B, C or D on your answer sheet to indicate the correct answer to each of the following questions.

His sister is always_______ dressed

Trang 1/5
63
A. conventionally B. conventionalized
C. convention D. conventional
12. Mark the letter A, B, C or D on your answer sheet to indicate the correct answer to each of the following questions.

I don’t think I am ________of doing it myself


A. capacity B. capability
C. capably D. capable
13. Mark the letter A, B, C or D on your answer sheet to indicate the correct answer to each of the following questions.

Surprisingly, the General Manager’s __________ occupation was serving in the military
A. former B. formerly
C. formal D. formality

14. Mark the letter A, B, C or D on your answer sheet to indicate the correct answer to each of the following questions.

The old man has been _________for several years now


A. dead B. died
C. death D. dying
15. Mark the letter A, B, C or D on your answer sheet to indicate the correct answer to each of the following questions.

If the directions are strictly _______, you won’t have any trouble operating the machine
A. follows B. followed
C. follow D. following

16. Mark the letter A, B, C or D on your answer sheet to indicate the correct answer to each of the following questions.

Thanks to her________ skills, she was offered the job


A. management B. managerial
C. manageable D. managerial or management

17. Mark the letter A, B, C or D on your answer sheet to indicate the correct answer to each of the following questions.

Where is that __________ dress that your grandma gave you?


A. long, pink, silk, lovely B. pink, long, lovely, silk
C. lovely, long, pink, silk D. lovely, pink, long, silk
18. Mark the letter A, B, C or D on your answer sheet to indicate the correct answer to each of the following questions.

We should control population growth as our resources are________.


A. limitation B. limit
C. limitless D. limited
19. Mark the letter A, B, C or D on your answer sheet to indicate the correct answer to each of the following questions.

It’s sometimes ________when you have to ask people for money


A. embarrass B. embarrassing
C. embarrassed D. embarrassment
20. Mark the letter A, B, C or D on your answer sheet to indicate the correct answer to each of the following questions.

Information must be stored so that it is _________ from accidental


A. secure B. secures
C. security D. securely
21. Mark the letter A, B, C or D on your answer sheet to indicate the correct answer to each of the following questions.

The country has had many ________ disasters recently


A. nature B. naturalist
C. natured D. natural
22. Mark the letter A, B, C or D on your answer sheet to indicate the correct answer to each of the following questions.

The system is_________ impossible to deal with most of the time


A. practically B. practicality
C. practice D. practical
Trang 2/5
64
23. Mark the letter A, B, C or D on your answer sheet to indicate the correct answer to each of the following questions.

Electronic emissions are _______ of causing damage to the health of young children and older adults
A. liable B. potential
C. capable D. possible
24. Mark the letter A, B, C or D on your answer sheet to indicate the correct answer to each of the following questions.

She has been played an _________part in the smooth running of the business
A. essential B. essence
C. essentially D. essentials
25. Mark the letter A, B, C or D on your answer sheet to indicate the correct answer to each of the following questions.

A(n) _____ _____ ______ dress


A. pink expensive summer B. expensive pink summer
C. summer expensive pink

26. Mark the letter A, B, C or D on your answer sheet to indicate the correct answer to each of the following questions.

This isolated area is _________ only by the helicopter


A. accessibility B. accessible
C. inaccessible D. access
27. Mark the letter A, B, C or D on your answer sheet to indicate the correct answer to each of the following questions.

I had never expected to get the job. I was really ________when I was offered it
A. amazed B. amaze
C. amazing D. amazement
28. Mark the letter A, B, C or D on your answer sheet to indicate the correct answer to each of the following questions.

Jim has been _________with his new job recently

A. occupying B. occupied
C. occupation D. occupational
29. Mark the letter A, B, C or D on your answer sheet to indicate the correct answer to each of the following questions.

We could make better ________ of our resources


A. useful B. usage
C. usefully D. use

30. Mark the letter A, B, C or D on your answer sheet to indicate the correct answer to each of the following questions.

He felt _________ with the results of his exam


A. disappointing B. disappointed
C. disappointedly D. disappointment
31. Mark the letter A, B, C or D on your answer sheet to indicate the correct answer to each of the following questions.

Thanks very much for your _______response


A. speedy B. speedily
C. speed D. speeded
32. Mark the letter A, B, C or D on your answer sheet to indicate the correct answer to each of the following questions.

The film gives us_________ insight into life in Brazil


A. valuable B. devalue
C. invaluable D. value
33. Mark the letter A, B, C or D on your answer sheet to indicate the correct answer to each of the following questions.

You cannot grow crops on _______ land


A. exhaust B. exhausting
C. exhaustion D. exhausted

34. Mark the letter A, B, C or D on your answer sheet to indicate the correct answer to each of the following questions.

Trang 3/5
65
I don’t think that film is_________.
A. bored B. bore
C. boring D. boringly
35. Mark the letter A, B, C or D on your answer sheet to indicate the correct answer to each of the following questions.

I feel so _________ and sleepy today because I worked late last night
A. tiredness B. tired
C. tiring D. tire
36. Mark the letter A, B, C or D on your answer sheet to indicate the correct answer to each of the following questions.

Smoke and waste from these _________areas has seriously polluted air
A. industrious B. industry
C. industrial D. industrialize

37. Mark the letter A, B, C or D on your answer sheet to indicate the correct answer to each of the following questions.

My teacher always smiles at me in a ________way whenever I see her


A. friend B. friendly
C. friendliness D. friendship
38. Mark the letter A, B, C or D on your answer sheet to indicate the correct answer to each of the following questions.

Jean is __________ and helpful in the classroom


A. cooperative B. operation
C. cooperate D. cooperatively

39. Mark the letter A, B, C or D on your answer sheet to indicate the correct answer to each of the following questions.

He said he wanted to speak to me ________.


A. private B. privately
C. privatize D. privacy
40. Mark the letter A, B, C or D on your answer sheet to indicate the correct answer to each of the following questions.

They were very________ when they choose the members of the academy team
A. selections B. selective
C. selected D. selectively

41. Mark the letter A, B, C or D on your answer sheet to indicate the correct answer to each of the following questions.

It would be _______ for you to stay here


A. danger B. endangers
C. dangers D. dangerous
42. Mark the letter A, B, C or D on your answer sheet to indicate the correct answer to each of the following questions.

The service was attentive, but the food was __________.


A. acceptance B. accepted
C. unacceptably D. unacceptable
43. Mark the letter A, B, C or D on your answer sheet to indicate the correct answer to each of the following questions.

A _____ ______ ______ spoon


A. brown soup wooden B. brown wooden soup
C. wooden brown soup
44. Mark the letter A, B, C or D on your answer sheet to indicate the correct answer to each of the following questions.

I’m afraid I’m not really __________ to comment on this matter


A. qualified B. qualifying
C. quality D. qualitative

45. Mark the letter A, B, C or D on your answer sheet to indicate the correct answer to each of the following questions.

He is much admired for his ________ skills

Trang 4/5
66
A. technically B. technical
C. techniques D. technician

46. Mark the letter A, B, C or D on your answer sheet to indicate the correct answer to each of the following questions.

Did you hear the ________ news? Anna is going to be the new manager!
A. surprise B. to surprise
C. surprising D. surprised
47. Mark the letter A, B, C or D on your answer sheet to indicate the correct answer to each of the following questions.

I am very ______by the complaints that have been made against you.
A. disturbance B. disturbing
C. disturb D. disturbed
48. Mark the letter A, B, C or D on your answer sheet to indicate the correct answer to each of the following questions.

A(n) _____ _____ ______ lady


A. old rich American B. American old rich
C. rich old American
49. Mark the letter A, B, C or D on your answer sheet to indicate the correct answer to each of the following questions.

The warranty doesn’t cover damage caused by __________ maintenance


A. properly B. improper
C. improperly D. property

50. Mark the letter A, B, C or D on your answer sheet to indicate the correct answer to each of the following questions.

The river has been polluted with toxic waste from _________factories.
A. local B. locality
C. locally D. locate

Trang 5/5
67
BÀI TẬP TỰ LUYỆN
BÀI. CÁC PHÉP SO SÁNH
PEN-C TIẾNG ANH - CÔ HƯƠNG FIONA

1. Mark the letter A, B, C, or D on your answer sheet to indicate the correct answer to each of the following questions.

Generally, the South of England is as __________ as the North


A. flatter B. the flattest
C. flat D. less flat
2. Choose the letter A, B, C or D to indicate the best answer to the following question.
This one is prettier, but it costs _____ as the other one.
A. twice as much B. as much as
C. twice as many D. as many

3. Choose the letter A, B, C or D to indicate the best answer to the following question.

The economic conditions today are _______ they were in the past.
A. much better than B. much more good
C. the best than D. much better
4. Choose the letter A, B, C or D to indicate the best answer to the following question.
Many chemicals react ______ in acid solutions.
A. more quick B. as quickly more
C. more quickly D. quicklier

5. Choose the letter A, B, C or D to indicate the best answer to the following question.
______ electricity you use, _______ your bill will be.
A. the most / the higher B. more / higher
C. the more / the higher D. the more / the high
6. Choose the letter A, B, C or D to indicate the best answer to the following question.

The faster we walk, ______ we will get there.


A. the more soon B. the soonest
C. the sooner D. the soon

7. Choose the letter A, B, C or D to indicate the best answer to the following question.

For _____, it is certain that in the future some things will be very different.
A. the good or the bad B. the better or the worse
C. good or bad D. better or worse
8. Mark the letter A, B, C or D on your answer sheet to indicate the correct answer to each of the following questions.

Laser disc provides images of ____ either television signals or video tapes.
A. better quality than those of B. better quality than of
C. better than D. better quality than
9. Choose the letter A, B, C or D to indicate the best answer to the following question.

My brother was feeling tired last night, so he went to bed ______ usual.
A. earlier than B. more earlier as
C. as early as D. more early than
10. Choose the letter A, B, C or D to indicate the best answer to the following question.

This shirt and that one ______.


A. as like as B. alike
C. are alike D. the same
11. Choose the letter A, B, C or D to indicate the best answer to the following question.
Of the two sisters, Linda _______ .
A. is beautiful B. is so beautiful as
C. the most beautiful D. is the more beautiful

Trang 1/4
68
12. Choose the letter A, B, C or D to indicate the best answer to the following question.
Despite its smaller size, the Indian Ocean _______ the Alantic Ocean.
A. is the same deep as B. is as deep as
C. as deep as D. deeper as
13. Choose the letter A, B, C or D to indicate the best answer to the following question.

No one else in the class plays the guitar ______ John.


A. as soon as B. so well as
C. as well D. as far as

14. Choose the letter A, B, C or D to indicate the best answer to the following question.

“Why did you buy these oranges?” - “They were _____ I could find. ”
A. cheapest ones B. the cheapest ones
C. the most cheapest D. cheapest
15. Choose the letter A, B, C or D to indicate the best answer to the following question.

She sat there quietly, but during all that time she was getting ______.Finally she exploded.
A. the most angry B. the more angry
C. more and more angry D. angrier and angrier
16. Choose the letter A, B, C or D to indicate the best answer to the following question.

The larger the apartment is, the _____ the rent is.
A. expensively B. more expensive
C. most expensive D. expensive
17. Choose the letter A, B, C or D to indicate the best answer to the following question.
The lab is _____ from the bus stop than the library.
A. further B. B and C
C. far D. farther
18. Choose the letter A, B, C or D to indicate the best answer to the following question.

The salary of a bus driver is much higher _______ a teacher.


A. than that of B. for
C. to compare with D. as that of
19. Mark the letter A,B,C,orD on your answer sheet to indicate the correct answer to each of the following questions

Last year Matt earned _________ his brother, who had a better position
A. twice as many as B. twice as more as
C. twice more than D. twice as much as

20. Choose the letter A, B, C or D to indicate the best answer to the following question.

______ you study for these exams, ______ you will do.
A. The more hard / the more good B. The more / the much
C. The harder / the better D. The hardest / the best
21. Choose the letter A, B, C or D to indicate the best answer to the following question.

Mr. Lam cannot earn _______ his wife.


A. as much money as B. as many money than
C. as much money than D. as many money as

22. Choose the letter A, B, C or D to indicate the best answer to the following question.

Which woman are you going to vote for? – I’m not sure. Everyone says that Joan is ____
A. the smarter B. smarter
C. more smart D. more smarter
23. Choose the letter A, B, C or D to indicate the best answer to the following question.

I would rather do any job ______ be jobless.

Trang 2/4
69
A. more than B. so than
C. than D. better than
24. Choose the letter A, B, C or D to indicate the best answer to the following question.
The ______ accident in the history of the city occurred last night on the Freeway.
A. worse B. badest
C. worst D. most bad
25. Choose the letter A, B, C or D to indicate the best answer to the following question.

He drives as ______ his father does.


A. the most careful B. careful as
C. more carefully D. carefully as
26. Choose the letter A, B, C or D to indicate the best answer to the following question.

I’ll be there _______ I can.


A. as soon as B. soonest as
C. sooner as D. no sooner as
27. Choose the letter A, B, C or D to indicate the best answer to the following question.
She plays the piano …………… as she sings.
A. the most beautifully B. as beautiful
C. as beautifully D. more beautifully
28. Choose the letter A, B, C or D to indicate the best answer to the following question.

My car is _______ yours.


A. more fast and more economical than B. faster and economical as
C. faster and more economical than D. more fast and economical than

29. Choose the letter A, B, C or D to indicate the best answer to the following question.

The town was nearer _____ we thought it would be.


A. than B. then
C. that D. as

30. Choose the letter A, B, C or D to indicate the best answer to the following question.
It’s becoming ______ to find a job.
A. more difficult than B. more and more difficult
C. more difficult and more D. most and more difficult
31. Mark the letter A, B, C, or D on your answer sheet to indicate the correct answer to each of the following questions.

The ______ North we go, the less likely we are to meet high temperatures
A. further B. farthest
C. far D. farther

32. Choose the letter A, B, C or D to indicate the best answer to the following question.

It’s too noisy here. Can we go somewhere _____?


A. more quieter B. quieter
C. noisier D. more noisy
33. Choose the best option to complete each sentence
The deep oceans contain some of the ______ of all living creatures.
A. as strange as B. strangest
C. strange D. stranger
34. Choose the letter A, B, C or D to indicate the best answer to the following question.

His house is _______ mine.


A. as two times big as B. as big as twice
C. twice as big as D. as twice big as
35. Choose the letter A, B, C or D to indicate the best answer to the following question.

Trang 3/4
70
Her grandfather’s illness was _____ we thought at first.
A. more seriously as B. as serious than
C. as seriously as D. more serious than
36. Choose the letter A, B, C or D to indicate the best answer to the following question.
He finished the test _______ of all.
A. most rapidly B. more rapidly
C. the most rapidly D. rapidly

37. Choose the letter A, B, C or D to indicate the best answer to the following question.
You can take ______ as you want.
A. as many B. too many
C. as much D. so much
38. Choose the letter A, B, C or D to indicate the best answer to the following question.

Of the four dresses, which is ____ expensive?


A. the greater B. the best
C. the more D. the most

39. Choose the letter A, B, C or D to indicate the best answer to the following question.
Sharon ______ from other women I know.
A. different B. more different
C. as different D. differs
40. Choose the letter A, B, C or D to indicate the best answer to the following question.

______ the time passes, ______ I feel ! The deadline of my thesis is coming, but I have just finished half of it.
A. The faster / the more nervous B. The more fast / the nervous
C. The fast / the more nervous D. The faster / the nervous

Trang 4/4
71
BÀI TẬP TỰ LUYỆN
BÀI. TECHNOLOGY
PEN-C TIẾNG ANH - CÔ HƯƠNG FIONA

1. Read the following passage and mark the letter A, B, C, or D on your answer sheet to indicate the correct word or phrase that best
fits each of the numbered blanks from 1 to 5.

In this age of (1) telephone networks and electronic mail, it seems that fewer and fewer people are taking time to sit
down and write letters to friends and relatives. For hundreds of years, letters were the only way to keep (2) people
who were any distance away and letter-writing was seen as an important skill for all learned people to master.
Gradually, (3) , the importance of writing letters is decreasing to a point that majority of us have to make a special
effort to turn out something worthwhile when we apply for a job or make a complaint. In business circles the tendency is for routine
communications to become shorter. Even though clients may appreciate a detailed letter, an employee who sends out long letters is often
regarded as (4) . Many people prefer the telephone in all circumstances and its speed is essential in many situations,
but (5) have you put the telephone down, dissatisfied with what you have managed to say? I don’t think I’ll throw
my pen away yet.

Question 1:
A. advanced
B. progressive
C. highly-developed
D. all are correct
Question 2:
A. on good terms with
B. in step with
C. in contact with
D. in favour of
Question 3:
A. for example
B. therefore
C. however
D. in short
Question 4:
A. impossible
B. unusual
C. inefficient
D. unimportant
Question 5:
A. how about
B. how often
C. how much
D. how long
2. Read the passage and choose A, B, c or D to indicate the correct answer to each of the questions.

Millions of people are using cellphones today. In many places, it is actually considered unusual not to use one. In many countries,
cellphones are very popular with young people. They find that the phones are more than a means of communication - having a mobile
phone shows that they are cool and connected.
The explosion in mobile phone use around the world has made some health professionals worried. Some doctors are concerned that in the
future many people may suffer health problems from the use of mobile phones. In England, there has been a serious debate about this
issue. Mobile phone companies are worried about the negative publicity of such ideas. They say that there is no proof that mobile phones
are bad for your health.
On the other hand, medical studies have shown changes in the brain cells of some people who use mobile phones. Signs of change in the
tissues of the brain and head can be detected with modem scanning equipment. In one case, a traveling salesman had to retire at young age
because of serious memory loss. He couldn't remember even simple tasks. He would often forget the name of his own son. This man used
to talk on his mobile phone for about six hours a day, every day of his working week, for a couple of years. His family doctor blamed his
mobile phone use, but his employer's doctor didn't agree.
What is it that makes mobile phones potentially harmful? The answer is radiation. High-tech machines can detect very small amounts of
radiation from mobile phones. Mobile phone companies agree that there is some radiation, but they say the amount is too small to worry
about.
As the discussion about their safety continues, it appears that it’s best to use mobile phones less often. Use your regular phone if you want
to talk for a long time. Use your mobile phone only when you really need it. Mobile phones can be very useful and convenient, especially
in emergencies. In the future, mobile phones may have a warning label that says they are bad for your health. So for now, it's wise not to
use your mobile phone too often.

Question 1: The most suitable title for the passage could be


A. “The Reasons Why Mobile Phones Are Popular”
B. “Mobile Phones: A Must of Our Time”
Trang 1/3
72
C. “The Way Mobile Phones Work”
D. “Technological Innovations and Their Price”
Question 2: According to the passage, cellphones are especially popular with young people because
A. they make them look more stylish
B. they are indispensable in every day communications
C. they keep the users alert all the time
D. they cannot be replaced by regular phones
Question 3: The changes possibly caused by the cellphones are mainly concerned with,
A. the mobility of the mind and the body
B. the resident memory
C. the arteries of the brain
D. the smallest units of the brain
Question 4: According to the passage, what makes mobile phones potentially harmful is
A. their radiant light
B. their power of attraction
C. their raiding power
D. their invisible rays
Question 5: According to the writer, people should
A. never use mobile phones in all cases
B. only use mobile phones in medical emergencies
C. keep off mobile phones regularly
D. only use mobile phones in urgent cases
Question 6: The man mentioned in the passage, who used his cellphone too often,
A. suffered serious loss of mental ability
B. had a problem with memory
C. abandoned his family
D. could no longer think lucidly
Question 7: The word "potentially" in the passage most closely means
A. obviously
B.possibly
C. certainly
D. privately

3. Read the following passage and type the letter A, B, C, or D into the blanks to indicate the correct word for each of the blanks.

Mobile phones emit microwave radio emissions. Researchers are questioning whether exposure to these radio waves might (1)
to brain cancer. So far, the data are not conclusive. The scientific evidence does not enable us to say with certainly
that mobile phones are categorically (2) . On the other hand, current research has not yet proved clear adverse effect
associated with the prolonged use of mobile phones
Numerous studies are now going (3) in various countries. Some of the results are contradictory but others have
shown an association between mobile phone use and cancer. (4) , these studies are preliminary and the issue needs
further, long - term investigation. Until the scientific data is more definite, it is prudent for people to try not to use mobile phone for long
periods of time. Don't think that hands free phones are any safer either. At the moment, research is in fact showing the opposite and they
may be just as dangerous. It is also thought that young people (5) bodies are still growing may be at particular risk.

Question 1.
A. bring
B. lead
C. cause
D. produce
Question 2.
A. risky
B. unhealthy
C. secure
D. safe
Question 3.
A. on
B. by
C. through
D. about
Question 4.
Trang 2/3
73
A. Additionally
B. However
C. While
D. Thought
Question 5.
A. whose
B. that
C. with
D. as

Trang 3/3
74
BÀI TẬP TỰ LUYỆN
BÀI. MẠO TỪ
PEN-C TIẾNG ANH - CÔ HƯƠNG FIONA

1. Mark the letter A, B, C or D on your answer sheet to indicate the correct answer to each of the following questions.
Jane had to go home from school because she had _______.
A. a headache B. the headache
C. headache D. one headache
2. Mark the letter A, B, C or D on your answer sheet to indicate the correct answer to each of the following questions.
I’ve noticed that.................. Spanish eat a lot of vegetables.
A. the B. ×
C. an D. a
3. Mark the letter A, B, C or D on your answer sheet to indicate the correct answer to each of the following questions.
I love this time in _____ when the sun is going down.
A. the morning B. evening
C. the evening D. morning
4. Mark the letter A, B, C or D on your answer sheet to indicate the correct answer to each of the following questions.
The judge sentenced the thief to six months in................... prison.
A. a B. the
C. × D. an

5. Mark the letter A, B, C or D on your answer sheet to indicate the correct answer to each of the following questions.
Reports are coming in of a major oil spill in _____Mediterranean.
A. a B. an
C. the D. no word

6. Mark the letter A, B, C or D on your answer sheet to indicate the correct answer to each of the following questions.
I like to listen to __________ in my free time.
A. the music B. one music
C. music D. a music
7. Mark the letter A, B, C or D on your answer sheet to indicate the correct answer to each of the following questions.
Barbara hopes to go to................ university next year.
A. × B. a
C. the D. an

8. Mark the letter A, B, C or D on your answer sheet to indicate the correct answer to each of the following questions.
Do you think that they’ll ever send a manned mission to _____ Venus ?
A. the B. no word
C. a D. an

9. Mark the letter A, B, C or D on your answer sheet to indicate the correct answer to each of the following questions.
The prize is _______ unique opportunity to travel the world!
A. a B. the
C. an D. no word
10. Mark the letter A, B, C or D on your answer sheet to indicate the correct answer to each of the following questions.
Oh, I didn’t tell you! We’ve got ___ new English teacher.
A. a B. the
C. an D. no word

11. Mark the letter A, B, C or D on your answer sheet to indicate the correct answer to each of the following questions.
Housewives find it easier to do domestic chores thanks to_____ invention of labour-saving devices.
A. a B. an
C. Some D. the

12. Mark the letter A, B, C or D on your answer sheet to indicate the correct answer to each of the following questions.
It’s true that................ rich lead a different life from the poor.
A. × B. a
C. an D. the
13. Mark the letter A, B, C or D on your answer sheet to indicate the correct answer to each of the following questions.
Here’s ______DVD you asked to borrow.

Trang 1/3
75
A. no word B. a
C. an D. the
14. Mark the letter A, B, C or D on your answer sheet to indicate the correct answer to each of the following questions.
It’s ______honour to be here this evening to speak to you.
A. an B. a
C. the D. no word
15. Mark the letter A, B, C or D on your answer sheet to indicate the correct answer to each of the following questions.
There’s _____ good chance we’ll be late for the meeting.
A. the B. no word
C. an D. a

16. Mark the letter A, B, C or D on your answer sheet to indicate the correct answer to each of the following questions.
Mary lives in................. Canada near Lake Ontario.
A. an B. a
C. × D. the
17. Mark the letter A, B, C or D on your answer sheet to indicate the correct answer to each of the following questions.
The robber was sent to______for a total of three years.
A. one prison B. prison
C. the prison D. a prison
18. Mark the letter A, B, C or D on your answer sheet to indicate the correct answer to each of the following questions.
The artist has ______ unusual name.
A. the B. no word
C. an D. a

19. Mark the letter A, B, C or D on your answer sheet to indicate the correct answer to each of the following questions.
The explorer crossed............... Pacific Ocean in a canoe.
A. no article B. an
C. a D. the

20. Mark the letter A, B, C or D on your answer sheet to indicate the correct answer to each of the following questions.
We’re out of _________coffee, so could you get some from the supermarket?
A. the B. an
C. a D. no word
21. Mark the letter A, B, C or D on your answer sheet to indicate the correct answer to each of the following questions.
We went to............... cinema twice a month.
A. a B. the
C. an D. some

22. Mark the letter A, B, C or D on your answer sheet to indicate the correct answer to each of the following questions.
................ examinations always make him nervous.
A. an B. ×
C. a D. the

23. Mark the letter A, B, C or D on your answer sheet to indicate the correct answer to each of the following questions.
She has been playing.............. flute for ten years.
A. × B. an
C. a D. the

24. Mark the letter A, B, C or D on your answer sheet to indicate the correct answer to each of the following questions.
Living in _____European city can be expensive.
A. no word B. the
C. a D. an
25. Mark the letter A, B, C or D on your answer sheet to indicate the correct answer to each of the following questions.
Sarah thinks.................. life is more difficult in a foreign country.
A. an B. ×
C. a D. the

26. Mark the letter A, B, C or D on your answer sheet to indicate the correct answer to each of the following questions.
A volcano has erupted in............. Philippines recently.
A. an B. the
C. many D. a

Trang 2/3
76
27. Mark the letter A, B, C or D on your answer sheet to indicate the correct answer to each of the following questions.
What time do you start................ work in the morning?
A. a B. ×
C. the D. an
28. Mark the letter A, B, C or D on your answer sheet to indicate the correct answer to each of the following questions.
For breakfast we usually have................... coffee and toast.
A. an B. ×
C. a D. the
29. Mark the letter A, B, C or D on your answer sheet to indicate the correct answer to each of the following questions.
It looks like _________ glass in your bedroom window is cracked.
A. a B. the
C. an D. no word
30. Mark the letter A, B, C or D on your answer sheet to indicate the correct answer to each of the following questions.
Salem doesn’t like.................. talkative people.
A. the B. many
C. × D. a

31. Mark the letter A, B, C or D on your answer sheet to indicate the correct answer to each of the following questions.
They went on a cruise down................. Nile and saw the Pyramids.
A. a B. the
C. an D. ×
32. Mark the letter A, B, C or D on your answer sheet to indicate the correct answer to each of the following questions.
I went to see the doctor because I’m finding it difficult to sleep at _____night.
A. no word B. an
C. the D. a
33. Mark the letter A, B, C or D on your answer sheet to indicate the correct answer to each of the following questions.
A recent survey has shown that _____________ increasing number of men are willing to share the housework with their wives.
A. an B. some
C. a D. the

34. Mark the letter A, B, C or D on your answer sheet to indicate the correct answer to each of the following questions.
My cousin works as _____in a café near where we live.
A. one waiter B. a waiter
C. the waiter D. waiter
35. Mark the letter A, B, C or D on your answer sheet to indicate the correct answer to each of the following questions.
It’s too far to walk so I think I’ll catch ___.
A. the bus B. buses
C. bus D. one bus

Trang 3/3
77
BÀI TẬP TỰ LUYỆN
BÀI. MODAL VERBS
PEN-C TIẾNG ANH - CÔ HƯƠNG FIONA

1. Choose the best answer among A, B, C or D to complete each sentence


We ……………. forget to shut the lift gates.
A. mustn’t B. should
C. must D. needn’t
2. Choose the best answer among A, B, C or D to complete each sentence
He helped her, but it was not necessary. He needn’t ________ her.
A. be helping B. have helped
C. help D. to help
3. Choose the best answer among A, B, C or D to complete each sentence
The fortune teller predicted that ……………. inherit a big fortune before the end of this year.
A. would B. she will
C. she would D. will
4. Choose the best answer among A, B, C or D to complete each sentence
You’ve been working non-stop for ten hours. You ________ be really tired.
A. would rather B. ought to
C. should D. must

5. Choose the best answer among A, B, C or D to complete each sentence


If you want to know about the time, pick up the receiver and dial 8081; you……………. say anything.
A. have to B. must
C. mustn’t D. needn’t

6. Choose the best answer among A, B, C or D to complete each sentence


The airline only allows two pieces of luggage. You……………. pack too much or you will have to take it out at the airport!
A. had not better B. had better not
C. had better D. better not
7. Choose the best answer among A, B, C or D to complete each sentence
"……………. you hand me that pair of scissors, please?" - "Certainly."
A. Shall B. May
C. Will D. Should

8. Choose the best answer among A, B, C or D to complete each sentence


I ……………. to take advantage of this opportunity to thank for your co-operation.
A. can B. am able to
C. might D. would like

9. Choose the best answer among A, B, C or D to complete each sentence


The waiter’s over there. I ……………. ask him for the bill.
A. am able to B. shall
C. will D. could
10. Choose the best answer among A, B, C or D to complete each sentence
“I left my bike here and now it’s gone. ” - “Someone ________ borrowed it. ”
A. should have B. needn’t have
C. couldn’t have D. must have

11. Choose the best answer among A, B, C or D to complete each sentence


You ……………. leave small objects lying around. Such objects ……………. be swallowed by children.
A. need, might B. needn’t, may
C. shouldn’t, must D. shouldn’t, may

12. Choose the best answer among A, B, C or D to complete each sentence


You……………. do the job if you didn’t speak Japanese fluently.
A. needn’t B. couldn’t
C. won’t be able to D. can’t
13. Choose the best answer among A, B, C or D to complete each sentence
"Can I borrow twenty dollars?" - "No, you know I……………. lend you money any more."

Trang 1/4
78
A. won’t B. might not
C. not going to D. shouldn’t have
14. Choose the best answer among A, B, C or D to complete each sentence
“I bought two bottles of milk. ” “You ________ have bought milk; we have heaps of it in the house. ”
A. needn’t B. couldn’t
C. hadn’t D. mustn’t
15. Choose the best answer among A, B, C or D to complete each sentence
I want this letter typed, but you ……………. do it today. Tomorrow you will do.
A. must B. mustn’t
C. couldn’t D. needn’t

16. Choose the best answer among A, B, C or D to complete each sentence


I……………. like to see the manager right now.
A. would B. shall
C. could D. will
17. Choose the best answer among A, B, C or D to complete each sentence
"You haven't eaten anything since yesterday afternoon. You……………. be really hungry!" - "I am."
A. will B. may
C. must D. should
18. Choose the best answer among A, B, C or D to complete each sentence
When I first went to England, I ……………. English, but I ……………. it.
A. could read / couldn’t speak B. could read / can’t speak
C. can read / can’t speak D. can read / couldn’t speak

19. Choose the best answer among A, B, C or D to complete each sentence


It's raining and I don't want to get my dress wet. I……………. an umbrella.
A. had better to bring B. had better not bring
C. hat better not bringing D. had better bring
20. Choose the best answer among A, B, C or D to complete each sentence
You ……………. go now. It’s getting late.
A. ought B. had rather
C. had better D. would rather
21. Choose the best answer among A, B, C or D to complete each sentence
They have plenty of time, so they need not……………..
A. hurry B. in hurry
C. to be hurried D. to hurry
22. Choose the best answer among A, B, C or D to complete each sentence
Dave loves chocolate, but he……………. too much or he will get fat.
A. shouldn't eat B. should eat not
C. should eat D. ought not eat
23. Choose the best answer among A, B, C or D to complete each sentence
You’d better ________ working so hard.
A. to stopping B. to stop
C. stop D. stopped
24. Choose the best answer among A, B, C or D to complete each sentence
We……………. reheat the pie. We can eat it cold.
A. C mustn’t B. must
C. needn’t D. should have
25. Choose the best answer among A, B, C or D to complete each sentence
She’s fainted. Throw some water on her face and she ……………. come round.
A. has to B. can have
C. may D. must

26. Choose the best answer among A, B, C or D to complete each sentence


“Whose car is this?” – “It ……………….. be Anton's. I saw him driving a red car like this one. ”
A. would B. could
C. must D. might

Trang 2/4
79
27. Choose the best answer among A, B, C or D to complete each sentence
“How old do you think Peter is?” - “I just looked at his driver’s license. He……………. 33. ”
A. must be B. should be
C. may be D. might be
28. Choose the best answer among A, B, C or D to complete each sentence
This is the key. You ……………. forget to take it with you when going out.
A. should B. needn’t
C. mustn’t D. have to
29. Choose the best answer among A, B, C or D to complete each sentence
You ……………….. leave work at 3. 30 today.
A. could B. can
C. will D. might
30. Choose the best answer among A, B, C or D to complete each sentence
Your wedding ring is very precious. You ……………. lose it.
A. needn’t B. mustn’t
C. must D. could

31. Choose the best answer among A, B, C or D to complete each sentence


I’m feeling very tired this morning. I ________ have stayed up late last night.
A. mustn’t B. to stopping
C. couldn’t D. shouldn’t
32. Choose the best answer among A, B, C or D to complete each sentence
You ……………. drink this; it’s poison.
A. mustn’t B. must
C. shouldn’t D. needn’t
33. Choose the best answer among A, B, C or D to complete each sentence
The fortune teller predicted that ________ inherit a big fortune before the end of this year.
A. will B. she would
C. would D. she will

34. Choose the best answer among A, B, C or D to complete each sentence


He had been working for more than 11 hours. He ……………. be tired after such hard work. He ……………. prefer to get some rest.
A. may, must B. must, need
C. must, should D. must, may
35. Choose the best answer among A, B, C or D to complete each sentence
If a letter comes for me, ……………. you please forward it to this address?
A. could B. should
C. shall D. must
36. Choose the best answer among A, B, C or D to complete each sentence
“________ you like to play a game of tennis?” “I’d love to. ”
A. Would B. Could
C. Do D. Will
37. Choose the best answer among A, B, C or D to complete each sentence
“Do you remember reading about it in the newspaper?” - “No, I ________ abroad at the time. ”
A. should be B. must have been
C. must be D. should have been
38. Choose the best answer among A, B, C or D to complete each sentence
His letter is full of mistakes. He ________ the mistakes carefully before sending it.
A. must have checked B. could have checked
C. should have checked D. can have checked
39. Choose the best answer among A, B, C or D to complete each sentence
You……………. forget to pay the rent tomorrow. The landlord is very strict about paying on time.
A. don’t have to B. can’t
C. couldn’t D. mustn’t
40. Choose the best answer among A, B, C or D to complete each sentence
It……………. rain this evening. Why don’t you take an umbrella?

Trang 3/4
80
A. could B. must
C. had better D. might
41. Choose the best answer among A, B, C or D to complete each sentence
Bob is absent; he ……………. sick again now.
A. must have been B. must being
C. must D. must be

42. Choose the best answer among A, B, C or D to complete each sentence


"Have you seen Jane?" - "No, but she ……………. be at her desk."
A. would B. ought
C. may D. can
43. Choose the best answer among A, B, C or D to complete each sentence
I'm going to visit your country. Where……………. if I want to go shopping?
A. ought to I go B. should I going
C. should I go D. I should go
44. Choose the best answer among A, B, C or D to complete each sentence
“……………. taking me downtown on your way to work this morning?” – “Not at all. ”
A. Could you please B. Can you
C. Why don’t you D. Would you mind
45. Choose the best answer among A, B, C or D to complete each sentence
His eyes were so bad that he……………. read the number on the board.
A. hadn’t to B. couldn’t
C. shouldn’t D. can’t
46. Choose the best answer among A, B, C or D to complete each sentence
You……………. ring the bell; I have a key.
A. mustn’t B. may not
C. needn’t D. couldn’t
47. Choose the best answer among A, B, C or D to complete each sentence
You……………. talk to other candidates during the exam.
A. needn’t B. shan’t
C. hadn’t better D. mustn’t
48. Choose the best answer among A, B, C or D to complete each sentence
You’ve been working non-stop for ten hours. You……………. be really tired.
A. should B. must
C. would rather D. ought to
49. Choose the best answer among A, B, C or D to complete each sentence
I need help, doctor. My baby doesn't sleep well. What……………. ?
A. I should do B. had I better do
C. should I do D. I'm so hot
50. Choose the best answer among A, B, C or D to complete each sentence
It’s too hot. You……………. your coat!
A. should to take off B. might putting on
C. should take off D. should put on

Trang 4/4
81
BÀI TẬP TỰ LUYỆN
BÀI. FOOD AND DRINKS
PEN-C TIẾNG ANH - CÔ HƯƠNG FIONA

1. Read the text carefully and then TYPE the correct answer (A, B, C or D) for each sentence from 1to 4.

If you are invited to someone’s house in America for dinner, you should bring a gift, such as a bunch of flowers or a box of chocolates. If
you give your host a wrapped gift, he/she may open it in front of you. Opening a gift in front of the gift-giver in considered polite. It shows
that the host is excited about receiving the gift and wants to show his/her appreciation to you immediately. Even if the host doesn’t like it,
he/she will tell “a white lie” and say how much they like the gift to prevent the guest from feeling bad.
If your host asks you to arrive at the particular time, you should not arrive exactly on time or earlier than expected time, because this is
considered to be inconvenient and therefore rude, as the host may not be ready.

Question 1: In America, if you are invited for dinner, you should bring as a gift.
A. Nothing
B. wine
C. a bunch of flowers
D. soft drink
Question 2: Opening a gift in front of the gift-giver is considered
A. rude
B. courteous
C. impolite
D. hospitable
Question 3: Why does the host open the gift in front of you ?
A. To show his feelings of the gift
B. To show his gratitude to you
C. To show his wish for a gift
D. To show his understanding
Question 4: When invited for dinner, you shouldn’t arrive exactly on time because
A. the host may be rude to you
B. it may be inconvenient for you
C. it may take you a lot of time
D. the host may not be ready

2. Read the following passage and TYPE the letter A, B, C, or D into the blanks to indicate the correct word or phrase that bestfits
each of the numbered blanks from 1 to 5.

In "Cerealizing America", Scott Bruce and Bill Crawford remark that the cereal industry uses 816 million pounds of sugar per year.
Americans buy 2.7 billion packages of breakfast cereal each year. If (1) end to end the empty cereal boxes from one
year's consumption would (2) to the moon and back. One point three (1.3) million advertisements for cereal are
broadcast on American television every year at a(n) (3) of $762 million for airtime. Only automobile manufacturers
spend more money on television advertising than the makers of breakfast cereal.
(4) of the boxed cereals found in supermarkets contain large amounts of sugar and some contain more than 50%
sugar. Cereal manufacturers are very clever in their marketing, making many cereals appear much healthier than they really are by
"fortifying" them with vitamins and minerals. Oh, (5) - you now have vitamin-fortified sugar!
Before you eat any cereal, read the ingredient list and see how high sugar appears on the ingredient list. Then check the "Nutrition facts"
panel.
From "Foods That Burn Fat, Foods That Turn to Fat" by Tom Ventulo.

Question 1.
A. laying
B. lay
C. laid
D. to lay
Question 2.
A. prolong
B. stretch
C. contact
D. reach
Question 3.
A. cost
B. charge
Trang 1/3
82
C. average
D. expense
Question 4.
A. Mostly
B. Furthermost
C. Most
D. Almost
Question 5.
A. beautiful
B. gorgeous
C. Lovely
D. charming
3. Read the following passage and blacken the letter A, B, C or D on your answer sheet to indicate the correct word for each of the
blanks in the following questions

Dining Etiquette in Japan: an honored guest sits an the entre of the table furthest from thr door and begins eating first. Learn to (1)
chopsticks – never point them, never pierce them, and rest them on the chopsticks – rest when breaking (2)
drink or chat. It is good etiquette to try a bit of everything.
Dining Etiquette in Turkey: Meals are a (3) affair, conversations are friendly and loud. The head of the family of
honored guest is served first. It is good etiquette to insist the kost senior is served first (4) of you. Asking for (5)
food is a compliment. If taken to a restaurant, Turkish dining etiquette has strict rules that the one extended the
invitation must pay.

Question 1:
A. handle
B. use
C. grasp
D. keep
Question 2:
A. with
B. on
C. as
D. for
Question 3:
A. socialize
B. society
C. social
D. because
Question 4:
A. instead
B. regardless
C. out
D. because
Question 5:
A. more
B. many
C. so many
D. the most
4. Read the following passage and TYPE the latter A, B, C or D into the blanks to indicate the word or phrase that best fits each of
the numbered blanks.

Sugar tastes sweet because of thousands of receptors on the tongue which connect the substance with the brain. The taste of sweetness is
universally accepted as the most pleasurable known, although it is a fructose. Abundant is the most common occurring sugar, (1)
of which include fruit and honey. Sucrose, which supplies glucose to the body, is (2) from the
sugar cane plant, and white sugar (pure sucrose) is used by food technologists to (3) sweetness in other substances.
Approximately a dozen artificial sweeteners have been discovered; one of the earliest was Sorbitol from France.
Manufacturers add large amounts of sugar to foodstuffs but never more than the (4) required to produce the optimum
pleasurable taste. Surprisingly, this amount is similar for different people and in different cultures. No one has (5)
discovered a way to predict whether a substance will taste sweet, and it was by chance alone that all the man-made chemical sweeteners
were found to be sweet.

Question 1:
A. roots
B. origins
C. sources

Trang 2/3
83
D. places
Question 2:
A. drawn
B. extracted
C. cited
D. made
Question 3:
A. smell
B. detect
C. taste
D. measure
Question 4:
A. maximum
B. excess
C. extremity
D. limit
Question 5:
A. just
B. yet
C. still
D. already

Trang 3/3
84
BÀI TẬP TỰ LUYỆN
BÀI. CÂU ĐIỀU KIỆN (P.1)
PEN-C TIẾNG ANH - CÔ HƯƠNG FIONA

1. Please let me know as soon as possible if he ………….. to come.


A. deciding B. had decided
C. should decide D. decides
2. Had the evidence not ………….. , the accused would have been found guilty.
A. been destroyed B. destroy
C. destroyed D. was destroyed
3. The rescue team wouldn't have found the lost boy scouts if the storm………………at night.
A. were being B. was
C. had been being D. had been
4. If I find it, I ………you.
A. would tell B. will tell
C. Told D. had told
5. If I had known you were in hospital, I …………to see you.
A. would go B. will go
C. went D. would have gone.
6. ………………you need any further assistance, do not hesitate to contact us.
A. Could B. Were
C. If should D. Should

7. Please ask to see the manager should you ……………any further questions.
A. had been having B. were having
C. should have D. have

8. If I ……….. a camera, I would have taken some pictures.


A. had B. have
C. had had D. would have

9. I would be very surprised if he……………..


A. would refuse. B. refuses
C. had refused D. refused
10. It would have been a good crop............ through.
A. didn't a storm sweep B. if the storm didn't sweep
C. would a storm not have swept D. had a storm not swept
11. Many people would be out of work if that factory……….. down.
A. closed B. would close.
C. had closed D. closes
12. If she sold her car, she ………… much money.
A. gets B. will get
C. would have got. D. would get
13. Mary can help herself to the refrigerator should she………….. hungry.
A. will get B. gets
C. got D. get
14. What would happen if you …………….. to work tomorrow?
A. don’t go B. won’t go
C. didn’t go D. wouldn’t go
15. If someone…………in here with a gun, I would be very frightened.
A. walks B. would walk
C. walked. D. had walked

16. You won’t pass the examination……………you study more.

Trang 1/2
85
A. Whether B. if
C. as long as D. unless
17. If I ………. a lot of money now, I ………….. a new car.
A. have / would buy B. had/ will buy
C. have /will buy D. had/ would buy.
18. …………………….. the book been cheaper, I would have bought them all.
A. If B. So that
C. Were to D. Had
19. If I were offered the job, I think I ………. it.
A. would take B. take
C. will take D. would have taken.
20. If I had enough money, I ……… abroad to improve my English.
A. would go B. will go
C. should have go to D. should go

21. We ‘ll get wet if we …………. out.


A. went B. had gone.
C. did go D. go
22. If I ………………… the job, I would have to think about it.
A. were offered B. offering
C. offer D. to be offered
23. I………….. out if I hadn’t been so tired.
A. went B. would have gone
C. would go D. will go
24. If I................. the competition, I............. on a world cruise.
A. won/ would go B. won/ would have gone
C. win/ will go D. will win/ go
25. If I go shopping, I ………some food.
A. buy B. will buy
C. would buy D. would have bought.
26. If I …………him, I would have said hello to him.
A. had seen B. saw
C. see D. would see
27. If I ……………you, I ……………. do that.
A. had been/ would. B. were/ will
C. am/ will D. were /would

28. They would be disappointed if we…………….


A. don’t come B. hadn’t come
C. didn’t come. D. wouldn’t come

29. If the Minister ……………. to get the necessary support, he would be forced to call a snap election.
A. fails B. failed
C. should fail D. had failed

30. If I ………….. you, I wouldn't spend the reward on things I don't need.
A. was B. being
C. were D. am

Trang 2/2
86
BÀI TẬP TỰ LUYỆN
BÀI. CÂU ĐIỀU KIỆN (P.2)
PEN-C TIẾNG ANH - CÔ HƯƠNG FIONA

1. Choose the best answer to complete each of the following questions

Had you not told me that this was going to happen, I________ it.
A. can’t believe B. hadn’t believed
C. would have never believed D. don’t believe
2. Choose the best answer to complete each of the following questions

Unless you _________ all of my questions, I can’t do anything to help you.


A. answered B. would answer
C. are answering D. answer
3. Mark the letter A, B, C, or D on your answer sheet to indicate the correct answer to each of the following questions.

If people paid a little more attention to the environment, the Earth _______ greener
A. had been B. will be
C. would have been D. would be

4. Choose the best answer to complete each of the following questions

If you will give me one good reason for your acting like this,_________ this incident again.
A. I will never mention B. will I never mention
C. I never mention D. I don’t mention
5. Choose the best answer to complete each of the following questions

You______ if you have taken a map.


A. will get lost B. would have got lost
C. would get lost D. won’t get lost
6. Mark the letter A, B, C, or D on your answer sheet to indicate the correct answer to each of the following questions.

________, he would have been able to pass the exam


A. If he studied hardly last year B. Had he studied harder last year
C. Studying harder last year D. Provided he studied hard last year
7. Choose the best answer to complete each of the following questions

If he had listened to me, he ________ trouble finding the way to the hotel at the moment.
A. wouldn’t have B. wouldn’t have had
C. won’t have D. hasn’t had

8. Mark the letter A, B, C or D on your answer sheet to indicate the correct answer to each of the following

________ the book been cheaper, I would have bought them all
A. Had B. So that
C. Were to D. If
9. Choose the best answer to complete each of the following questions

If Jake_________ to go on the trip, would you have gone?


A. didn’t agree B. doesn’t agree
C. hadn’t agreed D. wouldn’t agree

10. Mark the letter A, B, C or D on your answer sheet to indicate the correct answer to each of the following

Had the evidence not ______ , the accused would have been found guilty
A. destroy B. was destroyed
C. destroyed D. been destroyed
11. Choose the best answer to complete each of the following questions

______ then what I know today, I would have saved myself a lot of time and trouble over the years.

Trang 1/3
87
A. had I known B. did I know
C. If I would know D. If I know
12. Choose the best answer to complete each of the following questions

______ Mary to study hard, she would pass the exam.


A. Should B. Had
C. If D. Were
13. Choose the best answer to complete each of the following questions

If I ______ the same problem you had as a child, I might not have succeeded in life as well as you have.
A. have B. had had
C. should have D. would have

14. Mark the letter A, B, C or D on your answer sheet to indicate the correct answer to each of the following questions.

If everyone _______, how would we control the traffic?


A. had flown B. could fly
C. can flies D. fly
15. Choose the best answer to complete each of the following questions

We would save thousands of lives if we ______out the remedy for the flu.
A. find B. will find
C. found D. had not found

16. Mark the letter A, B, C or D on your answer sheet to indicate the correct answer to each of the following questions.

But for the fact that I had owed him a favor, I _______ to help him.
A. couldn’t agree B. would have agreed
C. would agree D. wouldn’t have agreed

17. Choose the best answer to complete each of the following questions

If you had told me earlier, I _____________ it to you now.


A. would give B. would have given
C. will give D. gave
18. Mark the letter A, B, C or D on your answer sheet to indicate the correct answer to each of the following questions.

If you hadn’t watched that late movie last night, you _____ sleepy now.
A. wouldn’t have been B. might not have been
C. wouldn’t have been being D. wouldn’t be
19. Mark the letter A, B, C or D on your answer sheet to indicate the correct answer to each of the following.

If the Minister _______ to get the necessary support, he would be forced to call a snap election
A. failed B. fails
C. should fail D. had failed
20. Choose the best answer to complete each of the following questions

I could have understood him if he _____ more slowly in the press conference
A. had spoken B. speaks
C. spoke D. would have spoken
21. Mark the letter A, B, C, or D on your answer sheet to indicate the correct answer to each of the following questions.

If I _________ my passport, I’ll be in trouble


A. will lose B. lost
C. Lose D. would lose
22. Choose the best answer to complete each of the following questions

_______ more help, I can call my neighbor.


A. I have needed B. should I need
C. I should need D. needed
Trang 2/3
88
23. Mark the letter A, B, C, or D on your answer sheet to indicate the correct answer to each of the following questions.

Without the traffic jam on the high way this morning, I _____ late for the meeting.

A. hadn't been B. wouldn't have been


C. would be D. would have been

24. Choose the best answer to complete each of the following questions

I ______ you sooner, had someone told me you were in the hospital.
A. visit B. would have visited
C. had visited D. visited
25. Choose the best answer to complete each of the following questions

Do you think there would be less conflict in the world if all people _______ the same language?
A. will speak B. had spoken
C. spoke D. speak

Trang 3/3
89
BÀI TẬP TỰ LUYỆN
BÀI. THỨC GIẢ ĐỊNH
PEN-C TIẾNG ANH - CÔ HƯƠNG FIONA

1. Choose the best answer


‘Do you mind if I smoke?’ ‘Well, I'd rather you____. ’
A. didn't B. don’t
C. did D. will be
2. Choose the best answer
It is essential that he ____ by your side.
A. stay B. would stay
C. stays D. stayed
3. Choose the best answer
I wish _____ still in use.
A. it were B. it is
C. it was D. it be
4. Choose the best answer
I insisted that he ____me money.
A. paid B. pays
C. is paying D. pay

5. Choose the best answer


She suggests that Mark _____ full time from Saturday.
A. works B. work
C. worked D. will work

6. Choose the best answer


Dr. Smith asked that Mark _____his research paper before the end of the month.
A. has submitted B. submitted
C. submits D. submit
7. Choose the best answer
"If I____two-faced, would I be wearing this one?“ (Abraham Lincoln)
A. be B. was
C. were D. am

8. Choose the best answer


I wish ____able to type faster.
A. he were B. was able
C. him to D. he to

9. Choose the best answer


They made a _____that we be early.
A. suggestion B. suggestive
C. suggest D. suggestions
10. Choose the best answer
I propose that Lee_______asked to play the guitar.
A. be B. x
C. will D. is

11. Choose the best answer


It is recommended that ______a gallon of water with him if he wants to hike to the bottom of the Grand Canyon.
A. he took B. he take
C. he takes D. she takes

12. Choose the best answer


I proposed that she ____a gym course.
A. should register B. register
C. "register" or "should register" D. registers
13. Choose the best answer
It's vital that I _____ the truth because I hate lies.

Trang 1/3
90
A. be told B. am told
C. tell D. was told
14. Choose the best answer
The company asked that employees _______ personal phone calls during business hours.
A. to accept B. do not accept
C. didn’t accept D. not accept
15. Choose the best answer
It is important that ______ immediately.
A. he arrive B. he arrived
C. he arrives D. he will arrive

16. Choose the best answer


They demanded that he _____ at once.
A. should leave B. leaves
C. "should leave" or "leave" D. leave
17. Choose the best answer
He said it was _____that Johan guard the main gate.
A. mandate B. madates
C. mandated D. mandatory
18. Choose the best answer
If I ____ a butterfly, I _____wings.
A. was – will have B. were – would be
C. were – will be D. were – would have

19. Choose the best answer


The teacher demands that we ____ on time.
A. is B. be
C. are D. will be

20. Choose the best answer


His requirement is that everyone________computer literate.
A. is B. x
C. will be D. be
21. Choose the best answer
I propose that we _____ in Tim's apartment when he gets home.
A. have all been waiting B. are all waiting
C. all waited D. all be waiting

22. Choose the best answer


We urge that he _____ immediate action.
A. takes B. took
C. take D. will take

23. Choose the best answer


I recommended that she ____ to the concert.
A. didn’t go B. doesn’t go
C. not go D. won’t go

24. Choose the best answer


It is ordinance that all pet owners _____ their animals on a leash.
A. are keeping B. keeps
C. kept D. keep
25. Choose the best answer
It is_____ that you be standing there when he gets off the plane.
A. advised B. advices
C. advice D. advising

26. Choose the best answer


The principal suggested that he_____ a scholarship.
A. is awarded B. as awarded
C. be awarded D. warded

Trang 2/3
91
27. Choose the best answer
It is crucial that you________there before Tom arrives.
A. were B. be
C. are D. will be
28. Choose the best answer
The boss insisted that Sam____at the meeting.
A. be not B. not be
C. isn’t D. doesn’t
29. Choose the best answer
Donna requested that Frank ____ to the party.
A. come B. came
C. should came D. comes

Trang 3/3
92
BÀI TẬP TỰ LUYỆN
BÀI. CÂU BỊ ĐỘNG
PEN-C TIẾNG ANH - CÔ HƯƠNG FIONA

1. We can’t go along here because the road ____________


A. is being repaired B. is repairing
C. repairs D. is repaired
2. Gold____________in California in the 19th century
A. they discover B. was discovered
C. has been discovered D. was discover
3. There ____ a number of reasons for the falloff the Roman Empire.
A. are said to have been B. was said being
C. are said being D. said to be
4. Why did Tom keep making jokes about me? – I don’t enjoy____________at
A. being laughed B. be laughed
C. laughing D. to be laughed
5. The preparation____________by the time the guest____________
A. have been finished - were arrived B. had been finished – arrived
C. have finished - arrived D. had finished - were arriving
6. No one enjoys _______ in public.
A. making fun of B. being made fun of
C. to be made fun of D. to make fun of

7. We____________by a loud noise during the night


A. were waking up B. were woken up
C. are woken up D. woke up

8. I must go to the dentist and___________.


A. take care of my teeth B. my teeth be taken care of
C. get my teeth to take care of D. get my teeth taken care of

9. Ted ______ by a bee while he was sitting in the garden.


A. got stung B. gets stung
C. got sting D. get stung
10. Football is thought ___________ in the world.
A. to be the most popular sport B. to have played the most popular sport
C. to play the most popular sport D. to have been the most popular sport
11. Bicycles____________in the driveway
A. must not be leaving B. must not leave
C. must not be left D. must not have left
12. References ____________ in the examination room.
A. not are used B. didn’t used
C. is not used D. are not used
13. The story I’ve just read____________Agatha Christie
A. wrote by B. was written from
C. was written D. was written by
14. The boy____________by the teacher yesterday.
A. was punished B. punishing
C. punish D. punished
15. Body language ________ communication effectively.
A. knew to have been enhanced B. was known to be enhanced
C. has known to enhance D. is known to enhance
16. “Your kitchen is fantastic! Did you do it all by yourself?” – “No, I ______ by a professional. ”

Trang 1/2
93
A. have it to be designed B. had designed it
C. had it designed D. designed it
17. I suggest the room ____ before Christmas.
A. were decorated B. be decorated
C. should decorate D. is decorated
18. James____________the news as soon as possible.
A. should be telled B. should tell
C. should told D. should be told
19. I’m going to go out and____________
A. have my hair cut B. have cut my hair
C. my hair be cut D. cut my hair
20. ____________ yet?
A. Have been the letters typed B. Have the letters typed
C. Have the letters been typed D. Had the letters typed

21. Today, many serious childhood diseases____________by early immunization


A. can prevent B. can be prevented
C. prevent D. are preventing
22. Lots of houses____________by the earthquake
A. destroyed B. are destroying
C. is destroyed D. were destroyed
23. Black, red, and even bright pink diamonds ______.
A. have occasionally been found B. occasionally found
C. occasionally to find D. have occasionally found
24. Some film stars____________difficult to work with
A. are said be B. are said to be
C. said to be D. say to be
25. Maria Jannings is said ________ her memory.
A. to lose B. to have lost
C. to have been lost D. to be lost
26. It seems that no-one predicted the correct result.
A. No-one seems have predicted the correct result. B. No-one seem to have predicted the correct result.
C. No-one seems to have predicted the correct result. D. No-one seems to has predicted the correct result.
27. The rumors go that Jack will be arrested. He is said _______ a bloody robbery.
A. to have joined in B. to take part in
C. to have taken part in D. to join in

28. Beethoven’s Fifth Symphony____________next weekend


A. will be performing B. is going to be performed
C. has been performed D. will have perform

29. Sarah is wearing a blouse. It____________of cotton.


A. are made B. is made
C. made D. be made

30. We got our mail ____________yesterday.


A. delivered B. to deliver
C. been delivered D. delivering
31. It was announced that neither the passengers nor the driver ______ in the crash.
A. were injured B. was injured
C. injured D. had injured

Trang 2/2
94
BÀI TẬP TỰ LUYỆN
BÀI. WOMEN RIGHTS
PEN-C TIẾNG ANH - CÔ HƯƠNG FIONA

1. Read the following passage and type the letter A, B, C, or D into the blanks to indicate the correct word or phrase that best fits
each of the numbered blanks from 1 to 5.

Many people today would like the traditional two-parent family back, that is to say, they want a man and a woman to (1)
for life; they also think the man should support the family and the woman should stay home with the children.
However, few families now (2) into this category. In fact, if more women decide to have children on their own, the
single-parent household may become more typical than the traditional family in many countries. Also, unmarried couples may decide to
have more children – or they might take in foster children or (3) . And because people are staying single and living
longer (often as widows), there may be more one-person households. (4) the other hand, some people believe similar
events happen again and again in history: if this is true, people may go back to the traditional (5) or nuclear family of
the past. Others think the only certainty in history is change: in other words, the structure of the future family could begin to change faster
and faster and in more and more ways.

Question 1:
A. marry
B.wed
C. engage
D. gather
Question 2:
A. belong
B. crowd
C. group
D. fall
Question 3:
A. bring up
B. feel
C. adapt
D. adopt
Question 4:
A. In
B. At
C. On
D. for
Question 5:
A. extended
B. extensive
C. enlarged
D. big
2. Read the following passage and type the letter A, B, C, or D into the blanks to indicate the correct answer to each of the questions
from 1 to 8

During the 19th century, women in the U. S organized and participated in a large number of reform movements, including movements to
reorganize the prison system, improve education, ban the sale of alcohol, and most importantly to free slaves. Some women saw
similarities in the social status of women and slaves. Women like Elizabeth Cady Stanton and Lucy Stone were feminists and abolitionists
who supported the rights of both women and blacks. A number of male abolitionists, including William Lloyd Garrison and Wendell
Phillips also supported the rights of women to speak and participate equally with men in anti- slavery activities. Probably more than any
other movement, abolitionism offered women a previously denied entry into politics. They became involved primarily in order to better
their living conditions and the conditions of others.
When the Civil war ended in 1865, the 14th, and 15th, Amendments to the Constitution adopted in 1868 and 1870 granted citizenship and
suffrage to blacks but not to women. Discouraged but resolved, feminists influenced more and more women to demand the right to vote. In
1869, the Wyoming Territory had yielded to demands by feminists, but eastern states resisted more stubbornly than ever before. A woman's
suffrage bill had been presented to every Congress since 1878 but it continually failed to pass until 1920, when the 19th Amendment
granted women the right to vote.

Question 1: What is the topic of the passage?


A. Women's suffrage
B. Abolitionists
C. The Wyoming Territory.
D. The 14th and 15th Amendment
Question 2: The word “ban” in line 2 most nearly means to
A. encourage
B. publish
C. prohibit
Trang 1/4
95
D. limit
Question 3: What is not among the reformation movements of women?
A. reorganizing the prison
B. passing the laws
C. freeing the slaves
D. prohibiting the sale of alcohol
Question 4: According to the passage, why did women become active in politics?
A. to improve the conditions of life that existed at the time.
B. to support Elizabeth Cady Stanton
C. to amend the Declaration of Independence
D. to be elected to public office.
Question 5: The word" primarily" in the first paragraph is closest in meaning to
A. somewhat
B. above all
C. always
D. finally
Question 6: What does the 19th Amendment guarantee?
A. Citizenship for women
B. Citizenship for blacks
C. Voting rights for women
D. Voting rights for blacks
Question 7: When were women allowed to vote throughout the US?
A. After 1920
B. After 1878
C. After 1870
D. After 1866
Question 8: What can not be inferred from the passage?
A. The blacks were given the right to vote before women.
B. The abolitionists believed in anti- slavery activities.
C. A women's suffrage bill had been discussed in the Congress for 50 years.
D. The eastern states did not like the idea of women's right to vote.
3. Read the following passage and mark the letter A, B, C or D on your answer sheet to indicate the correct answer ti each of the
questions from 1 to 8.

The issue of equality for women in British society first attracted national attention in the early 20th century, when the suffragettes won for
women the right to vote. In the 1960s feminism became the subject of intense debate when the women’s liberation movement encouraged
women to reject their traditional supporting role and to demand equal status and equal rights with men in areas such as employment and
pay. Since then, the gender gap between the sexes has been reduced. The Equal Pay Act of 1970, for instance, made it illegal for women
to be paid less than men for doing the same work, and in 1975 the Sex Discrimination Act aimed to prevent either sex having an unfair
advantage when applying for jobs. In the same year the Equal Opportunities Commission was set up to help people claim their rights to
equal treatment and to publish research and statistics to show where improvements in opportunities for women need to be made. Women
now have much better employment opportunities, though they still tend to get less well-paid jobs than men, and very few are appointed to
top jobs in industry. In the US the movement that is often called the “first wave of feminism” began in the mid 1800s. Susan B. Anthony
worked for the right to vote, Margaret Sanger wanted to provide women with the means of contraception so that they could decide whether
or not to have children, and Elizabeth Blackwell, who had to fight for the chance to become a doctor, wanted women to have greater
opportunities to study. Many feminists were interested in other social issues. The second wave of feminism began in the 1960s. Women
like Betty Friedan and Gloria Steinem became associated with the fight to get equal rights and opportunities for women under the law. An
important issue was the Equal Rights Amendment (ERA), which was intended to change the Constitution. Although the ERA was not
passed, there was progress in other areas. It became illegal for employers, schools, clubs, etc. to discriminate against women. But women
still find it hard to advance beyond a certain point in their careers, the so-called glass ceiling that prevents them from having high-level
jobs. Many women also face the problem of the second shift, i.e. the household chores. In the 1980s, feminism became less popular in the
US and there was less interest in solving the remaining problems, such as the fact that most women still earn much less than men.
Although there is still discrimination, the principle that it should not exist is widely accepted.

Question 1: It can be inferred from paragraph 1 that in the 19th century,


A. British women did not have the right to vote in political elections
B. most women did not wish to have equal status and equal rights
C. suffragettes fought for the equal employment and equal pay
D. British women did not complete their traditional supporting role
Question 2: The phrase “gender gap” in paragraph 2 refers to
A. the social relationship between the two sexes
B. the visible space between men and women
C. the social distance between the two sexes
D. the difference in status between men and women
Question 3: Susan B. Anthony, Margaret Sanger, and Elizabeth Blackwell are mentioned as
A. American women who were more successful than men
B. American women who had greater opportunities
Trang 2/4
96
C. pioneers in the fight for American women’s rights
D. American women with exceptional abilities
Question 4: The Equal Rights Amendment (ERA)
A. supported employers, schools and clubs
B. changed the US Constitution
C. was brought into force in the 1960s
D. was not officially approved
Question 5: In the late 20th century, some information about feminism in Britain was issued by
A. the Equal Rights Amendment
B. the Equal Pay Act of 1970
C. the Equal Opportunities Commission
D. the Sex Discrimination Act
Question 6: Which of the following is true according to the passage?
A. The US movement of feminism became the most popular in the late 20th century.
B. The British government passed laws to support women in the early 20th century.
C. The women’s liberation movement in the world first began in Britain.
D. The movement of feminism began in the US earlier than in Britain.
Question 7: The phrase “glass ceiling” in paragraph 4 mostly means
A. a ceiling made of glass
B. an imaginary barrier
C. a transparent frame
D. an overlooked problem
Question 8: Which of the following is NOT mentioned in the passage?
A. An American woman once had to fight for the chance to become a doctor.
B. British women now have much better employment opportunities.
C. There is now no sex discrimination in Britain and in the US.
D. Many American women still face the problem of household chores.
4. Read the following passage and type the letter A, B, C, or D into the blanks to indicate the correct answer to each of the questions.

Until recently, most American entrepreneurs were men. Discrimination against women in business, the demands of caring for families, and
lack of business training had kept the number of women entrepreneurs small. Now, however, businesses owned by women account for
more than $40 billion in annual revenues, and this figure is likely to continue rising throughout the 1990s. As Carolyn Doppelt Gray, an
official of the Small Business Administration, has noted, "The 1970s was the decade of women entering management, and the 1980s turned
out to be the decade of the woman entrepreneur". What are some of the factors behind this trend? For one thing, as more women earn
advanced degrees in business and enter the corporate world, they are finding obstacles. Women are still excluded from most executive
suites. Charlotte Taylor, a management consultant, had noted, "In the 1970s women believed if they got an MBA and worked hard they
could become chairman of the board. Now they've found out that isn't going to happen, so they go out on their own".
In the past, most women entrepreneurs worked in "women's" fields: cosmetics and clothing, for example. But this is changing. Consider
ASK Computer Systems, a $22-million-a-year computer software business. It was founded in 1973 by Sandra Kurtzig, who was then a
housewife with degrees in math and engineering. When Kurtzig founded the business, her first product was software that let weekly
newspapers keep tabs on their newspaper carriers-and her office was a bedroom at home, with a shoebox under the bed to hold the
company's cash. After she succeeded with the newspaper software system, she hired several bright computer-science graduates to develop
additional programs. When these were marketed and sold, ASK began to grow. It now has 200 employees, and Sandra Kurtzig owns $66.9
million of stock.
Of course, many women who start their own businesses fail, just as men often do. They still face hurdles in the business world, especially
problems in raising money; the banking and finance world is still dominated by men, and old attitudes die hard. Most businesses owned by
women are still quite small. But the situation is changing; there are likely to be many more Sandra Kurtzigs in the years ahead.

Question 1: What is the main idea of this passage?


A. The computer is especially lucrative for women today.
B. Women today are better educated than in the past, making them more attractive to the business world.
C. Women are better at small business than men are.
D. Women today are opening more business of their own.
Question 2: The word “excluded” is closest meaning to
A. often invited to
B. decorators of
C. not permitted in
D. charged admission to
Question 3: All of the following were mentioned in the passage as detriments to women in the business world EXCEPT

A. Women were required to stay at home with their families.


B. Women faced discrimination in business.
C. Women lacked ability to work in business.
D. Women were not trained in business.
Question 4: According to the passage, Charlotte Taylor believes that women in 1970s
A. were unrealistic about their opportunities in business management.
B. had fewer obstacles in business than they do today.
C. were unable to work hard enough to success in business. Trang 3/4
97
D. were still more interested in education than business opportunities
Question 5: The author mentions the “ shoesbox under the bed” in order to
A. show the resourcefulness of Sandra Kurtzig
B. Show the frugality of women in business
C. Point out that initially the financial resources of Sandra Kurtzig‟s business were limited
D. suggest that the company needed to expand
Question 6: The expression “ keep tabs on” is closest meaning to
A. pay the salaries of
B. keep records of
C. provide transportation for
D. recognize the appearance of
Question 7: The word “hurdles” can be best replaced by
A. fences
B. .obstacles
C. questions
D. small groups
Question 8: It can be inferred from the passage that the author believes that business operated by women are small because

A. many women fail at large businesses.


B. Women are not able to borrow money easily.
C. Women prefer a small intimate setting.
D. Women can’t deal with money.

Trang 4/4
98
BÀI TẬP TỰ LUYỆN
BÀI. CÂU GIÁN TIẾP
PEN-C TIẾNG ANH - CÔ HƯƠNG FIONA

1. Mark the letter A, B, C or D on your answer sheet to indicate the correct answer to each of the following questions.
He said that Ha and his friend ______ married ________
A. are getting / the next day B. were getting / tomorrow
C. will get! the day after D. were getting/ the next day
2. Mark the letter A, B, C or D on your answer sheet to indicate the correct answer to each of the following questions.
"Be aware of the dog," said Tom. ~ Tom warned us ______
A. being aware of the dog B. we should be aware of the dog
C. to be aware of the dog D. aware of the dog
3. Mark the letter A, B, C or D on your answer sheet to indicate the correct answer to each of the following questions.
The stranger asked me where I _____ from.
A. come B. coming
C. came D. to come
4. Mark the letter A, B, C or D on your answer sheet to indicate the correct answer to each of the following questions.
She asked me, "How high is the house?"
A. She asked me how high the house was. B. She asked me how high the house had been.
C. She asked me how high the house is. D. She asked me how high is the house.

5. Mark the letter A, B, C or D on your answer sheet to indicate the correct answer to each of the following questions.
"I'll keep it as a secret" , Robert said.
~ Robert promised _____ it as a secret.
A. keeping B. to keep
C. kept D. keep
6. Mark the letter A, B, C or D on your answer sheet to indicate the correct answer to each of the following questions.
The mother asked her son ______
A. where had he been B. where he had been
C. where has he been D. where he has been

7. Mark the letter A, B, C or D on your answer sheet to indicate the correct answer to each of the following questions.
I asked Mary ______ to study abroad.
A. if she is planning B. was she planning
C. are you planning D. if she was planning
8. Mark the letter A, B, C or D on your answer sheet to indicate the correct answer to each of the following questions.
"Please don't tell anyone what happened" An said to me.
A. An said me not to tell anyone what happened. B. An told me didn't tell anyone what had happened.
C. An told me not to tell anyone what had happened. D. An said to me please don't tell anyone what happened.
9. Mark the letter A, B, C or D on your answer sheet to indicate the correct answer to each of the following questions.
Mr. David said, "I played badminton yesterday." ~ Mr. David said that he _____ badminton the day before.
A. had played B. was playing
C. played D. has played

10. Mark the letter A, B, C or D on your answer sheet to indicate the correct answer to each of the following questions.
"How beautiful the dress you have just bought is!" Peter said to Mary.
A. Peter complimented Mary on her beautiful dress. B. Peter said thanks to Mary for her beautiful dress.
C. Peter asked Mary how she had just bought her beautiful dress. D. Peter promised to buy Marya beautiful dress.
11. Mark the letter A, B, C or D on your answer sheet to indicate the correct answer to each of the following questions.
"Would you like to go to the cinema with me tonight?" he said to me.
A. He asked me if I'd like to go to the cinema with him tonight. B. He offered me to go to the cinema with him tonight.
C. He invited me to go to the cinema with him that night. D. He would like me to go to the cinema with him this night.
12. Mark the letter A, B, C or D on your answer sheet to indicate the correct answer to each of the following questions.
Mary said she _____ there the day before.
A. is B. had been
C. will be D. would be

13. Mark the letter A, B, C or D on your answer sheet to indicate the correct answer to each of the following questions.

Trang 1/3
99
Mai asked Quang _______.
A. when he would come back B. when he comes back
C. when he is coming back D. when he will come back

14. Mark the letter A, B, C or D on your answer sheet to indicate the correct answer to each of the following questions.
The teacher said, "I didn't see her." ~ The teacher said ______ her.
A. I hadn't seen B. he hadn't seen
C. he didn't see D. he had seen
15. Mark the letter A, B, C or D on your answer sheet to indicate the correct answer to each of the following questions.
Mary told me that she ______ fish the day before.
A. have not eaten B. did eat
C. would not eat D. had not eaten
16. Mark the letter A, B, C or D on your answer sheet to indicate the correct answer to each of the following questions.
"Can I take a friend to the party?" Hung asked me.
A. Hung wanted to ask someone to take his friend to the party. B. Hung knew that taking a friend to the party was good.
C. I wanted to invite his friend to the party. D. Hung asked me if he could take a friend to the party.
17. Mark the letter A, B, C or D on your answer sheet to indicate the correct answer to each of the following questions.
He asked _____ him some money.
A. She lends B. she has lent
C. her lending D. her to lend

18. Mark the letter A, B, C or D on your answer sheet to indicate the correct answer to each of the following questions.
Ms Nga wanted to know what time _____
A. the movie began B. did the movie begin
C. the movie begins D. does the movie begin

19. Mark the letter A, B, C or D on your answer sheet to indicate the correct answer to each of the following questions.
"I'd like you to tell me the truth," Linda said. ~ Linda wanted ______ her the truth.
A. me tell B. me to tell
C. me telling D. I to tell
20. Mark the letter A, B, C or D on your answer sheet to indicate the correct answer to each of the following questions.
"Remember to pick me up at 5 o'clock tomorrow afternoon." she said.
A. She reminded me to remember to pick her up at 5 o'clock the next B. She told me to pick her up at 5 o'clock the next day afternoon.
afternoon.
C. She told me to remember to pick her up at 5 o'clock tomorrow D. She reminded me to pick her up at 5 o'clock the following
afternoon. afternoon.
21. Mark the letter A, B, C or D on your answer sheet to indicate the correct answer to each of the following questions.
"I'm really sorry for being late again." said Nga
A. I felt sorry for Nga's being late again. B. Nga was sorry for my being late again.
C. Nga excused for my being late again. D. Nga apologized for being late again.
22. Mark the letter A, B, C or D on your answer sheet to indicate the correct answer to each of the following questions.
I wondered"- ______ the right thing.
A. am I doing B. if I am doing
C. whether I was doing D. was I doing

23. Mark the letter A, B, C or D on your answer sheet to indicate the correct answer to each of the following questions.
John often says he _____ eating fish.
A. had not like B. does not like
C. did not like D. not liked

24. Mark the letter A, B, C or D on your answer sheet to indicate the correct answer to each of the following questions.
"If I were you, I'd marry him", she said to me.
A. She will marry him if she is me. • B. She suggested to marry him if she were me.
C. She advised me to marry him. D. She said to me that if I were you, I'd marry him.
25. Mark the letter A, B, C or D on your answer sheet to indicate the correct answer to each of the following questions.
He asked me when _____ there.
A. I can arrive B. I had arrived
C. did I arrive D. will I arrive

26. Mark the letter A, B, C or D on your answer sheet to indicate the correct answer to each of the following questions.

Trang 2/3
100
She asked Peter ______ tea or coffee.
A. that he preferred B. whether he preferred
C. if he prefers D. did he prefer
27. Mark the letter A, B, C or D on your answer sheet to indicate the correct answer to each of the following questions.
"Please come and join my party tonight" Lan said to her friends
~ Lan invited her friends ______
A. to my party tonight B. to my party that night
C. to come and join my party that night D. to come and join her party that night
28. Mark the letter A, B, C or D on your answer sheet to indicate the correct answer to each of the following questions.
She told me _______ her up at six o'clock.
A. please pick B. should pick
C. I can pick D. to pick
29. Mark the letter A, B, C or D on your answer sheet to indicate the correct answer to each of the following questions.
Tom ______ that his mother was in hospital.
A. asked me B. said me
C. told to me D. told me

30. Mark the letter A, B, C or D on your answer sheet to indicate the correct answer to each of the following questions.
"You should stop smoking and eat more vegetable", said the doctor.
-> The doctor advised him ______ more vegetable.
A. stopping smoking and eating B. stop smoking and eat
C. to stop smoking and eat D. to stop smoking illld to eating

Trang 3/3
101
BÀI TẬP TỰ LUYỆN
BÀI. MỆNH ĐỀ QUAN HỆ
PEN-C TIẾNG ANH - CÔ HƯƠNG FIONA

1. Choose the best answers:

What’s the name of the man ______ car you borrowed?


A. that B. which
C. who D. whose
2. Choose the correct answer to each question

The artist, ____ painting you bought, is a classmate of mine


A. from whose B. from whom
C. whom D. whose
3. Mark the letter A, B, C or D on your answer sheet to indicate the correct answer to each of the following questions.

The Lake District, _____ was made a national park in 1951, attracts a large number of tourists every year.
A. that B. which
C. what D. where

4. Choose the best answers:

My computer, ______ mouse doesn't work, cannot be used now.


A. because B. which
C. whose D. while
5. Mark the letter A, B, C, or D on your answer sheet to indicate the correct answer to each of the following questions.

People ________ outlook on life is optimistic are usually happy people


A. whom B. who
C. that D. whose
6. Choose the correct answer to each question

Some of the boys _____ didn’t come


A. when I invited B. I invited
C. I invited them D. whom I invited them
7. Choose the best answers:

Taj Mahal, _____ by Shah Janhan for his wife, is thought to be one of the great architectural wonders of the world.
A. being built B. which was built
C. was built D. been built

8. Choose the best answers:

I have two sisters, __________ are doctors.


A. both of whom B. whom both
C. both of which D. who both
9. Mark the letter A, B, C or D on your answer sheet to indicate the correct answer to each of the following

Brian said goodnight to his roommate ________ continued to play video games until his eyes were blurry with fatigue
A. Justin, who B. Justin who
C. Justin, that D. Justin, whom

10. Mark the letter A, B, C or D on your answer sheet to indicate the correct answer to each of the following

We have many blessings for those _______ we are deeply grateful to


A. whom B. that
C. which D. whose
11. Choose the correct answer to each question

The dress _____ Mary bought doesn’t fit her well

Trang 1/7
102
A. none B. which
C. All are correct D. that
12. Choose the correct answer to each question

Those _____ to join our club must fill in this form first
A. who want B. if they want
C. if wanting D. when tried
13. Choose the correct answer to each question

The man _____ I wanted to see was away on holiday


A. none B. All are correct
C. whom D. who

14. Choose the correct answer to each question

Tom passed the driving test, _____ surprised everyone


A. who B. which
C. that D. what
15. Choose the correct answer to each question

Louis Pasteur, ______ discovered a cure for rabies, was a French scientist
A. that B. whom
C. who D. he

16. Choose the correct answer to each question

There was a small room into ____ we all crowded


A. where B. it
C. that D. which

17. Choose the correct answer to each question

The police have to try to catch the men ____ drive dangerously
A. they B. who
C. whom D. which
18. Choose the correct answer to each question

Near the White House is another famous landmark _____ the Washington Monument
A. is which B. called
C. it is called D. which call
19. Choose the correct answer to each question

The woman ____ to Mai is my sister


A. Both "Who spoke" and "speaking" are correct B. spoke
C. speaking D. who spoke
20. Choose the best answers:

She didn’t tell me the reason _______ she came late.


A. when B. for which
C. both B and C D. why
21. Choose the correct answer to each question

Tom, ____ I introduced to you, is a doctor


A. that B. whom
C. which D. who
22. Mark the letter A, B, C or D on your answer sheet to indicate the correct answer to each of the following

To calm his angry girlfriend, John offered an apology _____ Jane did not accept
A. which B. whom
C. when D. who
Trang 2/7
103
23. Choose the correct answer to each question

I enjoyed talking to the people _____ I had dinner last night


A. with who B. with whom
C. whom with D. who with
24. Choose the correct answer to each question

They always give the available seats to ____ comes first


A. whom B. whomever
C. whoever D. which
25. Choose the correct answer to each question

The record _____, _____ has been produced by a completely new method, will be released next week.

A. which I told you about - none B. about which I told you - none
C. I told you about - which D. that I told you - which
26. Choose the correct answer to each question

Mr. Richards, _____ is standing at the door, is my teacher


A. whom B. who
C. that D. none
27. Choose the correct answer to each question

A home computer _____ an opportunity for convenient and efficient work at home is becoming more and more common
A. is provided B. provides
C. which provides D. providing or "which provides"

28. Choose the correct answer to each question

I have two sister, _____ are very good at computer science


A. who B. both of whom
C. both of them D. whom
29. Choose the correct answer to each question

The man _____ at the door is my teacher


A. both of B and C are correct B. stands
C. who is standing D. standing
30. Mark the letter A, B, C, or D on your answer sheet to indicate the correct answer to each of the following questions.

The boy _____ went to the hospital to ask for doctor's help.
A. whose sister sicked B. who his sister is sick
C. whose sick sister D. whose sister was sick
31. Choose the best answers:

This is the best play _____ I have ever seen.


A. whom B. what
C. that D. which
32. Choose the best answers:

There are a lot of people at my friend's wedding party, only a few of ________I had met before.
A. who B. whose
C. whom D. which
33. Mark the letter A, B, C or D on your answer sheet to indicate the correct answer to each of the following questions.

An adviser to both Franklin Delano Roosevelt and Harry Truman, _____ of Bethune-Cook man College
A. Dr. Mary Mcleod Bethune was the founder B. the founder was Dr, Mary Mcleod Bethune
C. Dr. Mary Mcleod Bethune, who was the founder D. did the founder Dr, Mary Mcleod Bethune
34. Mark the letter A, B, C, or D on your answer sheet to indicate the correct answer to each of the following questions.

Trang 3/7
104
My friends gave me a surprising party, _________ was good of them
A. what B. that
C. this D. which
35. Choose the correct answer to each question

Mexico City, ____ is the capital of Mexico, is a cosmopolitan city


A. that B. where
C. which D. what
36. Mark the letter A, B, C, or D on your answer sheet to indicate the correct answer to each of the following questions.

The picture _______ , was beautiful


A. at that she was looking B. at it she was looking
C. she was looking D. at which she was looking

37. Choose the correct answer to each question

Mrs. Hoa, _____ son is studying at the University of Law, is a farmer


A. who B. whom
C. her D. whose
38. Mark the letter A, B, C, or D on your answer sheet to indicate the correct answer to each of the following questions.

We should participate in the movements _______ the natural environment


A. organized to conserve B. organized conserving
C. organizing to conserve D. which organize to conserve

39. Mark the letter A, B, C or D on your answer sheet to indicate the correct answer to each of the following questions.

This is Sen village _____ Uncle Ho was born


A. which B. who
C. what D. where
40. Choose the correct answer to each question

The restaurant ____ Bob recommended was too expensive


A. at which B. where
C. which it D. that

41. Mark the letter A, B, C or D on your answer sheet to indicate the correct answer to each of the following

The children _______ skateboard in the street are especially noisy in the early evening
A. which B. who
C. whom D. whose
42. Choose the correct answer to each question

We came within sight of Everest, _____ has attracted so many climbers


A. which the summit B. the summit of which
C. of which the summit D. whose summit of
43. Choose the correct answer to each question

He told her not to eat it, ____ that there was a risk of food poisoning
A. explained B. explaining
C. he explained D. to explain
44. Mark the letter A, B, C or D on your answer sheet to indicate the correct answer to each of the following questions.

Some major causes of ocean pollution include oil spills, toxic waste and hazardous materials _____ into the ocean.
A. that dumped B. which dump
C. dumped D. are dumped

45. Mark the letter A,B,C,orD on your answer sheet to indicate the correct answer to each of the following questions

Mary,_______ mother is a doctor, intends to choose medical career.

Trang 4/7
105
A. that B. whose
C. who D. whom

46. Mark the letter A, B, C, or D on your answer sheet to indicate the correct answer to each of the following questions.

The Lake District, _______ was made a national park in 1951, attracts a large number of tourists every year
A. what B. where
C. that D. which
47. Mark the letter A, B, C or D on your answer sheet to indicate the correct answer to each of the following questions.

I haven’t got a passport, ______ means I can’t leave my country.


A. which B. it
C. that D. this
48. Choose the correct answer to each question

My computer, _____ mouse don’t work, cannot be used now


A. which B. because
C. while D. whose
49. Mark the letter A, B, C or D on your answer sheet to indicate the correct answer to each of the following

Michelle screamed when she saw the spider ______ dangled from the one clean bathroom towel
A. where B. whose
C. whom D. that

50. Choose the correct answer to each question

They are the people _____ houses were destroyed by the flood
A. what B. which
C. their D. whose
51. Choose the correct answer to each question

The manager called in my new colleagues, _____ I had already met


A. one of who B. all of them
C. one or two of whom D. both of them
52. Choose the correct answer to each question

A skilled workforce is essential, _____ why our training course program is so important
A. that is B. this is
C. which is D. which
53. Choose the best answers:

All of us are waiting for the man _______ son was lost.

A. whom B. whose
C. which D. who
54. Mark the letter A, B, C, or D on your answer sheet to indicate the correct answer to each of the following questions.

My bike __________ I had left at the gate, had disappeared


A. which B. when
C. that D. X

55. Mark the letter A, B, C, or D on your answer sheet to indicate the correct answer to each of the following questions

Books and magazines _______ around made his room very untidy
A. laying B. which lied
C. that lie D. lying

56. Choose the correct answer to each question

The students ____ by the teacher are lazy.

Trang 5/7
106
A. who punished B. punished
C. who were punished D. Both "who were punished" and "punished" are correct
57. Choose the correct answer to each question

Jane did not fill her car up with petrol before she left, ____ was careless of her
A. what B. that
C. which D. it
58. Choose the correct answer to each question

My father, ____ is 60 years old, has just retired


A. who B. whom
C. whose D. that

59. Choose the correct answer to each question

She is going to sell the business, ____ is a great shame


A. what B. it is
C. which D. that
60. Choose the correct answer to each question

Paul is always throwing his dirty clothes on the floor, ____ makes his mother annoyed
A. which B. whom
C. who D. that

61. Choose the correct answer to each question

The Taj Mahal, _____ is recognized as one of the wonders of the world, was built by an Indian King in memory of his beloved wife
A. that B. which
C. what D. where

62. Choose the correct answer to each question

The preservation of ancient sites and historical buildings is a job ____ requires a person ready to fight a long battle.
A. of which B. what
C. which D. whose
63. Choose the correct answer to each question

I recieved two job offers, _____ I accepted


A. neither of that B. neither of which
C. which neither D. of which neither
64. Mark the letter A, B, C, or D on your answer sheet to indicate the correct answer to each of the following questions.

Simple sails were made from canvas___________over a frame.


A. stretch B. was stretched
C. it was stretched D. stretched
65. Mark the letter A, B, C, or D on your answer sheet to indicate the correct answer to each of the following questions.

Jack has two elder brothers, ______ are famous actors


A. both of them B. both of which
C. both of whom D. both of who
66. Mark the letter A, B, C or D on your answer sheet to indicate the correct answer to each of the following questions.

The girl __________ design had been chosen stepped to the platform to receive the award.
A. which B. that
C. whom D. whose
67. Mark the letter A, B, C or D on your answer sheet to indicate the correct answer to each of the following questions.

The United States consists of fifty states, ____ has its own government.
A. they each B. each of which
C. hence each D. each of that
Trang 6/7
107
68. Mark the letter A, B, C or D on your answer sheet to indicate the correct answer to each of the following questions.

They are excellent students ______ I have complete confidence.


A. in whom B. that
C. who D. whom

69. Choose the correct answer to each question

We went to different places _____ you find people _____ language was hard to understand
A. where - whose B. where - which
C. which - whose D. that - whose

70. Choose the correct answer to each question

The man ____ spoke to Sue is her boss


A. whom B. who
C. All are correct D. none
71. Choose the correct answer to each question

I have three brothers, _____ are professional athletes


A. two of who B. two of that
C. two of whom D. two of them
72. Mark the letter A, B, C, or D on your answer sheet to indicate the correct answer to each of the following questions.

The medicine _______ had no effect at all


A. which given to me by the doctor B. the doctor gave me
C. The doctor gave it to me D. which the doctor gave it to me
73. Choose the best answers:

The banker to________ I gave my check was quite friendly.


A. that B. who
C. which D. whom
74. Choose the correct answer to each question

Some of the guests_____ I invited didn’t come


A. which B. them
C. whom D. whose

Trang 7/7
108
BÀI TẬP TỰ LUYỆN
BÀI. MỆNH ĐỀ TRẠNG TỪ
PEN-C TIẾNG ANH - CÔ HƯƠNG FIONA

1. Choose the best answer.


Both English and Vietnamese use Roman scripts' but the latter is a tonal language.
A. Later both Vietnamese and English use Roman scripts. B. English has tones but Vietnamese does not, although they both use
Roman scripts.
C. Although English and Vietnamese are both Roman scripts, the D. Vietnamese and English which use Roman scripts later become
former is not a tonal language while the latter is. tonal languages.
2. Some economists argue that new technology causes unemployment. Others feel that it allows more jobs to be created.
A. Some economists argue that new technology causes B. Arguing that new technology causes unemployment, other
unemployment, so others feel that it allows more jobs to be created. economists feel that it allows more jobs to be created.
C. Besides the argument that new technology causes unemployment, D. Some economists argue that new technology causes
some economists feel that it allows more jobs to be created. unemployment, whereas others feel that it allows more jobs to be
created.
3. The scene is set in Normandy, but most of the characters in this novel are Londoners.
A. In the novel, the action moves backwards and forwards between B. The main characters in the novel are Londoners on a sightseeing
Normandy and London. holiday in Normandy.
C. In this novel, the story takes place in Normandy but the majority D. The story is about Normandy, but the leading characters are all
of the characters are from London. Londoners
4. He cannot lend me the book now. He has not finished reading it yet.
A. Having finished reading the book, he cannot lend it to me. B. He cannot lend me the book until he has finished reading it.
C. Not having finished reading the book, he will lend it to me. D. As long as he cannot finish reading the book, he will lend it to me.
5. I did not arrive in time. I was notable to see her off.
A. She had left because I was not on time. B. I did not go there, so I could not see her off.
C. I was not early enough to see her off. D. I arrived very late to say goodbye to her.

6. Diana ran into her former teacher on the way to the stadium yesterday.
A. Diana caused an accident to her teacher while she was going to B. Diana's car ran over her teacher on the way to the stadium.
the stadium.
C. Diana happened to meet her teacher while she was going to the D. Diana's teacher got run over whole she was going to the stadium.
stadium.
7. It isn't just that the level of education of this school is high. It's that it's also been
consistent for years.
A. The level of education in this school, which is usually quite high, B. The standard of education is not high in this school, but at least all
shows only slight variations from year to year. the students are at the same level.
C. Not only are the standards of education good in this school, but it D. It isn't fair to deny that this school is successful, as it has had the
has maintained those standards over the years. same high standards for many years now.
8. We chose to find a place for the night. We found the bad weather very inconvenient.
A. Bad weather was approaching, so we started to look for a place to B. The bad weather prevented us from driving any further.
stay.
C. Seeing that the bad weather had set in, we decided to find D. Because the climate was so severe, we were worried about what
somewhere to spend the night. we'd do at night.
9. _______ he got top marks at high school, he never went to university.
A. nevertheless B. despite
C. although D. meanwhile
10. Although he was very tired, he agreed to help his child with his homework.
A. Despite of his tiredness, he was eager to help his child with his B. Tired as he was, he agreed to help his child with his homework.
homework.
C. Even if feeling very tired, he agreed to help his child with his D. He would have helped his child with his homework if he hadn’t
homework. been tired.
11. Ancient Egyptians mummified the dead bodies through the use of chemicals, __________ ancient Peruvians did through naturalTrang
processes.
1/4
109
A. because B. whereas
C. whether or not D. even though
12. I'd like to help you out, I’m afraid I just haven't got any spare money at the moment.
A. Even B. Despite
C. Much as D. Try as
13. We airived at the airport. We realized our passports were still at home.
A. It was until we arrived at the airport that we realize our passports B. We arrived at the airport and realized that our passports are still at
were still at home. home.
C. Not until had we arrived at the airport, we realized our passports D. Not until we arrived at the airport, did we realize that our
were still at home. passports were still at home.
14. The hotel is not spacious. The hotel is not comfortable.
A. The hotel is neither spacious nor comfortable B. The hotel is neither spacious or comfortable
C. The hotel is both spacious and comfortable D. The hotel is not spacious but comfortable.
15. Most scientists know him well. However, very few ordinary people have heard of him.
A. He is the only scientist that is not known to the general public. B. Not only scientists but also the general public know him as a big
name.
C. Although he is well known to scientists, he is little known to the D. Many ordinary people know him better than most scientists do.
general public.

16. _______ we work with her, we get confused because of her fast speaking pace.
A. So that B. Although
C. C.Whenever D. Lest
17. I read the contract again and again ______ avoiding making spelling mistakes.
A. In terms of B. By means of
C. With a view to D. In view of
18. We chose to find a place for the night. We found the bad weather very inconvenient.
A. Seeing that the bad weather had set in, we decided to find B. The bad weather prevented us from driving any further.
somewhere to spend the night.
C. Bad weather was approaching, so we started to look for a place to D. Because the climate was so severe, we were worried about what
stay. we'd do at night.

19. _______ our prepaid order, they failed to send us the items in time.
A. Regardless of B. Without any notice of
C. With respect to D. On behalf of

20. While I strongly disapproved of your behavior, I will help you this time.
A. Despite of my strong disapproval of your behavior, I will help you B. Although I strongly disapproved of your behavior, but I will help
this time. you this time.
C. Because of your behavior, I will help you this time. D. Despite my strong disapproval of your behavior, I will help you
this time.
21. The basketball team knew they lost the match. They soon started to blame each other.
A. Hardly had the basketball team known they lost the match when B. Not only did the basketball team lose the match but they blamed
they started to blame each other. each other as well.
C. No sooner had the basketball team started to blame each other D. As soon as they blamed each other, the basketball team knew they
than they knew they lost the match. lost the match.
22. You can stay in the flat for free as long as you pay the bills.
A. Whether you pay the bills or stay in the flat, it is free. B. Without the bills paid, you can stay in the free flat.
C. Unless the flat is free of bills, you cannot stay in it. D. Provided you pay the bills, you can stay in the flat for free.
23. She raised her hand high so that she could attract the teacher’s attention.
A. Because her teacher attracted her, she raised her hand again. B. Though she raised her hand high, she couldn’t attract her teacher’s
attention.
C. To attract her teacher’s attention, she raised her hand high. D. She had such a high raising of hand that she failed to attract her
teacher’s attention.

Trang 2/4
110
24. Mary doesn’t like sports. Her brother doesn’t, either.
A. Neither Mary or her brother likes sports. B. either Mary or her brotehr loikes sports.
C. Neither Mary nor her brother likes sports. D. both Mary and her brother like sports.
25. People should not throw rubbish in the park. People should not cut down the trees in the park.
A. People should either throw rubbish in the park or cut down the B. People should neither throw rubbish nor cut down the trees in the
trees in the park. park.
C. People should either throw rubbish nor cut down the trees in the D. People should neither throw rubbish or cut down the trees in the
park. park

26. He managed to win the race__________hurting his foot before the race.
A. in spite of B. despite of
C. although D. because of

27. He felt tired. However, he was determined to continue to climb up the mountain.
A. Tired as he might feel, he was determined to continue to climb up B. He felt so tired that he was determined to continue to climb up the
the mountain mountain.
C. Feeling very tired, he was determined to continue to climb up the D. As a result of his tiredness, he was determined to continue to
mountain. climb up the mountain.
28. Mike graduated with a good degree. However, he joined the ranks of the unemployed.
A. Mike joined the ranks of the unemployed because he graduated B. If Mike graduated with a good degree, he would join the ranks of
with a good degree. the unemployed.
C. Although Mike graduated with a good degree, he joined the ranks D. That Mike graduated with a good degree helped him join the ranks
of the unemployed. of the unemployed.
29. Put your coat on. You will get cold.
A. You will not get cold unless you put your coat on. B. Put your coat on, otherwise you will get cold.
C. It is not until you put your coat on that you will get cold. D. You not only put your coat on but also get cold
30. She is intelligent ____ lazy.
A. and B. so
C. but D. neither
31. On the one hand, I’d love to study Japanese. On the other hand, I really haven’t got the time.
A. Nevertheless I would love to study Japanese, I really haven’t got B. I really haven’t got the time; as a result, I would love to study
the time. Japanese.
C. Because I haven’t got the time, I would love to study Japanese. D. I haven’t got the time; therefore I would not love to study
Japanese.

32. The student works part-time at a bid restaurant. He also paints pictures to earn for living.
A. Apart from works part-time at a bid restaurant, the student also B. In addition working part-time at a big restaurant, the student also
paints pictures to earn for living. paints pictures to earn for living.
C. Besides working part-time at a big restaurant, the student also D. The student works part-time at a big restaurant as well as he paints
paints pictures to earn for living. pictures to earn for living.
33. It doesn’t matter how much you offer to pay, he won’t sell the antique vase
A. Although you offer to pay him a lot, he won’t sell the antique B. Despite a lot of money you offer to pay, he won’t sell the antique
vase. vase.
C. He won’t sell the antique vase if you offer to pay him a lot of D. However much you offer to pay, he won’t sell the antique vase.
money.
34. Although he was able to do the job, he wasn’t given the position.
A. The position wasn’t given to him in spite of his ability to do the B. He was given neither the job nor the position.
job.
C. Because he couldn’t do the job, he wasn’t given the position. D. He got the position despite being unable to do the job.
35. We will leave for the airport ________ he is ready.
A. while B. during
C. until D. as soon as
36. We didn’t get home until it was almost dark.
A. It was almost dark, which prevented us from getting home. B. We decided to go home when it grew dark.
C. Since it was almost dark, we decided not to go home. Trang 3/4
111 D. It was almost dark when we got home.
37. Peter woke up early because he didn’t want to miss his flight.
A. Peter woke up early so that he wouldn’t miss his flight. B. If Peter had woken up early, he wouldn’t have missed his flight.
C. Waking up early, Peter didn’t miss his flight. D. As Peter woke up early, he was unlikely to miss his flight.
38. Her living conditions were difficult. However, she studied very well.
A. Although she lived in difficult conditions, but she studied very B. She studied very well thanks to the fact that she lived in difficult
well. conditions.
C. Difficult as her living conditions, she studied very well. D. She studied very well in spite of her difficult living conditions.
39. It was cold and wet. _________, Paul put on his swimming suit and went to the beach.
A. Although B. Therefore
C. However D. Because
40. Sue and Brian met. Shortly after that, he announced they were getting married.
A. As soon as Sue and Brian met, they announced they were getting B. Right at the time Brian met Sue, he announced they were getting
married. married.
C. Scarcely had Sue and Brian met when he announced they were D. Until Sue and Brian met, they had announced they were getting
getting married married

Trang 4/4
112
BÀI TẬP TỰ LUYỆN
BÀI. TRANSPORTATION
PEN-C TIẾNG ANH - CÔ HƯƠNG FIONA

1. Read the following passage and mark the letter A, B, C, or D into the blanks to indicate the correct answer to each of the questions
from 1 to 7.

Arid regions in the southwestern United States have become increasingly inviting playgrounds for the growing number of recreation
seekers who own vehicles such as motorcycles or powered trail bikes and indulge in hill-climbing contests or in carving new trails in the
desert. But recent scientific studies show that these off-road vehicles can cause damage to desert landscapes that has long-range effects on
the area’s water-conserving characteristics and on the entire ecology, both plant and animal. Research by scientists in the western Mojave
Desert in California revealed that the compaction of the sandy arid soil resulting from the passage of just one motorcycle markedly reduced
the infiltration ability of the soil and created a stream of rain runoff water that eroded the hillside surface. In addition, the researchers
discovered that the soil compaction caused by the off-road vehicles often killed native plant species and resulted in the invasion of different
plant species within a few years. The native perennial species required many more years before they showed signs of returning. The
scientists calculated that roughly a century would be required for the infiltration capacity of the Mojave soil to be restored after being
compacted by vehicles.

Question 1: What is the main topic of the passage?


A. Problems caused by recreational vehicles
B. Types of off-road vehicles
C. Plants of the southwestern desert
D. The increasing number of recreation seekers
Question 2: According to the passage, what is being damaged?
A. Motorcycles
B. The desert landscape
C. Roads through the desert
D. New plant species
Question 3: According to the passage, the damage to plants is
A. unnoticeable
B. superficial
C. long-lasting
D. irreparable
Question 4: According to the passage, what happens when the soil is compacted?
A. Little water seeps through
B. Better roads are made
C. Water is conserved
D. Deserts are expanded
Question 5: What is happening to the desert hillsides?
A. The topsoil is being eroded
B. The surface is being irrigated
C. There are fewer types of plants growing on them
D. There are fewer streams running through them
Question 6: According to the passage, what is happening to native plants in these areas?
A. They are becoming more compact
B. They are adapting
C. They are invading other areas
D. They are dying
Question 7: It can be inferred that which of the following people would probably be most alarmed by the scientists’ findings?

A. Historians
B. Mapmakers
C. Farmer
D. Ecologists
2. Read the following passage and mark the letter A, B, C or D to indicate the correct answer to each of the questions.

Although people drive in all countries, the rules can be quite different between nations and areas. For this reason, you should always learn
the laws before you decide to drive in a foreign country. If you are not careful, you can get into trouble since the rules might be very
different from what you are used to. For example, you can find some big differences in laws related to the minimum driving age, the
appropriate side of the road to drive on, and mobile phone use while driving.

In the U.S., people who live in Alaska may obtain a learner’s permit (legal permission to drive while being supervised) at the age of
fourteen. This is quite different from the driving laws of Niger, where a person must be at least twenty-three years old to drive. If you are
inquisitive to know the minimum driving age in most countries, the answer is eighteen. It may also surprise you to learn that not every
country allows its citizens to drive, although most of them do. For example, women are not allowed to drive in Saudi Arabia no matter how
old they are. Instead, they must have a male family member or hired male driver to travel by car.
Trang 1/3
113
Driving on the right or left side of the road also varies. For instance, in Great Britain, Cyprus, Australia, India, and Malaysia people drive
on the left. However, in the U.S., Mexico, France, and Canada people are required to drive on the right. As a matter of fact, one country
can have different driving rules for different areas. People in Hong Kong drive on the left, while drivers in other parts of China use the
right side of the road.

Other driving laws that are different between countries include those related to using mobile phones. In Japan, using any kind of mobile
phone device is illegal, even if you do not need to hold the phone with your hands. However, in Argentina and Australia, drivers are
allowed to talk on their mobile phones as long as they do not use their hands.

You might also find it interesting to learn that some countries have very unusual laws. For example, in the country of Cyprus, it is against
the law to eat or drink anything while driving. Even more interesting is that in Germany, there is a famous road called the Autobahn, where
certain parts have no speed limit at all!

Question 1: Which of the following is TRUE according to the passage?


A. Females can drive a car in Saudi Arabia.
B. People in India drive on the right side of the road.
C. You can drive as fast as you want on all parts of the German Autobahn.
D. You are not allowed to hold your mobile phone and talk while you drive in Australia.
Question 2: Which of the following is NOT mentioned as a main difference in driving laws in this passage?
A. Drink driving limit
B. Legal driving age
C. Right or left hand traffic
D. Safety calls
Question 3: According to the passage, it is illegal to eat or drink while driving in
A. Cyprus
B. Great Britain
C. China
D. Australia
Question 4: It is important to learn the laws before driving in a foreign country because
A. It is fun to learn about other countries’ driving laws.
B. You can get into trouble when you are not used to another country’s rules.
C. You can have a car accident if you do not know the rules.
D. It helps you identify the appropriate side of the road to drive on.
Question 5: The word “inquisitive” in paragraph 2 can be best replaced by
A. indifferent
B. concerned
C. nosy
D. curious
Question 6: The word “those” in paragraph 4 refers to
A. drivers
B. mobile phones
C. driving laws
D. countries
Question 7: What is the main idea of the passage?
A. Driving in all countries shares a common purpose.
B. There are differences in laws related to driving among countries.
C. Countries have different ways to enact laws on driving.
D. People in different countries drive on different sides of the road.
3. Read the following passage and mark the letter A, B, C or D into the blanks to indicate the correct answer to each of the following
questions

The National Automobile Show in New York has been one of the top auto shows in the United States since 1900. On November 3 of that
year, about 8,000 people looked over the “horseless carriages.” It was the opening day and the first opportunity for the automobile industry
to show off its wares to a large crowd; however, the black-tie audience treated the occasion more as a social affair than as a sales
extravaganza. It was also on the first day of this show that William McKinley became the first U.S. president to ride in a car.

The automobile was not invented in the United States. That distinction belongs to Germany. Nicolaus Otto built the first practical internal-
combustion engine there in 1876. Then, German engineer Karl Benz built what are regarded as the first modern automobiles in the mid-
1880s. But the United States pioneered the merchandising of the automobile. The auto show proved to be an effective means of getting the
public excited about automotive products.

By happenstance, the number of people at the first New York show equaled the entire car population of the United States at that time. In
1900, 10 million bicycles and an unknown number of horse-drawn carriages provided the prime means of personal transportation. Only
about 4,000 cars were assembled in the United States in 1900, and only a quarter of those were gasoline powered. The rest ran on steam or
electricity.

Trang 2/3
114
After viewing the cars made by forty car makers, the show’s audience favored electric cars because they were quiet. The risk of a boiler
explosion turned people away from steamers, and the gasoline-powered cars produced smelly fumes. The Duryea Motor Wagon Company,
which launched the American auto industry in 1895, offered a fragrant additive designed to mask the smells of the naphtha that it burned.
Many of the 1900 models were cumbersome—the Gasmobile, the Franklin, and the Orient, for example, steered with a tiller like a boat
instead of with a steering wheel. None of them was equipped with an automatic starter.

These early model cars were practically handmade and were not very dependable. They were basically toys of the well-to-do. In fact,
Woodrow Wilson, then a professor at Princeton University and later President of the United States, predicted that automobiles would cause
conflict between the wealthy and the poor. However, among the exhibitors at the 1900 show was a young engineer named Henry Ford. But
before the end of the decade, he would revolutionize the automobile industry with his Model T Ford. The Model T, first produced in 1909,
featured a standardized design and a streamlined method of production—the assembly line. Its lower costs made it available to the mass
market.

Cars at the 1900 show ranged in price from $1,000 to $1,500, or roughly $14,000 to $21,000 in today’s prices. By 1913, the Model T was
selling for less than $300, and soon the price would drop even further. “I will build cars for the multitudes,” Ford said, and he kept his
promise.

Question 1. The passage implies that the audience viewed the 1900 National Automobile Show primarily as a(n)

A. formal social occasion.


B. chance to buy automobiles at low prices.
C. opportunity to learn how to drive.
D. chance to invest in one of thirty-two automobile manufacturers.
Question 2. According to the passage, who developed the first modern car?
A. Karl Benz
B. Nikolaus Otto
C. William McKinley
D. Henry Ford
Question 3. Approximately how many cars were there in the United States in 1900?
A. 4,000
B. 8,000
C. 10 million
D. An unknown number
Question 4. The phrase “by happenstance” in paragraph 3 is closest in meaning to
A. Generally
B. For example
C. Coincidentally
D. By design
Question 5. The word “they” in the paragraph 2 refers to
A. car makers
B. model cars
C. audience
D. electric cars
Question 6. Approximately how many of the cars assembled in the year 1900 were gasoline powered?
A. 32
B. 1,000
C. 2,000
D. 4,000
Question 7. Which of the following is NOT mentioned in the passage as steering with a tiller rather than with a steering wheel?

A. A Franklin
B. A Gasmobile
C. An Orient
D. A Duryea
Question 8. What was the highest price asked for a car at the 1900 National Automobile Show in the dollars of that time?

A. $300
B. $1,500
C. $14,000
D. $21,000

Trang 3/3
115
BÀI TẬP TỰ LUYỆN
PREPOSITION
PEN-C TIẾNG ANH - CÔ HƯƠNG FIONA

1. The people want a President they can believe _______.


A. for B. x
C. at D. in
2. After the Second World War Germany was divided _______ two separate countries.
A. by B. for
C. between D. into
3. Let me explain ________ you about Jackie.
A. with B. to
C. for D. about
4. The length of time spent exercising depends ________ the sport you are training for.
A. of B. on
C. to D. about
5. When are you moving ______ Memphis?
A. to B. at
C. on D. in
6. We'll break ________ coffee at ten.
A. x B. off
C. out D. for

7. She dreamed _______ having her own business


A. for B. of
C. at D. about

8. Both children escaped unharmed _______ the burning building.


A. from B. in
C. for D. out of

9. The warring sides have agreed ______ an unconditional ceasefire.


A. to B. on
C. about D. with
10. We are a moral, ethical people and therefore we do not approve _____ their activities.
A. for B. with
C. at D. of
11. I don't think she's ever quite forgiven me _____ getting her name wrong that time.
A. to B. with
C. for D. by
12. He repeatedly asked Bailey __________ the report.
A. for B. of
C. by D. at
13. She congratulated him ______ the birth of his son.
A. on B. in
C. at D. for
14. I do my ironing while listening ____ the radio.
A. for B. to
C. in D. at
15. I tried calling him, but he'd already left ______ work.
A. to B. at
C. for D. in

16. When the kitchen is finished I'm going to focus my attention _______ the back yard and get that straightened out.

Trang 1/5
116
A. to B. at
C. on D. with
17. The President said the agreement would allow other vital problems to be dealt ________.
A. out B. with
C. for D. to
18. You should be paid_______ the work you do.
A. about B. on
C. for D. in
19. This was the first I'd heard _____ any trouble in the area.
A. to B. at
C. for D. of
20. The basement is filling _________ water.
A. for B. with
C. to D. about

21. I was waiting at the bus stop and I saw him get _______ a bus with a baby.
A. in B. to
C. up D. on
22. Would you include a Walkman ______ your list of essentials?
A. to B. with
C. On D. at
23. We decided to invest ______ a new car.
A. in B. for
C. to D. of
24. Choose the answer (A, B, C, D) which best fits the space

The professor has been accused ________ stealing his student's ideas and publishing them.
A. about B. with
C. of D. for
25. We will have discussions _____ employee representatives ________ possible redundancies.
A. to - of B. with – about
C. about - with D. for - to
26. What's happened ______ my pen? I put it down there a few moments ago.
A. to B. for
C. at D. of
27. It would be difficult not to involve the child's father ______ the arrangements.
A. for B. in
C. on D. to

28. She often complains _______ not feeling appreciated at work.


A. about B. for
C. at D. of
29. I don't know how they manage to live _____ £55 a week.
A. to B. on
C. at D. in
30. Do you believe ______ anything enough to die _______ it?
A. x – for B. in – for
C. in – of D. x – of
31. Local tradesmen are objecting ______ plans for big new out-of-town shopping centre.
A. in B. to
C. at D. on
32. The first word begins _________ an F.

Trang 2/5
117
A. by B. of
C. with D. for
33. She knocked ______ the window to attract his attention.
A. to B. for
C. on D. in
34. The animals died _____ starvation in the snow.
A. for B. of
C. at D. in

35. You can't take the laptop home because it belongs _____ the company.
A. with B. by
C. x D. to
36. The vase broken ______ pieces was carefully wrapped in paper.
A. out B. into
C. for D. by
37. He was dismissed ________ his position for being lazy.
A. in B. from
C. for D. of
38. My dad said he would help _______ the costs of buying a house.
A. for B. to
C. with D. at
39. Have you contributed any money _______ this charity?
A. to B. at
C. in D. for
40. If he persists_______ asking awkward questions, then send him to the boss.
A. in B. at
C. on D. for
41. Tim was a clumsy boy, always bumping ________ the furniture.
A. with B. for
C. at D. into
42. She has devoted all her energies ______ the care of homeless people.
A. on B. to
C. with D. for
43. He works in a sports centre instructing people _______ the use of the gym equipment.
A. in B. to
C. for D. of

44. When we arrived _______ London the first thing we all wanted to look at was Big Ben tower.
A. in B. of
C. at D. to
45. Water consists ______ hydrogen and oxygen.
A. with B. of
C. in D. on
46. We are aiming _______ a 50 percent share of the German market.
A. about B. for
C. by D. to
47. People differ _______ one another in their ability to handle stress.
A. in B. of
C. to D. from
48. The government is being widely criticized _____ the press _______ failing to limit air pollution.
A. on – for B. on – of
C. in – for D. for – of

Trang 3/5
118
49. As a writer, he combined wit ______ passion.
A. into B. of
C. with D. for
50. Everyone in the class is expected to participate actively ______ these discussions.
A. in B. for
C. about D. on

51. It was amazing how my mother coped ________ bringing up three children on less than three pounds a week.
A. at B. to
C. for D. with

52. We only need one more player for this game. Can you persuade your sister to join ______?
A. to B. on
C. at D. in
53. When you apologize _______ someone, you say that you are sorry that you have hurt them or caused trouble for them.
A. for B. with
C. about D. to
54. The government's campaign is aimed _______ influencing public opinion.
A. to B. at
C. of D. with
55. I don't want my children associating ______ drug addicts and alcoholics.
A. for B. with
C. to D. by
56. Who should we invite _____ the party?
A. to B. for
C. of D. in
57. The van collided ________ a lorry.
A. on B. with
C. at D. off
58. I couldn't concentrate _______ my work - my mind was _____ other things.
A. at – on B. at – at
C. on – on D. on – at

59. Her eyes slowly adjusted _____ the dark.


A. with B. of
C. from D. to

60. She's applied _______ a job with an insurance company.


A. for B. of
C. in D. as
61. Police blamed the bus driver ______ the accident.
A. for B. of
C. with D. by
62. The doctor had to break the news _______ Jane about her husband's cancer.
A. on B. to
C. for D. with
63. "Look at that!" she said, pointing _____ the hole in the door.
A. on B. at
C. in D. for
64. When you glance ______ something, you look at it very quickly and then look away.
A. at B. to
C. for D. of
65. The ill-fated aircraft later crashed _______ the hillside.

Trang 4/5
119
A. at B. into
C. on D. for

66. She confessed ______ her husband that she had sold her wedding ring.
A. for B. on
C. with D. to

67. She insisted ______ telling me every single detail of what they did to her in hospital.
A. on B. of
C. to D. in
68. What did you dream _______last night?
A. of B. about
C. at D. for
69. She found it hard to engage _______ office life.
A. in B. to
C. with D. on
70. The British don't laugh ______ the same jokes as the French.
A. in B. at
C. to D. as

Trang 5/5
120
BÀI TẬP TỰ LUYỆN
QUALIFIERS
PEN-C TIẾNG ANH - CÔ HƯƠNG FIONA

1.
There are not _____ in that classroom.
A. many students B. a great deal of students
C. any student D. a lot of student
2.
He went to the supermarket and bought a _____ of soap.
A. sheet B. bar
C. tree D. cup
3.
I am in trouble. Would you mind giving me _____ ?
A. advices B. some advice
C. several advices D. a few advice
4.
He wrote the answer on _____ and threw it to me.
A. a piece of paper B. a pieces of paper
C. piece of papers D. pieces of papers
5.
– (Đề ĐH khối D 2012). Before going to bed, he wanted some tea but there was _____ left.
A. a few B. no
C. nothing D. any

6.
– (Đề TN 2008). “Would you like to have _____ coffee?”
A. many B. some
C. little D. few

7.
It requires _____ reading.
A. an amount of B. a few
C. many D. few

8.
– (Đề minh họa THPTQG 2017). A recent survey has shown that _____ increasing number of men are willing to share the housework with
their wives.
A. the B. some
C. an D. a

9.
Look! There are _____ left over there.
A. much seats B. many seat
C. a few seats D. a little

10.
They demolished the old building and _____ are unloaded to build a new one.
A. a heap of brick B. heaps of bricks
C. heap of bricks D. heaps of brick

11.
He was not offered the job because he knew _____ about Information Technology.
A. a little B. little
C. a few D. many

12.
She keeps _____ in the cupboard.

Trang 1/3
121
A. all rice B. several rice
C. many rice D. a lot of rices
13.
My mother needs _____ milk to make a cake.
A. some B. any
C. many D. a
14.
_____ necessary.
A. All the information is B. A lot of the information are
C. Some informations are D. Many of the informations are

15.
Her parents gave her a _____ of pearls on her wedding day.
A. rope B. piece
C. dune D. train

16.
I think she was merely looking for _____.
A. happiness B. many happiness
C. a few happiness D. some happinesses

17.
– (Đề CĐ 2007). It‟s said that he has _____ friends of his age.
A. few B. little
C. a little D. plenty

18.
I do not have _____ to make ____.
A. a potatoes / the soups B. much potatoes / a soup
C. enough potatoes / the soup D. several potato / soups

19.
She teaches _____ and does not have much time for her own _____.
A. a couple of classes / researchs B. many classes / researches
C. a lot of classes / research D. a great deal of classes / research
20.
_____ used to be very expensive when they first appeared.
A. An amount of radios B. Radio
C. A radios D. Radios
21.
Mr. Pike has just bought three _____.
A. ox B. oxen
C. oxes D. oxens

22.
– (Đề CĐ 2012). Susan, remember to apply this sun cream _____ two hours.
A. every B. some
C. each D. several

23.
There is not _____ left in my account.
A. some moneys B. many money
C. a few money D. much money

24.
– (Đề CĐ 2009). Although the exam was difficult, _____ the students passed it.
A. most of B. none of
C. a lot D. a few

Trang 2/3
122
25.
I'm sorry, but the repair will take _____ time.
A. a few B. few
C. a bit of D. many
26.
Peter never feels lonely. He has got _____ friends.
A. so much B. a great deal of
C. much D. many
27.
John wanted to buy _____.
A. some teethbrushes B. a toothbrushes
C. some toothbrushes D. a teethbrush
28.
– (Đề ĐH khối D 2008). Increasing _____ of fruit in the diet may help to reduce the risk of heart disease.
A. an amount B. the amount
C. a number D. the number
29.
I often enjoy _____ after lunch.
A. a sheet of chocolate B. any chocolate
C. a bar of chocolate D. chocolates
30.
We need _____ money to send our son to Oxford University.
A. a few of B. lot of
C. many D. a large sum of
31.
It is going to rain. _____ of dark clouds are gathering in the sky.
A. herds B. masses
C. crowds D. flocks
32.
Mr. Pike lived with her three _____ in the countryside.
A. childrens B. children
C. childs D. child
33.
Mark the letter A, B, C, or D on your answer sheet to indicate the correct answer to each of the following questions

(Đề TN 2007 lần 2). There is _____ water in the bottle.


A. a number of B. a little
C. many D. few
34.
– (Đề ĐH khối D 2007). While southern California is densely populated, _____ live in the northern part of the state.
A. many people B. few people
C. a few of people D. a number people
35.
Intelligence is the ability to use thought and _____ to understand things and solve _____.
A. knowledges / problem B. knowledge / problems
C. knowledge / a problems D. knowledges / problems

Trang 3/3
123
BÀI TẬP TỰ LUYỆN
PEOPLE AND SOCIETY
PEN-C TIẾNG ANH - CÔ HƯƠNG FIONA

1. Read the following passage and type the letter A, B, C, or D into the blanks to indicate the answer to each of the following
questions.

It is hard to get any agreement on the precise meaning of the term "social class". In everyday life, people tend to have a different approach
to those they consider their equals from which they assume with people they consider higher or lower than themselves in social scale. The
criteria we use to 'place' a new acquaintance, however, are a complex mixture of factors. Dress, way of speaking, area of residence in a
given city or province, education and manners all play a part.
In ancient civilizations, the Sumerian, for example, which flourished in the lower Euphrates valley from 2000 to 5000 B.C. social
differences were based on birth, status or rank, rather than on wealth. Four main classes were recognized. These were the rulers, the
priestly administrators, the freemen (such as craftsmen, merchants or farmers) and the slaves.
In Greece, after the sixth-century B.C., there was a growing conflict between the peasants and the aristocrats, and a gradual decrease in the
power of the aristocracy when a kind of ‘middle class’ of traders and skilled workers grew up. The population of Athens, for example, was
divided into three main classes which were politically and legally distinct. About one-third of the total population was slaves, who did not
count politically at all, a fact often forgotten by those who praise Athens as the nursery of democracy. The next main group consisted of
resident foreigners, the, ‘metics’ who were freemen, though they too were allowed no share in political life. The third group was the
powerful body of ‘citizens”, who were themselves divided into sub-classes.
In the later Middle Ages, however, the development of a money economy and the growth of cities and trade led to the rise of another class,
the ‘burghers’ or city merchants and mayors. These were the predecessors of the modern middle classes. Gradually high office and
occupation assumed importance in determining social position, as it became more and more possible for a person born to one station in life
to move to another. This change affected the towns more than the country areas, where remnants of feudalism lasted much longer.

Question 1. According to the passage, we evaluate other people's social position by


A. questioning them in great details
B. their dress, manners, area of residence and other factors
C. finding out how much their salary is
D. the kind of job they do
Question 2. The word "criteria" in the first paragraph is closest in meaning to
A. characteristics
B. words
C. standards of judgment
D. criticisms
Question 3. The word “which” in the paragraph 2 refers to
A. ancient civilizations
B. Sumerian
C. example
D. Euphrates valley
Question 4. The decline of the Greek aristocracy's power in the sixth century B.C
A. caused international conflicts in the area
B. coincided with the rise of a new "middle class" of traders and peasants
C. was assisted by a rise in the number of slaves
D. lasted for only a short time
Question 5. Athens is often praised as the nursery of democracy
A. even though slaves were allowed to vote
B. because its three main classes were politically and legally distinct.
C. in spite of its heavy dependence on slave labor
D. because even very young children could vote
Question 6. The word "predecessors" in the last paragraph is closest in meaning to
A. supporters
B. descendants
C. ancestors
D. authorities
Question 7. The passage is mainly about
A. the human history
B. the modern society
C. the division of social classes in the ancient world
D. the social life in ancient Greece
2. Read the following passage and TYPE the letter A, B, C or D into the blanks to indicate the correct answer to each of the questions
from 1 to 8

Being aware of one's own emotions - recognizing and acknowledging feelings as they happen - is at the very heart of Emotional
Intelligence. And this awareness encompasses not only moods but also thoughts about those moods. People who are able to monitor their
feelings as they arise are less likely to be ruled by them and are thus better able to manage their emotions.
Trang 1/3
124
Managing emotions does not mean suppressing them; nor does it mean giving free rein to every feeling. Psychologist Daniel Goleman, one
of several authors who have popularized the notion of Emotional Intelligence, insisted that the goal is balance and that every feeling has
value and significance. As Goleman said, "A life without passion would be a dull wasteland of neutrality, cut off and isolated from the
richness of life itself." Thus, we manage our emotions by expressing them in an appropriate manner. Emotions can also be managed by
engaging in activities that cheer us up, soothe our hurts, or reassure us when we feel anxious.
Clearly, awareness and management of emotions are not independent. For instance, you might think that individuals who seem to
experience their feelings more intensely than others would be less able to manage them. However, a critical component of awareness of
emotions is the ability to assign meaning to them - to know why we are experiencing a particular feeling or mood. Psychologists have
found that, among individuals who experience intense emotions, individual differences in the ability to assign meaning to those feelings
predict differences in the ability to manage them. In other words, if two individuals are intensely angry, the one who is better able to
understand why he or she is angry will also be better able to manage the anger.
Self-motivation refers to strong emotional self-control, which enables a person to get moving and pursue worthy goals, persist at tasks even
when frustrated, and resist the temptation to act on impulse. Resisting impulsive behavior is, according to Goleman, "the root of all
emotional self-control."
Of all the attributes of Emotional Intelligence, the ability to postpone immediate gratification and to persist in working toward some greater
future gain is most closely related to success - whether one is trying to build a business, get a college degree, or even stay on a diet. One
researcher examined whether this trait can predict a child's success in school. The study showed that 4-year-old children who can delay
instant gratification in order to advance toward some future goal will be "far superior as students" when they graduate from high school
than will 4-year-olds who are not able to resist the impulse to satisfy their immediate wishes.

Question 1: Which of the following can we infer from paragraph 1?


A. Some people can understand their feelings better than others.
B. People who can manage their emotions will be controlled by them.
C. If people pay attention to their feelings, they will not be able to manage them.
D. If people pay attention to their feelings, they can control their emotions better.
Question 2: The word "soothe" in paragraph 2 could be best replaced by
A. reduce
B. weaken
C. worsen
D. relieve
Question 3: According to paragraphs 1 to 3, people should be aware of their emotions so that they can
A. manage their emotions appropriately
B. stop feeling angry
C. experience feelings more intensively
D. explain their emotions to others
Question 4: From paragraph 2, we can see that Daniel Goleman
A. trained people to increase their Emotional Intelligence
B. treated patients who had emotional problems
C. wrote about Emotional Intelligence
D. studied how people manage their emotions
Question 5: All of the following are mentioned in paragraph 2 about our emotions EXCEPT
A. every feeling is important
B. we can manage our emotions
C. emotions are part of a satisfying life
D. we should ignore some feelings
Question 6: The word "them" in paragraph 3 refers to
A. psychologists
B. individuals
C. individual differences
D. intense emotions
Question 7: In paragraph 3, the author explains the concept of awareness and management of emotions by
A. describing how people learn to control their emotions
B. giving an example of why people get angry
C. comparing how two people might respond to an intense emotion
D. explaining why some people are not aware of their emotions
Question 8: According to paragraph 5, children might be more successful in school if they can resist impulses because they can

A. easily understand new information


B. be more popular with their teachers
C. have more friends at school
D. focus on their work and not get distracted
3. Read the following passage and type the letter A, B, C, or D into the blanks to indicate the correct answer to each of the following
questions from 1 to 8.

Trang 2/3
125
According to sociologists, there are several different ways in which a person may become recognized as the leader of a social group in the
United States. In the family, traditional cultural patterns confer leadership on one or both of the parents. In other cases, such as friendship
groups, one or more persons may gradually emerge as leaders, although there is no formal process of selection. In larger groups, leaders are
usually chosen formally through election or recruitment.
Although leaders are often thought to be people with unusual personal ability, decades of research have failed to produce consistent
evidence that there is any category of “natural leaders.” It seems that there is no set of personal qualities that all leaders have in common;
rather, virtually any person may be recognized as a leader if the person has qualities that meet the needs of that particular group.
Furthermore, although it is commonly supposed that social groups have a single leader, research suggests that there are typically two
different leadership roles that are held by different individuals. Instrumental leadership is leadership that emphasizes the completion of
tasks by a social group. Group members look to instrumental leaders to “get things” done. Expressive leadership, on the other hand, is
leadership that emphasizes the collective well-being of a social group’s member. Expressive leader are less concerned with the overall
goals of the group than with providing emotional support to group members and attempting to minimize tension and conflict among them.
Group members expect expressive leaders to maintain stable relationships within the group and provide support to individual members.
Instrumental leaders are likely to have a rather secondary relationship to other group members. They give orders and may discipline group
members who inhibit attainment of the group’s goals. Expressive leaders cultivate a more personal or primary relationship to others in the
group. They offer sympathy when someone experiences difficulties or is subjected to discipline, are quick to lighten a serious moment with
humor, and try to resolve issues that threaten to divide the group. As the differences in these two roles suggest, expressive leaders generally
receive more personal affection from group members; instrumental leaders, if they are successful in promoting group goals, may enjoy a
more distant respect

Question 1:What does the passage mainly discuss?


A. How leadership differs in small and large groups
B. The role of leaders in social groups
C. The problems faced by leaders
D. How social groups determine who will lead them
Question 2:The passage mentions all of the following ways by which people can become leaders EXCEPT
A. recruitment
B. specific leadership training
C. traditional cultural patterns
D. formal election process
Question 3:Which of the following statements about leadership can be inferred from paragraph 2?
A. A person can best learn how to be an effective leader by studying research on leadership.
B. Most people desire to be leaders but can produce little evidence of their qualifications.
C. A person who is an effective leader of a particular group may not be an effective leader in another group.
D. Few people succeed in sharing a leadership role with another person.
Question 4:The passage indicates that instrumental leaders generally focus on
A. sharing responsibility with group members
B. achieving a goal
C. ensuring harmonious relationships
D. identifying new leaders
Question 5:The word “collective” in paragraph 3 is closest in meaning to
A. necessary
B. group
C. particular
D. typical
Question 6:It can be understood that
A. There is lots of tension and conflict in an election of a leader in the family.
B. There is usually an election to choose leaders in a family as well as in larger groups.
C. It has been said that there must be a set of personal qualities that all leaders have in common.
D. Leaders are sometimes chosen formally or informally.
Question 7:The word “resolve” in paragraph 4 is closest in meaning to
A. talk about
B. find a solution for
C. avoid repeating
D. avoid thinking about
Question 8. Paragraphs 3 and 4 organize the discussion of leadership primarily in terms of
A. examples that illustrate a problem
B. narration of events
C. comparison and contrast
D. cause and effect analysis

Trang 3/3
126
BÀI TẬP TỰ LUYỆN
INVERSION
PEN-C TIẾNG ANH - CÔ HƯƠNG FIONA

1. ________, Mozart was an accomplished composer while still a child.


A. Whatever it seems remarkable how B. No matter how does it seem remarkable
C. No matter how it seems remarkable D. No matter how remarkable it seems
2. As soon as I entered the room, I noticed her.
A. No sooner had I entered the room than I noticed her. B. Hardly had I entered the room when I noticed her.
C. No sooner I had entered the room than I noticed her. D. Both A and B are right
3. Scarcely had Jake ______ the book report when Alan came in.
A. finish B. finishing
C. finished D. to finished
4. _________ but he also proved himself a good athlete
A. Not only he showed himself a good student B. He did not show himself only a good student
C. Not only did he show himself a good student D. A good student not only showed him
5. Only when you grow up________ the truth.
A. You will know B. You know
C. Do you know D. Will you know
6. Hardly ever _______ far from home.
A. she traveled B. she has traveled
C. did she travel D. she did travel

7. Peter doesn’t like scuba-diving. __________ does his brother.


A. Too B. Neither
C. Either D. So

8. I have seldom heard such a talented singer.


A. Seldom have I heard such a talented singer. B. Seldom I have heard such a talented singer.
C. Seldom has I heard such a talented singer. D. All are correct.

9. Had they arrived at the fair earlier, they _______ what they wanted.
A. had found B. have found
C. found D. would have found
10. In no circumstances_________ on campus.
A. Should smoking be allowed B. Should allow smoking
C. Smoking should be allowed D. We should allow smoking
11. No sooner ______ the feast will start tomorrow.
A. will the bell ring than B. the bell will ring than
C. the bell will ring when D. will the bell ring when
12. Not until a monkey is several years old___________to exhibit signs of independence from his mother.
A. does it begin B. beginning
C. and begin D. it begins
13. Scarcely had he stopped out of the room_______ he heard a loud laughter within.
A. Then B. until
C. than D. when
14. Only by working hard _______ your final exam.
A. will you pass B. you will pass
C. did you pass D. you passed
15. _______ that she does not want to stay any longer.
A. A little homesick does Beth feel B. Beth feels such homesick
C. Homesick though Beth may feel D. So homesick does Beth feel
16. Hardly _______ the phone down when the boss rang back.

Trang 1/2
127
A. did I put B. have I put
C. had I put D. I had put
17. _______ was the tea that I couldn’t drink it.
A. No longer B. So strong
C. Hardly ever D. How strong
18. Only after you obtain a driving licence, _______ a car.
A. you are able to drive B. can you drive
C. you will be able to drive D. did you drive
19. ________ make a good impression on her.
A. Only by doing so can I B. Only by so doing I can
C. Only by doing so I can D. Only so doing can I
20. Choose the best answer to complete each of the following question

No sooner had we arrived at the station ______ the announcement started.


A. than B. when
C. then D. last
21. ______ he arrived at the bus stop when the bus came.
A. Hardly had B. No sooner had
C. No longer has D. Not until had
22. “__________ see a play with such a strong message”
A. I have rarely B. Rarely did I
C. I rarely D. Rarely do I
23. No sooner _______ the soldiers leaped into the water.
A. had the ship touched the shore than B. had the ship touched the shore when
C. the ship had touched the shore then D. the ship touched the shore than
24. _______ circumstances should you call the police.
A. In B. Under
C. Under no D. In any
25. Only after food has been dried and canned__________.
A. Should it be stored for later consumption B. It can be stored for later consumption
C. That it is stored for later consumption D. Was it stored for later consumption
26. ______________ that she burst into tears.
A. So angry was she B. Such her anger
C. She was so anger D. Her anger was so

27. On the island ________ the only representation of the Indian’s handicraft
A. does it main B. remains
C. did it remain D. remains it

28. ______ he arrived at the bus stop when the bus came.
A. No longer has B. No sooner had
C. Not until had D. Hardly had
29. We spent nearly 3 hours waiting outside the station, then out ____________.
A. the star came B. did the star come
C. came the star D. under
30. As soon as he approached the house, the policeman stopped him.
A. No sooner had he approached the house than the policeman B. Hardly had he approached the house than the policeman stopped
stopped him. him.
C. No sooner had he approached the house when the policeman D. Hardly he had approached the house when the policeman stopped
stopped him. him.

Trang 2/2
128
BÀI TẬP TỰ LUYỆN
OTHER STRUCTURES
PEN-C TIẾNG ANH - CÔ HƯƠNG FIONA

1. If you saw a lawyer, he'd advise you _______ legal action.


A. taking B. take
C. to take D. for taking
2. The lecturer recommended ______ a number of books before the exam.
A. to have read B. reading
C. to read D. us reading
3. I can make myself _________ pretty well in English.
A. to understand B. understand
C. understanding D. understood
4. I was delighted _________ my old friends again.
A. to be seen B. seeing
C. seen D. to see
5. Hardly _______ to the bus-stop when the bus suddenly pulled away.
A. they had got B. had they got
C. did they get D. they got
6. _________ to friends is nice but _________ alone is also enjoyable.
A. Talk/ to be B. Talking/ being
C. Talk/ be D. To talk/ be

7. We saw our favorite ballet _________ at the theater last night.


A. perform B. performed
C. performing D. in performing

8. Jane appears _______ some weight. Has she been ill?


A. having been lost B. to have lost
C. having lost D. to have been lost

9. He was never heard _________ 'thank you' in his life.


A. to say B. say
C. saying D. said
10. We are going to get our house _______ next week.
A. to paint B. painting
C. painted D. to be painted
11. Now that we've finished painting the house, there is nothing left _______.
A. for doing B. to be done
C. for being done D. to do
12. Do you know what made so many people _________ their home?
A. evacuated B. evacuate
C. be evacuated D. to evacuate
13. Twenty-five people _______ killed in that traffic accident.
A. are reported being B. are reporting to be
C. reported to be D. are reported to be
14. US scientists claim that they _______ a new vaccine against malaria.
A. developed B. had developed
C. have developed D. was developing
15. I think your mother should let you _________ your own mind.
A. to make up B. made up
C. make up D. making up

16. I can hear a cat _________ at the window.

Trang 1/2
129
A. scratching B. scratches
C. to scratch D. was scratching
17. The police never found the money _________ in the robbery.
A. stealing B. be stolen
C. steal D. stolen
18. _________ bread, you usually need flour, salt, and yeast.
A. Making B. For make
C. Make D. To make
19. It's possible _________ a train across Canada.
A. to be taken B. taking
C. to take D. take
20. I'd rather _________ to Elvis than the Beatles.
A. to listen B. listen
C. listened D. listening

21. By the time you return here next week, we _______ the project.
A. will have finished B. will finish
C. has finished D. will be finishing
22. He advised me _______ anything about it for the time being.
A. didn't do B. not do
C. not to do D. don't do
23. The woman said she _______ in this village for over fifty years.
A. was living B. had been living
C. lives D. has been living
24. Before we leave, let's have Shelley __ a map for us so we won't get lost.
A. draw B. to draw
C. drawing D. drawn
25. Choose the correct answers.
Instead of buying a new pair of shoes, I had my old ones _________
A. repaired B. to repair
C. repairing D. repair

Trang 2/2
130
BÀI TẬP TỰ LUYỆN
HOME LIFE
PEN-C TIẾNG ANH - CÔ HƯƠNG FIONA

1. I like doing _______such as cooking, washing and cleaning the house.


A. house-keeper B. lord of house
C. white house D. household chores
2. My brothers are often very _______to what I say. They are really lovable.
A. obedience B. obstacle
C. obedient D. obey
3. Whenever problems come up, we discuss frankly and find _______quickly.
A. solve B. solved
C. to solve D. solution
4. In my free-time, I often help mom with _______the house.
A. clean B. to clean
C. cleaning D. cleans
5. I take responsibility to _______my little brothers because they are sometimes very naughty.
A. get up B. shut up
C. look after D. sit down
6. My father is very helpful. He is always _______give a hand with cleaning the house.
A. willing B. will
C. willing to D. will to

7. In my family, both my parents _______to give us a nice house and a happy home.
A. join hands B. clutch hands
C. shake hands D. get out

8. My father is a _______He often does researches with animals and plants.


A. biologist B. botanist
C. mathematician D. chemist

9. Mai is my closest friend. We are always willing to _______feelings to each other.


A. cut B. shake
C. tear D. share
10. My responsibility is to _______my little brothers.
A. take over B. work together
C. take care of D. join hands
11. Choose the best answer
My mother is a _______woman. She does all the household chores to make us more comfortable.
A. careless B. caring
C. boring D. harmful

12. My brothers love joining my dad in _______things around the house at weekends.
A. mending B. mend
C. mended D. to mend
13. My responsibility is to wash dishes and _______the garbage.
A. take over B. take off
C. take care of D. take out
14. My mother works _______a nurse in a big hospital. She examines the patients.
A. as B. in
C. at D. for
15. My family is the base from which we can go into the world with _______.
A. confident B. confidence
C. confide D. confiding

Trang 1/1
131
BÀI TẬP TỰ LUYỆN
BÀI. HOBBIES
PEN-C TIẾNG ANH - CÔ HƯƠNG FIONA

1. Read the following passage and mark the letter A, B, C, or D on your answer sheet to indicate correct word for each of the blanks.

Why read books?

Is it worth reading books, (1) nowadays there are so many other forms of entertainment? Some people say that even
paperback books are expensive, and not everyone can borrow books from a library. They might add that television is more exciting and
that viewers can relax as they watch their favourite (2) . All that may be true but books are still very popular. They
encourage the reader to use his or her imagination for a start. You can read a chapter of a book, or just a few pages, and then stop. Of
course, it may be so (3) that you can't stop! There are many different kinds of books, so you can choose a crime
novel or an autobiography, or a book which gives you interesting (4) , If you find it hard to choose, you can read
reviews, or ask friends for ideas. Personally, I can't do without books, but I can (5) up television easily enough. You
can’t watch television at bus stops!

Question 1:
A. in
B. or
C. why
D. since
Question 2:
A. ones
B. programmes
C. episodes
D. cereals
Question 3:
A. current
B. imagined
C. interest
D. gripping
Question 4:
A. advise
B. idea
C. information
D. fact
Question 5:
A. pick
B. look
C. give
D. tum
2. Read the following passage and type the letter A, B, C, or D into the blanks to indicate the correct answer to each of the questions

Colors and Emotions

Colors are one of the most exciting experiences in life. I love them, and they are just as important to me as emotions are. Have you ever
wondered how the two are so intimately related?
Color directly affects your emotions. Color both reflects the current state of your emotions, and is something that you can use to improve
or change your emotions. The color that you choose to wear either reflects your current state of being or reflects the color or emotion that
you need.
The colors that you wear affect you much more than they reflect the people around you. Of course, they also affect anyone who comes in
contact with you, but you are the one saturated with the color all day! I even choose items around me based on their color. In the morning,
I choose my clothes based on the color or emotion that I need for the day
Color, sound, and emotions are all vibrations. Emotions are literally energy in motion; they are meant to move and flow. This is the reason
that real feelings are the fastest way to get your energy in motion. Also, flowing energy is exactly what creates healthy cells in your body.
So, the fastest way to be healthy is to be open to your real feelings. Alternately, the fastest way to create disease is to inhibit your emotions.

Question 1: What is the main idea of the passage?


A. Emotions and colors are closely related to each other.
B. Colors are one of the most exciting experiences in life.
C. Colorful clothes can change your mood.
D. Colors can help you become healthy.
Question 2: Who is more influenced by the colors you wear?
A. You are more influenced
B. The people around you are more influenced
C. both A and B
Trang 1/2
132
D. neither A nor B
Question 3: Which of the following can be affected by color?
A. your need for thrills
B. your friend’s feelings
C. your appetite
D. your mood
Question 4: According to the passage, what creates disease?
A. wearing the color black
B. being open to your emotions
C. ignoring your emotions
D. exposing yourself to bright colors
Question 5: The term intimately in paragraph 1 is closest in meaning to
A. clearly
B. obviously
C. closely
D. simply
Question 6: The phrase saturated with in paragraph 3 is closest meaning to
A. covered with
B. bored with
C. in need of
D. lacking in
Question 7: What is the purpose of the passage?
A. to give an objective account of how colors affect emotions.
B. to prove the relationship between color and emotion
C. To persuade the reader that colors can influence emotions and give a person more energy.
D. to show that colors are important for a healthy life.

Trang 2/2
133
BÀI TẬP TỰ LUYỆN
BÀI. PHRASAL VERBS (1)
PEN-C TIẾNG ANH - CÔ HƯƠNG FIONA

1.
Choose the answer (A, B, C, D) which best fits the space

What a gorgeous dress - why don't you _______ it ______ for size (= to discover whether it fits)?
A. try…for B. try…on
C. try…out D. try…out for
2.
Choose the answer (A, B, C, D) which best fits the space

Can you lend me a fiver? I'll ______ you/it _____ tomorrow


A. pay…off B. pay…down
C. pay…out D. pay…back
3.
Choose the answer (A, B, C, D) which best fits the space

After dividing these candies into 3 parts, they _________ them ________.
A. put…on B. put…off
C. put up with D. put…together
4.
Choose the answer (A, B, C, D) which best fits the space

He tricked me and I'm going to make him ________ it!


A. pay off B. pay for
C. pay out D. pay back
5.
Choose the answer (A, B, C, D) which best fits the space

He's so moody - I don't know why she _________ with him


A. puts off B. puts up with
C. puts on D. puts away
6.
Choose the answer (A, B, C, D) which best fits the space

She _________ the opportunity to work in Paris


A. turn against B. turn in
C. turn off D. turn down
7.
Choose the answer (A, B, C, D) which best fits the space

Dad's ________ the whole family ________ to the cinema


A. taking…on B. taking…out
C. taking…after D. taking…off
8.
Choose the answer (A, B, C, D) which best fits the space

Most of the machines have to be ________ to be cleaned.


A. taken away B. taken back
C. take in D. taken apart
9.
Choose the answer (A, B, C, D) which best fits the space

Emma opened her bag and ______ her comb ______.

Trang 1/4
134
A. took…out B. took…in
C. took…on D. took…after
10.
Choose the answer (A, B, C, D) which best fits the space

I have a radio but I seldom ______ it ______ except for the concerts
A. turn…off B. turn…in
C. turn…over D. turn…on
11.
Choose the answer (A, B, C, D) which best fits the space

If you don't want to catch a train home at that time of night, you're welcome to ________.
A. sleep in B. sleep over
C. sleep with D. sleep on
12.
Choose the answer (A, B, C, D) which best fits the space

She _________ her notes before giving the lecture


A. run over B. run across
C. run after D. run on
13.
Choose the answer (A, B, C, D) which best fits the space

He _________ his mother/ his mother's side of the family.


A. takes over B. takes after
C. takes up D. takes on
14.
Choose the answer (A, B, C, D) which best fits the space

Why did you have to _______ me _______ in front of everybody like that?
A. put…by B. put…down
C. put…off D. put…at

15.
Choose the answer (A, B, C, D) which best fits the space

_______ that knife _______ before you hurt somebody!


A. put…down B. put…out
C. put…away D. put…on

16.
Choose the answer (A, B, C, D) which best fits the space

The swans__________ the lake.


A. took with B. took on
C. took off D. took over

17.
Choose the answer (A, B, C, D) which best fits the space

They've ___________ the water while they repair a burst pipe


A. taken off B. thrown off
C. gone off D. turned off

18.
Choose the answer (A, B, C, D) which best fits the space

Time is _________ for the trapped miners.

Trang 2/4
135
A. running away B. running out
C. running up D. running across
19.
Choose the answer (A, B, C, D) which best fits the space

I had to _______ the shirt _______ because it didn't fit me.


A. send…back B. send off
C. send in D. send for
20.
Choose the answer (A, B, C, D) which best fits the space

She was __________ from dozens of applicants for the job


A. picked at B. picked on
C. picked up D. picked out
21.
Choose the answer (A, B, C, D) which best fits the space

Mark and my sister are planning to __________ together to get married


A. run after B. run away
C. run out D. run across
22.
Choose the answer (A, B, C, D) which best fits the space

________ any clothes you want to throw away and give them to me.
A. Sort out B. Get out
C. Look out D. Check out

23.
Choose the answer (A, B, C, D) which best fits the space

__________ the heat and stir in the soy sauce.


A. turn in B. turn off
C. turn on D. turn up

24.
Choose the answer (A, B, C, D) which best fits the space

He _________ at the rehearsal the next day looking awful


A. turned on B. turned down
C. turned up D. turned out

25.
Choose the answer (A, B, C, D) which best fits the space

I said she could come and live with us, and she said she'd ______ it ______
A. think…over B. think…through
C. think…about D. think…up

26.
Choose the answer (A, B, C, D) which best fits the space

We almost _________ a fox that was crossing the road


A. run into B. run out
C. run over D. run on

27.
Choose the answer (A, B, C, D) which best fits the space

Don't forget to ________ the equipment before setting up the experiment

Trang 3/4
136
A. come out B. go out
C. try out D. look out
28.
Choose the answer (A, B, C, D) which best fits the space

When I hear the word ‘football’ I _________ (= because I am not interested in it).
A. look into B. switch on
C. look for D. switch off
29.
Choose the answer (A, B, C, D) which best fits the space

They decided to ________ the wedding ________ until his brother had returned from Brazil.
A. put by B. put down
C. put off D. put about
30.
Choose the answer (A, B, C, D) which best fits the space

Graham ________ someone he used to know at school the other day


A. run after B. run away
C. run over D. run into

Trang 4/4
137
BÀI TẬP TỰ LUYỆN
BÀI. PHRASAL VERBS (2)
PEN-C TIẾNG ANH - CÔ HƯƠNG FIONA

1. Choose the answer (A, B, C, or D) which best fits the space.


Now and then he frowned, _______ something _______ and rewrote it
A. crossed…at B. crossed…over
C. crossed...out D. crossed…off
2. Choose the answer (A, B, C, D) which best fits the space

We only had a couple of hours to ________ before the exhibition opened


A. set up B. set down
C. set aside D. set about

3. Choose the answer (A, B, C, D) which best fits the space

They claimed that they weren't selling drugs, but that they'd been _________ by the police
A. took up B. went up
C. set up D. woken up
4. Choose the answer (A, B, C, D) which best fits the space

Would you mind ________ your cigarette ________, please?


A. putting…together B. putting…up
C. putting…out D. putting…on
5. Choose the answer (A, B, C, D) which best fits the space

The guide __________ various historic monuments


A. point up B. point off
C. pointed out D. point in

6. Mark the letter A, B, C or D on your answer sheet to indicate the correct answer to each of the following questions.

It is difficult to ______ identical twins _______


A. tell/ apart B. speak/ over
C. speak/ out D. tell/ on

7. Choose the answer (A, B, C, D) which best fits the space

He ________ the letter ________ and threw it away


A. tore…up B. tore…away
C. tore…along D. tore…off
8. Choose the answer (A, B, C, D) which best fits the space

When you're buying a flight, you should always __________ for the best deal
A. look out B. shop around
C. come over D. keep on
9. Choose the answer (A, B, C, D) which best fits the space

The hall was silent as the examination papers were __________.


A. passed out B. caught out
C. gone out D. made out

10. Choose the answer (A, B, C, or D) which best fits the space.
You should save time for yourself by _______ your shopping _______ to twice a week
A. cutting….in B. cutting….away
C. cutting….down D. cutting….off
11. Choose the answer (A, B, C, D) which best fits the space

They stripped off his clothes and ________ a scarlet robe ________ him

Trang 1/3
138
A. put…aside B. put…down
C. put…on D. put…off
12. Mark the letter A, B, C or D on your answer sheet to indicate the correct answer to each of the following questions.

My mother told me to ______ for an electrician when her fan was out of order
A. send B. write
C. rent D. turn
13. Choose the answer (A, B, C, D) which best fits the space

I don't need this table —you can ________ it ________.


A. throw…together B. throw…away
C. throw…aside D. throw…in

14. Mark the letter A, B, C or D on your answer sheet to indicate the correct answer to each of the following questions.

People are advised to ____ smoking because of its harm to their health.
A. cut down B. cut down on
C. cut off D. cut in
15. Choose the answer (A, B, C, D) which best fits the space

After the poisoning scare, the product was __________ the shelves
A. taken after B. taken over
C. taken from D. taken on

16. Choose the answer (A, B, C, or D) which best fits the space.
With the weather worsening, they’ve ______ the search for survivors.
A. called up B. called on
C. called off D. called in
17. Choose the answer (A, B, C, D) which best fits the space

He ran up the stairs and ____________ the light on the landing


A. switched on B. got on
C. took on D. went on
18. Choose the answer (A, B, C, D) which best fits the space

We'd better __________ the main road, because the other roads are blocked with snow
A. stick at B. stick to
C. stick around D. stick by
19. Choose the answer (A, B, C, D) which best fits the space

Are the shoes too small? _______ them _______ and get a refund
A. take in B. take off
C. take on D. take…back
20. Choose the answer (A, B, C, or D) which best fits the space.
Brown said he would _______ the issue _______ again at the next climate convention.
A. bring/back B. bring/off
C. bring/up D. bring/out
21. Mark the letter A, B, C or D on your answer sheet to indicate the correct answer to each of the following questions.

The train was _____ by a heavy snowfall.


A. held up B. held out
C. held back D. held off
22. Mark the letter A, B, C or D on your answer sheet to indicate the correct answer to each of the following questions.

He did not share his secrets with many people but he _______ in her.
A. consented B. confessed
C. revealed D. confided
23. Choose the answer (A, B, C, D) which best fits the space

Trang 2/3
139
I was hit on the head and ___________.
A. passed out B. pass round
C. passed on D. passed away
24. Choose the answer (A, B, C, D) which best fits the space

Andy's just had a bath and _________ all the hot water
A. got up B. used up
C. looked up D. kept up

25. Choose the answer (A, B, C, D) which best fits the space

I can't believe she ____________ the chance to go to South America


A. caught up B. passed up
C. looked up D. set up
26. Choose the answer (A, B, C, D) which best fits the space

It might help you to understand Elaine if you ________ to when you were her age
A. think about B. think of
C. think on D. think back

27. Choose the answer (A, B, C, D) which best fits the space

I keep lots of food that's quickly thawed and easily __________.


A. warmed down B. warmed up
C. warmed to D. warm on

28. Choose the answer (A, B, C, or D) which best fits the space.
Your father thought she was nice but Camille and I didn't_______her.
A. care for B. care about
C. care of D. care with
29. Choose the answer (A, B, C, D) which best fits the space

He _______ the lights ______ low to sleep.


A. looked down on B. sat down
C. came down with D. turned…down
30. Choose the answer (A, B, C, D) which best fits the space

He's just ___________because that girl he likes is here


A. showed off B. showed up
C. showed through D. showed around

Trang 3/3
140
BÀI TẬP TỰ LUYỆN
BÀI. PHRASAL VERBS (3)
PEN-C TIẾNG ANH - CÔ HƯƠNG FIONA

1. Choose the best answer.


George has come ______ a brilliant idea for losing weight.
A. across to B. through for
C. up with D. out on
2. Mark the letter A, B, C, or D on your answer sheet to indicate the correct answer to each of the following questions

I know we had an argument, but now I’d quite like to _______


A. make up B. fall out
C. bring up D. look down

3. Mark the letter A, B, C or D on your answer sheet to indicate the correct answer to each of the following question
We expected her at nine but she finally _____ at midnight.
A. came to B. turned out
C. turned up D. came off
4. Mark the letter A, B, C, or D on your answer sheet to indicate the correct answer to each of the following questions.

While I was looking through my old albums the other day, I _______ this photograph of my parents’ wedding.

A. took after B. turned down


C. made up D. came across
5. Mark the letter A, B, C, or D on your answer sheet to indicate the correct answer to each of the following questions.

The new manager laid down very strict rules as soon as he had ______ the position
A. taken up B. come over
C. taken off D. taken over

6. Mark the letter A, B, C, or D on your answer sheet to indicate the correct answer to each of the following questions

The new manager ______ very strict rules as soon as he had taken over the position
A. settled down B. wrote down
C. put down D. laid down

7. Mark the letter A, B, C or D on your answer sheet to indicate the correct answer to each of the following questions.

Finally, we______her life secrets and felt proud of her.


A. gave up B. found out
C. gave out D. found for
8. Choose the best answer.
It’s difficult to tell identical twins ______.
A. out B. over
C. on D. apart
9. Choose the best answer.
The film didn’t really ______ our expectations, unfortunately.
A. put in for B. meet with
C. fall short of D. come up to
10. Mark the letter A, B, C, or D on your answer sheet to indicate the correct answer to each of the following questions.

I can’t _______ this noise any longer. I’m going to write a letter of complaint to the local authority about this problem
A. get back to B. take away from
C. make out of D. put up with

11. Choose the best answer.


It’s high time Bill got a steady job and ______. He’s almost 30.
A. turned in B. settled in
C. turned down D. settled down

Trang 1/4
141
12. Choose the answer (A, B, C, or D) which best fits the space.
The writer doesn't ______ his opinions with examples.
A. back up B. back down
C. back onto D. back out
13. Mark the letter A, B, C, or D on your answer sheet to indicate the correct answer to each of the following questions.

The summer power outage shut the air conditioning ________ throughout the whole neighborhood.
A. in B. out
C. up D. down

14. Mark the letter A, B, C or D on your answer sheet to indicate the correct answer to each of the following questions.

The effect of the painkiller is ____ and I begin to feel the soreness again.
A. doing without B. turning out
C. fading away D. wearing off
15. Mark the letter A, B, C, or D on your answer sheet to indicate the correct answer to each of the following questions.

It’s rude to _______ people while you are talking to them

A. point out B. smile at


C. look at D. point at

16. Mark the letter A, B, C or D on your answer sheet to indicate the correct answer to each of the following questions.

I’m sure when you’ve stopped looking for your keys, they’ll _______ up somewhere.
A. take B. look
C. pull D. turn
17. Mark the letter A, B, C, or D on your answer sheet to indicate the correct answer to each of the following questions.

Whenever a problem _______ , we try to discuss frankly and find solutions as soon as possible
A. comes off B. comes in
C. comes up D. comes by
18. Mark the letter A, B, C or D on your answer sheet to indicate the correct answer to each of the following questions.

He spent the entire night thinking and in the end ____ a brilliant idea.
A. put up with B. put through to
C. came up with D. came up to

19. Mark the letter A, B, C, or D on your answer sheet to indicate the correct answer to each of the
following questions.

You have to move this box to _____ the new television set.
A. make room for B. pay attention to
C. lose touch with D. take notice of

20. Mark the letter A, B, C, or D on your answer sheet to indicate the correct answer to each of the following questions.

Susan was alone in the house when the fire ______


A. broke out B. broke away
C. broke down D. broke up

21. Mark the letter A, B, C, or D on your answer sheet to indicate the correct answer to each of the following questions.

We must push the piano to the corner of the hall to______our party tonight.
A. make place for B. give place to
C. take up room to D. make room for
22. Mark the letter A, B, C, or D on your answer sheet to indicate the correct answer to each of the following questions.

It is important to have someone you can ________ in.


A. talk B. know
C. confide D. speak

Trang 2/4
142
23. Mark the letter A,B,C,orD on your answer sheet to indicate the correct answer to each of the following questions

We expected him at eight but he finally ________ at midnight.


A. came to B. turned out
C. came off D. turned up
24. Mark the letter A, B, C or D on your answer sheet to indicate the correct answer to each of the following questions.

He was completely _____ by her tale of hardship.


A. taken away B. taken down
C. taken up D. taken in
25. Mark the letter A, B, C or D on your answer sheet to indicate the correct answer to each of the following questions.

It is very important for a film or a company to keep_________the changes in the market.


A. track about B. up with
C. pace of D. touch with

26. Mark the letter A, B, C or D on your answer sheet to indicate the correct answer to each of the following questions

I can’t _______ this noise any longer. I am going to write a letter of complaint to the local authority about this problem.
A. take away from B. get back to
C. put up with D. make out of
27. Mark the letter A, B, C, or D on your answer sheet to indicate the correct answer to each of the following questions.

Deforestation ______ devastative floods in many parts of the world


A. makes up B. results in
C. brings up D. starts out
28. Mark the letter A, B, C, or D on your answer sheet to indicate the correct answer to each of the following questions.

He does not ________ his fellow-workers and there are often disagreements between them.
A. put up with B. take to
C. get on with D. go on with
29. Mark the letter A, B, C or D on your answer sheet to indicate the correct answer to each of the following questions.

She thinks they ______ her because she doesn’t have a job.
A. look down on B. look at
C. look up to D. look back on

30. Mark the letter A, B, C or D on your answer sheet to indicate the correct answer to each of the following questions.

Nothing can ____________ the loss of the child.


A. make up for B. make up with
C. come up with D. do with
31. Mark the letter A, B, C or D on your answer sheet to indicate the correct answer to each of the following questions.

My responsibility is to wash dishes and__________ the garbage.


A. take care of B. take over
C. take out D. take off

32. Mark the letter A, B, C, or D on your answer sheet to indicate the correct answer to each of the following questions.

When Tet holiday comes, Vietnamese people often feel inclined to _____ their houses.
A. do through B. do over
C. do in D. do up
33. Mark the letter A, B, C or D on your answer sheet to indicate the correct answer to each of the following questions.

Once again poor Colin has been _______ for promotion


A. locked out B. struck off
C. stood by D. passed over

34. Mark the letter A, B, C, or D on your answer sheet to indicate the correct answer to each of the following questions.

Trang 3/4
143
She should have been here but she’s ________ flu.

A. gone thought with B. gone down with


C. come in for D. come against
35. Mark the letter A, B, C or D on your answer sheet to indicate the correct answer to each of the following questions.

Jack was wearing blue jeans and a torn T-shirt, while all other guests had on formal dinner wear. He really _______.
A. stayed out B. came out
C. look out D. stood out

Trang 4/4
144
BÀI TẬP TỰ LUYỆN
BÀI. IDIOMS (1)
PEN-C TIẾNG ANH - CÔ HƯƠNG FIONA

1. Choose the letter A, B, C, or D to indicate the correct answer to the following question.
It was a joke! I was pulling your ________.
A. toe B. hair
C. leg D. thumb
2. Mark the letter A, B, C, or D to indicate the correct answer to each of the following questions.

“What’s wrong with Tom today? He’s unusually quiet,” - “He’s got something on his ______. I expect”.
A. thoughts B. brain
C. mind D. brow

3. Mark the letter A, B, C, or D on your answer sheet to indicate the correct answer to each of the following questions

My English is progressing in ______.


A. leaps and bounds B. odds and ends
C. ends and odds D. bounds and leaps
4. Mark the letter A, B, C or D on your answer sheet to indicate the correct answer to each of the following questions.

According to a recent survey, most people are on good _______ with their neighbours
A. acquaintance B. terms
C. relations D. relationships
5. Mark the letter A, B, C or D on your answer sheet to indicate the correct answer to each of the following questions.

Nothing ever seems to bother Colin. No matter what happens, he always seems to remain as cool as______.
A. as Eskimo B. cool feet
C. a cucumber D. ice-cream

6. Mark the letter A, B, C, or D to indicate the correct answer to each of the following questions.

Before she left for Australia she promised her parents that she would drop them _______ at least once a month
A. a note B. a word
C. a line D. the news

7. Mark the letter A, B, C or D on your answer sheet to indicate the correct answer to each of the following questions.

I’ve never really enjoyed going to the ballet or the opera; they’re not really my_________.
A. piece of cake B. cup of tea
C. biscuit D. sweets and candy
8. Mark the letter A, B, C, or D to indicate the correct answer to each of the following questions.

He has a quick temper and easily _______ off the handle.


A. runs B. goes
C. flies D. leaps
9. Choose the letter A, B, C, or D to indicate the correct answer to the following question.
I usually buy my clothes________. It’s cheaper than going to a dress- maker.
A. off the peg B. on the shelf
C. on the house D. in public
10. Mark the letter A, B, C, or D to indicate the correct answer to each of the following questions.

I’ve never enjoyed going to the ballet or the opera; they’re not really my ______.
A. chip off the old block B. cup of tea
C. piece of cake D. biscuit
11. Mark the letter A, B, C, or D on your answer sheet to indicate the correct answer to each of the following questions.

If I were you, I would take more ______ in my work

Trang 1/3
145
A. pride B. solution
C. confidence D. dedication
12. Mark the letter A, B, C, or D on your answer sheet to indicate the correct answer to each of the following questions.

Nadine:“I’ve been offered $550 for my stereo. Should I take it or wait for a better one?”
Kitty: “Take the $550. ________”
A. The early bird catches the worm B. A bird in the hand is worth two in the bush
C. Kill two birds with one stone D. Actions speak louder than words
13. Mark the letter A, B, C, or D to indicate the correct answer to each of the following questions.

I always get _______ in my stomach before visiting the dentist


A. crabs B. worms
C. butterflies D. hedgehogs
14. Mark the letter A, B, C, or D to indicate the correct answer to each of the following questions.

“What I’ve got to say to you now is strictly ___ and most certainly not for publication.” said the government official to the reporter.
A. off the record B. beside the point
C. by the way D. for the time being

15. Mark the letter A, B, C, or D to indicate the correct answer to each of the following questions.

He was wearing very shabby, dirty clothes and looked very _______.
A. down to earth B. down at heel
C. out of shape D. easy- going
16. Mark the letter A, B, C or D on your answer sheet to indicate the correct answer to each of the following questions.

Taxis don't follow any schedule: they come and go ________.


A. chronologically B. in sequence
C. punctually D. at random
17. Choose the letter A, B, C, or D to indicate the correct answer to the following question.
I really must go and lie down for a while; I’ve got a ________ headache.
A. ringing B. splitting
C. cracking D. cutting

18. Mark the letter A, B, C, or D on your answer sheet to indicate the correct answer to each of the following questions.

No one will fly off the ________ for no reason


A. head B. handles
C. hands D. roof
19. Choose the letter A, B, C, or D to indicate the correct answer to the following question.
Stop _____ about the bush, John! Just tell me exactly what the problem is.
A. hiding B. beating
C. moving D. rushing

20. Mark the letter A, B, C, or D to indicate the correct answer to each of the following questions.

Since he started his own business he has been making money hand over ________.
A. fist B. palm
C. heel D. head
21. Choose the letter A, B, C, or D to indicate the correct answer to the following question.
I just took it ________ that he’d always be available.
A. for granted B. easy
C. into account D. into consideration

22. Mark the letter A, B, C or D on your answer sheet to indicate the correct answer to each of the following questions.

I am not able to go anywhere this weekend because I am up to my _______ in work.


A. head B. eyes
C. necks D. nose
23. Mark the letter A, B, C or D on your answer sheet to indicate the correct answer to each of the following questions.

Trang 2/3
146
The increased pay offer was accepted although it_______ short of what the employees wanted.
A. arrived B. ended
C. came D. fell
24. Choose the letter A, B, C, or D to indicate the correct answer to the following question.
I was glad when he said that his car was ________.
A. at my disposal B. for my use
C. at my use D. for me use
25. Mark the letter A, B, C or D on your answer sheet to indicate the correct answer to each of the following questions.

When you do something, you should _____.


A. go down well with B. weigh up the pros and cons
C. get through to D. turn over a new leaf
26. Mark the letter A, B, C, or D on your answer sheet to indicate the correct answer to each of the following questions

The opposition will be elected into government at the next election, without a _____ of a doubt
A. shade B. hue
C. shadow D. benefit

27. Mark the letter A, B, C or D on your answer sheet to indicate the correct answer to each of the following questions.

There’s a lot more to Willie than one would think: still waters run _____
A. deeply B. deepness
C. deep D. depth
28. Mark the letter A, B, C, or D to indicate the correct answer to each of the following questions.

I was all set to take the job in Tokyo, but at the last minute I _____ and decided to stay in Britain
A. held my horses B. got cold feet
C. called it a day D. pulled my finger out

29. Mark the letter A, B, C or D on your answer sheet to indicate the correct answer to each of the following questions.

Despite all the evidence, he wouldn’t admit that he was in the ____.
A. fault B. wrong
C. slip D. error
30. Choose the letter A, B, C, or D to indicate the correct answer to the following question.
I caught the last bus by the skin of my ________.
A. leg B. neck
C. mouth D. teeth

Trang 3/3
147
BÀI TẬP TỰ LUYỆN
BÀI. IDIOMS (2)
PEN-C TIẾNG ANH - CÔ HƯƠNG FIONA

1.
Mark the letter A, B, C, or D to indicate the correct answer to each of the following questions.

The escaped prisoner fought ______ before he was finally overpowered.


A. head over heels B. heart and soul
C. foot and mouth D. tooth and nail
2.
Mark the letter A, B, C, or D to indicate the correct answer to each of the following questions.

I didn’t suspect anything at first, but when I noticed her going through the office drawers I began to smell______.
A. a pig B. a rat
C. a thief D. a culprit
3.
Mark the letter A, B, C, or D to indicate the correct answer to each of the following questions.

Nagging Susan to stop smoking has no effect on her. It’s like water off ________.
A. an umbrella B. a dripping tap
C. a windmill D. a duck’s back
4.
Mark the letter A, B, C, or D to indicate the correct answer to each of the following questions.

Of course you won’t become more intelligent if you eat a lot of fish – that’s just an old _____tale.
A. wives’ B. mothers’
C. maids’ D. ladies’
5.
Mark the letter A, B, C, or D to indicate the correct answer to each of the following questions.

My father refused to eat meat that had been fired. He had _______ in his bonnet about it causing cancer.
A. a bee B. a bull
C. a bug D. an ant
6.
Mark the letter A, B, C, or D to indicate the correct answer to each of the following questions.

He had a soft _____ for his granddaughter and thoroughly spoilt her.
A. spot B. way
C. smile D. heart
7.
Mark the letter A, B, C, or D to indicate the correct answer to each of the following questions.

I just couldn’t remember her name even though it was on the ______ of my tongue.
A. front B. top
C. tip D. edge
8.
Mark the letter A, B, C, or D to indicate the correct answer to each of the following questions.

The car swerved to avoid a cyclist and just missed hitting a passer- by by______.
A. a narrow escape B. a close thing
C. a slight edge D. a hair’s breadth
9.
Mark the letter A, B, C, or D to indicate the correct answer to each of the following questions.

“If only I hadn’t lent him that money!” - “Well, you did, so it’s no good crying over ________ milk.

Trang 1/4
148
A. wasted B. goat’s
C. split D. sour
10.
Mark the letter A, B, C, or D to indicate the correct answer to each of the following questions.

I can’t see us beating them at tennis this year- we’re so out of ______.
A. fitness B. step
C. practice D. breath
11.
Mark the letter A, B, C, or D to indicate the correct answer to each of the following questions.

I like my new job, the only fly in the ______ is the fact that I have to work every other weekend.
A. porridge B. fat
C. ointment D. soup
12.
Mark the letter A, B, C, or D to indicate the correct answer to each of the following questions.

Have you seen the new boss? She’s the _____image of Marilyn Monroe
A. spitting B. true
C. live D. same
13.
Mark the letter A, B, C, or D to indicate the correct answer to each of the following questions.

I’m afraid you’ve caught me on the ______. I wasn’t expecting you until this afternoon
A. hop B. grapevine
C. spot D. stove
14.
Mark the letter A, B, C, or D to indicate the correct answer to each of the following questions.

That’s exactly what I mean, Tom. You’ve _______!


A. hit the nail on the head B. put your foot in it
C. put two and two together D. killed two birds with one stone

15.
Mark the letter A, B, C, or D to indicate the correct answer to each of the following questions.

I can’t stand Mr. Brian. He’s always blowing his own _______- telling everyone how good he is at everything
A. mind B. trumpet
C. breath D. balloon

16.
Mark the letter A, B, C, or D to indicate the correct answer to each of the following questions.

I haven’t had an accident yet but I’ve had a number of ______ shaves
A. narrow B. close
C. near D. tiny

17.
Mark the letter A, B, C, or D to indicate the correct answer to each of the following questions.

I’m not surprised that Tom is ill. He’s been _____ for a long time. It was bound to affect his health sooner or later.
A. burning the candle at both ends B. playing with fire
C. going to town D. having his cake and eating it

18.
Mark the letter A, B, C, or D to indicate the correct answer to each of the following questions.

We looked in every ______and cranny for the missing ring, but we couldn’t find it anywhere

Trang 2/4
149
A. Niche B. nook
C. hole D. gap
19.
Mark the letter A, B, C, or D to indicate the correct answer to each of the following questions.

“I’m going for an interview for a job this afternoon.” - “Good luck! I’ll keep my _____crossed for you”
A. legs B. hands
C. arms D. fingers
20.
Mark the letter A, B, C, or D to indicate the correct answer to each of the following questions.

He may be shy now, but he’ll soon come out of his ______ when he meets the right girl.
A. hole B. shoe
C. shed D. shell
21.
Mark the letter A, B, C, or D to indicate the correct answer to each of the following questions.

You know times have been bad lately, Peter, but keep your _______up; things are bound to get better soon.
A. socks B. head
C. chin D. mind
22.
Mark the letter A, B, C, or D to indicate the correct answer to each of the following questions.

Everyone bosses me about at work, I’m nothing but a __________.


A. marked man B. blue- eye boy
C. general dogsbody D. lame duck

23.
Mark the letter A, B, C, or D to indicate the correct answer to each of the following questions.

Writing rhymes for birthday cards is really easy. It’s money for old ________.
A. rope B. rubbish
C. rags D. bread

24.
Mark the letter A, B, C, or D to indicate the correct answer to each of the following questions.

I agree that this is a bit of a _____ shot, but we’re desperate- we have to do something to try to save the company.
A. high B. wild
C. long D. hot

25.
Mark the letter A, B, C, or D to indicate the correct answer to each of the following questions.

He just wanted one ______ before setting down and getting married
A. last leap B. final fling
C. happy hop D. joyful jump

26.
Mark the letter A, B, C, or D to indicate the correct answer to each of the following questions.

Those smart phones are selling like________. If you want one, you’d better buy one now before they’re all gone.
A. shooting stars B. fresh bread
C. wild oats D. hot cakes

27.
Mark the letter A, B, C, or D to indicate the correct answer to each of the following questions.

Peter was born and brought up in Hastings and knows it like the _____.

Trang 3/4
150
A. nose on his face B. back of his hand
C. hair on his head D. tip of the tongue
28.
Mark the letter A, B, C, or D to indicate the correct answer to each of the following questions.

My aunt is a bit of a wet _____. She’s always spoiling everyone’s fun


A. blanket B. rat
C. sheep D. rag
29.
Mark the letter A, B, C, or D to indicate the correct answer to each of the following questions.

Well, well, if it isn’t Kathy Lewis! You’re a sight for _______eyes!


A. old B. sore
C. blue D. crocodile
30.
Mark the letter A, B, C, or D to indicate the correct answer to each of the following questions.

It’s a __________shame that so little is done nowadays to help the homeless in our large cities.
A. crying B. dying
C. sweeping D. pitying

Trang 4/4
151
BÀI TẬP TỰ LUYỆN
BÀI. COLLOCATIONS
PEN-C TIẾNG ANH - CÔ HƯƠNG FIONA

1. Choose the best answer.


Tell me what happened ______ your own words.
A. with B. by
C. in D. on
2. Mark the letter A, B, C, or D on your answer sheet to indicate the correct answer to each of the following questions.

My boss is going on vacation next month, __________ she’ll be away from the office for about four weeks.
A. for instance B. in other words
C. on the other hand D. such as

3. Mark the letter A, B, C, or D on your answer sheet to indicate the correct answer to each of the following questions

They still haven’t make a decision _______ to the new color scheme
A. with regard B. on reflection
C. by contrast D. in view
4. Choose A, B, C, or D to indicate the correct answer to the following question.
He was met by a ____of journalists and photographers.
A. pack B. throng
C. crowd D. mob

5. Mark the letter A, B, C or D on your answer sheet to indicate the correct answer to each of the following questions.

Don’t touch that wire or you’ll get an electric__________.


A. shock B. fire
C. current D. charge
6. Mark the letter A, B, C or D on your answer sheet to indicate the correct answer to each of the following questions.

The car had a(n) __________ tire, so we had to change the wheel
A. injured B. flat
C. bent D. cracked

7. Mark the letter A, B, C, or D on your answer sheet to indicate the correct answer to each of the following questions.

She _______ me a very charming compliment on my painting


A. showed B. made
C. took D. paid
8. Mark the letter A, B, C or D on your answer sheet to indicate the correct answer to each of the following questions.

I shouldn’t have drunk so much coffee last night. I was ____ awake till four in the morning.
A. wide B. full
C. well D. deep

9. Mark the letter A, B, C or D on your answer sheet to indicate the correct answer to each of the following questions.

He drove fast and arrived an hour _______ of schedule.


A. ahead B. on account
C. in front D. in advance
10. Choose the best answer.
I went up to read Marian a story, but she was ______ asleep.
A. well B. deeply
C. fast D. full

11. Choose the best answer.


Tina never comes here now. We only see her once in a ______ moon.
A. blue B. white
C. gold D. yellow

Trang 1/3
152
12. Choose A, B, C, or D to indicate the correct answer to the following question.
When I was a child, I collected a/an ______ of poems.
A. family B. anthology
C. set D. pack
13. Mark the letter A, B, C or D on your answer sheet to indicate the correct answer to each of the following questions.

You should make a(n) __________ to overcome this problem


A. apology B. trial
C. impression D. effort

14. Choose the best answer.


When the visitors from Japan arrived, the company gave them the ______ carpet treatment.
A. orange B. green
C. blue D. red
15. Mark the letter A, B, C, or D on your answer sheet to indicate the correct answer to each of the following questions.

On Christmas Eve, the______family gathers for dinner, usually at my grandmother’s house.


A. complete B. total
C. entire D. mere

16. Mark the letter A, B, C or D on your answer sheet to indicate the correct answer to each of the following questions.

My sister supports her living by _______ of evening jobs.


A. means B. measures
C. ways D. methods
17. Mark the letter A, B, C or D on your answer sheet to indicate the correct answer to each of the following questions.

The scientists introduced new farming methods which resulted in ________ crops.
A. bumper B. large
C. bump D. bumpy
18. Mark the letter A, B, C or D on your answer sheet to indicate the correct answer to each of the following questions.

Doctors have accepted that acupuncture can work for pain ___________.
A. killing B. liberation
C. relief D. release
19. Choose A, B, C, or D to indicate the correct answer to the following question.
A ____of pickpockets usually gathered at the local market.
A. crowd B. throng
C. gang D. crew
20. Choose the best answer.
When Bill saw my new car, he was ______ with envy.
A. yellow B. blue
C. green D. white

21. Choose A, B, C, or D to indicate the correct answer to the following question.


Look! A ____ of destroyers are running away.
A. crowd B. gang
C. fleet D. mob

22. Choose A, B, C, or D to indicate the correct answer to the following question.


I have made a/an _____ of cookies for my husband.
A. batch B. pack
C. fleet D. anthology
23. Mark the letter A, B, C or D on your answer sheet to indicate the correct answer to each of the following questions.

Although he supports the Council, he does not take an active _____ in politics
A. part B. charge
C. affair D. play
24. Mark the letter A, B, C, or D on your answer sheet to indicate the correct answer to each of the following questions.

Trang 2/3
153
My mother ________ the responsibility for running the household
A. takes B. holds
C. bears D. runs

Trang 3/3
154
Hocmai.vn – Website học trực tuyến số 1 tại Việt Nam
Khóa học Luyện thi THPT quốc gia PEN C –N3 Môn Tiếng Anh cô Hương Fiona

PHƯƠNG PHÁP LÀM BÀI PHÁT ÂM


( BÀI TẬP TỰ LUYỆN )
Giáo viên : Nguyễn Thanh Hương
Các bài tập trong tài liệu này được biên soạn kèm theo bài giảng “Phương pháp làm bài phát âm ” thuộc Khóa học Luyện thi
THPT quốc gia PEN - C: Môn Tiếng Anh (cô Hương Fiona )” tại website Hocmai.vn để giúp các Bạn kiểm tra, củng cố lại các kiến
thức được giáo viên truyền đạt trong bài giảng tương ứng. Để sử dụng hiệu quả, Bạn cần học trước bài giảng, sau đó làm đầy đủ các
bài tập trong tài liệu này.

Exercise 1.Mark the letter A, B, C, or D on your answer sheet to indicate the word whose underlined
part differs from the other three in pronunciation in each of the following questions.
1. A. misses B. goes C. leaves D. potatoes
2. A. adopted B. appealed C. dedicated D. wounded
3. A. grows B. tends C. roars D. sweeps
4. A. university B. duty C. early D. apply
5. A. indicate B. average C. application D. grade
6. A. chemical B. approach C. achieve D. challenge
7. A. bride B. confide C. determine D. oblige
8. A. humour B. honest C. honour D. hour
9. A. future B. resume C. enthusiasm D. pressure
10. A. character B. chores C. technical D. psychology
11. A. breath B. threaten C. great D. healthy

Exercise 2.Mark the letter A, B, C, or D on your answer sheet to indicate the word whose underlined
part differs from the other three in pronunciation in each of the following questions.

1. A. gather B. there C. ethnic D. although


2. A. secure B. shuttle C. future D. contribute
3. A. close-knit B. compulsory C. campus D. certificate
4. A. ordinary B. pollution C. doctor D. alcohol
5. A. straight B. celebrate C. break D. pleasure
6. A. endangered B. threatened C. recovered D. advanced
7. A. elephants B. decades C. poachers D. actions
8. A. southern B. athlete C. healthy D. enthusiast
9. A. chemical B. character C. technical D. charming
10. A. caps B. posts C. players D. roofs

Hocmai – Ngôi trường chung của học trò Việt Tổng đài tư vấn: 1900 69 33 - Trang | 1 -

155
Hocmai.vn – Website học trực tuyến số 1 tại Việt Nam
Khóa học Luyện thi THPT quốc gia PEN C –N3 Môn Tiếng Anh cô Hương Fiona

Exercise 3 .Mark the letter A, B, C, or D on your answer sheet to indicate the word whose underlined
part differs from the other three in pronunciation in each of the following questions.
1. A. spirit B. describe C. title D. final
2. A. honor B. exhibit C. hour D. habitat
3. A. forged B. noticed C. struggled D. composed
4. A. damaged B. increased C. destroyed D. proposed
5. A. protected B. developed C. balanced D. established
6. A. carried B. organized C. impressed D. involved
7. A. earthquakes B. countries C. epidemics D. delegates
8. A. sufferings B. disasters C. species D. advantages
9. A. politics B. beliefs C. rights D. pioneers
10. A. laugh B. cough C. rough D. plough

Exercise 4 .Mark the letter A, B, C, or D on your answer sheet to indicate the word whose underlined
part differs from the other three in pronunciation in each of the following questions.
1. A. identify B. carry C. typical D. penalty
2. A. mutual B. initiate C. picture D. question
3. A. legal B. medal C. level D. effort
4. A. pool B. good C. look D. book
5 A. spread B. measure C. weapon D. increase
6. A. accompany B. fascinating C. discriminate D. scoreboard
7. A. panda B. swallow C. parrot D. advocate
8. A. wounded B. combined C. considered D. believed
9. A. supports B. colonies C. leaders D. victims
10. A. color B. note C. go D. opponent

Exercise 5. Mark the letter A, B, C, or D on your answer sheet to indicate the word whose underlined
part differs from the other three in pronunciation in each of the following questions.
1. A. breather B. feather C. heather D. leather
2. A. pursuit B. suitable C. recruit D. guilty
3. A. parade B. apathetic C. companion D. marmalade
4. A. research B. resent C. resemble D. resist
5. A. item B. idiom C. idle D. ideal
6. A. pretty B. get C. send D. well
7. A. children B. child C. line D. sign
8. A. off B. of C. safe D. knife
9. A. sacred B. decided C. cooked D. contaminated
10. A. come B. comfort C. some D. comb

Nguồn : Hocmai.vn
Giáo viên : Hương Fiona

Hocmai – Ngôi trường chung của học trò Việt Tổng đài tư vấn: 1900 69 33 - Trang | 2 -

156
Hocmai.vn – Website học trực tuyến số 1 tại Việt Nam
Khóa học Luyện thi THPT quốc gia PEN C –N3 Môn Tiếng Anh cô Hương Fiona

PHƯƠNG PHÁP LÀM BÀI TRỌNG ÂM


(BÀI TẬP TỰ LUYỆN )
Giáo viên : Nguyễn Thanh Hương
Các bài tập trong tài liệu này được biên soạn kèm theo bài giảng “Phương pháp làm bài trọng âm ” thuộc Khóa học Luyện thi THPT
quốc gia PEN - C: Môn Tiếng Anh (cô Hương Fiona )” tại website Hocmai.vn để giúp các Bạn kiểm tra, củng cố lại các kiến thức được
giáo viên truyền đạt trong bài giảng tương ứng. Để sử dụng hiệu quả, Bạn cần học trước bài giảng, sau đó làm đầy đủ các bài tập trong
tài liệu này.

Exercise 1. Mark A, B, C, or D on your answer sheet to indicate the word that differs from the other
three in the position of primary stress in each of the following questions.
1. A. biologist B. counterpart C. compliment D. kindergarten
2. A. applicant B. maximum C. category D. inflation
3. A. renovation B. communication C. intervention D. expedition
4. A. different B. bamboo C. rainfall D. wildlife
5. A. personality B. rhinoceros C. gorilla D. opponent
6. A. leftovers B. windsurfing C. enthusiast D. scoreboard
7. A. comfortable B. tsunami C. precision D. involvement
8. A. struggle B. rubbish C. nonsense D. development
9. A. nursery B. certificate C. curriculum D. intelligence
10. A. industrial B. interfere C. develop D. activity
Exercise 2. Mark A, B, C, or D on your answer sheet to indicate the word that differs from the other
three in the position of primary stress in each of the following questions.

1. A. incredible B. unexpected C. unnoticed D. outstanding


2. A. overwhelming B. intellectual C. interesting D. economic
3. A. inaccurate B. illegal C. positive D. domestic
4. A. confident B. eternal C. dangerous D. healthier
5. A. independent B. academic C. compulsory D. unexpected
6. A. secondary B. immediate C. miraculous D. domestic
7. A. simple B. polite C. formal D. instant
8. A. beautiful B. terrific C. wonderful D. marvelous
9. A. conical B. different C. symbolic D. careful
10. A. cultural B. mischievous C. interesting D. responsible

Hocmai– Ngôi trường chung của học trò Việt Tổng đài tư vấn: 1900 69 33 - Trang | 1-

157
Hocmai.vn – Website học trực tuyến số 1 tại Việt Nam
Khóa học Luyện thi THPT quốc gia PEN C –N3 Môn Tiếng Anh cô Hương Fiona

Exercise 3. Mark A, B, C, or D on your answer sheet to indicate the word that differs from the other
three in the position of primary stress in each of the following questions.
1. A. finance B. service C. order D. company
2. A. interview B. agency C. addition D. customer
3. A. prisoner B. diffficult C. exercise D. disaster
4. A. improvement B. occasion C. believe D. interest
5. A influential B. opportunity C. temperament D. expectation
6. A. picture B. number C. water D. advice
7. A. organize B. embarrass C. discourage D. surrounding
8. A. podium B. title C. notice D. pioneer
9. A. nature B. marriage C. value D. belief
10. A. police B. spirit C. banquet D. culture

Exercise 4. Mark A, B, C, or D on your answer sheet to indicate the word that differs from the other
three in the position of primary stress in each of the following questions.
1. A. determine B. maintain C. sacrifice D. apologize
2. A. inversion B. miserable C. pleasant D. difficulty
3. A. swallow B. survive C. digest D. finish
4. A. product B. satisfy C. pleasure D. however
5. A. happiness B. rewrite C. eject D. oblige
6. A. compare B. compose C. company D. consist
7. A. manage B. recognize C. argue D. discriminate
8. A. entertain B. understand C. recommend D. develop
9. A. offer B. listen C. precede D. follow
10. A. interview B. appropriate C. employ D. describe

Exercise 5. Mark A, B, C, or D on your answer sheet to indicate the word that differs from the other
three in the position of primary stress in each of the following questions.
1. A. support B. fascinate C. accept D. believe
2. A. inform B. appeal C. attempt D. hesitate
3. A. integrate B. defeat C. advise D. compete
4. A. engrave B. categorize C. memorialize D. discover
5. A. extremely B. necessary C. necessity D. co-operate
6. A. politics B. historic C. electric D. specific

Hocmai– Ngôi trường chung của học trò Việt Tổng đài tư vấn: 1900 69 33 - Trang | 2-

158
Hocmai.vn – Website học trực tuyến số 1 tại Việt Nam
Khóa học Luyện thi THPT quốc gia PEN C –N3 Môn Tiếng Anh cô Hương Fiona

7. A. committee B. referee C. employee D. refugee


8. A. stimulate B. subtract C. announce D. maintain
9. A. determine B. appearance C. develop D. difference
10. A. romantic B. following C. summary D. physical

Nguồn : Hocmai.vn
Giáo viên : Hương Fiona

Hocmai– Ngôi trường chung của học trò Việt Tổng đài tư vấn: 1900 69 33 - Trang | 3-

159
Hocmai.vn – Website học trực tuyến số 1 tại Việt Nam
Khóa học Luyện thi THPT quốc gia PEN C –N3 Môn Tiếng Anh cô Hương Fiona

PHƯƠNG PHÁP LÀM DẠNG BÀI TÌM LỖI SAI


(BÀI TẬP TỰ LUYỆN )
Giáo viên : Nguyễn Thanh Hương
Các bài tập trong tài liệu này được biên soạn kèm theo bài giảng “Phương pháp làm dạng bài tìm lỗi sai” thuộc Khóa học Luyện thi
THPT quốc gia PEN - C: Môn Tiếng Anh (cô Hương Fiona )” tại website Hocmai.vn để giúp các Bạn kiểm tra, củng cố lại các kiến thức
được giáo viên truyền đạt trong bài giảng tương ứng. Để sử dụng hiệu quả, Bạn cần học trước bài giảng, sau đó làm đầy đủ các bài tập
trong tài liệu này.

I. Mark the letter A, B, C or D on your answer sheet to show the underlined part that needs correction
Question 1: There will always be a job for Mike if he change his mind.
A B C D
Question 2:Language is important factor in the accumulation of culture.
A B C D
Question 3: That cats have nine lives have been believed for centuries
A B C D
Question 4: Despite the disappearance of the Mayan empire , there are still Mayans in the region
A B C
that they once inhabited
D
Question 5: After writing it , the essay must be duplicated by the student himself and handed into the
A B C
department secretary before the end of the month .
D
II. Mark the letter A, B C or D on your answer sheet to indicate the underlined part that needs correction
in each of the following questions .
Question 1: Men and women in the Peace Corps work with people in the developing countries
A B
to help them improving their living conditions.
C D
Question 2: Although not widely sold, that book is considered to be best book on the subject.
A B C D
Question 3: Because his sickness he didn’t take part in the English competition held last Sunday.
A B C D
Question 4: I found my new contact lenses strangely at first, but I got used to them in the end.

Hocmai– Ngôi trường chung của học trò Việt Tổng đài tư vấn: 1900 69 33 - Trang | 1-

160
Hocmai.vn – Website học trực tuyến số 1 tại Việt Nam
Khóa học Luyện thi THPT quốc gia PEN C –N3 Môn Tiếng Anh cô Hương Fiona

A B C D
Question 5: Dreaming, like all other mental processes, it is a product of the brain and its activity.
A B C D
III. Mark the letter A, B, C, or D on your answer sheet to show the underlined part that needs
correction.
Question 1: My father asked me where had I gone the night before.
A B C D
Question 2: The harder he tried, the worst he danced before the large audience.
A B C D
Question 3: Students suppose to read all the questions carefully and find out the answers to
A B C
them.
D
Question 4: Lake Superior, that lies on the US Canadian border, is the largest lake in North America.
A B C D
Question 5: Public health experts say that the money one spends avoiding illness is less than the cost
A B C
of to treat sickness.
D
IV. Mark the letter A, B, C, or D on your answer sheet to show the underlined part that needs
correction.
Question 1. One of the students who are being considered for the scholarship are from this university.
A B C D
Question 2. More than 600 million individual bacteria lives on the skin of humans.
A B C D
Question 3. I'm very glad that you've done lots of progress this semester.
A B C D
Question 4. Not only did he broke two glasses, but also he left the table dirty.
A B C D
Question 5. Alike all other mammals, dolphins have lungs.
A B C D

Hocmai– Ngôi trường chung của học trò Việt Tổng đài tư vấn: 1900 69 33 - Trang | 2-

161
Hocmai.vn – Website học trực tuyến số 1 tại Việt Nam
Khóa học Luyện thi THPT quốc gia PEN C –N3 Môn Tiếng Anh cô Hương Fiona

V. Mark the letter A, B, C, or D on your answer sheet to show the underlined part that needs correction.
Question 1. Wage rates depend in part from the general prosperity of the economy.
A B C D
Question 2. If either of you take a vacation, we will not be able to finish the work.
A B C D
Question 3. Several people have apparent tried to change the man's mind, but he refuses to listen.
A B C D
Question His father told him to apologise their neighbour for being rude.
A B C D
Question 5. The physicians appeared nervously when he talked to the patient.
A B C D
VI. Mark the letter A, B, C, or D on your answer sheet to show the underlined part that needs
correction.
Question 1. Many theory on how the Earth began its existence have been proposed.
A B C D
Question 2. The task of the cartographer is to represent the Earth's surface at a reduced greatly scale.
A B C D

Question 3. They couldn't hear the music down there because it wasn't too loud.
A B C D
Question 4. The headmaster, together with the teaching staff, are away on holiday.
A B C D
Question 5. Automobiles begun to be equipped with built-in radios around 1930.
A B C D
VII. Mark the letter A, B, C, or D on your answer sheet to show the underlined part that needs
correction.
Question 1. Although a number of police officers was guarding the priceless treasures in the
A B
museum, the director worried that someone would try to steal them.
C D
Question 2 . One of the primary cause of road accidents is driving after drinking.
A B C D
Question 3. Insurance rates are not the same for different people because they are not likely
A B

Hocmai– Ngôi trường chung của học trò Việt Tổng đài tư vấn: 1900 69 33 - Trang | 3-

162
Hocmai.vn – Website học trực tuyến số 1 tại Việt Nam
Khóa học Luyện thi THPT quốc gia PEN C –N3 Môn Tiếng Anh cô Hương Fiona

have the same risk.


C D
Question 4. It is vitally important that she takes this medication night and morning.
A. B. C. D.
Question 5. We had better to review this chapter carefully because we will have some questions on it on
A B C D
our test tomorrow.

VIII. Mark the letter A, B, C, or D on your answer sheet to show the underlined part that needs
correction.
Question 1. American painter Georgia O’Keeffe is well known (A) as her large paintings of flowers (B) in
which single (C) blossoms are presented as if in (D) close-up.
Question 2. Although this car (A) appears (B) to be manufactured by a (C) different company, it has the
same body style, size, and (D) perform as that one.
Question 3. Farm (A) animals have been (B) regardless by nearly all societies as a (C) valuable economic
(D) resource.
Question 4. (A) Although her (B) severe pain, she tried (C) to walk to the auditorium (D) to attend the
meeting.
Question 5. (A) In my opinion, (B) our world view is quite different from (C) the writers in (D) the fourth
century BC.
IX.Mark the letter A, B, C, or D on your answer sheet to show the underlined part that needs correction.
Question 1. (A) The electric (B) toaster was one of the earliest (C) appliance to be developed for the (D)
kitchen.
Question 2. The ability of (A) speech is one of the (B) skills that make (C) humans difference from the
rest of (D) the animal world.
Question 3. Each of (A) the students (B) in the accounting class (C) has to type their own research paper
(D) this semester.
Question 4. In the United States (A) among 60 percent (B) of the space on the pages of newspapers (C) is
reserved for (D) advertising.
Question 5. Daisy has (A) such many things (B) to do that she (C) has no time (D) to go out.
X.Mark the letter A, B, C, or D on your answer sheet to show the underlined part that needs correction.
Choose A, B, C, or D to show the underlined part that needs correction.

Hocmai– Ngôi trường chung của học trò Việt Tổng đài tư vấn: 1900 69 33 - Trang | 4-

163
Hocmai.vn – Website học trực tuyến số 1 tại Việt Nam
Khóa học Luyện thi THPT quốc gia PEN C –N3 Môn Tiếng Anh cô Hương Fiona

Question 1. Objects (A) falling freely in a (B) vacuum have the same rate of (C) speed is regardless of
differences in (D) size and weight.
Question 2. One of the (A) majority causes of (B) tides is the (C) gravitational (D) attraction of the
Moon.
Question 3. The oldest and (A) most widespread celebrations are (B) that connected with the (C)
harvesting of the (D) first fruits.
Question 4. From bones (A) finding in the United States, we (B) have learned that many animals which
(C) no longer existed in the world once (D) made their homes there.
Question 5. (A) All of my students are longing (B) for having a nice summer (C) holiday (D) at the
seaside.

Nguồn : Hocmai.vn
Giáo viên : Hương Fiona

Hocmai– Ngôi trường chung của học trò Việt Tổng đài tư vấn: 1900 69 33 - Trang | 5-

164
Hocmai.vn – Website học trực tuyến số 1 tại Việt Nam
Khóa học Luyện thi THPT quốc gia PEN C –N3 Môn Tiếng Anh cô Hương Fiona

VOCABULARY – CULTURE AND LANGUAGE


( TÀI LIỆU BÀI GIẢNG)
Giáo viên : Nguyễn Thanh Hương
Các bài tập trong tài liệu này được biên soạn kèm theo bài giảng “Culture and language” thuộc Khóa học Luyện thi THPT quốc
gia PEN - C: Môn Tiếng Anh (cô Hương Fiona )” tại website Hocmai.vn để giúp các Bạn kiểm tra, củng cố lại các kiến thức được
giáo viên truyền đạt trong bài giảng tương ứng. Để sử dụng hiệu quả, Bạn cần học trước bài giảng, sau đó làm đầy đủ các bài tập
trong tài liệu này.

EXERCISE 1. Read the following passage and mark the letter A, B, C or D on your answer sheet to
indicate the correct word for each of the blanks.
THE HISTORY OF WRITING
The development of writing (1) ________ a huge difference to the world and might see it as the beginning
of the media. Pieces of pottery with marks on that are probably numbers have been discovered in China
that date from around 4000 BC. Hieroglyphics and other forms of "picture writing" developed in the (2)
________ around Mesopotamia (mordem-day Iraq), where the ancent Sumerian civilization was based,
from around 3300 BC onwards. However, the first (3) ________ alphabet was used by the Phoenicians
around 1050BC. Their alphabet had 22 letters and it is estimated that it lasted for 1000 years. The first two
signs were called "aleph" and "beth", which in Greek became "alpha" and "beta"* which gave us the
modem word "alphabet" The modem European alphabet is based on the Greek and (4) to other
European countries under the
Romans. A number of changes took place as time passed. The Romans added the letter G, and the letter J
and V were unknown to people in Shakespear's time.
If we (5) ________ the history of punctuation, we also find some interesting facts. The Romans
used to write quaesto at the end of a sentence in order to show that it was a Question, they started to write
Qo in place of the whole word, and then put the Q above the 0. In the end, that became the question mark
"?"
Question 1: A. did B. had C. made D. took
Question 2: A. distance B. area C. length D. earth
Question 3: A. true B. accurate C. exact D. precise
Question 4: A. spread B. appeared C. was D. occuưed
Question 5: A. look into B. bring on C. make off D. hold up

EXERCISE 2. Read the following passage and mark the letter A, B, c or D on your answer sheet to
indicate the correct word for each of the blanks.
Basic to any understanding of Canada in the 20 years after the Second World War is the country’s
impressive population growth. For every three Canadians in 1945, there were over five in 1966. In
September 1966 Canada’s population passed the 20 million mark. Most of this surging growth came from

Hocmai – Ngôi trường chung của học trò Việt Tổng đài tư vấn: 1900 69 33 - Trang | 1 -

165
Hocmai.vn – Website học trực tuyến số 1 tại Việt Nam
Khóa học Luyện thi THPT quốc gia PEN C –N3 Môn Tiếng Anh cô Hương Fiona

natural increase. The depression of the 1930s and the war had held back marriages, and the catching-up
process began after 1945. The baby boom continued through the decade of the 1950s, producing a
population increase of nearly fifteen percent in the five years from 1951 to 1956 This rate of increase had
been exceeded only once before in Canada’s history, in the decade before 1911 when the prairies were
being settled. Undoubtedly, the good economic conditions of the
1950s supported a growth in the population, but the expansion also derived from a trend toward earlier
marriages and an increase in the average size of families. In 1957 the Canadian birth rate stood at 28 per
thousand, one of the highest in the worlk
After the peak year of 1957, the birth rate in Canada began to decline. It continued falling until in
1966 it stood at the lowest level in 25 years. Partly this decline reflected the low level of births during the
depression and the war, but it was also caused by changes in Canadian society. Young people were staying
at school longer more women were working; young married couples were buying automobiles or houses
before starting families; rising living standards were cutting down the size of families It appeared that
Canada was once more falling in step with the trend toward smaller families that had occurred all through
the Western world since the time of the Industrial Revolution.
Although the growth in Canada’s population had slowed down by 1966 (the increase in the first
half of the 1960s was only nine percent) another large population wave was coming over the horizon. It
would be composed of the children who were born during the period of the high birth rate prior to 1957.

Question 1. What does the passage mainly discuss?


A. Educational changes in Canadian society B. Canada during the Second World War
C. Population trends in postwar Canada D. Standards of living in Canada
Question 2: The word “five” in bold refers to
A. Canadians B. years C. decades D. marriages
Question 3: The word “surging” in bold is closest in meaning to
A. new B. extra C. accelerating D. surprising
Question 4: The author suggests that in Canada during the 1950's
A. the urban population decreased rapidly B. fewer people married
C. economic conditions were poor D. the birth rate was very high
Question 5: The author mention all of the following as causes of declines in population growth after 1957
EXCEPT
A. people being better educated B. people getting married earlier
C. better standards of living D. couples buying houses
Question 6: It can be inferred from the passage that before the Industrial Revolution
A. families were larger B. population statistics were unreliable
C. the population grew steadily D. economic conditions were bad
Question 7: The word “it” in bold refers to

Hocmai – Ngôi trường chung của học trò Việt Tổng đài tư vấn: 1900 69 33 - Trang | 2 -

166
Hocmai.vn – Website học trực tuyến số 1 tại Việt Nam
Khóa học Luyện thi THPT quốc gia PEN C –N3 Môn Tiếng Anh cô Hương Fiona

A. horizon B. population wave C. nine percent D. first half

EXERCISE 3.
Read the following passage and mark the letter A, B, C, or D on your answer sheet to indicate the correct
word or phrase that best fits each of the numbered blanks from 31 to 35.
Early writing and Alphabets
When people first began to write, they did not use an alphabet. Instead, they drew small pictures to
(1) _______ the objects they were writing about. This was very slow because there was a different picture
for any word.
The Ancient Egyptians had a system of picture writing that was described hieroglyphics. The
meaning of this writing was forgotten for a very long time but in 1799 some scientists (2) _______ a stone
near Alexandria, in Egypt. The stone had been there for (3) _______ a thousand years. It had both Greek
and hieroglyphics on it and researchers were finally able to understand what the hieroglyphics meant.
An alphabet is quite different from picture writing. It (4) _______ of letters or symbols that
represent a sound and each sound is just part of one word. The Phoenicians, who lived about 3,000 years
ago, developed the modern alphabets. It was later improved by the Roman’s and this alphabet is now used
(5) _______ throughout the world.
Question 1: A. notice B. show C. appear D. mark
Question 2: A. discovered B. realized C. delivered D. invented
Question 3: A. quite B. more C. over D. already
Question 4: A. consistsof B. includes C. contains D. involves
Question 5: A. broadly B. widely C. deeply D. hugely

EXERCISE 4.
Read the following passage and mark the letter A, B, C or D on your answer sheet to indicate the correct
answer to each of the questions below.
Because writing has become so important in our culture, we sometimes think of it as more real
than speech. A little thought, however, will show w hy speech is primary and writing secondary to
language. Human beings have been writing (as far as we can tell from surviving evidence) for at least 5000
years; but they have been talking for much longer, doubtless ever since there have been human beings.

Hocmai – Ngôi trường chung của học trò Việt Tổng đài tư vấn: 1900 69 33 - Trang | 3 -

167
Hocmai.vn – Website học trực tuyến số 1 tại Việt Nam
Khóa học Luyện thi THPT quốc gia PEN C –N3 Môn Tiếng Anh cô Hương Fiona

When writing did develop, it was derived from and represented speech, although
imperfectly. Even today there are spoken languages that have no written form. Furthermore, we all learn to
talk well before we learn to write; any human child who is not severely handicapped physically
or mentally will learn to talk: a normal human being cannot be prevented from doing so. On the other
hand, it takes a special effort to learn to write. In the past many intelligent and useful members of
society did not acquire the skill, and even today many who speak languages with writing systems
never learn to read or write, while some who learn the rudiments of those skills do so only
imperfectly.
To affirm the primacy of speech over writing is not, however, to disparage the latter. One
advantage writing has over speech is that it is more permanent and makes possible the records that any
civilization must have. Thus, if speaking makes us human, writing makes us civilized.
Question 1:We sometimes think of writing as more real than speech because ______.
A. writing is secondary to language
B. human beings have been writing for at least 5000 years
C. it has become very important in our culture
D. people have been writing since there have been human beings
Question 2:The author of the passage argues that ______.
A. speech is more basic to language than writing
B. writing has become too important in today’s society
C. everyone who learns to speak must learn to write
D. all languages should have a written form
Question 3: According to the passage, writing ______.
A. is represented perfectly by speech B. represents speech, but not perfectly
C. developed from imperfect speech D. is imperfect, but less so than speech
Question 4:Normal human beings ______.
A. learn to talk after learning to write B. learn t o write before learning to talk
C. learn to write and to talk at the same time D. learn to talk before learning to write
Question 5:Learning to write is ______.
A. easy B. too difficult C. not easy D. very easy
Question 6:In order to show that learning to write requires effort, the author gives the example of
______.

Hocmai – Ngôi trường chung của học trò Việt Tổng đài tư vấn: 1900 69 33 - Trang | 4 -

168
Hocmai.vn – Website học trực tuyến số 1 tại Việt Nam
Khóa học Luyện thi THPT quốc gia PEN C –N3 Môn Tiếng Anh cô Hương Fiona

A. people who learn the rudiments of speech B. severely handicapped children


C. intelligent people who couldn’t write D. people who speak many languages
Question 7:In the author’s judgment, ______.
A. writing has more advantages than speech
B. writing is more real than speech
C. speech conveys ideas less accurately than writing does
D. speech is essential but writing has important benefits
Question 8. The word “advantage” in the last paragraph most closely means ______.
A. “rudiments” B. “skill” C. “domination” D. “benefit”

EXERCISE 5
Read the following pasage and mark the letter A, B, C or D on your answer sheet ti indicate the correct
answer to each of the questions
Over the past 600 years, English has grown from a language of few speakers to become the
dominant language of international communication. English as we know it today emerged around 1350,
after having incorporated many elements of French that were introduced following the Norman invasion
off 1066. Until the 1600s, English was, for the most part, spoken only in England and had not expanded
even as far as Wales, Scotland, or Ireland. However, during the course of the next two century, English
began to spread around the globe as a result of exploration, trade (including slave trade), colonization, and
missionary work. Thus, small enclaves of English, speakers became established and grew in various parts
of the world. As these communities proliferated, English gradually became the primary language of
international business, banking, and diplomacy.
Currently, about 80 percent of the information stored on computer systems worldwide is in
English. Two thirds of the world's science writing is in English, and English is the main language of
technology, advertising, media, international airport, and air traffic controllers. Today there are more than
700 million English users in the world, and over half of these are non-native speakers, constituting the
largest number of non-native users than any other language in the world.
Question 1:What is the main topic of the passage?
A. The number of non-native users of English.
B. The French influence on the English language.
C. The expansion of English as an international language.
D. The use of English for science and technology.
Question 2:: In the first paragraph, the word "emerged" is closest in meaning to
A. appeared B. hailed C. frequented D. engaged

Hocmai – Ngôi trường chung của học trò Việt Tổng đài tư vấn: 1900 69 33 - Trang | 5 -

169
Hocmai.vn – Website học trực tuyến số 1 tại Việt Nam
Khóa học Luyện thi THPT quốc gia PEN C –N3 Môn Tiếng Anh cô Hương Fiona

Question 3:: In the first paragraph, the word "elements" is closest in meaning to
A. declaration B. features C. curiosities D. customs
Question 4:: Approximately when did English begin to be used beyond England?____________ .
A. In 1066 B. around 1350 C. before 1600 D. after 1600
Question 5:: According to the passage, all of the following contributed to the spread of English around the
world except ____________ .
A. the slave trade B. the Norman invasion
C. missionaries D. colonization
Question 6: In the second paragraph, the word "stored" is closest in meaning to____________ .
A. bought B. saved C. spent D. valued
Question 7:According to the passage, approximately how many non-native users of English are there in
the world today ? ____________ .
A. a quarter million B. half a million
C. 350 million D. 700 million

Nguồn : Hocmai.vn

Giáo viên : Hương Fiona

Hocmai – Ngôi trường chung của học trò Việt Tổng đài tư vấn: 1900 69 33 - Trang | 6 -

170
Hocmai.vn – Website học trực tuyến số 1 tại Việt Nam
Khóa học Luyện thi THPT quốc gia PEN C –N2 Môn Tiếng Anh thầy Phạm Trọng Hiếu

PHƯƠNG PHÁP LÀM BÀI CÂU GIAO TIẾP


( BÀI TẬP TỰ LUYỆN )
Giáo viên : Phạm Trọng Hiếu

Mark the letter A, B, C, or D to indicate the most suitable response to complete the following exchanges
Question 1. - Kate:" How lovely your cats are!" - David:" ________ "
A. Really? They are B. Thank you, it is nice of you to say so
C. Can you say it again? D. I love them, too
Question 2. - Anne:" Make yourself at home" - John: “ ________ ”
A. Thanks! Same to you B. That's very kind. Thank you
C. Not at all. Don't mention it D. Yes, Can I help you?
Question 3. - Dad: ‘I have bought you a toy. Happy birthday to you!’ - Son- ‘_______’
A. Have a nice day! B. The same to you!
C. What a lovely toy! Thanks. D. What a pity!
Question 4. ‘When can you get it all done?' – ‘_______’
A. How does next Monday sound? B. Two hours ago
C. I used to do it on Monday D. How much time do 1 need?
Question 5. - Joan: Our friends are coming _______, Mike?’
- Mike: I’m sorry, I can't do it now.'
A. Why don’t we cook some coffee B. Shall I make you like some coffee
C. Shall you make some coffee, please D. Would you mind making some coffee
Question 6: 'Would you like to have dinner with me?’ ‘_______’
A. Yes, it is B. Yes, so do I
C. I’m very happy D. Yes, I'd love to
Question 7: A: I don’t like meat. - B: ________.
A. Nor I do B. I don’t too. C. Neither don’t I. D. Neither do I
Question 8: Cindy wanted to drive her best friend to have dinner at an Italian restaurant but her car
couldn’t start. Therefore, she had to borrow one from Nancy. Choose the most suitable response to fill in
the blank in the following exchange.
Cindy: “Would you mind lending me your car?” - Nancy: “________.”
A. Yes. Here it is B. Yes, let’s
C. No, not at all D. Great

Hocmai– Ngôi trường chung của học trò Việt Tổng đài tư vấn: 1900 69 33 - Trang | 1-

171
Hocmai.vn – Website học trực tuyến số 1 tại Việt Nam
Khóa học Luyện thi THPT quốc gia PEN C –N2 Môn Tiếng Anh thầy Phạm Trọng Hiếu

Question 9: “Can I have a look at that pullover, please?”


“_________________________.”
A. Sorry, it is out of stock. B. Which one? This one?
C. It’s much cheaper. D. Can I help you?
Question 10: “Thanks for helping me with my homework, John.”
“_________________________.”
A. Don’t mention it. B. Please, don’t blame yourself.
C. No harm done. D. I don’t mean so.
Question 11: “What's your birthday, Ann?” – “It's _____.”
A. on the thirty-one of July B. on July the thirty-one
C. on the thirty-first of July D. in July the thirty- first
Question 12: “Sugar?” – “________”
A. No, no me! B. No, I don't. C. Thanks, I'd like. D.Yes,please
Question 13: A: “Can I help you, madam?”
B: “______________________”
A. Yes, it's in our summer sale B. It's very cheap.
C. No, thanks. I'm just looking D. Right. It looks a bit small
Question 14: “Do you mind if I switch the light off?”
- “_________________”
A. Yes, I mind it, sorry. B. What if I don’t mind it?
C. Yes, please do it. D. I’d rather you didn’t, if you don’t mind.
Question 15: Harry:" ______ "
Kate: "Yes. I'd like to buy a computer. "
A. Do you look for something? B. Good morning. Can I help you?
C. Excuse me. Do you want to buy it? D. Can you help me buy something?
Question 16: Peter: “I’ve been awarded a scholarship to study in America.”
Kate: “Uh. really?_________!”
A. Take care of yourself B. Congratulations
C. You are always lucky D. Lucky as you are
Question 17: - "Mr. Adams is going to retire next month."
- “_____________.”
A. Oh, I have no idea C. Right, you’d probably be the next.
B. You don't say! D. Congratulations!

Hocmai– Ngôi trường chung của học trò Việt Tổng đài tư vấn: 1900 69 33 - Trang | 2-

172
Hocmai.vn – Website học trực tuyến số 1 tại Việt Nam
Khóa học Luyện thi THPT quốc gia PEN C –N2 Môn Tiếng Anh thầy Phạm Trọng Hiếu

Question 18: - “________________”


- "Never mind, better luck next time."
A. I’ve broken your precious vase. B. I have a lot on my mind,
C. I couldn’t keep my mind on work. D. I didn't get the vacant position
Question 19: - Jordan: "________"
- Jim: "No, thanks."
A. Would you want another drink? B. Would you care for another drink?
C. Can you help me with this? D. Come in, please! breath
Question 20: Jane: “It's going to rain”.
Mary: “________.”
A. I hope not so B. I hope not C. I don't hope so D. I don't hope either
Question 21: A: I’ve been working at this school for 15 years.
B: __________________
A. Why did you leave? B. Are you going to retire soon?
C. Why had you left? D. Will you retire?
Question 22: “A: Would you bother if I had a look at your paper?’
B: ______________
A. You’re welcome B. That’s a good idea
C. Oh, I didn’t realize it. D. Well, actually I’d rather you didn’t
Question 23: - “You must be Jane's sister. Glad to meet you.
“______________”
A. I am, either B. So I am. I'm glad C. What do you do D. Me too
Question 24: - “How lovely your pets are!”
“_______________”
A. Thank you, it's nice of you to say no B. Really? They are
C. can you say that again D. I love them, too
Question 25: Mary is talking to a porter in the hotel lobby.
Porter: “Shall I help you with your suitcase?” - Mary: “_______”
A. Not a chance. B. That's very kind of you.
C. I can’t agree more. D. What a pity!
Question 26: Hoa is asking Hai, who is sitting at a corner of the room, seeming too shy.
Hoa: “Why aren’t you taking part in our activities? _______” - Hai: “Yes. I can. Certainly.”
A. Could you please show me how to get to the nearest post office?

Hocmai– Ngôi trường chung của học trò Việt Tổng đài tư vấn: 1900 69 33 - Trang | 3-

173
Hocmai.vn – Website học trực tuyến số 1 tại Việt Nam
Khóa học Luyện thi THPT quốc gia PEN C –N2 Môn Tiếng Anh thầy Phạm Trọng Hiếu

B. Shall I take your hat off?


C. Can I help you?
D. Can you help me with these decorations?
Question 27: “I don’t think I can do this.” – “_______”
A. Sure, no way! B. Yeah. It’s not easy.
C. Oh, come on! Give it a try! D. No, I hope not
Question 28: Peter: “Is it important?” - Thomas: “_______”
A. Not on your life! B. It’s matter of life and death!
C. No worry, that’s nothing. D. It’s ridiculous
Question 29: Bob: “Our team has just won the last football match.”
Michael: “_______!”
A. Good idea. Thanks for the news B. Yes. I guess it’s very good
C. Well, that’s very surprising D. Yes, it’s our pleasure
Question 30: “_______” - “Oh, it’s great!”
A. How is the English competition like?
B. Would you like the English competition?
C. What do you like about the English competition?
D. What do you think of the English competition?

Nguồn : Hocmai.vn
Giáo viên : Phạm Trọng Hiếu

Hocmai– Ngôi trường chung của học trò Việt Tổng đài tư vấn: 1900 69 33 - Trang | 4-

174
Hocmai.vn – Website học trực tuyến số 1 tại Việt Nam
Khóa học Luyện thi THPT quốc gia PEN C –N3 Môn Tiếng Anh côHương Fiona

PHƯƠNG PHÁP LÀM BÀI TỪ ĐỒNG NGHĨA-TRÁI NGHĨA


( BÀI TẬP TỰ LUYỆN )
Giáo viên : Nguyễn Thanh Hương
Các bài tập trong tài liệu này được biên soạn kèm theo bài giảng “Phương pháp làm bài từ đồng nghĩa-trái nghĩa” thuộc Khóa học
Luyện thi THPT quốc gia PEN - C: Môn Tiếng Anh (cô Hương Fiona )” tại website Hocmai.vn để giúp các Bạn kiểm tra, củng cố lại
các kiến thức được giáo viên truyền đạt trong bài giảng tương ứng. Để sử dụng hiệu quả, Bạn cần học trước bài giảng, sau đó làm đầy
đủ các bài tập trong tài liệu này.

Mark the letter A, B, C or D on your answer sheet to indicate the word(s) CLOSEST in meaning to the
underlined word(s) in each of the following questions.
Question 1: Readers are required to abide by the rules of the lybrary and mind their manners.
A. memorize B. obey C. compose D. review
Question 2: The whole village was wiped out in the earthquake last night.
A. Cleaned well B. destroyed C. changed completely D. removed quickly
Mark the letter A, B, C or D on your answer sheet to indicate the word(s) OPPOSITE in meaning to
the underlined word(s) in each of the following questions
Question 3: They have not made any effort to intergrate with the local community.
A. Put together B. separate C. connect D. cooperate
Question 4: Experts hope that the vaccine will be mass-produced soon.
A. Produced with high cost B. Produced in great numbers
C. Produced cheaply D. Produced in small numbers
Mark the letter A,B,C or D to indicate the word or phrase that is OPPOSITE in meaning to the
underlined part in each of the following questions.
Question 5: I find it hard to work at home because there’re too many distractions.
A. Unawareness B. unconcern C. carelessness D. attention
Question 6: In 1864 George Pullman designed a sleeping car that eventually saw widespread use.
A. previously B. ultimately C. familiarly D. simultaneously
Mark the letter A, B, C, or D on your answer sheet to indicate the word CLOSEST in meaning to the
underlined word in each of the following questions.
Question 7: The situation seems to be changing minute by minute.
A. from time to time B. time after time C. again and again D. very rapidly
Question 8: An interesting feature of this park is the Orphanage where lots of orphaned or abandoned animals
are taken care of.
A. immoral B. wicked C. shameless D. deserted

Hocmai– Ngôi trường chung của học trò Việt Tổng đài tư vấn: 1900 69 33 - Trang | 1-

175
Hocmai.vn – Website học trực tuyến số 1 tại Việt Nam
Khóa học Luyện thi THPT quốc gia PEN C –N3 Môn Tiếng Anh côHương Fiona

Mark the letter A, B, C, or D on your answer sheet to indicate the word(s) OPOSITE in meaning to the
underlined word(s) in each of the following questions.
Question 9: We can’t make any conclusion right now with this paucity of information.
A. excess B. certainty C. timidity D. scarcity
Question 10: The law restricting pollution in the city are very rigid.
A. stiff B. strict C. strict D. tolerant
Mark the letter A, B, C, or D on your answer sheet to indicate the word(s) CLOSEST in meaning to the
underlined word(s) in each of the following questions.
Question 11: The air is naturally contaminated by foreign matter such as plant pollens and dust.
A. filled B. concentrated C. polluted D. occupied
Question 12: Shake a leg or you will miss the train.
A. Hurry up B. Slow down C. Watch out D. Put down
Mark the letter A, B, C, or D on your answer sheet to indicate the word(s) OPPOSITE in meaning to the
underlined word(s) in each of the following questions.
Question 13: People sometimes choose partners who compensate for their own shortcomings.
A. strengths B. disadvantages C. weaknesses D. benefits
Question 14: You should pat yourself on the back for having achieved such a high score in the graduation
exam.
A. praise yourself B. criticize yourself C. check up your back D. wear a backpack
Mark the letter A, B, C or D on your answer sheet to indicate the word or phrase that is OPPOSITE is
meaning to the underlined part in each of the following questions.
Question 15: My friend tends to look on the bright side in any circumstances.
A. be smart B. be confident C. be pessimistic D. be optimistic
Question 16: All children can attend without paying fees at state school.
A. high schools B. primary schools
C. private schools D. secondary schools
Mark the letter A, B, C or D on your answer sheet to indicate the word or phrase that is CLOSEST is
meaning to the underlined part in each of the following questions.
Question 17: They decided to postpone their journey till the end of the month because of epidemic.
A. put off B. turn round C. do with D. carry on
Question 18: In the future many large corporations will be wiped out an millions of jobs will be lost.
A. destroyed B. developed C. broadened D. opened

Hocmai– Ngôi trường chung của học trò Việt Tổng đài tư vấn: 1900 69 33 - Trang | 2-

176
Hocmai.vn – Website học trực tuyến số 1 tại Việt Nam
Khóa học Luyện thi THPT quốc gia PEN C –N3 Môn Tiếng Anh côHương Fiona

Choose A, B, C or D to indicate the word that is OPPOSITE in meaning to the underlined part in each
of the following questions:
Question 19: Mr.Smith’s new neighbors appear to be very friendly.
A. Inapplicable B. hostile C. amiable D. futile
Question 20: Henry has found a temporary job in a factory.
A. Genuine B. eternal C. permanent D. satisfactory

Choose A, B ,C or D to indicate the word that is CLOSEST in meaning to the underlined part in each
sentence.
Question 21: Carpets from countries such as Persia and Afghanistan often fetch high prices in the United
States.
A. Artifacts B. Textile C. Pottery D. Rugs
Question 22: Though many scientific breakthroughs have resulted from mishaps it has taken brilliant thinkers
to recognize their potential.
A. Incidental B. misunderstandings C. accidents D. misfortunes

Mark the letter A, B. C, or D on your answer sheet to indicate the word(s) CLOSEST in meaning to the
underlined word(s) in each of the following
Question 23: An international medical conference initiated by Davison resulted in the birth of the League of
Red Cross Societies in 1991.
A. treated B. dedicated C. helped D. started
Question 24: He drove so fast that I really felt my life was in danger.
A. at the target B. in accident C. at stake D. in comfort

Mark the letter A,B,C or D on your answer sheet to indicate the word(s) OPPOSITE in meaning to the
underlined word(s) in each of the following questions
Question 25: Be quick! We must speed up if we don’t want to miss the flight”
A. put forward B. look up C. slow down D. turn down
Question 26: Although it’s a long day for us, we feel we are contented with what we do
A. interested B. dissatisfied C. excited D. shocked
Mark the letter A, B, C, or D on your answer sheet to indicate the word(s) CLOSEST in meaning to the
underlined word (s) in each of the following questions
Question 27: Some studies have shown a strong association between pesticide and certain diseases.

Hocmai– Ngôi trường chung của học trò Việt Tổng đài tư vấn: 1900 69 33 - Trang | 3-

177
Hocmai.vn – Website học trực tuyến số 1 tại Việt Nam
Khóa học Luyện thi THPT quốc gia PEN C –N3 Môn Tiếng Anh côHương Fiona

A. cooperation B. collaboration C. consequence D. connection


Question 28: Although they hold similar political views, their religious beliefs present a striking contrast.
A. minor comparison B. interesting resemblance
C. significant discrepancy D. complete coincidence
Mark the letter A, B, C, or D on your answer sheet to indicate the word(s) OPPOSITE in meaning to the
underlined word (s) in each of the following questions
Question 29: The research, contrary to common belief, revealed some rather unexpected results.
A. impressive B. predictable C. surprising D. positive
Question 30: I tried to patch things up after the argument, but they wouldn't speak to me.
A. fall out with B. make up with C. spice up D. straighten out
Mark the letter A, B, C or D on your answer sheet to indicate the word (s) OPPOSITE in meaning to
the underlined word (s) in each of the following questions
Question 31: If he gets to university, his parents will be walking on air
A. disgusted B. promising C. upset D. hopeful
Question 32: She rarely smiles because she’s shy about exposing her crooked teeth.
A. pulling B. hiding C. showing D. brushing
Mark the letter A, B, C or D on your answer sheet to indicate the word (s) CLOSEST in meaning to the
underlined word (s) in each of the following questions.
Question 33: Did anyone acknowledge responsibility for the outbreak of the fire ?
A. inquire about B. accept C. report D. find out
Question 34: There is a set of expensive spoons, forks and knives in the cupboard.
A. cutlery B. instrument C. equipment D. utensil
Choose the letter A, B, C, or D on your answer sheet to indicate the word that is CLOSEST in meaning
to the underlined part in each of the following questions
Question 35: The newscaster have a concise account of tradegy.
A. complicated and intricate B. short and clear
C. long and detailed D. sad and depressing
Question 36: At last, we succeeded in persuading those boys and girls to join our picnic.
A. At the end B. In the end C. Lastly D. Endlessly
Choose the letter A, B, C, or D on your answer sheet to indicate the word that is OPPOSITE in meaning
to the underlined part in each of the following questions
Question 37: “That is a well-behaved boy whose behavior has nothing to complain about.”
A. behaving cleverly B. behaving nice

Hocmai– Ngôi trường chung của học trò Việt Tổng đài tư vấn: 1900 69 33 - Trang | 4-

178
Hocmai.vn – Website học trực tuyến số 1 tại Việt Nam
Khóa học Luyện thi THPT quốc gia PEN C –N3 Môn Tiếng Anh côHương Fiona

C. good behavior D. behaving improperly


Question 38: He mentioned in particular electronics, his major at university.
A. one and all B. in general C. on whole D. in all
Mark the letter A, B, C, or D on your answer sheet to indicate the word or phrase CLOSEST in meaning
to the underlined word(s) in each of the two following questions.
Question 39. The shop assistant was totally bewildered by the customer’s behavior.
A. disgusted B. puzzled C. angry D. Upset
Question 40. The newspaper reporters bear out what the Minister told yesterday.
A. define B. confirm C. support D. complain
Mark the letter A, B, C, or D on your answer sheet to indicate the word or phrase OPPOSITE in
meaning to the underlined word(s) in each of the two following questions
Question 41. Unless you have been very lucky, you have undoubtedly experienced events in your life that
made you cry.
A. cei tainly B. questionably C. absolutely D. definitely
Question 42. His boss has had enough of his impudence and doesn't want to hire him anymore.
A. respect B. rudeness C. obedience D. agreement
Mark the letter A, B, C, or D on your answer sheet to indicate the word or phrase CLOSEST in meaning
to the underlined word(s) in each of the two following questions.
Question 43. The student apologized to his teacher for submitting the essay late.
A. handing in B. dropping out of
C. carrying out D. bringing in
Question 44. I wonder when I’m finally going to receive news from Joe.
A. to hear of B. to hear from
C. to get in touch with D. turn away from
Mark the letter A, B, C, or D on your answer sheet to indicate the word or phrase OPPOSITE in
meaning to the underlined word(s) in each of the two following questions.
Question 45. We run a very tight ship here, and we expect all our employees to be at their desks by eight
o'clock and take good care of their own business.
A. manage an inflexible system B. have a good voyage
C. run faster than others D. organize things inefficiently
Question 46. He usually stays in peace in stressful situations but this time he really lost his head.
A. kept calm B. excited
C. took leave of his sense D. lost touch

Hocmai– Ngôi trường chung của học trò Việt Tổng đài tư vấn: 1900 69 33 - Trang | 5-

179
Hocmai.vn – Website học trực tuyến số 1 tại Việt Nam
Khóa học Luyện thi THPT quốc gia PEN C –N3 Môn Tiếng Anh côHương Fiona

Mark the letter A, B, C, or D on your answer sheet to indicate the word or phrase CLOSEST in meaning
to the underlined word(s) in each of the two following questions.
Question 47. We decided to pay for the furniture on the installment plan.
A. monthly payment B. cash and carry
C. credit card D. piece by piece
Question 48. Teletext is continuously sent out at all times when regular television programs are broadcast.
A. transmitted B. electrified C. automated D. aired
Mark the letter A, B, C, or D on your answer sheet to indicate the word or phrase OPPOSITE in
meaning to the underlined word(s) in each of the two following questions.
Question 49. Any student who neglects his or her homework is unlikely to do well at school.
A. puts off B. looks for C. attends to D. approves of
Question 50. Many political radicals advocated that women should not be discriminated on the basic of their
sex.
A. openly criticized B. rightly claimed
C. publicly said D. protested

Nguồn : Hocmai.vn
Giáo viên : Hương Fiona

Hocmai– Ngôi trường chung của học trò Việt Tổng đài tư vấn: 1900 69 33 - Trang | 6-

180
Hocmai.vn – Website học trực tuyến số 1 tại Việt Nam
Khóa học Luyện thi THPT quốc gia PEN C –N3 Môn Tiếng Anh côHương Fiona

PHƯƠNG PHÁP LÀM BÀI CÂU ĐỒNG NGHĨA – NỐI CÂU


( BÀI TẬP TỰ LUYỆN )
Giáo viên : Nguyễn Thanh Hương
Các bài tập trong tài liệu này được biên soạn kèm theo bài giảng “Phương pháp làm bài câu đồng nghĩa – nối câu” thuộc Khóa học
Luyện thi THPT quốc gia PEN - C: Môn Tiếng Anh (cô Hương Fiona )” tại website Hocmai.vn để giúp các Bạn kiểm tra, củng cố lại
các kiến thức được giáo viên truyền đạt trong bài giảng tương ứng. Để sử dụng hiệu quả, Bạn cần học trước bài giảng, sau đó làm đầy
đủ các bài tập trong tài liệu này.

Mark the letter A, B, C, or D on your answer sheet to indicate the sentence that is CLOSEST in meaning
to each of the following sentences
Question 1: The scene is set in Normandy, but most of the characters in this novel are Londoners.
A. In the novel, the action moves backwards and forwards between Normandy and London.
B. The main characters in the novel are Londoners on a sightseeing holiday in Normandy.
C. In this novel, the story takes place in Normandy but the majority of the characters are from London.
D. The story is about Normandy, but the leading characters are all Londoners
Question 2: The airline requested a confirmation call to ensure a seat on my flight back home.
A. The airline made sure I got a seat on my flight home
B. I secured a seat on my flight home.
C. The airline confirmed a seat on my flight home
D. It was necessary to confirm seat on my flight back home.
Question 3. Mary should never have been allowed to try to swim in the sea alone.
A When Mary left to swim in the sea alone, she said she knew what she was doing.
B. It would probably be wrong to let Mary swim in the sea on her own.
C. No one could have stopped Mary from trying to swim in the sea by herself.
D. Someone ought to have stopped Mary from attempting to swim in the sea on her own
Question 4: The last time when I saw her was three years ago.
A. I have often seen her for the last three years.
B. About three years ago, I used to meet her.
C. I have not seen her for three years.
D. I saw her three years ago and will never meet her again
Question 5: Somebody cleans that room every day.
A. The room every day is cleaned.
B. The room is everyday cleaned.
C. The room is cleaned every day.

Hocmai– Ngôi trường chung của học trò Việt Tổng đài tư vấn: 1900 69 33 - Trang | 1-

181
Hocmai.vn – Website học trực tuyến số 1 tại Việt Nam
Khóa học Luyện thi THPT quốc gia PEN C –N3 Môn Tiếng Anh côHương Fiona

D. The room is cleaned by somebody everyday.


Question 6: As we were heading out of the café, we bumped into our plumber, who we still owed money
to.
A. Our plumber, whom we hadn't paid back yet, was the person who we encountered by chance at the moment
we were exiting the café.
B. When we met our plumber in the café as we were about to leave, we realised that we hadn't yet paid him
all the money we owed him.
C. Our plumber, when we ran into him as he was entering the cafe that we were leaving, didn't bring up our
debt to him.
D. Not wanting to see the plumber to whom we still owed a debt, we quickly headed out of the café when we
saw him coming in.
Question 7: They said that Paula had quit five jobs before working for us.
A. They said that five jobs had been quit by Paula before working for us.
B. Paula worked for us then she quit five other jobs.
C. It was said that Paula had quit five jobs before working for us
D. Paula said that she had quit five jobs before working for us.
Question 8: “You're always making terrible mistakes", said the teacher.
A. The teacher complained about his student making terrible mistakes.
B. The teacher made his students not always make terrible mistakes
C. The teacher asked his students why they always made terrible mistakes.
D. The teacher realized that his students always made terrible mistakes
Question 9. Diana ran into her former teacher on the way to the stadium yesterday.
A. Diana caused an accident to her teacher while she was going to the stadium.
B. Diana's car ran over her teacher on the way to the stadium.
C. Diana happened to meet her teacher while she was going to the stadium.
D. Diana's teacher got run over whole she was going to the stadium.
Question 10: “Why I haven’t thought of this before?” Tony said to himself.
A. Tony advised himself not to have thought of that before.
B. Tony suggested himself not thinking of that before.
C. Tony said that why he hasn’t thought of that before.
D. Tony wondered why he hadn’t thought of that before.
Question 11: She is the most intelligent woman I have ever met.
A. I have never met a more intelligent woman than her.

Hocmai– Ngôi trường chung của học trò Việt Tổng đài tư vấn: 1900 69 33 - Trang | 2-

182
Hocmai.vn – Website học trực tuyến số 1 tại Việt Nam
Khóa học Luyện thi THPT quốc gia PEN C –N3 Môn Tiếng Anh côHương Fiona

B. She is not as intelligent as the women I have ever met.


C. I have ever met such an intelligent woman.
D. She is more intelligent than I am.
Question 12: She broke down the moment she heard the news.
A. She was broken for a moment when she heard the news.
B. She broke her leg when hearing the news.
C. On hearing the news, she broke down.
D. When she heard the news, she was sick.
Question 13: Although he was very tired, he agreed to help his child with his homework.
A. Despite of his tiredness, he was eager to help his child with his homework.
B. Tired as he was, he agreed to help his child with his homework.
C. Even if feeling very tired, he agreed to help his child with his homework.
D. He would have helped his child with his homework if he hadn’t been tired
Question 14: It’s Nick’s job to look after the tender plants in the garden.
A. Nick enjoys looking after the tender plants in the garden.
B. Nick is responsible for looking after the tender plants in the garden.
C. Looking after the tender plants in the garden is liable for Nick.
D. The tender plants in the garden give Nick opportunity to work.
Question 15: My suit needs to be cleaned before the interview but I’m too busy to do that.
A. I must have my suit cleaned before the interview.
B. I must have my suit to be cleaned before the interview.
C. I must clean my suit before the interview.
D. I must get my mother to clean my suit before the interview
Question 16: He seemed very reluctant to take my advice.
A. He seemed quite willing to take my advice.
B. It seemed he was not ready to give me advice
C. It seemed he was not willing to take my advice.
D. He seemed very anxious to take my advice.
Question 17: It was very impolite of him to leave without saying a word.
A.He was very impolite to leave without saying a word.
B.He didn’t say nothing when he left.
C. He didn’t say anything as he left, which was impolite.
D. Both A and c are correct.

Hocmai– Ngôi trường chung của học trò Việt Tổng đài tư vấn: 1900 69 33 - Trang | 3-

183
Hocmai.vn – Website học trực tuyến số 1 tại Việt Nam
Khóa học Luyện thi THPT quốc gia PEN C –N3 Môn Tiếng Anh côHương Fiona

Question 18: She reminded her daughter of their table manners.


A. She wanted her daughter to be more polite while eating.
B. She wanted her daughter to leave the dinner table.
C. She wanted her daughter to eat a little more slowly.
D. She wanted her daughter to remember all meal time.
Question 19: He was driving so fast that he could have had an accident.
A. He wasn’t driving fast enough to avoid an accident.
B. He didn’t have an accident although he was driving very fast
C. If he had been driving very fast, he would have had an accident.
D. An accident happened, and it was caused by his very fast driving
Question 20: Mrs. Jones told me that her neighbors were moving to Florida
A. Mrs. Jones and her neighbors live in Florida.
B. Mrs. Jones is planning to move to Florida with her neighbors.
C. I knew that Mrs. Jones had moved to Florida because her neighbors told me.
D. “My neighbors are moving to Florida,” said Mrs. Jones.
Question 21: No matter how hard Fred tried to start the car, he didn’t succeed.
A. Fried tried very hard to start the car, and succeeded.
B. Fried tried hard to start the car, and with success,
C. However hard he fried, Fried couldn’t start the car.
D. It’s hard for Fried to start the car because he never succeeded
Question 22: When the unemployment rate is high, the crime rate is usually also high.
A.The unemployment rate and the crime rate are both higher.
B. The higher the unemployment rate is, the higher the crime rate is.
C. The unemployment rate is as high as the crime rate.
D. The high rate of unemployment depends on the high rate of crime
Question 23: I wish you hadn't said that
A. I wish you not to say that. B. If only you didn't say tot
C. I hope you will not say that. D. It would be nice if you hadn’t said that.
Question 24: “You're always making terrible mistakes”, said the teacher.
A. The teacher asked his students why they always made terrible mistakes.
B. The teacher realized that his students always made terrible mistakes.
C. The teacher complained about his students making terrible mistakes.
D. The teacher made his students not always make terrible mistakes

Hocmai– Ngôi trường chung của học trò Việt Tổng đài tư vấn: 1900 69 33 - Trang | 4-

184
Hocmai.vn – Website học trực tuyến số 1 tại Việt Nam
Khóa học Luyện thi THPT quốc gia PEN C –N3 Môn Tiếng Anh côHương Fiona

Question 25: "It can't be Mike who leaked the document, it might be Tom.” said our manager.
A. Our manager suspected Tom of having leaked the document, not Mike.
B. Our manager blamed Tom for having leaked the document instead of Mike.
C. Our manager showed his uncertainty about who leaked the document: Mike or Tom
D. Our manager made it clear that Tom was the one who leaked the document, not Mike
Question 26: Their holiday plans fell through because there was a strike at the airport.
A. They couldn’t go on holiday as planned as a result of a strike at the airport
B. They failed to go on the holiday like they had planned because a strike took place at the airport
C. A strike at the airport almost put a stop to their holiday plans.
D. Disappointingly, a strike at the airport forced their holiday plans to nothing
Question 27: He was driving so fast that he could have had an accident.
A. An accident happened, and it was caused by his very fast driving
B. He didn’t have an accident although he was driving very fast
C. If he had been driving very fast, he would have had an accident.
D. He wasn't driving slowly enough to avoid the accident
Question 28: "Leave my house now or I'll call the police!" shouted the lady to the man.
A. The lady threatened to call the police if the man didn't leave her house.
B. The lady said that she would call the police if the man didn't leave her house.
C. The lady told the man that she would call the police if he didn't leave her house.
D. The lady informed the man that she would call the police if he didn't leave her house
Question 29: He last had his eyes tested ten months ago.
A. He had tested his eyes ten months before.
B. He had not tested his eyes for ten months then.
C. He hasn't had his eyes tested for ten months.
D. He didn't have any test on his eyes in ten months
Question 30: The children couldn't go swimming because the sea was too rough.
A. The children were not calm enough to swim in the sea.
B. The sea was rough enough for the children to swim in.
C. The sea was too rough for the children to go swimming.
D. The sea was too rough to the children's swimming
Question 31. Both English and Vietnamese use Roman scripts' but the latter is a tonal language.
A. Later both Vietnamese and English use Roman scripts.
B. English has tones but Vietnamese does not, although they both use Roman scripts.

Hocmai– Ngôi trường chung của học trò Việt Tổng đài tư vấn: 1900 69 33 - Trang | 5-

185
Hocmai.vn – Website học trực tuyến số 1 tại Việt Nam
Khóa học Luyện thi THPT quốc gia PEN C –N3 Môn Tiếng Anh côHương Fiona

C. Although English and Vietnamese are both Roman scripts, the former is not a tonal language while the
latter is.
D. Vietnamese and English which use Roman scripts later become tonal languages
Question 32. If I had known about their wedding plan earlier I would have been able to make time to
attend the reception party.
A. I knew their wedding would be planned earlier so I made some time to attend the morning reception.
B. I wish I had known their wedding plan sooner so that I could arrange time to attend the morning reception.
C. I don t know their wedding plan earlier so I can't make time to attend their morning reception.
D. When I knew their wedding party, it was too late to attend the morning reception.
Question 33. "You did a great job! I'm proud of your achievement" said the woman to her grandchild.
A. The woman said that her grandchild's job was great and site was proud of his work achievement.
B. The woman told her grandchild that she was proud of his achievement at work.
C. The woman told her grandchild to do a great job so that she could be proud of his achievement.
D. The woman complimented her grandchild on his achievement
Mark the letter A, B, C, or D on your answer sheet to indicate the sentence that best combines each pair
of sentences in the following questions
Question 34. He cannot lend me the book now. He has not finished reading it yet.
A. Having finished reading the book, he cannot lend it to me.
B. He cannot lend me the book until he has finished reading it.
C. Not having finished reading the book, he will lend it to me.
D. As long as he cannot finish reading the book, he will lend it to me
Question 35. I did not arrive in time. I was notable to see her off.
A. She had left because I was not on time.
B. I did not go there, so I could not see her off.
C. I was not early enough to see her off.
D. I arrived very late to say goodbye to her.
Question 36: I'm sorry I wasn’t in the office when you phoned. I know I promised to be.
A. I should be in the office when you phoned.
B. I should have been in the office when you phoned.
C. I must have been in the office when you phoned.
D. I might be in the office when you phoned.
Question 36: There was a terrible flood. All villagers, who had received a warning of the impending
flood, escaped to safety.

Hocmai– Ngôi trường chung của học trò Việt Tổng đài tư vấn: 1900 69 33 - Trang | 6-

186
Hocmai.vn – Website học trực tuyến số 1 tại Việt Nam
Khóa học Luyện thi THPT quốc gia PEN C –N3 Môn Tiếng Anh côHương Fiona

A. All of the villagers had been warned but only some escaped.
B. All of the villagers had been warned and all escaped.
C. Only some of the villagers had been warned and only some escaped.
D. Only some of the villagers had been warned and all escaped.
Question 37. It isn't just that the level of education of this school is high. It's that it's also been consistent
for years.
A. The level of education in this school, which is usually quite high, shows only slight variations from year
to year.
B. The standard of education is not high in this school, but at least all the students are at the same level.
C. Not only are the standards of education good in this school, but it has maintained those standards over the
years.
D. It isn't fair to deny that this school is successful, as it has had the same high standards for many years now.
Question 38. No one but the experts was able to realize that the painting was an imitation. It greatly
resembled the original.
A. It was obvious that only a person with great talent could fake a painting so successfully.
B. It was hard for ordinary people to judge between the fake painting and the real one, but not for the experts.
C. It was almost impossible for amateurs to realize that the painting was not authentic, though the experts
could judge it quite easily.
D. The painting looked so much like the authentic one that only the experts could tell it wasn't genuine.
Question 39: The basketball team knew they lost the match. They soon started to blame each other.
A. Hardly had the basketball team known they lost the match when they started to blame each other.
B. No sooner had the basketball team started to blame each other than they knew they lost the match.
C. As soon as they blamed each other, the basketball team knew they lost the match.
D. Not only did the basketball team lose the match but they blamed each other as well
Question 40: We chose to find a place for the night. We found the bad weather very inconvenient.
A. Bad weather was approaching, so we started to look for a place to stay.
B. The bad weather prevented us from driving any further.
C. Seeing that the bad weather had set in, we decided to find somewhere to spend the night.
D. Because the climate was so severe, we were worried about what we'd do at night
Question 41: He had finished the report. He submitted it to the director.
A. Finishing the report, it was submitted to the director.
B. Having finished the report, it was submitted to the director.
C. Having finished the report, he submitted to the director.

Hocmai– Ngôi trường chung của học trò Việt Tổng đài tư vấn: 1900 69 33 - Trang | 7-

187
Hocmai.vn – Website học trực tuyến số 1 tại Việt Nam
Khóa học Luyện thi THPT quốc gia PEN C –N3 Môn Tiếng Anh côHương Fiona

D. Having finished the report, he submitted it to the director.


Question 42: The plan may be ingenious. It will never work in practice.
A. Ingenious as it may be, the plan will never work in practice.
B. Ingenious as may the plan, it will never work in practice.
C. The plan may be too ingenious to work in practice.
D. The plan is as impractical as it is ingenious.
Question 43: John was not here yesterday. Perhaps he was ill.
A. John needn't be here yesterday because he was ill.
B. Because of his illness, John shouldn't have been here yesterday.
C. John might have been ill yesterday, so he was not here.
D. John must have been ill yesterday, so he was not here.
Question 44: "Cigarette?", he said. "No, thanks.", I said.
A. He asked for a cigarette, and I immediately refitted.
B. He mentioned a cigarette, so I thanked him.
C. He offered me a cigarette, but I promptly declined.
D. He asked if I was smoking, and I denied at once.
Question 45: It was an interesting novel. I stayed up all night to finish it.
A. I stayed up all night to finish the novel so it was interesting.
B. Unless it were an interesting novel, I would not stay up all night to finish it
C. Though it was an interesting novel, I stayed up all night to finish it.
D. So interesting was the novel that I stayed up all night to finish it
Question 46: We arrived at the airport. We realized our passports were still at home.
A. It was until we arrived at the airport that we realize our passports were still at home.
B. We arrived at the airport and realized that our passports are still at home.
C. Not until had we arrived at the airport, we realized our passports were still at home.
D. Not until we arrived at the airport, did we realize that our passports were still at home
Question 47: John is studying hard. He doesn’t want to fail the exam.
A. John is studying hard in Oder not to fail the next exam
B. John is studying hard in Oder that he not fail the next exam
C. John is studying hard so as to fail the next exam
D. John is studying hard in Oder to not to fail the next exam
Question 48: She gave in her notice. She planned to start her new job in January
A. She gave in her notice, plan to start her new job in January

Hocmai– Ngôi trường chung của học trò Việt Tổng đài tư vấn: 1900 69 33 - Trang | 8-

188
Hocmai.vn – Website học trực tuyến số 1 tại Việt Nam
Khóa học Luyện thi THPT quốc gia PEN C –N3 Môn Tiếng Anh côHương Fiona

B. She gave in her notice with a view to starting her new job in January
C. Her notice was given in with an aim to start her new job in January
D. Her notice was given in order for her to start her new job in January.
Question 49: Joe does a lot of exercise. He’s still very fat
A. Despite the fact that doing a lot of exercise, Joe’s still very fat
B. Joe does a lot of exercise, so he’s very fat
C. Even though joe does a lot of exercise, he’s very fat.
D. Joe’s very fat, but he does a lot of exercise.
Question 50: Canada does not require us citizens to obtain passports to enter the country. Mexico does
not require US citizens to do the same.
A. Canada does not require us citizens to obtain passports to enter the country, and Mexico does either
B. Canada does not require us citizens to obtain passports to enter the country, and Mexico does not, either
C. Canada does not require us citizens to obtain passports to enter the country, and neither Mexico does
D. Canada does not require us citizens to obtain passports to enter the country while Mexico does
Question 51. I do my homework and schoolwork in separate books. I don't get muddled up.
A. Having two separate books at home and at work helps me avoid getting muddled up.
B. I do my homework and schoolwork in separate hooks so that I don't get muddled up.
C. I do not get muddled up due to the separation between homework and schoolwork.
D. I would get muddled up if I did not separate homework from schoolwork.
Question 52. Some economists argue that new technology causes unemployment. Others feel that it
allows more jobs to be created.
A. Some economists argue that new technology causes unemployment, so others feel that it allows more jobs
to be created.
B. Arguing that new technology causes unemployment, other economists feel that it allows more jobs to be
created.
C. Besides the argument that new technology causes unemployment, some economists feel that it allows more
jobs to be created.
D. Some economists argue that new technology causes unemployment, whereas others feel that it allows more
jobs to be created.

Nguồn : Hocmai.vn
Giáo viên : Hương Fiona

Hocmai– Ngôi trường chung của học trò Việt Tổng đài tư vấn: 1900 69 33 - Trang | 9-

189
Hocmai.vn – Website học trực tuyến số 1 tại Việt Nam
Khóa học Luyện thi THPT quốc gia PEN C –N3 Môn Tiếng Anh côHương Fiona

Hocmai– Ngôi trường chung của học trò Việt Tổng đài tư vấn: 1900 69 33 - Trang | 10-

190
Hocmai.vn – Website học trực tuyến số 1 tại Việt Nam
Khóa học Luyện thi THPT quốc gia PEN C –N3 Môn Tiếng Anh côHương Fiona

PHƯƠNG PHÁP CHUNG LÀM BÀI HOÀN THÀNH CÂU


(BÀI TẬP TỰ LUYỆN )
Giáo viên : Nguyễn Thanh Hương
Các bài tập trong tài liệu này được biên soạn kèm theo bài giảng “Phương pháp chung làm bài hoàn thành câu” thuộc Khóa học Luyện
thi THPT quốc gia PEN - C: Môn Tiếng Anh (cô Hương Fiona )” tại website Hocmai.vn để giúp các Bạn kiểm tra, củng cố lại các kiến
thức được giáo viên truyền đạt trong bài giảng tương ứng. Để sử dụng hiệu quả, Bạn cần học trước bài giảng, sau đó làm đầy đủ các bài
tập trong tài liệu này.

Mark the letter A, B, C or D on your answer sheet to indicate the correct answer to each of the following
questions.
Question 1: Since he failed his exam, he had to_________for it again.
A. pass B. make C. take D. sit
Question 2: Is there_________at all I can help?
A. everything B. anything C. something D. one thing
Question 3: Don’t worry. He’ll do the job as_________as possible.
A. economizing B. econimic C. economical D. economically
Question 4: Only when you grow up _________the truth.
A. you will know B. you know C. do you know D. will you know
Question 5: My brother left his job last week because he did not have any_________to travel.
A. position B. chance C. ability D. location
Question 6: John paid $2 for his meal, _________he had thought it would cost.
A. not as much B. not so much as C. less as D. not so many as
Question 7: It is very important for a film or a company to keep_________the changes in the market.
A. pace of B. track about C. touch with D. up with
Question 8: I’m sure you’ll have no_________the exam.
A. difficulty passing B. difficulties to pass
C. difficulty to pass D. difficulties of passing
Question 9: I _________this letter around for days without looking at it.
A. carry B. must carry C. have been carrying D. am carrying
Question 10: Vietnam’s rice export this year will decrease_________about 10%, compared with that of last
year.
A. with B. at C. by D. on
Question 11: I won’t change my mind_________what you say.
A. whether B. no matter C. because D. although
Question 12: My car isn’t_________. It’s always letting me down.
A. believable B. reliable C. colorable D. conceivable

Question 13: Many applicants find a job interview__________ if they are not well-prepared for it.

Hocmai– Ngôi trường chung của học trò Việt Tổng đài tư vấn: 1900 69 33 - Trang | 1-

191
Hocmai.vn – Website học trực tuyến số 1 tại Việt Nam
Khóa học Luyện thi THPT quốc gia PEN C –N3 Môn Tiếng Anh côHương Fiona

A. impressive B. stressful C. threatening D. time-consuming


Question 14: The manager__________ him for a minor mistake.
A. accused B. charged C. complained D. blamed
Question 15: I __________ hurry. It’s nearly 8.00, and my first class starts at 8.15.
A. would prefer B. can’t help C. would rather D. had better
Question 16: He managed to keep his job__________ the manager had threatened to sack him.
A. although B. despite C. unless D. therefore
Question 17: Don’t touch that wire or you’ll get an electric__________.
A. shock B. fire C. charge D. current
Question 18: The car had a(n) __________ tire, so we had to change the wheel.
A. bent B. flat C. cracked D. injured
Question 19: Does television adequately reflect the ethnic and cultural__________ of the country.
A. costom B. diversity C. alternations D. article
Question 20: You should make a(n) __________ to overcome this problem.
A. trial B. impression C. effort D. apology
Question 21: –“You look beautiful with your new hairstyle!”
– “____________”.
A. Not at all B. It’s kind of you to say so
C. Very kind of your part D. Willingly
Question 22: It was only__________ he told me his surname that I realized that we had been to the same
school.
A. then B. until C. as soon as D. when
Question 23: The girl __________ design had been chosen stepped to the platform to receive the award.
A. whose B. whom C. that
D. which
Question 24: My responsibility is to wash dishes and__________ the garbage.
A. take care of B. take out C. take off D. take over
Question 25: Don’ t forget to ________ the alarm clock for 5 o‟clock tomorrow morning.
A. ring B. put C. wind D. set
Question 26: Books are no longer the only _________ of stories and information.
A. basis B. site C. source D. style
Question 27: ________ for our health.
A. One’s diet is helpful in extra fiber. B. Helpful one‟s diet is extra fiber
C. Extra fiber is one‟s helpful diet D. Extra fiber in one‟s diet is helpful
Question 28: The growth of two-income families in the United States ______ of people moving to a new
social class.
A. has resulted in millions B. resulting in millions
C. results of millions D. millions of results

Hocmai– Ngôi trường chung của học trò Việt Tổng đài tư vấn: 1900 69 33 - Trang | 2-

192
Hocmai.vn – Website học trực tuyến số 1 tại Việt Nam
Khóa học Luyện thi THPT quốc gia PEN C –N3 Môn Tiếng Anh côHương Fiona

Question 29: Black, red, and even bright pink diamonds ______ .
A. occasionally found B. have occasionally been found
C. have occasionally found D. occasionally to find
Question 30: An adviser to both Franklin Delano Roosevelt and Harry Truman, _____ of Bethune-Cook
man College.
A. the founder was Dr, Mary Mcleod Bethune
B. did the founder Dr, Mary Mcleod Bethune
C. Dr. Mary Mcleod Bethune, who was the founder
D. Dr. Mary Mcleod Bethune was the founder
Question 31: Before _______ , they used horse drawn wooden carts
A. farmers had tractors B. farmers have had tractors
C. tractors owned by farmers D. having tractors farmers
Question 32: The door is unlocked; _______ here last night.
A. Something strange was happened
B. Something strange should have happened
C. Something strange had happened
D. Something strange could have happened
Question 33: Although he supports the Council, he does not take an active _____ in politics.
A. affair B. play C. part D. charge
Question 34: Why don‟t you wear that blue dress of yours? It _______ you.
A. agrees B. goes with C. suits D. watches
Question 35: Although he claims to have left his job voluntarily, he was actually______ for misconduct.
A. dismissed B. dispelled C. resigned D. released
Question 36: Because aluminum is lighter and cheaper_______, it is frequently used for high tension
power transmission.
A. as copper B. more copper C. for copper D. than copper
Question 37: Since the flood the number of homeless people _______ dramatically.
A. are increasing B. had increased C. increase D. has increased
Question 38: While everybody else in our class prefers working in groups, Mina likes working ______
A. on herself B. on her own C. of her ow D. in herself
Question 39: ________, the young mother appeared visibly very happy after the birth of her child.
A. Tired as she was B. She was tired C. As tired D. Despite tired
Question 40: Could you please tell me __________ ?
A. where does my uncle's room B. where is my uncle’s room
C. where my uncle’s room is D. where my uncle's room

Hocmai– Ngôi trường chung của học trò Việt Tổng đài tư vấn: 1900 69 33 - Trang | 3-

193
Hocmai.vn – Website học trực tuyến số 1 tại Việt Nam
Khóa học Luyện thi THPT quốc gia PEN C –N3 Môn Tiếng Anh côHương Fiona

Nguồn : Hocmai.vn
Giáo viên : Hương Fiona

Hocmai– Ngôi trường chung của học trò Việt Tổng đài tư vấn: 1900 69 33 - Trang | 4-

194
Hocmai.vn – Website học trực tuyến số 1 tại Việt Nam
Khóa học Luyện thi THPT quốc gia PEN C –N3 Môn Tiếng Anh côHương Fiona

PHƯƠNG PHÁP CHUNG LÀM BÀI HOÀN THÀNH ĐOẠN VĂN


( BÀI TẬP TỰ LUYỆN )
Giáo viên : Nguyễn Thanh Hương
Các bài tập trong tài liệu này được biên soạn kèm theo bài giảng “Phương pháp chung làm bài hoàn thành đoạn văn” thuộc Khóa học
Luyện thi THPT quốc gia PEN - C: Môn Tiếng Anh (cô Hương Fiona )” tại website Hocmai.vn để giúp các Bạn kiểm tra, củng cố lại
các kiến thức được giáo viên truyền đạt trong bài giảng tương ứng. Để sử dụng hiệu quả, Bạn cần học trước bài giảng, sau đó làm đầy
đủ các bài tập trong tài liệu này.

I. Read the following passage and mark the letter A, B, C, or D on your answer sheet to indicate
the correct answer to each of the blanks.
When the weather is cold, it is not very much fun to (1)______ a bus. People have to
stand at a bus-stop for minutes or even hours. They talk about the bus and (2) _______
about the weather. Most of them feel cold. Some have to drink coffee to stay warm.
Traveling by bus (3) ___ pollution, but people often would rather drive their cars. Many
people are not used to the bus schedules, and they do not like to wait.
On the other hand, many people have been (4) _ the bus every day for many years. They
are used to it. They say the bus has been coming on time every day, and they have never
been late for work. In (5) ______, they do not need a parking place in all that time. Buses
are very convenient when you are used to them.
Question 1:A. search B. look for C. stand for D. wait for
Question 2:A. tell B. complain C. comment D. judge
Question 3:A. cuts B. reduces C. stops D. eliminates
Question 4:A. looking B. waiting C. getting D. taking
Question 5:A. end B. last C. addition D. conclusion
II. Read the following passage and mark the letter A,B,C or D on your answer sheet to indicate
the correct word or phrase that best fits each of the numbered blanks.
A trend that has emerged recently is the sharing of childcare (1) __________ between husband and
wife. Young couples will try to arrange their work schedules so that they work opposite hours or
shifts in order that one parent is always home with the children. Since childcare is expensive, this
saves money for the young couple trying to establish themselves and provide a secure environment
for the family. Husband and wife may also share household chores. Some fathers are just as capable
as mothers at cooking dinner, changing and bathing the baby, and doing the laundry.
In some cases, the woman’s salary is for family (2) __________ and the father becomes the
“househusband." These cases are still fairly rare. One positive trend, however, is that fathers seem
to be spending more time with their children. In a recent survey, 41% of the children sampled said
they spend equal time with their mothers and fathers. “This is one of our most significant cultural
changes,” says Dr. Leon Hoffman, who co-directs the Parent Child Center at the New York

Hocmai– Ngôi trường chung của học trò Việt Tổng đài tư vấn: 1900 69 33 - Trang | 1-

195
Hocmai.vn – Website học trực tuyến số 1 tại Việt Nam
Khóa học Luyện thi THPT quốc gia PEN C –N3 Môn Tiếng Anh côHương Fiona

Psychoanalytic Society. In practice, for over 30 years, Hoffman has found "a very dramatic
difference in the involvement of the father in everything from care-taking to general decision (3)
__________ around kids' lives.”
Another factor has recently been added to the childcare formula. The number of people who work
from home nearly full-time rose 23% from the last decade. The (4) _________ of technology -
computers, faxes, teleconferencing - has made it easier for at-home workers to be constantly in
touch. Will this new flexibility in the workforce bring a positive change for the (5) __________ of
children? Only time will tell.
Question1: A. abilities B.possibilities C.techniques D.responsibilities
Question2: A. payment B. expenses C. fares D. fees
Question3: A. making B. creating C. holding D. giving
Question4: A. accessible B. accessibly C. access D. accessibility
Question5: A.well being B. security C. comfort D. interests
III. Read the following passage and mark the letter A, B, C, or D on your answer sheet to indicate
the correct word or phases that best fits each of the numbered blanks from 1 to 5.
People have always dreamt of living forever. Although we all know this will never happen, we will
want to live as long as possible.
__(1)__, there are advantages and disadvantages of a long life.In the first place, people who live longer
can spend more time with their family and friends. Secondly, __(2)__have busy working lives look
forward to a long, relaxing life, when they can do the things they‘ve never had time for.__(3)__, there
are some serious disadvantages. Firstly, many people become ill and consequently have to spend time
in hospital or become burden __(4)__ the children and friends. Many of them find this dependence
annoying or embarrassing. In addition to this, __(5)__ the fewer friends they seem to have because old
friends die or become and it’s often difficult to make new friends.
To sum up, living to a very old age is worthwhile for those who stay healthy enough to remain
independent and enjoy life.
Question 1:A. Naturally B. Really C. Consequently D. Surprisingly
Question 2:A. people who B. people C. people D. everyone who
Question 3:A. In other words B. For example C. On the other hand D. Contrary to
Question 4:A. of B. for C. to D. with
Question 5:A. when people get older B. when the older people get
C. the old people get D. the older people get
IV. Read the following passage and choose A, B, C, or D to indicate the correct answer to each of
the questions.
Tyler Perry

Hocmai– Ngôi trường chung của học trò Việt Tổng đài tư vấn: 1900 69 33 - Trang | 2-

196
Hocmai.vn – Website học trực tuyến số 1 tại Việt Nam
Khóa học Luyện thi THPT quốc gia PEN C –N3 Môn Tiếng Anh côHương Fiona

Perry had a rough childhood. He was physically and sexually abused growing up, got kicked out of
high school, and tried to commit suicide ___(1)___once as a preteen and again at 22. At 23 he move
to Atlanta and took ___(2)___ odd jobs as he started working on his stage career.
In 1992 he wrote, produced, and starred in his first theater ___(3)___, I Know I’ve Been Changed,
somewhat informed by his difficult upbringing. Perry put all his savings into the show and it failed
miserably; the run lasted just one weekend and only 30 people came to watch. He kept up with the
production, working more odd jobs and often slept in his car to get by. Six years later, Perry finally
___(4)___ through when, on its seventh run, the show became a success. He‘s since gone on to have
an extremely successful career ___(5)___ a director, writer, and actor. In fact, Perry was named Forbes’
highest paid man in the field.
Question 1: A. twice B. two C. second D. double
Question 2: A. up B. off C. in D. to
Question 3: A. producer B. productivity C. production D. productive
Question 4: A. went B. broke C. got D. put
Question 5: A. same B. as C. like D. as soon as
V. Read the following passage and mark the letter A, B,C or D on your answer sheet to indicate the
correct word or phrase that best first each of the numbered blank from 1 to 5

American folk music originated with (1) _____ people at a time when the rural population was isolated
and music was not (2)______spread by radio, records, or music video. It was (3)_____by oral
traditional and is noted for its energy ,humor, and emotional impact. The major source of aerly
American folk songs was music from the Bristish Isles , but songs from Africa as songs of the American
Indians have significant part in its heritage. Later settler from other countries also contributed songs.
In the nineteenth century, composer Steven Foster wrote some of the most enduringly popular of all
American songs ,(4)______soon became part of the folk tradition. Beginning in the 1930s , Woody
Guthrie gained great popularity by adapting melodies and lyrics and supplying new ones as well. In
the 1950s and 1960s , singer – composers such as Peter Seeger , Bob Dylan , Joan Baez continued this
tradition by „urban’ folk music. Many of these songs deal (5)_____important social issue, such as
racial intergration and the war in Vietnam.
Question 1:A. typical B. ordinary C. common D. popular
Question 2:A. yet B. still C. until D. even
Question 3:A. transferred B. transited C. transmitted D. transformed
Question 4:A. who B. which C. that D. this
Question 5:A. with B. in C. by D. at
VI. Read the following passage and circle the letter A, B, C or D on your answer sheet to indicate
the correct word for each of the blanks .
When you read something in a foreign language, you frequently come across words you do not fully
understand. Sometimes you (1) ______ the meaning in a dictionary and sometimes you guess. The

Hocmai– Ngôi trường chung của học trò Việt Tổng đài tư vấn: 1900 69 33 - Trang | 3-

197
Hocmai.vn – Website học trực tuyến số 1 tại Việt Nam
Khóa học Luyện thi THPT quốc gia PEN C –N3 Môn Tiếng Anh côHương Fiona

strategy you adopt depends very much upon the degree of accuracy you require and the time at your
disposal.
If you are the sort of person who tends to turn to the dictionary frequently, it is (2) ______ remembering
that every dictionary has its limitations. Each definition is only an approximation and one builds up an
accurate picture of the meaning of a word only after meeting it in a (3) ______ of contexts. It is also
important to recognize the special dangers of dictionaries that translate from English into your native
language and vice versa. If you must use a dictionary, it is usually far safer to consult an English-
English dictionary.
In most exams you are not permitted to use a dictionary. (4) ______ you are allowed to use one, it is
very time-consuming to look up words, and time in exams is usually limited. You are, therefore , forced
to guess the meaning of unfamiliar words.
When you come across unknown words in an exam text, it is very easy to panic. However, if you
develop efficient techniques for guessing the meaning, you will overcome a number of possible
problems and help yourself to understand far more of the text than you at first thought likely.
Two strategies which may help you guess the meaning of a word are: using contextual clues, both
within the sentence and outside, and making use of clues (5) ______ from the formation of the word

Question 1:A. control B. inspect C. check D. examine


Question 2:A. valuable B. worth C. essential D. vital
Question 3:A. variation B. multiple C. diversity D. variety
Question 4:A. Even if B. Provided C. Although D. In case
Question 5:A. originated B. extracted C. derived D. coming

VII. Read the following passage and mark the letter A, B, C, or D on your answer sheet to indicate
the correct word or phrase that best fits each of the numbered blanks from 1 to 5.

We know what happens when we don't have water - we die. We use water, other animals use it, plants
use it... but the world never loses it. Basically, the (1) _______ of water on, above and within the planet
never changes.
Our use of it is part of an immense (2) _______, known simply enough as the water cycle. The cycle
can happen because water is one of the few substances that exists naturally, and at temperatures which
people can tolerate, as a liquid, a gas and a solid.
We all know something about the water cycle; it is part of our daily scene. Water (3) _______ from
lakes and oceans and from the leaves of plants. From there it forms clouds which come back to earth
(4) _______ rain, ice or snow. That water soaks into the ground or runs off. And the cycle continues
with rivers running into the sea, plants transpiring and so on. Animals like us use it on the way through
the cycle.

Hocmai– Ngôi trường chung của học trò Việt Tổng đài tư vấn: 1900 69 33 - Trang | 4-

198
Hocmai.vn – Website học trực tuyến số 1 tại Việt Nam
Khóa học Luyện thi THPT quốc gia PEN C –N3 Môn Tiếng Anh côHương Fiona

For millions of years, changes in the water cycle were slow, but now, with the (35) _______ increase
in the number of people on earth and the development of technology, we are having a dramatic impact
on the way the water cycle behaves and on the quality of the water itself.
Question 1. A. amount B. number C. sum D. figure
Question 2. A. method B. process C. procedure D. route
Question 3. A. steams B. evaporates C. dissolves D.disperses
Question 4. A. like B. similar C. for D. as
Question 5. A. quick B. abrupt C. rapid D. hasty
VIII. Read the following passage and mark the letter A, B, C, or D on your answer sheet to indicate
the correct word or phrase that best fits each of the numbered blanks from 1 to 5.
Easter is a holiday in late March or early April, the first Sunday after the first full moon after 21 March.
Many people (1)_________it with their family or have a short holiday/ vacation. It is also an important
Christian festival. Easter Sunday, the day of the Resurrection, is the end of Lent (2) ________ the
most important date in the Christian year. Many people who do not go to church at other times go on
Easter Sunday. It was once (3)_________for people to wear new clothes to church on this day. Women
(4)_________new hats, called Easter bonnets. Today, people sometimes make elaborately decorated
Easter bonnets for fun. A few people send Easter cards with religious symbols on them or pictures of
small chickens, lambs, rabbits and spring flowers, all traditionally associated (5)_________Easter.
Question 1. A. take B. use C. spend D. expend
Question 2. A. for B. as C. and D. nor
Question 3. A. familiar B. ordinary C. common D. regular
Question 4. A. put on B. take on C. bore D. wore
Question 5. A. toward B. with C. to D. from
IX. Read the following passage and mark the letter A, B, C, or D on your answer sheet to indicate
the correct word or phrase that best fits each of the numbered blanks from 1 to 5.
Early writing and Alphabets
When people first began to write, they did not use an alphabet. Instead, they drew small pictures to (1)
_______ the objects they were writing about. This was very slow because there was a different picture
for any word.
The Ancient Egyptians had a system of picture writing that was described hieroglyphics. The meaning
of this writing was forgotten for a very long time but in 1799 some scientists (2) _______ a stone near
Alexandria, in Egypt. The stone had been there for (3) _______ a thousand years. It had both Greek
and hieroglyphics on it and researchers were finally able to understand what the hieroglyphics meant.
An alphabet is quite different from picture writing. It (4) _______ of letters or symbols that represent
a sound and each sound is just part of one word. The Phoenicians, who lived about 3,000 years ago,
developed the modern alphabets. It was later improved by the Roman’s and this alphabet is now used
(5) _______ throughout the world.
Question 1:A. notice B. show C. appear D. mark
Hocmai– Ngôi trường chung của học trò Việt Tổng đài tư vấn: 1900 69 33 - Trang | 5-

199
Hocmai.vn – Website học trực tuyến số 1 tại Việt Nam
Khóa học Luyện thi THPT quốc gia PEN C –N3 Môn Tiếng Anh côHương Fiona

Question 2:A. discovered B. realized C. delivered D. invented


Question 3:A. quite B. more C. over D. already
Question 4:A. consistsof B. includes C. contains D. involves
Question 5:A. broadly B. widely C. deeply D. hugely

Nguồn : Hocmai.vn
Giáo viên : Hương Fiona

Hocmai– Ngôi trường chung của học trò Việt Tổng đài tư vấn: 1900 69 33 - Trang | 6-

200
Hocmai.vn – Website học trực tuyến số 1 tại Việt Nam
Khóa học Luyện thi THPT quốc gia PEN C –N3 Môn Tiếng Anh côHương Fiona

PHƯƠNG PHÁP LÀM BÀI ĐỌC HIỂU


( BÀI TẬP TỰ LUYỆN )
Giáo viên : Nguyễn Thanh Hương
Các bài tập trong tài liệu này được biên soạn kèm theo bài giảng “Phương pháp làm bài đọc hiểu” thuộc Khóa học Luyện thi THPT quốc
gia PEN - C: Môn Tiếng Anh (cô Hương Fiona )” tại website Hocmai.vn để giúp các Bạn kiểm tra, củng cố lại các kiến thức được giáo
viên truyền đạt trong bài giảng tương ứng. Để sử dụng hiệu quả, Bạn cần học trước bài giảng, sau đó làm đầy đủ các bài tập trong tài
liệu này.

Reading 1.
Read the following passage and mark the letter A, B, C or D on your answer sheet to indicate the correct
answer to each of the questions that follow.
In the primary school, a child is in a comparatively simple setting and most of the time forms and
relationships with one familiar teacher. On entering secondary school, a new world opens up and frequently
it is much more difficult world. The pupil soon learns to be less free in the way he speaks to teachers and
even to his fellow pupils. He beins to lose radually the free and easy ways of the primary school, for he
senses the need for a more cautious approach in the secondary school where there are older pupils. Secondary
staff and pupils suffer from the pressures of academic work and seem to have less time to stop and talk.
Teachers with specialist roles may see hundreds of children in a week, and a pupil may be able to form
relationships with very few of the staff.
He has to decide which adults are approachable; good schools will make clear to every young person from
the first year what guidance and personal help is available - but whether the reality of life in the institution
actually encourages requests for help is another matter.
Adults often forget what a confusing picture school can offer to a child. He sees a great deal of movement,
a great number of people - often rather frightening - looking people - and realizes that an increasing number
of choices and decisions have to be made. As he progresses through the school the confusion may become
less but the choices and decisions required will increase. The school will rightly expect the pupil to take the
first steps to obtain the help he needs, for this is the pattern of adult of life for which he has to be prepared
but all the time the opportunities for personal and group advice must be presented in a way which makes
them easy to understand and within easy reach of pupils.
Question 1: According to the passage one of the problems for pupils entering secondary school is
that ...........
A. they are taught by many different teachers.
B. they do not attend lessons in every subject.
C. the teachers do not want to be friendly.
D. the teachers give most attention to the more academic pupils.
Question 2: The teachers at secondary school do not talk much to the pupils because ................
A. they want to keep a certain distance with the pupils.
B. they are too busy with their academic work.
C. the pupils are afraid of them.
D. it is the regulation of the school.
Question 3: In secondary schools every pupil having problems should................
Hocmai– Ngôi trường chung của học trò Việt Tổng đài tư vấn: 1900 69 33 - Trang | 1-

201
Hocmai.vn – Website học trực tuyến số 1 tại Việt Nam
Khóa học Luyện thi THPT quốc gia PEN C –N3 Môn Tiếng Anh côHương Fiona

A. know how to ask for help.


B. be freed from any pressure of academic work.
C. be able to discuss his problems in class.
D. be able to discuss his problems with any teacher.
Question 4: What will the school rightly expect the pupils to do?
A. they firstly obtain the help. B. they think carefully.
C. they understand well. D. they study hard.
Question 5: The word “adults” in the second paragraph has the nearest meaning with ...................
A. the pupils at secondary school. B. the secondary pupils’ parents
C. the staff at secondary school. D. the teachers and pupils at secondary school.
Question 6: How many pupils do teachers see in a week?
A. one pupil B. A few of pupils C. Hundreds of pupils D. Many pupils
Question 7: Who do the pupils make relationship with?
A. Few of the staff B. Other parents C. Other pupils D. A few of the staff
Question 8: In this passage about secondary schools, the author is mainly concerned about ................
A. academic standards B. the role of specialist teachers.
C. the training of the individual teachers. D. the personal development of pupils.

Reading 2.

Read the following passage and mark the letter A, B, C or D on your answer sheet to indicate the correct
answer to each of the questions that follow.
Although only a small percentage of the electromagnetic radiation that is emitted by the Sun is ultraviolet
(UV) radiation, the amount that is emitted would be enough to cause severe damage to most forms of life on
Earth were it all to reach the surface of the Earth. Fortunately, all of the Sun’s ultraviolet radiation does not
reach the Earth because of a layer of oxygen, called the ozone layer encircling the Earth in the stratosphere
at an altitude of about 15 miles above the Earth. The ozone layer absorbs much of the Sun’s ultraviolet
radiation and prevents it from reaching the Earth.
Ozone is a form of oxygen in which each molecule consists of three atoms (O3) instead of the two atoms
(O2) usually found in an oxygen molecule. Ozone forms in the stratosphere in a process that is initiated by
ultraviolet radiation from the Sun. UV radiation from the Sun splits oxygen molecules with two atoms into
free oxygen atoms, and each of these unattached oxygen atoms then joins up with an oxygen molecule to
form ozone. UV radiation is also capable of splitting up ozone molecules; thus, ozone is constantly forming,
splitting, and reforming in the stratosphere. When UV radiation is absorbed during the process of ozone
formation and reformation, it is unable to reach Earth and cause damage there.
Recently, however, the ozone layer over parts of the Earth has been diminishing. Chief among the culprits
in the case of the disappearing ozone, those that are really responsible, are the chlorofluorocarbons (CFCs).
CFCs meander up from Earth into the stratosphere, where they break down and release chlorine. The released
chlorine reacts with ozone in the stratosphere to form chlorine monoxide (CIO) and oxygen (O2). The
chlorine then becomes free to go through the cycle over and over again. One chlorine atom can, in fact,

Hocmai– Ngôi trường chung của học trò Việt Tổng đài tư vấn: 1900 69 33 - Trang | 2-

202
Hocmai.vn – Website học trực tuyến số 1 tại Việt Nam
Khóa học Luyện thi THPT quốc gia PEN C –N3 Môn Tiếng Anh côHương Fiona

destroy hundreds of thousands of ozone molecules in this repetitious cycle, and the effects of this destructive
process are now becoming evident.
Question 1: According to the passage, ultraviolet radiation from the Sun..............
A. is causing severe damage to the Earth's ozone layer
B. is only a fraction of the Sun's electromagnetic radiation
C. creates electromagnetic radiation
D. always reaches the Earth
Question 2: The word “encircling” in paragraph 1 is closest in meaning to…………….
A. attacking B. raising C. rotating D. surrounding
Question 3: It is stated in the passage that the ozone layer……………
A. enables ultraviolet radiation to reach the Earth
B. reflects ultraviolet radiation
C. shields the Earth from a lot of ultraviolet radiation
D. reaches down to the Earth
Question 4: The pronoun “it” in paragraph 2 refers to……………….
A. radiation B. process C. formation D. damage
Question 5: The word “culprits'' in paragraph 3 is closest in meaning to which of the following?
A. Guilty parties B. Detectives C. Group members D. Leaders
Question 6: According to the passage, what happens after a chlorine molecule reacts with an ozone
molecule?
A. The ozone breaks down into three oxygen atoms.
B. Two different molecules are created.
C. The two molecules combine into one molecule.
D. Three distinct molecules result.
Question 7: The paragraph following the passage most likely discusses……….........
A. the negative results of the cycle of ozone destruction
B. where chlorofluorocarbons (CFCs) come from
C. the causes of the destruction of ozone molecules
D. how electromagnetic radiation is created

Reading 3.
Read the following passage and mark the letter A, B, C or D to indicate the correct answer to each of
the questions.
Although people drive in all countries, the rules can be quite different between nations and areas. For this
reason, you should always learn the laws before you decide to drive in a foreign country. If you are not
careful, you can get into trouble since the rules might be very different from what you are used to. For
example, you can find some big differences in laws related to the minimum driving age, the appropriate side
of the road to drive on, and mobile phone use while driving.
In the U.S., people who live in Alaska may obtain a learner’s permit (legal permission to drive while being
supervised) at the age of fourteen. This is quite different from the driving laws of Niger, where a person
must be at least twenty-three years old to drive. If you are inquisitive to know the minimum driving age in

Hocmai– Ngôi trường chung của học trò Việt Tổng đài tư vấn: 1900 69 33 - Trang | 3-

203
Hocmai.vn – Website học trực tuyến số 1 tại Việt Nam
Khóa học Luyện thi THPT quốc gia PEN C –N3 Môn Tiếng Anh côHương Fiona

most countries, the answer is eighteen. It may also surprise you to learn that not every country allows its
citizens to drive, although most of them do. For example, women are not allowed to drive in Saudi Arabia
no matter how old they are. Instead, they must have a male family member or hired male driver to travel by
car.
Driving on the right or left side of the road also varies. For instance, in Great Britain, Cyprus, Australia,
India, and Malaysia people drive on the left. However, in the U.S., Mexico, France, and Canada people are
required to drive on the right. As a matter of fact, one country can have different driving rules for different
areas. People in Hong Kong drive on the left, while drivers in other parts of China use the right side of the
road.
Other driving laws that are different between countries include those related to using mobile phones. In
Japan, using any kind of mobile phone device is illegal, even if you do not need to hold the phone with your
hands. However, in Argentina and Australia, drivers are allowed to talk on their mobile phones as long as
they do not use their hands.
You might also find it interesting to learn that some countries have very unusual laws. For example, in the
country of Cyprus, it is against the law to eat or drink anything while driving. Even more interesting is that
in Germany, there is a famous road called the Autobahn, where certain parts have no speed limit at all!
Question 1: Which of the following is TRUE according to the passage?
A. Females can drive a car in Saudi Arabia.
B. People in India drive on the right side of the road.
C. You can drive as fast as you want on all parts of the German Autobahn.
D. You are not allowed to hold your mobile phone and talk while you drive in Australia.
Question 2: Which of the following is NOT mentioned as a main difference in driving laws in this
passage?
A. Drink driving limit B. Legal driving age
C. Right or left hand traffic D. Safety calls
Question 3: According to the passage, it is illegal to eat or drink while driving in_______.
A. Cyprus B. Great Britain C. China D. Australia
Question 4: It is important to learn the laws before driving in a foreign country because_______.
A. It is fun to learn about other countries’ driving laws.
B. You can get into trouble when you are not used to another country’s rules.
C. You can have a car accident if you do not know the rules.
D. It helps you identify the appropriate side of the road to drive on.
Question 5: The word “inquisitive” in paragraph 2 can be best replaced by _______.
A. indifferent B. concerned C. nosy D. curious
Question 6: The word “those” in paragraph 4 refers to_______.
A. drivers B. mobile phones C. driving laws D. countries
Question 7: What is the main idea of the passage?
A. Driving in all countries shares a common purpose.
B. There are differences in laws related to driving among countries.
C. Countries have different ways to enact laws on driving.
D. People in different countries drive on different sides of the road.
Reading 4
Hocmai– Ngôi trường chung của học trò Việt Tổng đài tư vấn: 1900 69 33 - Trang | 4-

204
Hocmai.vn – Website học trực tuyến số 1 tại Việt Nam
Khóa học Luyện thi THPT quốc gia PEN C –N3 Môn Tiếng Anh côHương Fiona

Read the following passage and mark the letter A, B, C or D to indicate the correct answer to each of
the questions.
There is a strange paradox to the success of the Asian education model. On the one hand, class sizes are
huge by Western standards with between 30 and 40 students per class, in countries like Japan and Korea. On
the other hand, school children in developed Asian economies rank among the highest in the world for
academic achievement in the areas of science and mathematics, especially on standardised tests. Meanwhile,
British secondary school students fail to shine in conditions most educational researchers would say are far
more likely to help them succeed.
Classroom management seems to be easier in places like Korea, and perhaps lessons are more effective as a
direct consequence. After all, we are only too aware of the decline in discipline standards in our own school:
belligerent and disrespectful students appear to be the norm these days. Teachers in Britain seem powerless
to control what happens anymore. Surely this situation cannot create a very effective learning environment,
so perhaps the number of students is far less relevant than is the manner in which they conduct themselves.
But there are other factors to consider, too. There is the home environment. The traditional family unit still
remains relatively intact in Korea. Few children come from broken homes, so there is a sense of security,
safety and trust both at home and at school. In Britain meanwhile, one in every two marriages fails and
divorce rates are sky high. Perhaps children struggle to cope with unstable family conditions and their only
way to express their frustration is by misbehaving at school.
But while the Japanese, Korean and Asian models generally do seem to produce excellent results, the
statistics don’t tell the whole truth. You see, behind those great maths and science scores, there is a quite
remarkable work ethic. Asian students tend to put their education before literally everything else. They do
very few extracurricular activities and devote far more time to their studies than their British peers.
There has been a lot of attention and praise given to these Asian models and their “impressive” statistics of
late. And without question, some of this praise is justified, but it seems to be a case of two extremes in
operation here. At one end, there is the discipline and unbelievably hard work ethic of the Asian students –
success in education before all else. At the other end, British students at times appear careless and extremely
undisciplined by comparison, but at least they Do have the free time to enjoy their youth and explore their
interests. Is either system better outright? Or is it perhaps about time we stopped comparing and started
trying to combine the best bits of both, so that we can finally offer our students a balanced, worthwhile
education
Question 1: The word “They” in paragraph 4 refers to_______.
A. British students B. Asian students C. Korean students D. Japanese students
Question 2: British secondary school students_______.
A. have larger class sizes B. fail at school more than they succeed
C. do better on stadardised tests D. enjoy better classroom conditions
Question 3: What can be implied from the writer’s opinion of the two educational systems discussed?
A. The Asian system is clearly better.
B. The British system is too strict.
C. Neither system is perfect.
D. Both systems are quite satisfactory for different reasons.
Question 4: The traditional family unit_______.
A. is more common in Korean than in Britain
Hocmai– Ngôi trường chung của học trò Việt Tổng đài tư vấn: 1900 69 33 - Trang | 5-

205
Hocmai.vn – Website học trực tuyến số 1 tại Việt Nam
Khóa học Luyện thi THPT quốc gia PEN C –N3 Môn Tiếng Anh côHương Fiona

B. is disappearing in Korean due to high divorce rates


C. is bad for children that come from broken homes
D. is unstable in Korean due to conditions in the home
Question 5: What does the writer mean when he says there is a “paradox” in the Asian education model?
A. There are too many students in each class.
B. You would expect larger classes to get poorer results but they do not.
C. Class sizes are much smaller in other parts of the world.
D. Asian students outperform their peers in other countries.
Question 6: What does the writer suggest might make lessons in Korean schools more successful than in
Britain?
A. Better teacher B. Better school Boards of Management
C. More effective lesson planning D. Better discipline
Question 7: The word “unstable ” in paragraph 3 can be best replaced by_______.
A. unsteady B. unchangeable C. unpredictable D. unimportant
Question 8: According to the writer, Asian students_______.
A. focus too much on recreational activities
B. don’t have as good a work ethic as British ones
C. don’t allow themselves much time to relax and have fun
D. make a big deal of their good results

Reading 5.
Read the following passage and mark the letter A, B, C, or D on your answer sheet to indicate the
correct answer to each of the questions
Ambient divers are, unlike divers who go underwater in submersible vehicles of pressure resistant
suits, exposed to the pressure and temperature of the surrounding ambient water. Of all types of diving, the
oldest and simplest is free diving. Free divers may use no equipment at all, but most use a face mask, foot
fins, and a snorkel. Under the surface, free divers must hold their breath. Most free divers can only descend
30 to 40 feet, but some skilled divers can go as deep as 100 feet
Scuba diving provides greater range than free diving. The word scuba stands for self-contained
underwater breathing apparatus. Scuba divers wear metal tanks with compressed air or other breathing
gases. When using open-circuit equipment, a scuba diver simply breathes air form the tank through a hose
and releases the exhaled air into the water. A closed-circuit breathing device, also called a rebreather, filters
out carbon dioxide and other harmful gases and automatically adds oxygen. This enables the diver to breathe
the same air over and over. In surface-supplied diving, divers wear helmets and waterproof canvas
suits.Today, sophiticated plastic helmets have replaced the heavy copper helmets used in the past.These
divers get their air from a hose connected to compressors on a boat.Surface-supplied divers can go deeper
than any other type of ambient diver.
Question 1: Ambient divers are ones who_____
A. can descend to extreme depths B. use submersible vehicles
C. use no equipment D. are exposed to the surrounding water
Question 2: According to the passage, a free diver may use any of the following EXCEPT______

Hocmai– Ngôi trường chung của học trò Việt Tổng đài tư vấn: 1900 69 33 - Trang | 6-

206
Hocmai.vn – Website học trực tuyến số 1 tại Việt Nam
Khóa học Luyện thi THPT quốc gia PEN C –N3 Môn Tiếng Anh côHương Fiona

A. a rebreather B. a snorkel C. foot fins D. a mask


Question 3: According to the passage, the maximum depth for free divers is around ______
A. 40 feet B. 100 feet C. 200 feet D. 1,000 feet
Question 4: When using closed-circuit devices, divers_______
A. exhale air into the water B. hold their breath
C. breathe the same air over and over D. receive air from the surface
Question 5: What does the word “descend” in the paragraph probably mean?
A. move to a lower level B. climb to a higher place
C. swim on the river bed D. go up and down slowly
Question 6: The word “apparatus” in the passage is closet in meaning to _______
A. clothes B. gases C. mask D. equipment
Question 7: What are the helmets that surface-supplied divers use today made from, according to the
passage?
A. glass B. copper C. plastic D. canvas

Reading 6.

Read the following passage and mark the letter A, B, C, or D on your answer sheet to indicate the
correct answer to each of the questions from 43 to 50
There are a number of natural disasters that can strike across the globe. Two that are frequently linked to one
another are earthquakes and tsunamis. Both of them can cause a great amount of devastation when they hit.
However, tsunamis are the direct result of earthquakes and cannot happen without them.
The Earth has three main parts. They are the crust, the mantle, and the core. The crust is the outer layer of
the Earth. It is not a single piece of land. Instead, it is comprised of a number of plates. There are a few
enormous plates and many smaller ones. These plates essentially rest upon the mantle, which is fluid. As a
result, the plates are in constant – yet slow – motion. The plates may move away from or toward other plates.
In some cases, they collide violently with the plates adjoining them. The movement of the plates causes
tension in the rock. Over a long time, this tension may build up. When it is released, an earthquake happens.
Tens of thousands of earthquakes happen every year. The vast majority are so small that only scientific
instruments can perceive them. Others are powerful enough that people can feel them, yet they cause little
harm or damage. More powerful earthquakes, however, can cause buildings, bridges, and other structures to
collapse. They may additionally injure and skill thousands of people and might even cause the land to change
it appearance.
Since most of the Earth’s surface is water, numerous earthquakes happen beneath the planet’s oceans.
Underwater earthquakes cause the seafloor to move. This results in the displacement of water in the ocean.
When this occurs, a tsunami may form. This is a wave that forms on the surface and moves in all directions
from the place where the earthquake happened. A tsunami moves extremely quickly and can travel thousnads
of kilometres. As it approaches land, the water near the coast gets sucked out to sea. This causes the tsunamis
to increase in height. Minutes later, the tsunami arrives. A large tsunami – one more than ten meters in height
– can travel far inland. As it does that, it can flood the land, destroy human settlements, and kill large numbers
of people.
Question 1: Which of the following statements does paragraph 1 support?
Hocmai– Ngôi trường chung của học trò Việt Tổng đài tư vấn: 1900 69 33 - Trang | 7-

207
Hocmai.vn – Website học trực tuyến số 1 tại Việt Nam
Khóa học Luyện thi THPT quốc gia PEN C –N3 Môn Tiếng Anh côHương Fiona

A. A tsunami happens in tandem with an earthquake.


B. The most severe type of natural disaster is an earthquake.
C. Earthquakes cause more destruction than tsunamis.
D. Earthquakes frequently take place after tsunamis do.
Question 2: The word “it” in bold in paragraph 2 refers to_________.
A. The core B. The crust C. The Earth D. The mantle
Question 3: What is the passage mainly about?
A. When earthquakes are the most likely to happen.
B. What kind of damage natural disasters can cause.
C. How earthquakes and tsunamis occur.
D. Why tsunamis are deadlier than earthquakes.
Question 4: The word “adjoining” in bold in paragraph 2 is closest in meaning to________.
A. residing B. approaching C. bordering D. appearing
Question 5: The word “perceive” in bold in paragraph 3 is closest in meaning to________.
A. detect B. prevent C. comprehend D. locate
Question 6: Which of the following is true regarding the crust?
A. It is the smallest of the Earth’s three layers
B. It is thicker on land than it is under the water.
C. There many separate pieces that make it up
D. The mantle beneath it keeps it from moving too much.
Question 7: Based on the passage, what is probably true about tsunamis?
A. They kill more people each year than earthquakes.
B. They are able to move as fast as the speed of sound.
C. They cannot damage ships sailing on the ocean.
D. They can be deadly to people standing near shore.
Question 8: Which of the following is NOT mentioned in paragraph 3 about earthquakes?
A. How severe the majority of them are B. What kind of damage they can cause
C. How often powerful ones take place D. How many people they typically kill

Reading 7. Read the following passage and mark the letter A, B, C, or D on your answer sheet
to indicate the correct answer to each of the questions from 36 to 42.
Though Edmund Halley was most famous because of his achievements as an astronomer, he was a
scientist of diverse interests and great skill. In addition to studying the skies, Halley was also deeply
interested in exploring the unknown depths of the oceans. One of his lesser-known accomplishments that
were quite remarkable was his design for a diving bell that facilitated exploration of the watery depths.
The diving bell that Halley designed had a major advantage over the diving bells that were in use prior
to his. Earlier diving bells could only make use of the air contained within the bell itself, so divers had to
surface when the air inside the bell ran low. Halley’s bell was an improvement in that its design allowed for
an additional supply of fresh air that enabled a crew of divers to remain underwater for several hours.

Hocmai– Ngôi trường chung của học trò Việt Tổng đài tư vấn: 1900 69 33 - Trang | 8-

208
Hocmai.vn – Website học trực tuyến số 1 tại Việt Nam
Khóa học Luyện thi THPT quốc gia PEN C –N3 Môn Tiếng Anh côHương Fiona

The diving contraption that Halley designed was in the shape of a bell that measured three feet across
the top and five feet across the bottom and could hold several divers comfortably; it was open at the bottom
so that divers could swim in and out at will. The bell was built of wood, which was first heavily tarred to
make it water repellent and was then covered with a half-ton sheet of lead to make the bell heavy enough to
sink in water. The bell shape held air inside for the divers to breathe as the bell sank to the bottom.
The air inside the bell was not the only source of air for the divers to breathe, and it was this improvement
that made Halley's bells superior to its predecessors. In addition to the air already in the bell, air was also
supplied to the divers from a lead barrel that was lowered to the ocean floor close to the bell itself. Air flowed
through a leather pipe from the lead barrel on the ocean floor to the bell. The diver could breathe the air from
a position inside the bell, or he could move around outside the bell wearing a diving suit that consisted ofa
lead bell-shaped helmet with a glass viewing window and a leather body suit, with a leather pipe carrying
fresh air from the diving bell to the helmet.
Question 1. Which of the following best expresses the subject of this passage?
A. Halley’s work as an astronomer
B. Halley’s many different interests
C. Halley’s invention of a contraption for diving
D. Halley's experiences as a diver
Question 2. Halley’s bell was better than its predecessors because it _______.
A. was bigger B. provided more air
C. weighed less D. could rise more quickly
Question 3. The expression ‘ran low’ in paragraph 2 is closest in meaning to _______.
A. moved slowly B. had been replenished
C. sank to the bottom D. was almost exhausted
Question 4. How long could divers stay underwater in Halley’s bell?
A. Just a few seconds B. Only a few minutes
C. For hours at a time D. For days on end
Question 5. It is NOT stated in the passage that Halley's bell _______.
A. was wider at the top than at the bottom
B. was made of tarred wood
C. was completely enclosed
D. could hold more than one diver
Question 6. The expression "at will" in paragraph 3 could best be replaced by _______.
A. in the future B. as they wanted
C. with great speed D. upside down
Question 7. It can be inferred from the passage that, were Halley’s bell not covered with lead it would
_______.
A. float B. get wet
C. trap the divers D. suffocate the divers

Reading 8.

Hocmai– Ngôi trường chung của học trò Việt Tổng đài tư vấn: 1900 69 33 - Trang | 9-

209
Hocmai.vn – Website học trực tuyến số 1 tại Việt Nam
Khóa học Luyện thi THPT quốc gia PEN C –N3 Môn Tiếng Anh côHương Fiona

Read the following passage and mark the letter A, B, C, or D on your answer sheet to indicate
the correct answer to each of the questions from 43 to 50.
The Timber rattlesnakes, once widespread throughout the eastern United States, is now on the
endangered species list and is extinct in two eastern states in which it once thrived. Compared to its Western
cousins the Timber rattlesnakes may be especially vulnerable because of certain behaviors adapted for
coping with the cold climate in which it lives.
Rattlesnakes are generally found in warm climates because, like all reptiles, they cannot generate or
regulate their own body temperature internally and must rely on the sun’s warmth for heat. But Timber
rattlesnakes migrated into colder northern areas about 8,000 years ago when glaciers retreated. In these
northern regions, the snakes developed a number of adaptive strategies to survive, but ultimately these
behaviors make them more vulnerable to human predation, their main threat.
One survival strategy the snakes have developed is hibernation. For approximately eight months of the
year, the rattlers remain motionless in deep frost-free crevices, with their body temperature dropping as low
as 40 degrees. In the spring, when they emerge, they must warm their chilled bodies by sunning for three or
four days on rocks in the open. This behavior coupled with the fact that Timber rattlesnakes tend to
concentrate in large numbers at their wintering sites, make them easy prey. Gestating females are
particularly vulnerable because they spend much of their time basking in the sun in order to produce live
young from eggs. In addition, females have very long interbirth intervals, producing live young only every
three to five years. If a frost or cold spell comes late in the year, the entire litter of 6 to 12 young may die.
Efforts are underway to protect the Timber rattlesnakes and its habitats from further human depredation,
but in many states it is already too late.
Question 1. What is the main topic of the passage?
A. Why Timber rattlesnakes hibernate
B. How Timber rattlesnakes are surviving
C. How Timber rattlesnakes adapted to northern climates
D. Why Timber rattlesnakes are endangered
Question 2. Which of the following is closest in meaning to the word “vulnerable” in the first paragraph?
A. Unprotected B. Impervious C. Insensitive D. Deprived
Question 3. Which of the following in the true about Timber rattlesnakes?
A. They migrated to eastern states
B. They migrated northward during a warming climate
C. They migrated to escape a cold climate
D. They migrated to the South to seek a warmer climate
Question 4. Which of the following could best replace the word "emerge" in the third paragraph?
A. Come out B. Set off C. Get up D. See through
Question 5. In which of the following places might a person be most likely to find Timber rattlesnakes in
the spring?
A. in the woods B. In meadows C. In bushy areas D. In canyons
Question 6. The phrase “easy prey” in the third paragraph could best be replaced by which of the
following?
A. Relaxed B. Protective C. Victims D. Sociable

Hocmai– Ngôi trường chung của học trò Việt Tổng đài tư vấn: 1900 69 33 - Trang | 10-

210
Hocmai.vn – Website học trực tuyến số 1 tại Việt Nam
Khóa học Luyện thi THPT quốc gia PEN C –N3 Môn Tiếng Anh côHương Fiona

Question 7. Which of the following word can best replace the word "basking" in the third paragraph?
A. Washing B. Eating C. Sleeping D. Lying
Question 8. Which of the following does NOT contribute to the rattlesnake being an endangered animal?
A. Hibernating for eight months B. Basking in the sun
C. Congregating together D. Having long intervals between births
Reading 9.

Read the following passage and mark the letter A, B, C, or D on your answer sheet to indicate
the correct answer to each of the questions from 36 to 42.
I read with interest the article on American families. In general, I agree with it, but there are some
important things it left out. It didn’t tell the reader much about the life of a househusband. It’s not an easy
life. I know, because I’m now a househusband myself. A househusband has to change many of his ideas and
his ways.
First of all, he has to change the way he thinks about time. Before I was a househusband, I worked full-
time for the New York Times. I was a reporter, and time was always important. We had to finish our articles
quickly and give them to the editor. Everyone was always in a hurry. This is the way, many other men work
too. Businessmen, lawyers, bankers, and doctors all have to work quickly.
At home it's different. The househusband cannot be in a hurry all the time. If you rush around, you will
make everyone unhappy! The children will be unhappy because they don't understand. For them, time is not
important. Your wife will be unhappy because the children are unhappy. You will be unhappy, too, because
they are all unhappy. So you have to learn to slow down. That is the first and most important rule for a
househusband.
There is something else the househusband must learn. You must learn to show how you feel about
things. At work, men usually do not talk about feelings. If they do, people think they are strange. So, many
men are not used to telling anyone about their feelings. They do not know how to talk about their anger,
worries, or love. But children need to know how you feel. They need to know how much you love them. If
you are angry, they need to know why. Your wife also needs to know about your feelings. If you do not say
anything, your family may get the wrong idea. Then there may be serious problems.
People talk a lot about househusbands these days. Usually they talk about men doing the housework,
the cooking, cleaning, and shopping. But in my opinion, these are the easiest things to learn. It was much
harder for me to change the way I think and the way I act with my family. I think, other men will also find
this harder, but, like me, will find it necessary if they want to have a happy family!
Ted Diamond Hartwell, New Jersey
Question 1. What is this article is mainly about?
A. fathers and children B. life as a househusband
C. ideas about time D. American families
Question 2. At work, most people _______.
A. have to hurry B. take their time
C. have lots of time D. have to slow down
Question 3. The word rush in paragraph 3 is closest in meaning to _______.
A. move about very slowly B. move about hurriedly

Hocmai– Ngôi trường chung của học trò Việt Tổng đài tư vấn: 1900 69 33 - Trang | 11-

211
Hocmai.vn – Website học trực tuyến số 1 tại Việt Nam
Khóa học Luyện thi THPT quốc gia PEN C –N3 Môn Tiếng Anh côHương Fiona

C. shout loudly and angrily D. spend time idly


Question 4. What does the househusband has to learn?
A. to do things more slowly. B. to do things more quickly.
C. the importance of time. D. how to understand his work.
Question 5. According to the passage, children _______.
A. are usually unhappy. B. are always in a hurry.
C. usually don’t think time is important. D. don’t know how to show their feelings.
Question 6. What does the word they in paragraph 4 refers to?
A. strange people at work B. men who talk about feelings
C. men as househusbands D. men who do not talk about feelings
Question 7. It can be inferred from the passage that Ted thinks _______.
A. learning about housework is easy.
B. learning about housework is a problem.
C. cooking is the easiest thing to learn.
D. being a househusband is easy.
Reading 10.
Read the following passage and mark the letter A, B, C, or D on your answer sheet to indicate
the correct answer to each of the questions from 43 to 50.
According to some accounts, the first optical telescope was accidentally invented in the 1600s by
children who put two glass lenses together while playing with them in a Dutch optical shop. The owner of
the shop, Hans Lippershey, looked through the lenses and was amazed by the way they made the nearby
church look so much larger. Soon after that, he invented a device that he called a "looker," a long thin tube
where light passed in a straight line from the front lens to the viewing lens at the other end of the tube. In
1608 he tried to sell his invention unsuccessfully. In the same year, someone described the "looker’’ to the
Italian scientist Galileo, who made his own version of the device. In 1610 Galileo used his version to make
observations of the Moon, the planet Jupiter, and the Milky Way. In April of 1611, Galileo showed his device
to guests at a banquet in his honor. One of the guests suggested a name for the device: telescope.
When Isaac Newton began using Galileo's telescope more than a century later, he noticed a problem.
The type of telescope that Galileo designed is called a refractor because the front lens bends, or refracts the
light. However, the curved front lens also caused the light to be separated into colors. This meant that when
Newton looked through the refracting telescope, the images of bright objects appeared with a ring of colors
around them. This sometimes interfered with viewing. He solved this problem by designing a new type of
telescope that used a curved mirror. This mirror concentrated the light and reflected a beam of light to the
eyepiece at the other end of the telescope. Because Newton used a mirror, his telescope was called a reflector.
Very much larger optical telescopes can now be found in many parts of the world, built on hills and
mountains far from city lights. The world's largest refracting telescope is located at the Yerkes Observatory
in Williams Bay, Wisconsin, Another telescope stands on Mount Palomar in California. This huge reflecting
telescope was for many years the largest reflecting telescope in the world until an even larger reflecting
telescope was built in the Caucasus Mountains. A fourth famous reflector telescope, the Keck Telescope
situated on a mountain in Hawaii, does not use a single large mirror to collect the light. Instead, the Keck
uses the combined light that falls on thirty- six mirrors.

Hocmai– Ngôi trường chung của học trò Việt Tổng đài tư vấn: 1900 69 33 - Trang | 12-

212
Hocmai.vn – Website học trực tuyến số 1 tại Việt Nam
Khóa học Luyện thi THPT quốc gia PEN C –N3 Môn Tiếng Anh côHương Fiona

Radio telescopes, like optical telescopes, allow astronomers to collect data from outer space, but they
are different in important ways. First of all, they look very different because instead of light waves, they
collect radio waves. Thus, in the place of lenses or mirrors, radio telescopes employ bowl-shaped disks that
resemble huge TV satellite dishes. Also, apart from their distinctive appearance, radio telescopes and optical
telescopes use different methods to record the information they collect. Optical telescopes use cameras to
take photographs of visible objects, while radio telescopes use radio receivers to record radio waves from
distant objects in space.
Question 1. What can be inferred about the first optical telescope?
A. It was bought by children. B. It was invented in America.
C. It was sold by a shop owner. D. It was invented by accident.
Question 2. Which of the following is NOT TRUE about Hans Lippershey?
A. He owned a shop.
B. He was Dutch.
C. He sold his invention in 1608.
D. He got his idea of a telescope from the kids in his shop.
Question 3. When was Galileo's invention called "telescope"?
A. in 1611 B. in 1610 C. in 1608 D. in 1600
Question 4. What did Newton notice about Galileo's telescope when he used it?
A. It had many problems B. It refracted the light
C. It was called a refractor D. It had a curved mirror
Question 5. What did Newton do with Galileo's telescope?
A. He called it reflector. B. He sent it back to Galileo.
C. He improved it. D. He stopped using it after his notice.
Question 6. When did Newton start to use Galileo's telescope?
A. in the 17th century B. in the 18th century
C. in the 16th century D. in the 15th century
Question 7. Where does the largest reflecting telescope stand?
A. in Wisconsin B. in California
C. in Hawaii D. in Caucasus Mountains
Question 8. Which of the following can both optical and radio telescope do?
A. Taking photographs of visible objects.
B. Allowing a collection of data from outer space.
C. Collecting radio waves.
D. Collecting light waves.

Nguồn : Hocmai.vn
Giáo viên : Hương Fiona

Hocmai– Ngôi trường chung của học trò Việt Tổng đài tư vấn: 1900 69 33 - Trang | 13-

213
Hocmai.vn – Website học trực tuyến số 1 tại Việt Nam
Khóa học Luyện thi THPT quốc gia PEN C –N3 Môn Tiếng Anh cô Hương Fiona

VOCABULARY – CONSERVATION
( BÀI TẬP TỰ LUYỆN)
Giáo viên : Nguyễn Thanh Hương
Các bài tập trong tài liệu này được biên soạn kèm theo bài giảng “Conservation” thuộc Khóa học Luyện thi THPT quốc gia PEN -
C: Môn Tiếng Anh (cô Hương Fiona )” tại website Hocmai.vn để giúp các Bạn kiểm tra, củng cố lại các kiến thức được giáo viên
truyền đạt trong bài giảng tương ứng. Để sử dụng hiệu quả, Bạn cần học trước bài giảng, sau đó làm đầy đủ các bài tập trong tài liệu
này.

EXERCISE 1

Read the following passage and mark the letter A, B, C, or D on your answer sheet to indicate the
correct answer to each of the questions from 1 to 8.
The Timber rattlesnakes, once widespread throughout the eastern United States, is now on the
endangered species list and is extinct in two eastern states in which it once thrived. Compared to its
Western cousins the Timber rattlesnakes may be especially vulnerable because of certain behaviors
adapted for coping with the cold climate in which it lives.
Rattlesnakes are generally found in warm climates because, like all reptiles, they cannot generate or
regulate their own body temperature internally and must rely on the sun’s warmth for heat. But Timber
rattlesnakes migrated into colder northern areas about 8,000 years ago when glaciers retreated. In these
northern regions, the snakes developed a number of adaptive strategies to survive, but ultimately these
behaviors make them more vulnerable to human predation, their main threat.
One survival strategy the snakes have developed is hibernation. For approximately eight months of the
year, the rattlers remain motionless in deep frost-free crevices, with their body temperature dropping as
low as 40 degrees. In the spring, when they emerge, they must warm their chilled bodies by sunning for
three or four days on rocks in the open. This behavior coupled with the fact that Timber rattlesnakes tend
to concentrate in large numbers at their wintering sites, make them easy prey. Gestating females are
particularly vulnerable because they spend much of their time basking in the sun in order to produce live
young from eggs. In addition, females have very long interbirth intervals, producing live young only every
three to five years. If a frost or cold spell comes late in the year, the entire litter of 6 to 12 young may die.
Efforts are underway to protect the Timber rattlesnakes and its habitats from further human
depredation, but in many states it is already too late.
Question 1. What is the main topic of the passage?
A. Why Timber rattlesnakes hibernate
B. How Timber rattlesnakes are surviving
C. How Timber rattlesnakes adapted to northern climates
D. Why Timber rattlesnakes are endangered
Question 2. Which of the following is closest in meaning to the word “vulnerable” in the first paragraph?
A. Unprotected B. Impervious C. Insensitive D. Deprived
Question 3. Which of the following in the true about Timber rattlesnakes?

Hocmai – Ngôi trường chung của học trò Việt Tổng đài tư vấn: 1900 69 33 - Trang | 1 -

214
Hocmai.vn – Website học trực tuyến số 1 tại Việt Nam
Khóa học Luyện thi THPT quốc gia PEN C –N3 Môn Tiếng Anh cô Hương Fiona

A. They migrated to eastern states


B. They migrated northward during a warming climate
C. They migrated to escape a cold climate
D. They migrated to the South to seek a warmer climate
Question 4. Which of the following could best replace the word "emerge" in the third paragraph?
A. Come out B. Set off C. Get up D. See through
Question 5. In which of the following places might a person be most likely to find Timber rattlesnakes in
the spring?
A. in the woods B. In meadows C. In bushy areas D. In canyons
Question 6. The phrase “easy prey” in the third paragraph could best be replaced by which of the
following?
A. Relaxed B. Protective C. Victims D. Sociable
Question 7. Which of the following word can best replace the word "basking" in the third paragraph?
A. Washing B. Eating C. Sleeping D. Lying
Question 8. Which of the following does NOT contribute to the rattlesnake being an endangered animal?
A. Hibernating for eight months B. Basking in the sun
C. Congregating together D. Having long intervals between births

Exercise 2.
Read the following passage and mark the letter A, B, C or D on your answer sheet to indicate the
correct answer to each of the questions.
Endangered species
Aesthetic justification contends that biodiversity contributes to the quality of life because many of the
endangered plants and animals are particularly appreciated for their unique physical beauty. The aesthetic
role of nature in all its diverse forms is reflected in the art and literature of every culture attaining symbolic
status in the spiritual life of many groups. According to the proponents of the aesthetic argument, people
need nature in all its diverse and beautiful forms as part of the experience of the world.
Another argument that has been put forward, especially by groups in the medical and pharmacological
fields, is that of ecological self-interest. By preserving all species, we retain a balance of nature that is
ultimately beneficial to humankind. Recent research on global ecosystems has been cited as evidence that
every species contributes important or even essential functions that may be necessary to the survival of our
own species. Some advocates of the ecological argument contend that important chemical compounds
derived from rare plants may contain the key to a cure for one of the diseases currently threatening human
beings. If we do not protect other species, then they cannot protect us.
Apart from human advantage in both the aesthetic and ecological arguments, the proponents of a moral
justification contend that all species have the right to exist, a viewpoint stated in the United Nations World

Hocmai – Ngôi trường chung của học trò Việt Tổng đài tư vấn: 1900 69 33 - Trang | 2 -

215
Hocmai.vn – Website học trực tuyến số 1 tại Việt Nam
Khóa học Luyện thi THPT quốc gia PEN C –N3 Môn Tiếng Anh cô Hương Fiona

Charter for Nature, created in 1982. Furthermore, if humankind views itself as the stewards of all the
creatures on Earth, then it is incumbent upon human beings to protect them, and to ensure the continued
existence of all species. Moral justification has been extended by a movement called "deep ecology," the
members of which rank the biosphere higher than people because the continuation of life depends on this
larger perspective. To carry their argument to its logical conclusion, all choices must be made for the
biosphere, not for people.
Question 1: Which of the following is the main topic of the passage?
A. The beauty of the world B. The quality of life
C. The preservation of species D. The balance of nature
Question 2: Which of the arguments supports animal rights?
A. Aesthetic justification B. Ecological argument
C. Self-interest argument D. Moral justification
Question 3: The word "perspective" in the 3rd paragraph could best be replaced by
A. idea B. event C. truth D. view
Question 4: The word "them" in the 3rd paragraph refers to
A. humankind B. stewards C. creatures D. human beings
Question 5: Where in the passage does the author explain how rare species contribute to the health of the
human species?
A. in the underlined sentence B. in the second paragraph
C. in the first paragraph D. in the last paragraph
Question 6: According to the passage, what do we know from research on global ecosystems?
A. Nature is very diverse. B. A balance of nature is important
C. Humans have a responsibility to nature. D. Nature represents spiritual values
Question 7: The author mentions all of the following as justifications for the protection of endangered
species EXCEPT
A. the natural compounds needed for medicines. B. the intrinsic value of the beauty of nature,
C. the control of pollution in the biosphere. D. the right to life implied by their existence.
Question 8: It can be inferred from the passage that the author
A. is a member of the "deep ecology" movement
B. does not agree with ecological self-interest,
C. supports all of the arguments to protect species.
D. participated in drafting the Charter for Nature.

Hocmai – Ngôi trường chung của học trò Việt Tổng đài tư vấn: 1900 69 33 - Trang | 3 -

216
Hocmai.vn – Website học trực tuyến số 1 tại Việt Nam
Khóa học Luyện thi THPT quốc gia PEN C –N3 Môn Tiếng Anh cô Hương Fiona

Exercise 3.
Circle the word whose underlined part is pronounced differently from that of the others
1.A. bamboo B. good C. foot D. cook
2. A. cake B. panda C. face D. late
3. A. social B. science C. sour D. sure
Circle the word whose stress part is placed differently from that of the others
4. A. reserve B. schedule C. wildlife D. beauty
5. A. derive B. contain C. leopard D. prevent
Exercise 4.
Make the correct choice:
6. Many plants and endangered species are now endangering of .
A. expression B. expulsion C. extinction D. extension
7. is destroying larger areas of tropical rain forests.
A. Disforestation B. Deforestation C. Anti-forestation D. forests
8. A lot of different conservation efforts have been made to endangered species.
A. save B. kills C. make D. do
9. The raw sewage needs to be treated.
A. chemically B. chemical C. chemist C. chemistry
10. There are more than 20 working on the water treatment project.
A. researches B. researcher C. researchers D. research
11. More and more people of food poisoning nowadays.
A. exits B. survive C. die D. starve
12. The government the flood victims with food, clothers and money.
A. gave B. provided C. offered D. presented
13. Many people that natural resources will never be used up.
A. view B. consider C. believe D. regard
14. Conservation is the protection of the environment.
A. nature B. natural C. naturally D. naturalize
15. There are many of pollution in our modern world
A. resources B. sources C. foundations D. bases
Nguồn : Hocmai.vn

Giáo viên : Hương Fiona

Hocmai – Ngôi trường chung của học trò Việt Tổng đài tư vấn: 1900 69 33 - Trang | 4 -

217
Hocmai.vn – Website học trực tuyến số 1 tại Việt Nam
Khóa học Luyện thi THPT quốc gia PEN C –N3 Môn Tiếng Anh cô Hương Fiona

Hocmai – Ngôi trường chung của học trò Việt Tổng đài tư vấn: 1900 69 33 - Trang | 5 -

218
Hocmai.vn – Website học trực tuyến số 1 tại Việt Nam
Khóa học PEN C – N3 (Cô Nguyễn Thanh Hương)

FRIENDSHIP
Bài tập tự luyện
Giáo viên: Nguyễn Thanh Hương

I. PRONUNCIATION

1. A. machine B. change C. teacher D. choose


2. A. condition B. option C. suggestion D. relation
3. A. believe B. readily C. friend D. pleasure
4. A. good B. gossip C. game D. geometry
5. A. trust B. mutual C. number D. uncertain

II. Complete these sentences with the correct form of the word in the box.

constant capable selfish loyal suspicion


acquaint enthusiast sympathize hospitable friend

1. His ________ for music has stayed strong, throughout his 23 years in radio.
2. He showed unswerving ___________ to his friends.
3. We all have great __________ for the victims of the flood.
4. __________ is very important because uncertain people cannot have a lifelong friendship.
5. It was very ___________ of him to offer us his room.
6. We were delighted by the wonderful _________ of the local people.
7. The wine had made him ________ of thinking clearly.
8. Playing a game with the children is a good way of getting them _________.
9. Inform the police immediately if you see anything ___________.
10. ________ need time to develop.
III. Complete each of the sentences with an appropriate preposition.
1. A selfish person is incapable __________ true friendship.
2. He was accused _________ being disloyal _________ the government.
3. How much time do you spend ________ homework?
4. Good friendship should be based ______ mutual understanding.

Hocmai – Ngôi trường chung của học trò Việt Tổng đài tư vấn: 1900 6933 - Trang | 1 -
219
Hocmai.vn – Website học trực tuyến số 1 tại Việt Nam
Khóa học PEN C – N3 (Cô Nguyễn Thanh Hương)

5. Do you keep in touch _________ any school friends?


6. Some adults have a total lack ______ sympathy _______ young people.
7. He's more concerned ______ what people think about him than anything else.
8. She had been briefly acquainted ______ him more than 20 years earlier.
9. She thought _________ him very often.
10. Some people take __________ interest with enthusiasm, but they are soon tired ________it.

IV. Choose the correct words or phrases that best complete the passage.
Dear Jane,

I'm sorry for not (1) ______ to you for two months. In my postcard sent to you when I was on vacation
in July, I told you about a new friend of (2) ____, Lisa Moran, who lives near my uncle's farm in Napan
Valley, California. You may be curious to know what she is (3) ______.

Well, Lisa is of our age - 17 - and not very tall or slim. In fact, she has (4) ______ rounded shoulders
and a plump face. The best (5) ______ on Lisa's face are her green eyes and the two dimples on her
checks. She (6) ______ lively with her hair cut short and dressed in (7) ______ clothes - jeans and a
T-shirt. When I first met her at my cousin's birthday party, I was attracted by her (8) ______ and
pleasingly direct manners. Curiously enough, Lisa has the same (9) ______ in music as you and I.

That's (10) ______ about Lisa for the moment. How have you been doing with your school work?
Please write soon.
Best wishes
Ann Maxfield
1. A. write B. to write C. writing D. wrote
2. A. I B. me C. my D. mine
3. A. looking B. like C. seen D. similar
4. A. slightly B. hardly C. scarcely D. reasonably
5. A. aspects B. points C. features D. marks
6. A. feels B. seems C. appears D. looks
7. A. casual B. formal C. sports D. trendy
8. A. light-hearted B. high-handed C. self-centered D. easy-going
9. A. taste B. hearing C. sense D. way
10. A. all B. enough C. end D. most

Hocmai – Ngôi trường chung của học trò Việt Tổng đài tư vấn: 1900 6933 - Trang | 2 -
220
Hocmai.vn – Website học trực tuyến số 1 tại Việt Nam
Khóa học PEN C – N3 (Cô Nguyễn Thanh Hương)

V. Read the passage carefully, then choose the correct answers.


Having a best friend to confide in can bring a positive effect on our emotional health. An evening out
with the closest friend may be the best guarantee of a good time. In fact, our best friend can prevent us
from developing serious psychological problems such as depression and anxiety.
Best friendship evolves with time - we cannot go out and pick our best friend. We become friends with
people who share common interests – at school or through hobbies, for example.
Best friends have usually known each other for years and stuck together through good and bad times.
If you haven't got one, perhaps you are being too distant from people, or focusing too much on your
work.
11. A best friend can ________.
A. give us a healthy life B. go out with us in the evening
C. spend much time finding D. share joy and sadness with us
12. Close friends need to ________.
A. study at the same school B. have the same interests
C. pursue the same hobbies D. spend time together
13. According to the passage, ________.
A. it takes a lot of time to make close friendships
B. we can go out and choose a good friend easily
C. best friends have good and bad times
D. It’s very difficult to make lasting friendships
14. The word 'one' in the last paragraph refers to ________.
A. good time B. bad time C. a friendship D. a close friend
15. Which of the following sentences is not mentioned?
A. We often have satisfaction being with a close friend.
B. the ties of close friendship will never be broken.
C. An unsociable person may not have a close friend.
D. Friendships need time to develop.

Giáo viên: Nguyễn Thanh Hương


Nguồn : Hocmai

Hocmai – Ngôi trường chung của học trò Việt Tổng đài tư vấn: 1900 6933 - Trang | 3 -
221
Hocmai.vn – Website học trực tuyến số 1 tại Việt Nam
Khóa học Luyện thi THPT quốc gia PEN C –N3 Môn Tiếng Anh cô Hương Fiona

PHƯƠNG PHÁP LÀM BÀI PHÁT ÂM


( ĐÁP ÁN BÀI TẬP TỰ LUYỆN )
Giáo viên : Nguyễn Thanh Hương
Các bài tập trong tài liệu này được biên soạn kèm theo bài giảng “Phương pháp làm bài phát âm ” thuộc Khóa học Luyện thi
THPT quốc gia PEN - C: Môn Tiếng Anh (cô Hương Fiona )” tại website Hocmai.vn để giúp các Bạn kiểm tra, củng cố lại các kiến
thức được giáo viên truyền đạt trong bài giảng tương ứng. Để sử dụng hiệu quả, Bạn cần học trước bài giảng, sau đó làm đầy đủ các
bài tập trong tài liệu này.

Exercise 1.Mark the letter A, B, C, or D on your answer sheet to indicate the word whose underlined
part differs from the other three in pronunciation in each of the following questions.
1. A. misses B. goes C. leaves D. potatoes
Đáp án A phiên âm là /iz/, các phương án khác phiên âm là /z/
2. A. adopted B. appealed C. dedicated D. wounded
Đáp án B phiên âm là /d/, các phương án khác phiên âm là /id/
3. A. grows B. tends C. roars D. sweeps
Đáp án D phiên âm là /s/, các phương án khác phiên âm là /z
4. A. university B. duty C. early D. apply
Đáp án D phiên âm là /ai/, các phương án khác phiên âm là /i/
5. A. indicate B. average C. application D. grade
Đáp án B phiên âm là /i/, các phương án khác phiên âm là /ei/
6. A. chemical B. approach C. achieve D. challenge
Đáp án A phiên âm là /k/, các phương án khác phiên âm là / tʃ/
7. A. bride B. confide C. determine D. oblige
Đáp án C phiên âm là /i/, các phương án khác phiên âm là /ai/
8. A. humour B. honest C. honour D. hour
Đáp án A phiên âm là /h/, các phương án khác “h” bị câm
9. A. future B. resume C. enthusiasm D. pressure
Đáp án D phiên âm là / juː/, các phương án khác phiên âm là / ə/
10. A. character B. chores C. technical D. psychology
Đáp án B phiên âm là / tʃ/các phương án khác phiên âm là /k/
11. A. breath B. threaten C. great D. healthy
Đáp án C phiên âm là /ei/, các phương án khác phiên âm là / e/

Hocmai – Ngôi trường chung của học trò Việt Tổng đài tư vấn: 1900 69 33 - Trang | 1 -

222
Hocmai.vn – Website học trực tuyến số 1 tại Việt Nam
Khóa học Luyện thi THPT quốc gia PEN C –N3 Môn Tiếng Anh cô Hương Fiona

Exercise 2.Mark the letter A, B, C, or D on your answer sheet to indicate the word whose underlined
part differs from the other three in pronunciation in each of the following questions.
1. A. gather B. there C. ethnic D. although
Đáp án C phiên âm là / θ /, các phương án khác phiên âm là / ð /
2. A. secure B. shuttle C. future D. contribute
Đáp án B phiên âm là /ʌ/các phương án khác phiên âm là / juː/
3. A. close-knit B. compulsory C. campus D. certificate
Đáp án D phiên âm là /s/, các phương án khác phiên âm là /k/
4. A. ordinary B. pollution C. doctor D. alcohol
Đáp án A phiên âm là / ɔː /, các phương án khác phiên âm là / ə/
5. A. straight B. celebrate C. break D. pleasure
Đáp án D phiên âm là /e/, các phương án khác phiên âm là /ei/
6. A. endangered B. threatened C. recovered D. advanced
Đáp án D phiên âm là /t/, các phương án khác phiên âm là / d/
7. A. elephants B. decades C. poachers D. actions
Đáp án A phiên âm là /s/, các phương án khác phiên âm là /z/
8. A. southern B. athlete C. healthy D. enthusiast
Đáp án A phiên âm là / ð /, các phương án khác phiên âm là / θ /
9. A. chemical B. character C. technical D. charming
Đáp án D phiên âm là / tʃ/, các phương án khác phiên âm là / k/
10. A. caps B. posts C. players D. roofs
Đáp án C phiên âm là / z/các phương án khác phiên âm là /s/

Exercise 3 .Mark the letter A, B, C, or D on your answer sheet to indicate the word whose underlined
part differs from the other three in pronunciation in each of the following questions.
1. A. spirit B. describe C. title D. final
Đáp án A phiên âm là / i /, các phương án khác phiên âm là /ai /
2. A. honor B. exhibit C. hour D. habitat
Đáp án D phiên âm là /h/các phương án khác “h” bị câm
3. A. forged B. noticed C. struggled D. composed
Đáp án B phiên âm là /t/, các phương án khác phiên âm là /d/
4. A. damaged B. increased C. destroyed D. proposed
Đáp án B phiên âm là /t /, các phương án khác phiên âm là / d/

Hocmai – Ngôi trường chung của học trò Việt Tổng đài tư vấn: 1900 69 33 - Trang | 2 -

223
Hocmai.vn – Website học trực tuyến số 1 tại Việt Nam
Khóa học Luyện thi THPT quốc gia PEN C –N3 Môn Tiếng Anh cô Hương Fiona

5. A. protected B. developed C. balanced D. established


Đáp án A phiên âm là /id/, các phương án khác phiên âm là /t/
6. A. carried B. organized C. impressed D. involved
Đáp án C phiên âm là /t/, các phương án khác phiên âm là / d/
7. A. earthquakes B. countries C. epidemics D. delegates
Đáp án B phiên âm là /z/, các phương án khác phiên âm là /s/
8. A. sufferings B. disasters C. species D. advantages
Đáp án D phiên âm là / iz /, các phương án khác phiên âm là / z /
9. A. politics B. beliefs C. rights D. pioneers
Đáp án D phiên âm là / z/, các phương án khác phiên âm là / s/
10. A. laugh B. cough C. rough D. plough
Đáp án D “ph” bị câm, các phương án khác phiên âm là /f/
Exercise 4 .Mark the letter A, B, C, or D on your answer sheet to indicate the word whose underlined
part differs from the other three in pronunciation in each of the following questions.
1. A. identify B. carry C. typical D. penalty
Đáp án A phiên âm là / ai /, các phương án khác phiên âm là /i /
2. A. mutual B. initiate C. picture D. question
Đáp án B phiên âm là / s/, các phương án khác phiên âm là / tʃ/,
3. A. legal B. medal C. level D. effort
Đáp án A phiên âm là /i/, các phương án khác phiên âm là /e/
4. A. pool B. good C. look D. book
Đáp án A phiên âm là /u: /, các phương án khác phiên âm là / u/
5 A. spread B. measure C. weapon D. increase
Đáp án D phiên âm là /i:/, các phương án khác phiên âm là /e/
6. A. accompany B. fascinating C. discriminate D. scoreboard
Đáp án B phiên âm là /s/, các phương án khác phiên âm là / k/
7. A. panda B. swallow C. parrot D. advocate
Đáp án B phiên âm là /ɑː/, các phương án khác phiên âm là /æ/
8. A. wounded B. combined C. considered D. believed
Đáp án A phiên âm là / id /, các phương án khác phiên âm là / d /
9. A. supports B. colonies C. leaders D. victims
Đáp án A phiên âm là / s/, các phương án khác phiên âm là / z/

Hocmai – Ngôi trường chung của học trò Việt Tổng đài tư vấn: 1900 69 33 - Trang | 3 -

224
Hocmai.vn – Website học trực tuyến số 1 tại Việt Nam
Khóa học Luyện thi THPT quốc gia PEN C –N3 Môn Tiếng Anh cô Hương Fiona

10. A. color B. note C. go D. opponent


Đáp án A phiên âm là /ʌ/,các phương án khác phiên âm là / ou/
Exercise 5. Mark the letter A, B, C, or D on your answer sheet to indicate the word whose underlined
part differs from the other three in pronunciation in each of the following questions.
1. A. breather B. feather C. heather D. leather
Đáp án A phiên âm là / i: /, các phương án khác phiên âm là /e /
2. A. pursuit B. suitable C. recruit D. guilty
Đáp án D phiên âm là / i/, các phương án khác phiên âm là / u:/,
3. A. parade B. apathetic C. companion D. marmalade
Đáp án C phiên âm là / æ/, các phương án khác phiên âm là / ə /
4. A. research B. resent C. resemble D. resist
Đáp án A phiên âm là /s /, các phương án khác phiên âm là / z/
5. A. item B. idiom C. idle D. ideal
Đáp án B phiên âm là /i/, các phương án khác phiên âm là /ai/
6. A. pretty B. get C. send D. well
Đáp án A phiên âm là /i/, các phương án khác phiên âm là / e/
7. A. children B. child C. line D. sign
Đáp án A phiên âm là /i/, các phương án khác phiên âm là /ai/
8. A. off B. of C. safe D. knife
Đáp án B phiên âm là / v /, các phương án khác phiên âm là / id /
9. A. sacred B. decided C. cooked D. contaminated
Đáp án C phiên âm là / t/, các phương án khác phiên âm là / id/
10. A. come B. comfort C. some D. comb
Đáp án D phiên âm là/ ou//, các phương án khác phiên âm là ʌ/,

Nguồn : Hocmai.vn
Giáo viên : Hương Fiona

Hocmai – Ngôi trường chung của học trò Việt Tổng đài tư vấn: 1900 69 33 - Trang | 4 -

225
Hocmai.vn – Website học trực tuyến số 1 tại Việt Nam
Khóa học Luyện thi THPT quốc gia PEN C –N3 Môn Tiếng Anh cô Hương Fiona

PHƯƠNG PHÁP LÀM BÀI TRỌNG ÂM


( ĐÁP ÁN BÀI TẬP TỰ LUYỆN )
Giáo viên : Nguyễn Thanh Hương
Các bài tập trong tài liệu này được biên soạn kèm theo bài giảng “Phương pháp làm bài trọng âm ” thuộc Khóa học Luyện thi THPT
quốc gia PEN - C: Môn Tiếng Anh (cô Hương Fiona )” tại website Hocmai.vn để giúp các Bạn kiểm tra, củng cố lại các kiến thức được
giáo viên truyền đạt trong bài giảng tương ứng. Để sử dụng hiệu quả, Bạn cần học trước bài giảng, sau đó làm đầy đủ các bài tập trong
tài liệu này.

Exercise 1. Mark A, B, C, or D on your answer sheet to indicate the word that differs from the other
three in the position of primary stress in each of the following questions.
1. A. biologist B. counterpart C. compliment D. kindergarten
Đáp án A. Trọng âm rơi số 2, các phương án còn lại trọng âm rơi số 1.

A. biologist /baɪˈɒlədʒɪst/
B. counterpart /ˈkaʊntəpɑːt/
C. compliment /ˈkɒmplɪmənt/
D. kindergarten/ˈkɪndəɡɑːtn

2. A. applicant B. maximum C. category D. inflation


Đáp án D. Trọng âm rơi số 2, các phương án còn lại trọng âm rơi số 1.

A. applicant /ˈæplɪkənt/
B. maximum /ˈmæksɪməm/
C. category /ˈkætəɡəri/
D. inflation /ɪnˈfleɪʃn/

3. A. renovation B. communication C. intervention D. expedition


Trọng âm rơi số 4, các phương án còn lại trọng âm rơi số 3.

A. renovation /ˌrenəˈveɪʃn/
B. communication /kəˌmjuːnɪˈkeɪʃn/
C. intervention /ˌɪntəˈvenʃn/
D. expedition /ˌekspəˈdɪʃn/
4. A. different B. bamboo C. rainfall D. wildlife
Đáp án B. Trọng âm rơi số 2, các phương án còn lại trọng âm rơi số 1.

A. different /ˈdɪfrənt/
B. bamboo /ˌbæmˈbuː/
C. rainfall /ˈreɪnfɔːl/

Hocmai– Ngôi trường chung của học trò Việt Tổng đài tư vấn: 1900 69 33 - Trang | 1-

226
Hocmai.vn – Website học trực tuyến số 1 tại Việt Nam
Khóa học Luyện thi THPT quốc gia PEN C –N3 Môn Tiếng Anh cô Hương Fiona

D. wildlife /ˈwaɪldlaɪf/

5. A. personality B. rhinoceros C. gorilla D. opponent


Đáp án A. Trọng âm rơi số 3, các phương án còn lại trọng âm rơi số 2.

A. personality /ˌpɜːsəˈnæləti/
B. rhinoceros /raɪˈnɒsərəs/
C. gorilla /ɡəˈrɪlə/
D. opponent /əˈpəʊnənt/

6. A. leftovers B. windsurfing C. enthusiast D. scoreboard


Đáp án C. Trọng âm rơi số 2, các phương án còn lại trọng âm rơi số 1.

A. leftovers /ˈleftəʊvə(r)/

B. windsurfing /ˈwɪndsɜːfɪŋ/

C. enthusiast /ɪnˈθjuːziæst/

D. scoreboard /ˈskɔːbɔːd/

7. A. comfortable B. tsunami C. precision D. involvement


Đáp án A. Trọng âm rơi số 1, các phương án còn lại trọng âm rơi số 2.

A. comfortable /ˈkʌmftəbl/

B. tsunami /tsuːˈnɑːmi/

C. precision /prɪˈsɪʒn/

D. involvement /ɪnˈvɒlvmənt/

8. A. struggle B. rubbish C. nonsense D. development


Đáp án D. Trọng âm rơi số 2, các phương án còn lại trọng âm rơi số 1.

A. struggle /ˈstrʌɡl/

B. rubbish /ˈrʌbɪʃ/

C. nonsense /ˈnɒnsns/

D. development /dɪˈveləpmənt/
9. A. nursery B. certificate C. curriculum D. intelligence
Đáp án A. Trọng âm rơi số 1, các phương án còn lại trọng âm rơi số 2.

A. nursery /ˈnɜːsəri/

B. certificate /səˈtɪfɪkət/

Hocmai– Ngôi trường chung của học trò Việt Tổng đài tư vấn: 1900 69 33 - Trang | 2-

227
Hocmai.vn – Website học trực tuyến số 1 tại Việt Nam
Khóa học Luyện thi THPT quốc gia PEN C –N3 Môn Tiếng Anh cô Hương Fiona

C. curriculum /kəˈrɪkjələm/

D. intelligence /ɪnˈtelɪdʒəns/

10. A. industrial B. interfere C. develop D. activity


Đáp án B. Trọng âm rơi số 3, các phương án còn lại trọng âm rơi số 2.
A. industrial /ɪnˈdʌstriəl/
B. interfere /ˌɪntəˈfɪə(r)/
C. develop /dɪˈveləp/
D. activity /ækˈtɪvəti/
Exercise 2. Mark A, B, C, or D on your answer sheet to indicate the word that differs from the other
three in the position of primary stress in each of the following questions.

1. A. incredible B. unexpected C. unnoticed D. outstanding


B. Trọng âm rơi số 3, các phương án còn lại trọng âm rơi số 2.

2. A. overwhelming B. intellectual C. interesting D. economic


2. C. Trọng âm rơi số 1, các phương án còn lại trọng âm rơi số 3.

3. A. inaccurate B. illegal C. positive D. domestic


3. C. Trọng âm rơi số 1, các phương án còn lại trọng âm rơi số 2.

4. A. confident B. eternal C. dangerous D. healthier


4. B. Trọng âm rơi số 2, các phương án còn lại trọng âm rơi số 1.

5. A. independent B. academic C. compulsory D. unexpected


5. C. Trọng âm rơi số 2, các phương án còn lại trọng âm rơi số 3.

6. A. secondary B. immediate C. miraculous D. domestic


6. A. Trọng âm rơi số 1, các phương án còn lại trọng âm rơi số 2.

7. A. simple B. polite C. formal D. instant


7. B. Trọng âm rơi số 2, các phương án còn lại trọng âm rơi số 1.

8. A. beautiful B. terrific C. wonderful D. marvelous


8. B. Trọng âm rơi số 2, các phương án còn lại trọng âm rơi số 1.

9. A. conical B. different C. symbolic D. careful


9. C. Trọng âm rơi số 2, các phương án còn lại trọng âm rơi số 1.

10. A. cultural B. mischievous C. interesting D. responsible


10. D. Trọng âm rơi số 2, các phương án còn lại trọng âm rơi số 1.

Hocmai– Ngôi trường chung của học trò Việt Tổng đài tư vấn: 1900 69 33 - Trang | 3-

228
Hocmai.vn – Website học trực tuyến số 1 tại Việt Nam
Khóa học Luyện thi THPT quốc gia PEN C –N3 Môn Tiếng Anh cô Hương Fiona

Exercise 3. Mark A, B, C, or D on your answer sheet to indicate the word that differs from the other
three in the position of primary stress in each of the following questions.
1. A. finance B. service C. order D. company
1 A. Trọng âm rơi số 2, các phương án còn lại trọng âm rơi số 1.

2. A. interview B. agency C. addition D. customer


2. C. Trọng âm rơi số 2, các phương án còn lại trọng âm rơi số 1.

3. A. prisoner B. diffficult C. exercise D. disaster


3. D. Trọng âm rơi số 2, các phương án còn lại trọng âm rơi số 1.

4. A. improvement B. occasion C. believe D. interest


4. D. Trọng âm rơi số 1, các phương án còn lại trọng âm rơi số 2.

5. A influential B. opportunity C. temperament D. expectation


5. C. Trọng âm rơi số 1, các phương án còn lại trọng âm rơi số 3.

6. A. picture B. number C. water D. advice


6. D. Trọng âm rơi số 2, các phương án còn lại trọng âm rơi số 1.

7. A. organize B. embarrass C. discourage D. surrounding


7. A. Trọng âm rơi số 1, các phương án còn lại trọng âm rơi số 2.

8. A. podium B. title C. notice D. pioneer


8. D. Trọng âm rơi số 3, các phương án còn lại trọng âm rơi số 1.

9. A. nature B. marriage C. value D. belief


9. D. Trọng âm rơi số 2, các phương án còn lại trọng âm rơi số 1.

10. A. police B. spirit C. banquet D. culture


10. A. Trọng âm rơi số 2, các phương án còn lại trọng âm rơi số 1.

Exercise 4. Mark A, B, C, or D on your answer sheet to indicate the word that differs from the other
three in the position of primary stress in each of the following questions.
1. A. determine B. maintain C. sacrifice D. apologize
1 C. Trọng âm rơi số 1, các phương án còn lại trọng âm rơi số 2.

2. A. inversion B. miserable C. pleasant D. difficulty


2. A. Trọng âm rơi số 2, các phương án còn lại trọng âm rơi số 1.

3. A. swallow B. survive C. digest D. finish


3. B. Trọng âm rơi số 2, các phương án còn lại trọng âm rơi số 1.

Hocmai– Ngôi trường chung của học trò Việt Tổng đài tư vấn: 1900 69 33 - Trang | 4-

229
Hocmai.vn – Website học trực tuyến số 1 tại Việt Nam
Khóa học Luyện thi THPT quốc gia PEN C –N3 Môn Tiếng Anh cô Hương Fiona

4. A. product B. satisfy C. pleasure D. however


4. D. Trọng âm rơi số 2, các phương án còn lại trọng âm rơi số 1.

5. A. happiness B. rewrite C. eject D. oblige


5. A. Trọng âm rơi số 1, các phương án còn lại trọng âm rơi số 3.

6. A. compare B. compose C. company D. consist


6. C. Trọng âm rơi số 1, các phương án còn lại trọng âm rơi số 2.

7. A. manage B. recognize C. argue D. discriminate


7. D. Trọng âm rơi số 2, các phương án còn lại trọng âm rơi số 1.

8. A. entertain B. understand C. recommend D. develop


8. D. Trọng âm rơi số 2, các phương án còn lại trọng âm rơi số 3.

9. A. offer B. listen C. precede D. follow


9. C. Trọng âm rơi số 2, các phương án còn lại trọng âm rơi số 1.

10. A. interview B. appropriate C. employ D. describe


10. A. Trọng âm rơi số 1, các phương án còn lại trọng âm rơi số 2.

Exercise 5. Mark A, B, C, or D on your answer sheet to indicate the word that differs from the other
three in the position of primary stress in each of the following questions.
1. A. support B. fascinate C. accept D. believe
1 B. Trọng âm rơi số 1, các phương án còn lại trọng âm rơi số 2.

2. A. inform B. appeal C. attempt D. hesitate


2. D. Trọng âm rơi số 1, các phương án còn lại trọng âm rơi số 2.

3. A. integrate B. defeat C. advise D. compete


3. A. Trọng âm rơi số 1, các phương án còn lại trọng âm rơi số 2.

4. A. engrave B. categorize C. memorialize D. discover


4. B. Trọng âm rơi số 1, các phương án còn lại trọng âm rơi số 2.

5. A. extremely B. necessary C. necessity D. co-operate


5. B. Trọng âm rơi số 1, các phương án còn lại trọng âm rơi số 2.

6. A. politics B. historic C. electric D. specific


6. A. Trọng âm rơi số 1, các phương án còn lại trọng âm rơi số 2.

7. A. committee B. referee C. employee D. refugee


7. A. Trọng âm rơi số 2, các phương án còn lại trọng âm rơi số 3.

Hocmai– Ngôi trường chung của học trò Việt Tổng đài tư vấn: 1900 69 33 - Trang | 5-

230
Hocmai.vn – Website học trực tuyến số 1 tại Việt Nam
Khóa học Luyện thi THPT quốc gia PEN C –N3 Môn Tiếng Anh cô Hương Fiona

8. A. stimulate B. subtract C. announce D. maintain


8. A. Trọng âm rơi số 1, các phương án còn lại trọng âm rơi số 2.

9. A. determine B. appearance C. develop D. difference


9. D. Trọng âm rơi số 1, các phương án còn lại trọng âm rơi số 2.

10. A. romantic B. following C. summary D. physical


10. A. Trọng âm rơi số 2, các phương án còn lại trọng âm rơi số 1

Nguồn : Hocmai.vn
Giáo viên : Hương Fiona

Hocmai– Ngôi trường chung của học trò Việt Tổng đài tư vấn: 1900 69 33 - Trang | 6-

231
Hocmai.vn – Website học trực tuyến số 1 tại Việt Nam
Khóa học Luyện thi THPT quốc gia PEN C –N3 Môn Tiếng Anh cô Hương Fiona

PHƯƠNG PHÁP LÀM BÀI TÌM LỖI SAI


(ĐÁP ÁN BÀI TẬP TỰ LUYỆN )
Giáo viên : Nguyễn Thanh Hương
Các bài tập trong tài liệu này được biên soạn kèm theo bài giảng “Phương pháp làm bài tìm lỗi sai” thuộc Khóa học Luyện thi THPT
quốc gia PEN - C: Môn Tiếng Anh (cô Hương Fiona )” tại website Hocmai.vn để giúp các Bạn kiểm tra, củng cố lại các kiến thức được
giáo viên truyền đạt trong bài giảng tương ứng. Để sử dụng hiệu quả, Bạn cần học trước bài giảng, sau đó làm đầy đủ các bài tập trong
tài liệu này.

I.Mark the letter A, B, C or D on your answer sheet to show the underlined part that needs correction
Question 1: There will always be a job for Mike if he change his mind.
A B C D
1 D if he change his mind-> if he changes his mind
Question 2:Language is important factor in the accumulation of culture.
A B C D
2 B important factor-> An important factor
Question 3: That cats have nine lives have been believed for centuries
A B C D
3 D have been -> has been (That+ S+V…: Việc mà… chia chủ ngữ số ít:
Dich: Việc mà mèo có 9 mạng từ lâu đã được tin tưởng trong nhiều thế kỉ)
Question 4: Despite the disappearance of the Mayan empire , there are still Mayans in the region
A B C
that they once inhabited
D
4 D that they -> where they
Question 5: After writing it , the essay must be duplicated by the student himself and handed into the
A B C
department secretary before the end of the month .
D
5 A after writing it -> after being written
II.Mark the letter A, B C or D on your answer sheet to indicate the underlined part that needs correction
in each of the following questions .
Question 1: Men and women in the Peace Corps work with people in the developing countries
A B
to help them improving their living conditions.

Hocmai– Ngôi trường chung của học trò Việt Tổng đài tư vấn: 1900 69 33 - Trang | 1-

232
Hocmai.vn – Website học trực tuyến số 1 tại Việt Nam
Khóa học Luyện thi THPT quốc gia PEN C –N3 Môn Tiếng Anh cô Hương Fiona

C D
1 C improving -> (to) improve, cấu trúc help sb do/ to do sth
Question 2: Although not widely sold, that book is considered to be best book on the subject.
A B C D
2 C best-> the best
Question 3: Because his sickness he didn’t take part in the English competition held last Sunday.
A B C D
3 A Because -> Because of
Question 4: I found my new contact lenses strangely at first, but I got used to them in the end.
A B C D
4 B strangely -> strange, cấu trúc find sb/ sth adj: thấy ai/ cái gì như thế nào
Question 5: Dreaming, like all other mental processes, it is a product of the brain and its activity.
A B C D
5 B it is-> is , câu này thừa từ it, do S chính của câu là “Dreaming”
III. Mark the letter A, B, C, or D on your answer sheet to show the underlined part that needs correction.
Question 1: My father asked me where had I gone the night before.
A B C D
1 B . had I -> I had
Question 2: The harder he tried, the worst he danced before the large audience.
A B C D
2 C. the worst -> the worse
Question 3: Students suppose to read all the questions carefully and find out the answers to
A B C
them.
D
3 A suppose-> are supposed, cấu trúc S+ be + supposed to V: có trách nhiệm, được mong chờ làm gì.
Câu này dịch: Các em học sinh được khuyên hãy đọc tất cả câu hỏi cẩn thận…
Question 4: Lake Superior, that lies on the US Canadian border, is the largest lake in North America.
A B C D
4 A that -> which
Question 5: Public health experts say that the money one spends avoiding illness is less than the cost
A B C
of to treat sickness.

Hocmai– Ngôi trường chung của học trò Việt Tổng đài tư vấn: 1900 69 33 - Trang | 2-

233
Hocmai.vn – Website học trực tuyến số 1 tại Việt Nam
Khóa học Luyện thi THPT quốc gia PEN C –N3 Môn Tiếng Anh cô Hương Fiona

D
5 D to treat sickness-> treating sickness, sau giới từ OF động từ dùng ở dạng Ving: preposition+ Ving.
IV. Mark the letter A, B, C, or D on your answer sheet to show the underlined part that needs correction.
Question 1. One of the students who are being considered for the scholarship are from this university.
A B C D
D. are -> is, hòa hợp chủ-vị do chủ ngữ chính là . “One of the students: một trong số những bạn học
sinh là chủ ngữ số ít nên động từ theo sau phải ở số ít “is”

Question 2. More than 600 million individual bacteria lives on the skin of humans.
A B C D
C. lives-> live, hòa hợp chủ-vị do chủ ngữ chính là 600 million individual bacteria: 600 triệu con vi
khuẩn, “bacteria” : vi khuẩn (số nhiều), bacterium: vi khuẩn (số ít)
Question 3. I'm very glad that you've done lots of progress this semester.
A B C D
B. done-> made, cụm từ cố định make progress: tạo được thành tựu
Question 4. Not only did he broke two glasses, but also he left the table dirty.
A B C D
B broke -> break, đảo ngữ, xuất hiện trợ động từ did thì động từ chính phải giữ dạng nguyên là “break”
Question 5. Alike all other mammals, dolphins have lungs.
A B C D
A .alike-> like, alike: giống nhau, không thể đứng đầu câu, thường đi look alike: trông giống, vd: They
look alike.
Like: giống>< Unlike: không giống: có thể đứng đầu câu.

V.Mark the letter A, B, C, or D on your answer sheet to show the underlined part that needs correction.
Question 1. Wage rates depend in part from the general prosperity of the economy.
A B C D
C from-> on. Depend on: phụ thuộc, B không sai do đi cùng cụm in part: một phần,
câu này dịch là Tỉ lệ tiền lương phụ thuộc một phần vào sự thịnh vượng của nền kinh tế.
Question 2. If either of you take a vacation, we will not be able to finish the work.
A B C D
B take -> takes, hài hòa chủ vị, either of you: là một trong 2 bạn: Chủ ngữ số ít nên động từ takes phải
chia số ít
Question 3. Several people have apparent tried to change the man's mind, but he refuses to listen.

Hocmai– Ngôi trường chung của học trò Việt Tổng đài tư vấn: 1900 69 33 - Trang | 3-

234
Hocmai.vn – Website học trực tuyến số 1 tại Việt Nam
Khóa học Luyện thi THPT quốc gia PEN C –N3 Môn Tiếng Anh cô Hương Fiona

A B C D
B apparent -> apparently, trạng từ đứng trước V tried
Question 4. His father told him to apologise their neighbour for being rude.
A B C D
B apologise -> apologise to, cấu trúc apologise to Sb for Sth/ doing sth
Question 5. The physicians appeared nervously when he talked to the patient.
A B C D
C. nervously-> nervous, appear+ adj: dường như
VI. Mark the letter A, B, C, or D on your answer sheet to show the underlined part that needs correction.
Question 1. Many theory on how the Earth began its existence have been proposed.
A B C D
A. theory -> theories, sau many phải dùng N số nhiều theories
Question 2. The task of the cartographer is to represent the Earth's surface at a reduced greatly scale.
A B C D

D. reduced greatly-> greatly reduced, cấu trúc adv+ P2


Question 3. They couldn't hear the music down there because it wasn't too loud.
A B C D
D. too loud-> loud enough, nghĩa câu: họ không thể nghe thấy tiếng nhạc vì nó không đủ lớn.
Question 4. The headmaster, together with the teaching staff, are away on holiday.
A B C D
C. are-> is, hài hòa chủ vị, câú trúc có chủ ngữ chính phụ, chia theo chủ ngữ chính số ít The headmaster:
is
Question 5. Automobiles begun to be equipped with built-in radios around 1930.
A B C D
A. begun-> began, thì quá khứ
VII.Mark the letter A, B, C, or D on your answer sheet to show the underlined part that needs correction.
Question 1. Although a number of police officers was guarding the priceless treasures in the
A B
museum, the director worried that someone would try to steal them.
C D
B was guarding-> were guarding, hài hòa chủ vị, a number of + N số nhiều nên động từ cũng phải ở
dạng số nhiều

Hocmai– Ngôi trường chung của học trò Việt Tổng đài tư vấn: 1900 69 33 - Trang | 4-

235
Hocmai.vn – Website học trực tuyến số 1 tại Việt Nam
Khóa học Luyện thi THPT quốc gia PEN C –N3 Môn Tiếng Anh cô Hương Fiona

Question 2 . One of the primary cause of road accidents is driving after drinking.
A B C D
B. cause => causes. Sau cụm one of the thì N phải ở dạng số nhiều: một trong số những…
Question 3. Insurance rates are not the same for different people because they are not likely
A B
have the same risk.
C D
C. have=> to have. Cấu trúc S+ be likely+ to V: có xu hướng,khuynh hướng làm gì.
Question 4. It is vitally important that she takes this medication night and morning.
A. B. C. D.
C. takes=> take, cấu trúc thức giả định It’s+ ADJ (important, necessary…)+ S+ Vo
Question 5. We had better to review this chapter carefully because we will have some questions on it on
our test tomorrow.
A B. C. D.
A. to review => review. Had better+ Vo
VIII.Mark the letter A, B, C, or D on your answer sheet to show the underlined part that needs correction.
Question 1. American painter Georgia O’Keeffe is well known (A) as her large paintings of flowers (B) in
which single (C) blossoms are presented as if in (D) close-up.
A. well- known as (biết đến/ nổi tiếng như là)=> . well- known for: nổi tiếng cho/ nhờ, nghĩa câu:
Georgia O’Keeffe nổi tiếng về những bức tranh hoa…
Question 2. Although this car (A) appears (B) to be manufactured by a (C) different company, it has the
same body style, size, and (D) perform as that one.
D perform=> performance, cấu trúc song hành, các N body style, size, and performance đi cùng với
nhau.
Question 3. Farm (A) animals have been (B) regardless by nearly all societies as a (C) valuable economic
(D) resource.
B. regardless=> regarded by: được coi như là (regardless of: bất chấp)
Question 4. (A) Although her (B) severe pain, she tried (C) to walk to the auditorium (D) to attend the
meeting.
A. Although => Despite vì tiếp sau đó là cụm danh từ her severe pain, although+ S+V
Question 5. (A) In my opinion, (B) our world view is quite different from (C) the writers in (D) the fourth
century BC.

Hocmai– Ngôi trường chung của học trò Việt Tổng đài tư vấn: 1900 69 33 - Trang | 5-

236
Hocmai.vn – Website học trực tuyến số 1 tại Việt Nam
Khóa học Luyện thi THPT quốc gia PEN C –N3 Môn Tiếng Anh cô Hương Fiona

C. the writers-> that of the writers: so sánh đối tượng bị khập khiễng, thế giới quan của chúng ta ngày
nay khác với thế giới quan của những nhà văn thời xưa
IX.Mark the letter A, B, C, or D on your answer sheet to show the underlined part that needs correction.
Question 1. (A) The electric (B) toaster was one of the earliest (C) appliance to be developed for the (D)
kitchen.
1 C. appliance => appliances

Question 2. The ability of (A) speech is one of the (B) skills that make (C) humans difference from the
rest of (D) the animal world.
2 C. humans difference => humans different, cấu trúc make Sb/ Sth ADJ

Question 3. Each of (A) the students (B) in the accounting class (C) has to type their own research paper
(D) this semester.
3 C has to type their own research => has to type his/ her own research, chủ ngữ là số ít “each of the
students”: mỗi bạn học sinh nên dùng his/ her

Question 4. In the United States (A) among 60 percent (B) of the space on the pages of newspapers (C) is
reserved for (D) advertising.
4 A. among=> about nghĩa câu: khoảng 60% không gian ở các trang báo là dành cho quảng cáo.

Question 5. Daisy has (A) such many things (B) to do that she (C) has no time (D) to go out.
5.A such=> so cấu trúc so+many/much/ few/ little + N that S+V

X.Mark the letter A, B, C, or D on your answer sheet to show the underlined part that needs correction.
Choose A, B, C, or D to show the underlined part that needs correction.
Question 1. Objects (A) falling freely in a (B) vacuum have the same rate of (C) speed is regardless of
differences in (D) size and weight.
C speed is => VSHHGWURQJFkXÿmFy9FKtQKOjKDYHWKHVDPHVSHHGOj2EMHFWUHJDUGOHVVRIOjSUHS
Question 2. One of the (A) majority causes of (B) tides is the (C) gravitational (D) attraction of
the Moon.
A. majority => major, trước N: causes cần tính từ, dịch: các nguyên nhân chính
Question 3. The oldest and (A) most widespread celebrations are (B) that connected with the (C)
harvesting of the (D) first fruits.
B. that=> those. Vì cần hài hòa với chủ ngữ số nhiều celebrations đứng trước.
Question 4. From bones (A) finding in the United States, we (B) have learned that many animals which
(C) no longer existed in the world once (D) made their homes there.
A. finding => found: chia bị động, xương được tìm thấy ở Mỹ

Hocmai– Ngôi trường chung của học trò Việt Tổng đài tư vấn: 1900 69 33 - Trang | 6-

237
Hocmai.vn – Website học trực tuyến số 1 tại Việt Nam
Khóa học Luyện thi THPT quốc gia PEN C –N3 Môn Tiếng Anh cô Hương Fiona

Question 5. (A) All of my students are longing (B) for having a nice summer (C) holiday (D) at the
seaside.
B. for having => to have: long to V= want to V: muốn.

Nguồn : Hocmai.vn
Giáo viên : Hương Fiona

Hocmai– Ngôi trường chung của học trò Việt Tổng đài tư vấn: 1900 69 33 - Trang | 7-

238
Hocmai.vn – Website học trực tuyến số 1 tại Việt Nam
Khóa học Luyện thi THPT quốc gia PEN C –N3 Môn Tiếng Anh cô Hương Fiona

VOCABULARY – CULTURE AND LANGUAGE


( ĐÁP ÁN BÀI TẬP TỰ LUYỆN)
Giáo viên : Nguyễn Thanh Hương
Các bài tập trong tài liệu này được biên soạn kèm theo bài giảng “Culture and language” thuộc Khóa học Luyện thi THPT quốc
gia PEN - C: Môn Tiếng Anh (cô Hương Fiona )” tại website Hocmai.vn để giúp các Bạn kiểm tra, củng cố lại các kiến thức được
giáo viên truyền đạt trong bài giảng tương ứng. Để sử dụng hiệu quả, Bạn cần học trước bài giảng, sau đó làm đầy đủ các bài tập
trong tài liệu này.

EXERCISE 1. Read the following passage and mark the letter A, B, C or D on your answer sheet to
indicate the correct word for each of the blanks.
THE HISTORY OF WRITING
The development of writing (1) ________ a huge difference to the world and might see it as the beginning
of the media. Pieces of pottery with marks on that are probably numbers have been discovered in China
that date from around 4000 BC. Hieroglyphics and other forms of "picture writing" developed in the (2)
________ around Mesopotamia (mordem-day Iraq), where the ancent Sumerian civilization was based,
from around 3300 BC onwards. However, the first (3) ________ alphabet was used by the Phoenicians
around 1050BC. Their alphabet had 22 letters and it is estimated that it lasted for 1000 years. The first two
signs were called "aleph" and "beth", which in Greek became "alpha" and "beta"* which gave us the
modem word "alphabet" The modem European alphabet is based on the Greek and (4) to other
European countries under the
Romans. A number of changes took place as time passed. The Romans added the letter G, and the letter J
and V were unknown to people in Shakespear's time.
If we (5) ________ the history of punctuation, we also find some interesting facts. The Romans
used to write quaesto at the end of a sentence in order to show that it was a Question, they started to write
Qo in place of the whole word, and then put the Q above the 0. In the end, that became the question mark
"?"
Question 1: A. did B. had C. made D. took
Make a diference: tạo sự khác biệt
Question 2: A. distance B. area C. length D. earth
Area around Mesopotamia: khu vực xung quanh Mesopotamia.
 Distance (n): khoảng cách
 Area (n) (vùng, khu vực)
 Length (n) (chiều dài)
 Earth (n) (trái đất)
Phía sau chỗ trống là đanh từ "Mesopotamia" có nghĩa là vùng đồng bằng Lưỡng Hà. Do đó, "area" là từ
phù hợp nhất để điền vào chỗ trống.
Question 3: A. true B. accurate C. exact D. precise

Hocmai – Ngôi trường chung của học trò Việt Tổng đài tư vấn: 1900 69 33 - Trang | 1 -

239
Hocmai.vn – Website học trực tuyến số 1 tại Việt Nam
Khóa học Luyện thi THPT quốc gia PEN C –N3 Môn Tiếng Anh cô Hương Fiona

 True (adj); thực, chân thực, đúng với sự thật (thường dùng với một sự kiện, sự việc)
 Accurate (adj): chính xác, xác đáng (theo kiểu đúng với mọi chi tiết)
 Exact (adj): đúng đắn, chính xác (đưa ra tẩt cả các chi tiết một cách đúng đắn)
Precise (adj): rõ ràng, chính xác, tỉ mi, nghiêm ngặt (đưa ra các chi tiểt một cách rõ ràng, chính xác và
thường dùng trong đo hrâmg)
Question 4: A. spread B. appeared C. was D. occurred
Tạm dịch: "The modem European alphabet is based on the Greek and spread to other European countries
under the Romans": Bảng chữ cái của người Châu Ấu hiện đại có nền tảng là tiếng Hy Lạp và lan truyền
sang các mước Châu Ấu khác dưới thời La Mã
To spread: truyền ra, rải, truyền bả
Question 5: A. look into B. bring on C. make off D. hold up
 Look into: điều tra.
Tạm dịch: "If we look into the history of punctuation, we also find some interesting facts": Nếu chúng ta
xem xét lại lịch sử của hệ thống chẩm câu, chúng ta cũng sẽ tìm thấy một vài thực tế thú vị.

EXERCISE 2. Read the following passage and mark the letter A, B, c or D on your answer sheet to
indicate the correct word for each of the blanks.
Basic to any understanding of Canada in the 20 years after the Second World War is the country’s
impressive population growth. For every three Canadians in 1945, there were over five in 1966. In
September 1966 Canada’s population passed the 20 million mark. Most of this surging growth came from
natural increase. The depression of the 1930s and the war had held back marriages, and the catching-up
process began after 1945. The baby boom continued through the decade of the 1950s, producing a
population increase of nearly fifteen percent in the five years from 1951 to 1956 This rate of increase had
been exceeded only once before in Canada’s history, in the decade before 1911 when the prairies were
being settled. Undoubtedly, the good economic conditions of the
1950s supported a growth in the population, but the expansion also derived from a trend toward earlier
marriages and an increase in the average size of families. In 1957 the Canadian birth rate stood at 28 per
thousand, one of the highest in the worlk
After the peak year of 1957, the birth rate in Canada began to decline. It continued falling until in
1966 it stood at the lowest level in 25 years. Partly this decline reflected the low level of births during the
depression and the war, but it was also caused by changes in Canadian society. Young people were staying
at school longer more women were working; young married couples were buying automobiles or houses
before starting families; rising living standards were cutting down the size of families It appeared that
Canada was once more falling in step with the trend toward smaller families that had occurred all through
the Western world since the time of the Industrial Revolution.

Hocmai – Ngôi trường chung của học trò Việt Tổng đài tư vấn: 1900 69 33 - Trang | 2 -

240
Hocmai.vn – Website học trực tuyến số 1 tại Việt Nam
Khóa học Luyện thi THPT quốc gia PEN C –N3 Môn Tiếng Anh cô Hương Fiona

Although the growth in Canada’s population had slowed down by 1966 (the increase in the first
half of the 1960s was only nine percent) another large population wave was coming over the horizon. It
would be composed of the children who were born during the period of the high birth rate prior to 1957.

Question 1. What does the passage mainly discuss?


A. Educational changes in Canadian society B. Canada during the Second World War
C. Population trends in postwar Canada D. Standards of living in Canada
Ta làm câu hỏi sau khi đã hoàn thành các câu hỏi khác.
Key words: mainly discuss
Câu hỏi: Bài vãn chủ yếu bàn luận vấn đề gì?
Clue: the Kinetoscope... was designed for use ỉn Kinetoscope parlors, or arcades, which contained only a
few individual machines and permitted only one customer to view a short... at any one time... customers
moved from machine to machine to watch five different films (or, in the case of famous prizefights,
successive rounds of a single fight)”: Thiết bị Kinestiscope... được thiết kế để sử dụng trong phòng chiếu
Kinestiscope chứa được chỉ vài máy và cho phép chi một khách hàng xem một bộ phim ngẳn... một lần...
các khách hàng dì chuyển từ máy này sang máy khác để lần lượt xem toàn bộ phim khác nhau (hoặc, khi
xem các giải đẩu quyền anh, hoặc các trận đẩu đơn hay liên tiếp)
Phân tích: Đoạn văn có nhắc đến việc mỗi một khán giả xem một bộ phim, khách hàng có thể xem hết
phim này đển phim khác và các bộ phim thì rất ngắn. Tranh giải đấu quyền anh dược nhắc đến là một chủ
đề của phim nhưng không được đề cập đến là bộ phim phổ biển nhất. Vậy chọn đáp án C.Population trends
in postwar Canada: Những xu hướng phổ biến ở Canada sau chiến tranh
Các đáp án khác đều có thông tin trong đoạn:
A: Eucational changes in Canadian society: Thay đồi giáo dục trong xã hội Canada
B: Canada during the Second World War: Đất nước Canada trong Thể Chiến thứ II
C: Population trends in postwar Canada: Những xu hướng phổ biến ở Canada sau chiển tranh
D: Standards of living in Canada: Mức sống ờ Canada
Question 2: The word “five” in bold refers to
A. Canadians B. years C. decades D. marriages
Key words: five, refer to
Câu hỏi: từ “five" in đậm trong đoạn văn đề cập đến gỉ?
Clue: “Basic to any understanding of Canada in the 20 years after flic Second World War is the country’s
impressive population growth. For every three Canadians in 1945, there were over five in 1966”: Lẽ cơ
bản về Canada 20 năm sau Thế chiến thứ hai là sự gia tăng dân số ẩn tượng của đất nước này. Cứ mỗi ba
người Canada vào năm 1945 thì có năm người vào năm 1966.
Do đó, chọn đáp án A. Canadians: người Canada. Các đáp án khác không phù hợp:
 Years: các năm
 Decadcs: các thập kỳ

Hocmai – Ngôi trường chung của học trò Việt Tổng đài tư vấn: 1900 69 33 - Trang | 3 -

241
Hocmai.vn – Website học trực tuyến số 1 tại Việt Nam
Khóa học Luyện thi THPT quốc gia PEN C –N3 Môn Tiếng Anh cô Hương Fiona

 Marriages: các cuộc hôn nhân


Như vậy, đáp án B là đáp án đúng nhất
Question 3: The word “surging” in bold is closest in meaning to
A. new B. extra C. accelerating D. surprising
Key words: surging, meaning
Câu hỏi: Từ “surging” in đậm gần nghĩa nhất với từ nào?
Clue: “In September 1966 Canada’s population passed the 20 million mark. Most of this surging growth
came from natural increase”: Vào năm 1966, dân số của Canada đã vượt mốc 20 triệu người. Phần lớn
của làn sóng gia tăng này là gia tăng tự nhiên.
Phân tích: “Surging” là trào lên, dâng ỉên như một làn sóng, ám chỉ sự gia tăng rất nhanh. Vậy chọn đáp
án C. accelerating: nhanh chóng. Các đáp án khác không phù hợp.
(“Surging” in bold is closest in meaning to ...)
 New (mới) Extra (thêm mới)
 Surprising (đáng ngạc nhiên)
Question 4: The author suggests that in Canada during the 1950's
A. the urban population decreased rapidly B. fewer people married
C. economic conditions were poor D. the birth rate was very high
Keywords: Canada, 1950’s
Câu hỏi: Tác giả cho rằng Canada trong những năm 1950 như thế nào?
Clue: “The baby boom continued through the decade of the 1950s, producing a population increase of
nearly fifteen percent in the five years from 1951 to 1956... Undoubtedly, the good economic conditions of
the 1950s supported a growth in the population”: Thể hệ sinh ra sau Thế chiến thứ hai tiếp tục tăng cuối
những thập kỷ của những năm 1950, sản sinh ra lượng dân số tăng gần 15% trong vòng 5 năm từ 1951 đến
1956... Không nghi ngờ răng, điều kiện kinh tế thuận lợi của những năm 1950 đã hỗ trợ cho sự gia tăng
trong dân so.
Phân tích: Các con số là keywords thuộc về những năm 1950 đều được gắn liền với chi tiết gia tăng dân
số. Vậy chọn đáp án D. the birth rate was very high: tỉ lệ sinh rất cao.
Các đáp án khác không phù hợp:
A: The urban population decreased rapidly: dân số thành thị giảm nhanh chồng - Không có thông tin
B: Fewer people married: it người kết hôn hơn - Đây là chi tiết thuộc về năm 1930 khi “The depression of
the 1930s and the war had held back marriages”: Cuộc khủng hoàng những năm 1930 và cuộc chiền tranh
đã ngăn cản hôn nhân
C: Economic conditions were poor: các điều kiện kinh tể nghèo nàn - Sau những năm 1950 có điều kiện
kinh tế tốt
Như vậy, đáp án D là đáp án hợp lý nhất.
Question 5: The author mention all of the following as causes of declines in population growth after 1957
EXCEPT

Hocmai – Ngôi trường chung của học trò Việt Tổng đài tư vấn: 1900 69 33 - Trang | 4 -

242
Hocmai.vn – Website học trực tuyến số 1 tại Việt Nam
Khóa học Luyện thi THPT quốc gia PEN C –N3 Môn Tiếng Anh cô Hương Fiona

A. people being better educated B. people getting married earlier


C. better standards of living D. couples buying houses
Key words: causes of declines in population growth, after 1957, EXCEPT
Câu hỏi: Tác giả đề cập tới tất cả những điều sau đều là nguyên nhân của sự giảm gia tăng dân số sau năm
1957 NGOẠI TRỪ?
Clue: “….this decline... was also caused by changes in Canadian society. Young people were staying at
school longer, more women were working; young married couples were buying automobiles or houses
before star ting families; rising living standards were cutting down the size, of families”: Những người trẻ
tuổi học cao hơn, nhiều phụ nữ đi làm hơn; các cặp đôi đã kết hôn đang tiết kiệm mua xe và nhà trước khi
sinh con; mức sổng tăng cao đang cắt giảm quy mô các gia đình.
Phân tích: Tất cả các đáp án A, C, D đều xuất hiện trong clue. Đáp án B. People getting married earlier:
con ngirời kết hôn sớm hơn Không có thông tin.
A: People being better educated: con người được giáo dục tốt hơn
B: People getting married earlier: con người kết hôn sớm hơn
C: Better standards of living: mức sống tốt hơn
D: Couples buying houses: các cặp đôi mua nhà
Question 6: It can be inferred from the passage that before the Industrial Revolution
A. families were larger B. population statistics were unreliable
C. the population grew steadily D. economic conditions were bad
Key words: inferred, before the Industrial Revolution
Câu hỏi: Có thể suy đoán điều gì từ đoạn văn về thời điêm trước khi xảy ra Cách mạng Công nghiệp?
Clue: It appeared that Canada was once more falling in step with the trend toward smaller families that
had occurred all through the Western world since the time of the Industrial Revolution”: Có vẻ như
Canada lại một lần nữa rơi vào xu hướng gia đình nhỏ hơn, xu hướng đã từng xảy ra khắp phương Tây từ
cuộc Cách mạng Công nghiệp.
Phân tích: Sau cuộc Cách mạng Công nghiệp gia đình có xu hướng nhỏ đi. Vậy trước đó các gia đình có
quy mô lớn hơn. Vậy có thể suy luận về thời điểm trước cuộc Cách mạng Công nghiệp là các gia đình khi
đó lớn hơn. Ta chọn đáp án A. families were larger: những gia đình đông hơn.Scanned by CamScanner
Các đáp án khác không có thông tin.
B: Population statistics were unreliable: thống kê dân số không đáng tin cậy
C: The population grew steadily: dân số tăng đều đặn
D: Economic conditions were bad: điều kiện kỉnh tế tồi tệ
Question 7: The word “it” in bold refers to
A. horizon B. population wave C. nine percent D. first half
Keywords: it, refer to
Câu hỏi: Từ “it” in đậm trong đoạn văn đề cập đến gì?
Clue: “Although the growth in Canada’s population had slowed down by 1966... another large population

Hocmai – Ngôi trường chung của học trò Việt Tổng đài tư vấn: 1900 69 33 - Trang | 5 -

243
Hocmai.vn – Website học trực tuyến số 1 tại Việt Nam
Khóa học Luyện thi THPT quốc gia PEN C –N3 Môn Tiếng Anh cô Hương Fiona

wave was coming over the horizon. It would be composed of the children...”: Mặc dù gia tâng dân số tại
Canada đã giảm xuống vào năm 1966. . . một làn sóng dân số khác dự tinh sắp xuất hiện. Nó có thành
phần là các trẻ em. .
Phân tích: Ta phải xét xem từ nào có thể đứng làm chủ ngữ và đi được với “be composed of”. Xét các đáp
án:
 Horizon: đường chân trời: đường chân trời có thành phần là các trẻ em: Không hợp lý - Loại
 Population wave: làn sóng dân số: làn sóng dân sổ có thành phần là trẻ em: Hợp lý.- Chọn
 Nine percent: chín phần trăm: Chín phần trăm có thành phần là trè em: Không hợp lý - Loại
First half: nửa đầu: nửa đầu có thành phần là trẻ em: Không hợp lý. - LoạI

EXERCISE 3.
Read the following passage and mark the letter A, B, C, or D on your answer sheet to indicate the correct
word or phrase that best fits each of the numbered blanks from 31 to 35.
Early writing and Alphabets
When people first began to write, they did not use an alphabet. Instead, they drew small pictures to
(1) _______ the objects they were writing about. This was very slow because there was a different picture
for any word.
The Ancient Egyptians had a system of picture writing that was described hieroglyphics. The
meaning of this writing was forgotten for a very long time but in 1799 some scientists (2) _______ a stone
near Alexandria, in Egypt. The stone had been there for (3) _______ a thousand years. It had both Greek
and hieroglyphics on it and researchers were finally able to understand what the hieroglyphics meant.
An alphabet is quite different from picture writing. It (4) _______ of letters or symbols that
represent a sound and each sound is just part of one word. The Phoenicians, who lived about 3,000 years
ago, developed the modern alphabets. It was later improved by the Roman’s and this alphabet is now used
(5) _______ throughout the world.
Question 1: A. notice B. show C. appear D. mark
Notice (v): chú ý
Show (v): cho biết, chỉ ra
Appear (v): có vẻ, giống như
Mark (v): đánh dấu
Dịch: “… Thay vào đó, họ vẽ những bức vẽ nhỏ để chỉ ra những vật thể mà họ đang viết…”
Question 2: A. discovered B. realized C. delivered D. invented
Discover (v): khám phá, phát hiện

Hocmai – Ngôi trường chung của học trò Việt Tổng đài tư vấn: 1900 69 33 - Trang | 6 -

244
Hocmai.vn – Website học trực tuyến số 1 tại Việt Nam
Khóa học Luyện thi THPT quốc gia PEN C –N3 Môn Tiếng Anh cô Hương Fiona

Realize (v): nhận ra


Deliver (v): phân phát
Invent (v): phát minh, sáng chế
Dịch: “… Ý nghĩa của bài viết này đã bị lãng quên suốt một thời gian dài nhưng vào năm 1799 một số nhà
khoa học đã phát hiện ra một tảng đá gần Alexandria, ở Ai Cập…”
Question 3: A. quite B. more C. over D. already
Over + a period of time: hơn …
Dịch: “… Tảng đá đã ở đó khoảng hơn 6 nghìn năm…”
Question 4: A. consists of B. includes C. contains D. involves
Consist of: bao gồm
Dịch: “… Nó bao gồm nhiều ký tự hoặc biểu tượng mà thể hiện một âm thanh và mỗi âm thanh chỉ là một
phần của một từ…”
Question 5: A. broadly B. widely C. deeply D. hugely
Broadly (adv): chung, đại khái, đại thể
Widely (adv): một cách rộng rãi
Deeply (adv): một cách sâu sắc
Hugely (adv): rất đỗi, hết sức
Dịch: “… bảng chữ cái này hiện đang được sử dụng một cách rộng rãi khắp thế giới.”

EXERCISE 4.
Read the following passage and mark the letter A, B, C or D on your answer sheet to indicate the correct
answer to each of the questions below.
Because writing has become so important in our culture, we sometimes think of it as more real
than speech. A little thought, however, will show w hy speech is primary and writing secondary to
language. Human beings have been writing (as far as we can tell from surviving evidence) for at least 5000
years; but they have been talking for much longer, doubtless ever since there have been human beings.
When writing did develop, it was derived from and represented speech, although
imperfectly. Even today there are spoken languages that have no written form. Furthermore, we all learn to
talk well before we learn to write; any human child who is not severely handicapped physically
or mentally will learn to talk: a normal human being cannot be prevented from doing so. On the other
hand, it takes a special effort to learn to write. In the past many intelligent and useful members of
society did not acquire the skill, and even today many who speak languages with writing systems
never learn to read or write, while some who learn the rudiments of those skills do so only
imperfectly.
Hocmai – Ngôi trường chung của học trò Việt Tổng đài tư vấn: 1900 69 33 - Trang | 7 -

245
Hocmai.vn – Website học trực tuyến số 1 tại Việt Nam
Khóa học Luyện thi THPT quốc gia PEN C –N3 Môn Tiếng Anh cô Hương Fiona

To affirm the primacy of speech over writing is not, however, to disparage the latter. One
advantage writing has over speech is that it is more permanent and makes possible the records that any
civilization must have. Thus, if speaking makes us human, writing makes us civilized.
Question 1:We sometimes think of writing as more real than speech because ______.
A. writing is secondary to language
B. human beings have been writing for at least 5000 years
C. it has become very important in our culture
D. people have been writing since there have been human beings
Chúng ta thường nghĩ viết thật hơn nói bởi vì => nó rất quan trọng trong nền văn hóa của chúng ta.
Because writing has become so important in our culture, we sometimes think of it as more real than
speech.
Question 2:The author of the passage argues that ______.
A. speech is more basic to language than writing
B. writing has become too important in today’s society
C. everyone who learns to speak must learn to write
D. all languages should have a written form
Tác giả trong bài đọc tranh luận rằng => Nói là ngôn ngữ cơ bản hơn viết
Furthermore, we all learn to talk well before we learn to write; any human child who is not severely
handicapped physically or mentally will learn to talk: a normal human being cannot be prevented from
doing so.
Question 3: According to the passage, writing ______.
A. is represented perfectly by speech B. represents speech, but not perfectly
C. developed from imperfect speech D. is imperfect, but less so than speech
Theo bài đọc, kỹ năng viết thì => thể hiện lời nói nhưng không hoàn hảo.even today many who speak
languages with writing systems never learn to read or write, while some who learn the rudiments of those
skills do so only imperfectly
Question 4:Normal human beings ______.
A. learn to talk after learning to write B. learn t o write before learning to talk
C. learn to write and to talk at the same time D. learn to talk before learning to write
Người bình thường => Học nói trước khi học viết
we all learn to talk well before we learn to write
Question 5:Learning to write is ______.

Hocmai – Ngôi trường chung của học trò Việt Tổng đài tư vấn: 1900 69 33 - Trang | 8 -

246
Hocmai.vn – Website học trực tuyến số 1 tại Việt Nam
Khóa học Luyện thi THPT quốc gia PEN C –N3 Môn Tiếng Anh cô Hương Fiona

A. easy B. too difficult C. not easy D. very easy


Học viết thì => không dễ
On the other hand, it takes a special effort to learn to write
Question 6:In order to show that learning to write requires effort, the author gives the example of
______.
A. people who learn the rudiments of speech B. severely handicapped children
C. intelligent people who couldn’t write D. people who speak many languages
Để thể hiên rằng học viết cần phải nỗ lực, tác giả đưa ra ví dụ về => những người thông minh không thể
viết
In the past many intelligent and useful members of society did not acquire the skill
Question 7:In the author’s judgment, ______.
A. writing has more advantages than speech
B. writing is more real than speech
C. speech conveys ideas less accurately than writing does
D. speech is essential but writing has important benefits
Kết luận của tác giả => Nói rất cần thiết nhưng kỹ năng viết cũng có những lợi ích quan trọng
Question 8. The word “advantage” in the last paragraph most closely means ______.
A. “rudiments” B. “skill” C. “domination” D. “benefit”
Advantage = benefit: lợi ích, thuận lợi

EXERCISE 5
Read the following pasage and mark the letter A, B, C or D on your answer sheet ti indicate the correct
answer to each of the questions
Over the past 600 years, English has grown from a language of few speakers to become the
dominant language of international communication. English as we know it today emerged around 1350,
after having incorporated many elements of French that were introduced following the Norman invasion
off 1066. Until the 1600s, English was, for the most part, spoken only in England and had not expanded
even as far as Wales, Scotland, or Ireland. However, during the course of the next two century, English
began to spread around the globe as a result of exploration, trade (including slave trade), colonization, and
missionary work. Thus, small enclaves of English, speakers became established and grew in various parts
of the world. As these communities proliferated, English gradually became the primary language of
international business, banking, and diplomacy.

Hocmai – Ngôi trường chung của học trò Việt Tổng đài tư vấn: 1900 69 33 - Trang | 9 -

247
Hocmai.vn – Website học trực tuyến số 1 tại Việt Nam
Khóa học Luyện thi THPT quốc gia PEN C –N3 Môn Tiếng Anh cô Hương Fiona

Currently, about 80 percent of the information stored on computer systems worldwide is in


English. Two thirds of the world's science writing is in English, and English is the main language of
technology, advertising, media, international airport, and air traffic controllers. Today there are more than
700 million English users in the world, and over half of these are non-native speakers, constituting the
largest number of non-native users than any other language in the world.
Question 1:What is the main topic of the passage?
A. The number of non-native users of English.
B. The French influence on the English language.
C. The expansion of English as an international language.
D. The use of English for science and technology.
Tiêu đề phù hợp nhất của bài viết là The expansion of English as an international language. (Sự phát
triển của Tiếng anh như một ngôn ngữ quốc tế)
Thông tin ở 2 câu đầu tiên của 2 đoạn:
- Over the past 600 years, English has grown from a language of few speakers to become the dominant
language of international communication.
- Currently, about 80 percent of the information stored on computer systems worldwide is in English.
Question 2:: In the first paragraph, the word "emerged" is closest in meaning to
A. appeared B. hailed C. frequented D. Engaged
“emerge” = “appear”: xuất hiện, nổi lên
Question 3:: In the first paragraph, the word "elements" is closest in meaning to
A. declaration B. features C. curiosities D. Customs
“elements” = “features”: những yếu tố, những nguyên tố, những đặc trưng
Question 4:: Approximately when did English begin to be used beyond England?____________ .
A. In 1066 B. around 1350 C. before 1600 D. after 1600
Thông tin ở câu thứ 3 đoạn đầu tiên: Until the 1600s, English was, for the most part, spoken only in
England (Cho đến những năm 1600, Tiếng anh, ở hầu hết khu vực, được sử dụng duy nhất ở Anh)
Question 5:: According to the passage, all of the following contributed to the spread of English around the
world except ____________ .
A. the slave trade B. the Norman invasion
C. missionaries D. Colonization
A, C, D đều được nhắc đến trong câu: However, during the course of the next two century, English
began to spread around the globe as a result of exploration, trade (including slave trade), colonization,
and missionary work.
Chỉ có duy nhất B không được đề cập đến
Question 6: In the second paragraph, the word "stored" is closest in meaning to____________ .
A. bought B. saved C. spent D. Valued
stored” = saved: được lưu, được lưu trữ

Hocmai – Ngôi trường chung của học trò Việt Tổng đài tư vấn: 1900 69 33 - Trang | 10 -

248
Hocmai.vn – Website học trực tuyến số 1 tại Việt Nam
Khóa học Luyện thi THPT quốc gia PEN C –N3 Môn Tiếng Anh cô Hương Fiona

Question 7:According to the passage, approximately how many non-native users of English are there in
the world today ? ____________ .
A. a quarter million B. half a million
C. 350 million D. 700 million
Thông tin ở câu cuối của đoạn 2: Today there are more than 700 million English users in the world,
and over half of these are non-native speakers (Có hơn 700 triệu người nói tiếng anh, và hơn 1 nửa số
đó không phải người bản địa)

Nguồn : Hocmai.vn

Giáo viên : Hương Fiona

Hocmai – Ngôi trường chung của học trò Việt Tổng đài tư vấn: 1900 69 33 - Trang | 11 -

249
Hocmai.vn – Website học trực tuyến số 1 tại Việt Nam
Khóa học Luyện thi THPT quốc gia PEN C –N2 Môn Tiếng Anh thầy Phạm Trọng Hiếu

PHƯƠNG PHÁP LÀM CÂU GIAO TIẾP


( ĐÁP ÁN BÀI TẬP TỰ LUYỆN )
Giáo viên : Phạm Trọng Hiếu

Mark the letter A, B, C, or D to indicate the most suitable response to complete the following exchanges
Question 1. - Kate:" How lovely your cats are!" - David:" ________ "
A. Really? They are B. Thank you, it is nice of you to say so
C. Can you say it again? D. I love them, too
Dịch:
- Kate: "Những con mèo của bạn thật dễ thương làm sao” -David:" ________ "
A. Thật sao?
B. Cảm ơn bạn, bạn thật tốt khi đã nói như thế
C. Bạn có thế nói lại lần nữa được không?
D. Tôi cũng yêu chúng.
Question 2. - Anne:" Make yourself at home" - John: “ ________ ”
A. Thanks! Same to you B. That's very kind. Thank you
C. Not at all. Don't mention it D. Yes, Can I help you?
Dịch:
- Anne:" Cứ tự nhiên như ở nhà nhé." -John:" ________ "
A. Cảm ơn. Bạn cũng thế. B. Thật là tốt. Cảm ơn bạn.
C. Không có gì. Đừng lưu ý đến nó D. Vâng. Tôi có thế giúp gì cho bạn?
Question 3. - Dad: ‘I have bought you a toy. Happy birthday to you!’ - Son- ‘_______’
A. Have a nice day! B. The same to you!
C. What a lovely toy! Thanks. D. What a pity!
Dịch:
“Bố mua đồ chơi mới cho con này, Chúc mừng sinh nhật con!!!!” “ Đẹp quá bố, cảm ơn bố ạh”
Question 4. ‘When can you get it all done?' – ‘_______’
A. How does next Monday sound? B. Two hours ago
C. I used to do it on Monday D. How much time do 1 need?
Dịch:
Câu cho sẵn hỏi về thời gian và câu trả lời gợi ý dịch là "Thế thứ hai thì sao?"
Question 5. - Joan: Our friends are coming _______, Mike?’

Hocmai– Ngôi trường chung của học trò Việt Tổng đài tư vấn: 1900 69 33 - Trang | 1-

250
Hocmai.vn – Website học trực tuyến số 1 tại Việt Nam
Khóa học Luyện thi THPT quốc gia PEN C –N2 Môn Tiếng Anh thầy Phạm Trọng Hiếu

- Mike: I’m sorry, I can't do it now.'


A. Why don’t we cook some coffee B. Shall I make you like some coffee
C. Shall you make some coffee, please D. Would you mind making some coffee
Dịch:
Đề nghị ai làm điều gì bằng cấu trúc Wound you mind + V ing
Question 6: 'Would you like to have dinner with me?’ ‘_______’
A. Yes, it is B. Yes, so do I
C. I’m very happy D. Yes, I'd love to
→ Đáp lại lời đề nghị lịch sự
“ Tối nay bạn ăn tối với mình nhé”
“ Được, mình rất thích”
Question 7: A: I don’t like meat. - B: ________.
A. Nor I do B. I don’t too. C. Neither don’t I. D. Neither do I
Giải thích:
Để diễn tả sự giống nhau “cũng”, ta có thể dùng câu trả lời ngắn với so, too, either, neither.
Khẳng định So + tobe/ trợ động từ + S S tobe/ trợ động từ, too.
Phủ định Neither + tobe/ trợ động từ + S S tobe/ trợ động từ (có not), either.

Ở đây người thứ nhất dùng phù định don’t like nên đối chiếu ta có neither do I hoặc I don’t, either.
Nor = neither.
Dịch nghĩa: Tôi không thích thịt. - Tôi cũng không
Question 8: Cindy wanted to drive her best friend to have dinner at an Italian restaurant but her car
couldn’t start. Therefore, she had to borrow one from Nancy. Choose the most suitable response to fill in
the blank in the following exchange.
Cindy: “Would you mind lending me your car?” - Nancy: “________.”
A. Yes. Here it is B. Yes, let’s
C. No, not at all D. Great
Giải thích:
Would/ do you mind - Bạn có phiền không - dùng để xin phép. Để đồng ý cho người đó làm, ta sẽ trả lời
“không phiền”, chẳng hạn như no, not at all/ go ahead/ I don’t mind.
Còn nếu không đồng ý, nghĩa là bạn có phiền, thì sẽ trả lời bằng cách đưa ra các lí do từ chối, thỉnh thoảng
cũng sử dụng Yes, I would, nhưng không phổ biến vì hơi mất lịch sự.
Dịch nghĩa: Bạn có phiền nếu cho tôi mượn xe không? - Không, không phiền tý nào

Hocmai– Ngôi trường chung của học trò Việt Tổng đài tư vấn: 1900 69 33 - Trang | 2-

251
Hocmai.vn – Website học trực tuyến số 1 tại Việt Nam
Khóa học Luyện thi THPT quốc gia PEN C –N2 Môn Tiếng Anh thầy Phạm Trọng Hiếu

Question 9: “Can I have a look at that pullover, please?”


“_________________________.”
A. Sorry, it is out of stock. B. Which one? This one?
C. It’s much cheaper. D. Can I help you?
Dịch:

- Tôi có thể xem qua cái áo phông đó được không?

A. Xin lỗi, hết hàng rồi ạ.

B. Cái nào? Cái này à?

C. Nó rẻ hơn nhiều.

D. Tôi có thể giúp gì cho ngài?

Question 10: “Thanks for helping me with my homework, John.”


“_________________________.”
A. Don’t mention it. B. Please, don’t blame yourself.
C. No harm done. D. I don’t mean so.
- Cảm ơn vì đã giúp tôi làm bài tập về nhà nhé John.

A. Đừng bận tâm làm gì.

B. Làm ơn đừng tự đổ lỗi cho mình nữa.

C. Không thiệt hại gì.

D. Tôi không cố ý.

Question 11: “What's your birthday, Ann?” – “It's _____.”


A. on the thirty-one of July B. on July the thirty-one
C. on the thirty-first of July D. in July the thirty- first
“thirty – first of July”: ngày 31 tháng 7.
Các đáp án còn lại không có nghĩa.
Dịch: “Khi nào sinh nhật bạn, Ann?”
“Nó vào ngày 31 tháng 7.”
Question 12: “Sugar?” – “________”
A. No, no me! B. No, I don't. C. Thanks, I'd like. D.Yes,please
No, no me!: Không, không phải tôi
No, I don't: từ chối của câu hỏi Y/N question
Thanks, I'd like: cảm ơn, tôi rất thích

Hocmai– Ngôi trường chung của học trò Việt Tổng đài tư vấn: 1900 69 33 - Trang | 3-

252
Hocmai.vn – Website học trực tuyến số 1 tại Việt Nam
Khóa học Luyện thi THPT quốc gia PEN C –N2 Môn Tiếng Anh thầy Phạm Trọng Hiếu

Yes, please: Vâng, làm ơn


Dịch: “Thêm đường nhé!”
“Vâng, làm ơn”
Question 13: A: “Can I help you, madam?”
B: “______________________”
A. Yes, it's in our summer sale B. It's very cheap.
C. No, thanks. I'm just looking D. Right. It looks a bit small
Tạm dịch: A: Tôi cổ thể giúp gì cho bà không ạ?
-Ố không, tôi chi xem thôi
Question 14: “Do you mind if I switch the light off?”
- “_________________”
A. Yes, I mind it, sorry. B. What if I don’t mind it?
C. Yes, please do it. D. I’d rather you didn’t, if you don’t mind.
Tạm dịch: Bạn có thấy phiền không nếu tôi tắt đèn đi. Would rather somebody did not do something: muốn
ai đó không làm gì thì hơn
“Bạn có phiền nếu tôi tảt điện không?’’-“bạn không cần làm như thế đâu, nếu như bạn không thấy phiền.
Do đó đáp án chính xác là D.
Các lựa chọn còn lại không phù hợp.
A. Xin lỗi, nhưng tôi thấy phiền lắm.
B. Nếu tôi không phiền thì làm sao.
C. Vâng, cứ thể làm đi.
Question 15: Harry:" ______ "
Kate: "Yes. I'd like to buy a computer. "
A. Do you look for something? B. Good morning. Can I help you?
C. Excuse me. Do you want to buy it? D. Can you help me buy something?
Lời đáp của khách hàng là muốn mua một chiếc máy tính nên câu hỏi mang tính chào mời của người bán
sẽ là “Good morning. Can I help you?”
Question 16: Peter: “I’ve been awarded a scholarship to study in America.”
Kate: “Uh. really?_________!”
A. Take care of yourself B. Congratulations

Hocmai– Ngôi trường chung của học trò Việt Tổng đài tư vấn: 1900 69 33 - Trang | 4-

253
Hocmai.vn – Website học trực tuyến số 1 tại Việt Nam
Khóa học Luyện thi THPT quốc gia PEN C –N2 Môn Tiếng Anh thầy Phạm Trọng Hiếu

C. You are always lucky D. Lucky as you are


Peter thông báo rằng anh vừa được trao học bổng đi học tại Úc. Vì vậy, Kate đáp lại bằng lời chúc mừng
“Congratulations! ”
Question 17: - "Mr. Adams is going to retire next month."
- “_____________.”
A. Oh, I have no idea C. Right, you’d probably be the next.
B. You don't say! D. Congratulations!
“You don’t say!”: a general response to something that someone has said (expresses a little polite surprise
or interest, but not disbelief): Một câu trả lời chung chung cho điều gì ai đó nói, thế hiện một sự ngạc hiên
hoặc quan tâm một cách lịch sự, không phải để thể hiện thiểu tin tưởng vào lời nói của người kia
Question 18: - “________________”
- "Never mind, better luck next time."
A. I’ve broken your precious vase. B. I have a lot on my mind,
C. I couldn’t keep my mind on work. D. I didn't get the vacant position
Tạm dịch câu trả lời: Không sao đâu, lần sau may mắn hơn nhé!
Dễ dàng thấy câu đáp lại này là một câu an ủi, động viên. Do đó, người đầu tiên phải nói một câu mang
Question 19: - Jordan: "________"
- Jim: "No, thanks."
A. Would you want another drink? B. Would you care for another drink?
C. Can you help me with this? D. Come in, please! Breath
“No, thanks” thường được sử dụng để từ chổi một lời mời. Trong các đáp án chỉ có đáp án A và B là lời
mời. Tuy nhiên đáp án A sử dụng từ “want” không được lịch sự. Mặt khác, Care for = like. Do đó ta chọn
đáp án B
Question 20: Jane: “It's going to rain”.
Mary: “________.”
A. I hope not so B. I hope not C. I don't hope so D. I don't hope either
Để thể hiện hi vọng trái với ý kiến được đưa ra sử dụng cấu trúc I hope not (Tôi hi vọng là không). Thể
hiện hi vọng đồng với ý kiến được đưa ra sử dụng I hope so (Tôi hy vọng vậy).
tính chất mang lại tin buồn của chính người đó.
I've broken your precious vase: Tôi đã làm vỡ lọ hoa quý báu của anh rồi
I have a lot on my mind: Tôi đang có nhiều

chuyện phiền não quá


I couldn't keep my mind on work: Tôi không thề tập trung vào công việc được

Hocmai– Ngôi trường chung của học trò Việt Tổng đài tư vấn: 1900 69 33 - Trang | 5-

254
Hocmai.vn – Website học trực tuyến số 1 tại Việt Nam
Khóa học Luyện thi THPT quốc gia PEN C –N2 Môn Tiếng Anh thầy Phạm Trọng Hiếu

I didn't get the vacant position: Tôi đã không nhận được vị trí đang còn trống đó ”
“Vacant” ở đây không liên quan tới “vacation”, không phải là “xuất đi nghi”, tránh nhầm lẫn “ vacant” và
“ vacation”. “Vacant” (a): bị bỏ không, trống rỗng, không có người làm
Question 21: A: I’ve been working at this school for 15 years.
B: __________________
A. Why did you leave? B. Are you going to retire soon?
C. Why had you left? D. Will you retire?
Vì câu đầu sử dụng thì hiện tại hoàn thành tiếp diễn (=> vẫn còn đang làm việc ở trường) nên không thể
chọn A hay C. Còn D “Will you...” chỉ dùng để yêu cầu ai làm gì đó.
Tạm dịch: Tôi đã làm việc ở trường này được 15 năm rồi -Anh có dự định nghỉ việc không?
Question 22: “A: Would you bother if I had a look at your paper?’
B: ______________
A. You’re welcome B. That’s a good idea
C. Oh, I didn’t realize it. D. Well, actually I’d rather you didn’t
Tạm dịch: Bạn có phiền không nếu tôi nhìn bài của bạn? - Ừm thực ra tôi nghĩ bạn đừng làm thế thì hơn.
Nhiều bạn có thể thắc mắc rằng tại sao đáp án A “You're welcome” không đúng. Đó là do ta dùng câu
này trong trường hợp đáp lại lời cảm ơn thôi. Còn để nói đồng ý với câu đã cho thì phải trả lời “No, not at
all” hay “No problem” - Tôi không phiền đâu
Question 23: - “You must be Jane's sister. Glad to meet you.
“______________”
A. I am, either B. So I am. I'm glad C. What do you do D. Me too
Người đầu tiên nói “Rất vui được gặp bạn” - Đáp lại;
❖ Me too nghĩa là Tôi cũng vậy (Tôi cũng rất vui được gặp bạn).
Các đáp án khác không phù hợp:
❖ I am, either Sai, either đứng trong câu phủ định
❖ So I am. I'm glad: Sai vì phải đảo ngữ là So am I
What do you do: Câu hỏi nghề nghiệp. Dùng đáp trả trong trường hợp này là không lịch sự
Question 24: - “How lovely your pets are!”
“_______________”
A. Thank you, it's nice of you to say no B. Really? They are
C. can you say that again D. I love them, too
Thank you, It’s nice of you to say so: là lời phúc đáp thích hợp nhất cho câu cảm thán phía trên

Hocmai– Ngôi trường chung của học trò Việt Tổng đài tư vấn: 1900 69 33 - Trang | 6-

255
Hocmai.vn – Website học trực tuyến số 1 tại Việt Nam
Khóa học Luyện thi THPT quốc gia PEN C –N2 Môn Tiếng Anh thầy Phạm Trọng Hiếu

Question 25: Mary is talking to a porter in the hotel lobby.


Porter: “Shall I help you with your suitcase?” - Mary: “_______”
A. Not a chance. B. That's very kind of you.
C. I can’t agree more. D. What a pity!
❖ Dịch: Mary đang nói chuyện với người trông cửa trong khách sạn.
❖ Người trông cửa: “Để tôi giúp cô với cái vali nhé?” – Mary: “Anh thật tốt bụng.”
Question 26: Hoa is asking Hai, who is sitting at a corner of the room, seeming too shy.
Hoa: “Why aren’t you taking part in our activities? _______” - Hai: “Yes. I can. Certainly.”
A. Could you please show me how to get to the nearest post office?
B. Shall I take your hat off?
C. Can I help you?
D. Can you help me with these decorations?
❖ Dịch: Hoa đang hỏi Hải, ai đang ngồi ở góc phòng kia, đang có vẻ rất xấu hổ.
Hoa: “Tại sao bạn không tham gia vào hoạt động của tụi mình? Bạn có thể giúp tôi với phần trang trí này
không?” – Hải: “Được, chắc chắn rồi.”
Question 27: “I don’t think I can do this.” – “_______”
A. Sure, no way! B. Yeah. It’s not easy.
C. Oh, come on! Give it a try! D. No, I hope not
Dịch: “Tôi không nghĩ mình có thể làm điều này.” – “Ồ, thôi nào! Hãy cứ thử xem!”
Question 28: Peter: “Is it important?” - Thomas: “_______”
A. Not on your life! B. It’s matter of life and death!
C. No worry, that’s nothing. D. It’s ridiculous
Peter: “Nó có quan trọng không?” – Thomas: “Đó là vấn đề sống còn đấy!”
Question 29: Bob: “Our team has just won the last football match.”
Michael: “_______!”
A. Good idea. Thanks for the news B. Yes. I guess it’s very good
C. Well, that’s very surprising D. Yes, it’s our pleasure
Bob: “Đội của chúng ta đã thắng trận đá bóng chung kết rồi.”
Michael: “Ồ, thật là đáng ngạc nhiên!”
Question 30: “_______” - “Oh, it’s great!”
A. How is the English competition like?
B. Would you like the English competition?
C. What do you like about the English competition?

Hocmai– Ngôi trường chung của học trò Việt Tổng đài tư vấn: 1900 69 33 - Trang | 7-

256
Hocmai.vn – Website học trực tuyến số 1 tại Việt Nam
Khóa học Luyện thi THPT quốc gia PEN C –N2 Môn Tiếng Anh thầy Phạm Trọng Hiếu

D. What do you think of the English competition?


Dịch: “Bạn nghĩ gì về kỳ thi tiếng Anh?” – “Oh, nó thật tuyệt!”

Nguồn : Hocmai.vn
Giáo viên : Phạm Trọng Hiếu

Hocmai– Ngôi trường chung của học trò Việt Tổng đài tư vấn: 1900 69 33 - Trang | 8-

257
Hocmai.vn – Website học trực tuyến số 1 tại Việt Nam
Khóa học Luyện thi THPT quốc gia PEN C–N3 Môn Tiếng Anh côHương Fiona

PHƯƠNG PHÁP LÀM BÀI TỪ ĐỒNG NGHĨA-TRÁI NGHĨA


( BÀI TẬP TỰ LUYỆN )
Giáo viên : Nguyễn Thanh Hương
Các bài tập trong tài liệu này được biên soạn kèm theo bài giảng “Phương pháp làm bài từ đồng nghĩa-trái nghĩa” thuộc Khóa học
Luyện thi THPT quốc gia PEN - C: Môn Tiếng Anh (cô Hương Fiona )” tại website Hocmai.vn để giúp các Bạn kiểm tra, củng cố lại
các kiến thức được giáo viên truyền đạt trong bài giảng tương ứng. Để sử dụng hiệu quả, Bạn cần học trước bài giảng, sau đó làm đầy
đủ các bài tập trong tài liệu này.

Mark the letter A, B, C or D on your answer sheet to indicate the word(s) CLOSEST in meaning to the
underlined word(s) in each of the following questions.
Question 1: Readers are required to abide by the rules of the lybrary and mind their manners.
A. memorize B. obey C. compose D. review
→ Abide by: tuân thủ = obey: vâng lời.
Dịch: Người đọc cần phải tuân thủ các quy tắc của người lybrary và thận trọng với cách cư xử của họ
Question 2: The whole village was wiped out in the earthquake last night.
A. Cleaned well B. destroyed C. changed completely D. removed quickly
→ Wipe out = destroy: phá hủy
Dịch: Toàn bộ ngôi làng đã bị phá hủy trong trận bão tối qua.
Mark the letter A, B, C or D on your answer sheet to indicate the word(s) OPPOSITE in meaning to
the underlined word(s) in each of the following questions
Question 3: They have not made any effort to intergrate with the local community.
A. Put together B. separate C. connect D. cooperate
→ intergrate: hòa nhập, hợp tác >< separate: tách ra, phân chia
Put together :đặt cùng với nhau
Separate: chia rẽ
Connect: kết nối
Dịch: Họ vẫn chưa có nhiều nỗ lực để hòa nhập với cộng đồng địa phương
Question 4: Experts hope that the vaccine will be mass-produced soon.
A. Produced with high cost B. Produced in great numbers
C. Produced cheaply D. Produced in small numbers
→ Dịch: Các chuyên gia hy vọng rằng vacxin sẽ được sản xuất hàng loạt sớm.
A. Sản xuất với chi phí cao B. Sản xuất với số lượng lớn
C. sản xuất với giá rẻ D. Được sản xuất với số lượng nhỏ
mass-produced: sản xuất hàng loạt >< Produced in small numbers

Hocmai– Ngôi trường chung của học trò Việt Tổng đài tư vấn: 1900 69 33 - Trang | 1-

258
Hocmai.vn – Website học trực tuyến số 1 tại Việt Nam
Khóa học Luyện thi THPT quốc gia PEN C–N3 Môn Tiếng Anh côHương Fiona

Mark the letter A,B,C or D to indicate the word or phrase that is OPPOSITE in meaning to the
underlined part in each of the following questions.
Question 5: I find it hard to work at home because there’re too many distractions.
A. Unawareness B. unconcern C. carelessness D. attention
→ Giải thích : distraction (n) : sự phân tâm >< attention (n) : sự tập trung
Question 6: In 1864 George Pullman designed a sleeping car that eventually saw widespread use.
A. previously B. ultimately C. familiarly D. simultaneously
→ Giải thích : eventually (adv) = ultimately = at last = finally : cuối cùng >< previously : ban đầu
Familiarly: một cách quen thuộc
Simultaneously: đồng thời.
Mark the letter A, B, C, or D on your answer sheet to indicate the word CLOSEST in meaning to the
underlined word in each of the following questions.
Question 7: The situation seems to be changing minute by minute.
A. from time to time B. time after time C. again and again D. very rapidly
→ minute by minute: từng phút từng phút một, thể hiện sự nhanh chóng
đồng nghĩa là very rapidly: một cách rất nhanh, một cách nhanh chóng
from time to time = sometimes: thỉnh thoảng
time after time: nhiều lần, thường xuyên
again and again: nhiều lần
Dịch: Tình hình dường như đang thay đổi từng phút một
Question 8: An interesting feature of this park is the Orphanage where lots of orphaned or abandoned animals
are taken care of.
A. immoral B. wicked C. shameless D. deserted
→ Abandoned: bị bỏ rơi, bị vứt bỏ
Đồng nghĩa là deserted: bị bỏ, bị vứt đi
Immoral: vô đạo
Wicked: xấu, tệ, đồi bại, độc ác, tội lỗi
Shameless: không biết xấu hổ, trơ trẽn, vô liêm sỉ
Dịch câu: Một tính năng thú vị của công viên này là trại mồ côi nơi nhiều động vật mồ côi hoặc bị bỏ rơi được
đưa về chăm sóc.
Mark the letter A, B, C, or D on your answer sheet to indicate the word(s) OPOSITE in meaning to the
underlined word(s) in each of the following questions.
Question 9: We can’t make any conclusion right now with this paucity of information.

Hocmai– Ngôi trường chung của học trò Việt Tổng đài tư vấn: 1900 69 33 - Trang | 2-

259
Hocmai.vn – Website học trực tuyến số 1 tại Việt Nam
Khóa học Luyện thi THPT quốc gia PEN C–N3 Môn Tiếng Anh côHương Fiona

A. excess B. certainty C. timidity D. scarcity


→ Paucity: sự ít, sự ít ỏi
Trái nghĩa là excess: vượt quá, nhiều vượt quá
Certainty: sự chắc chắn
Timidity: sự rụt rè
Scarcity: sự khan hiếm
Dịch câu: Chúng tôi không thể đưa ra bất kỳ kết luận ngay bây giờ với số lượng ít ỏi thông tin này
Question 10: The law restricting pollution in the city are very rigid.
A. stiff B. strictly C. strict D. tolerant
→ Rigid: cứng nhắc, cứng
Trái nghĩa là flexible: linh hoạt
Stiff: cứng, cứng đơ, đặc quánh
Strict: cứng nhắc, nghiêm khắc
Tolerant: khoan dung; tha thứ
Dịch: Luật hạn chế ô nhiễm trong thành phố rất cứng nhắc
Mark the letter A, B, C, or D on your answer sheet to indicate the word(s) CLOSEST in meaning to the
underlined word(s) in each of the following questions.
Question 11: The air is naturally contaminated by foreign matter such as plant pollens and dust.
A. filled B. concentrated C. polluted D. occupied
fill: điền
concentrate: tập trung
pollute: ô nhiễm
occuppied: chiếm giữ
Contaminate = pollute: ô nhiễm
Dịch: Không khí bị ô nhiễm một cách tự nhiên bởi những tác nhân bên ngoài như phấn hoa và bụi bẩn.
Question 12: Shake a leg or you will miss the train.
A. Hurry up B. Slow down C. Watch out D. Put down
A: nhanh lên B: chậm lại
C: trông chừng D: đặt xuống
Shake a leg = hurry up: nhanh lên
Dịch: Nhanh lên hoặc là anh sẽ lỡ tàu đó.
Mark the letter A, B, C, or D on your answer sheet to indicate the word(s) OPPOSITE in meaning to the
underlined word(s) in each of the following questions.

Hocmai– Ngôi trường chung của học trò Việt Tổng đài tư vấn: 1900 69 33 - Trang | 3-

260
Hocmai.vn – Website học trực tuyến số 1 tại Việt Nam
Khóa học Luyện thi THPT quốc gia PEN C–N3 Môn Tiếng Anh côHương Fiona

Question 13: People sometimes choose partners who compensate for their own shortcomings.
A. strengths B. disadvantages C. weaknesses D. benefits
A: điểm mạnh B: bất lợi
C: điểm yêu D: lợi ích
Shortcoming: điểm yếu >< strength
Dịch: mọi người thỉnh thoảng chọn bạn đồng hành – người có thể bù đắp những điểm yếu của họ
Question 14: You should pat yourself on the back for having achieved such a high score in the graduation
exam.
A. praise yourself B. criticize yourself C. check up your back D. wear a backpack
A: tự khen mình
B: tự chỉ trích mình
C: kiểm tra lưng mình
D: đeo 1 cái ba lô
Pat yourself on the back : khen ngợi chính mình >< criticize yourself
Dịch: Bạn nên tự khen mình vì đã đạt được điểm cao trong kì thi tốt nghiệp
Mark the letter A, B, C or D on your answer sheet to indicate the word or phrase that is OPPOSITE is
meaning to the underlined part in each of the following questions.
Question 15: My friend tends to look on the bright side in any circumstances.
A. be smart B. be confident C. be pessimistic D. be optimistic
Giải thích: look on the bright side: lạc quan >< C. be pessimistic: bi quan
Dịch: Bạn tôi thường rất lạc quan trong mọi trường hợp
Question 16: All children can attend without paying fees at state school.
A. high schools B. primary schools
C. private schools D. secondary schools
Giải thích: State chool: trường công.>< private schools: trường tư
Dịch: Tất cả học sinh học trường công đều không phải trả phí
Mark the letter A, B, C or D on your answer sheet to indicate the word or phrase that is CLOSEST is
meaning to the underlined part in each of the following questions.
Question 17: They decided to postpone their journey till the end of the month because of epidemic.
A. put off B. turn round C. do with D. carry on
Giải thích: postpone: trì hoãn = put off.
turn round: quay người lại
do with: làm cùng ai/ cái gì

Hocmai– Ngôi trường chung của học trò Việt Tổng đài tư vấn: 1900 69 33 - Trang | 4-

261
Hocmai.vn – Website học trực tuyến số 1 tại Việt Nam
Khóa học Luyện thi THPT quốc gia PEN C–N3 Môn Tiếng Anh côHương Fiona

carry on: tiếp tục


Dịch: Họ quyết định hoãn chuyến đi đến tận cuối tháng vì bệnh dịch
Question 18: In the future many large corporations will be wiped out an millions of jobs will be lost.
A. destroyed B. developed C. broadened D. opened
Giải thích: be wiped out = be destroyed.
Dịch: Trong tương lai nhiều công ty sẽ bị mất hàng ngàn công việc sẽ bị mất
Choose A, B, C or D to indicate the word that is OPPOSITE in meaning to the underlined part in each
of the following questions:
Question 19: Mr.Smith’s new neighbors appear to be very friendly.
A. Inapplicable B. hostile C. amiable D. futile
A: Không áp dụng được
B: Không thân thiện, thù địch
C: Đáng yêu
D: Vô ích
Dịch : Hàng xóm mới của ông Smith có vẻ rất thân thiện.
Friendly >< hostile
Question 20: Henry has found a temporary job in a factory.
A. Genuine B. eternal C. permanent D. satisfactory
A: Thông minh, thiên tài
B: Vĩnh cửu
C: mãi mãi
D: hài lòng
Dịch: Henry vừa tìm được một công việc tạm thời trong nhà máy.
Temporary: tạm thời >< permanent
Choose A, B ,C or D to indicate the word that is CLOSEST in meaning to the underlined part in each
sentence.
Question 21: Carpets from countries such as Persia and Afghanistan often fetch high prices in the United
States.
A. Artifacts B. Textile C. Pottery D. Rugs
Carpet = rug : tấm thảm
A: Đồ nhân tạo B: Hàng dệt
C: gốm D: thảm
Dịch: Những tấm thảm từ các quốc gia như Persia và Afghanista thường có giá cao ở Mỹ

Hocmai– Ngôi trường chung của học trò Việt Tổng đài tư vấn: 1900 69 33 - Trang | 5-

262
Hocmai.vn – Website học trực tuyến số 1 tại Việt Nam
Khóa học Luyện thi THPT quốc gia PEN C–N3 Môn Tiếng Anh côHương Fiona

Question 22: Though many scientific breakthroughs have resulted from mishaps it has taken brilliant thinkers
to recognize their potential.
A. Incidental B. misunderstandings C. accidents D. misfortunes
Mishap = accident: tai nạn, rủi ro,
A: tình cờ
B: sự hiểu làm
C: tai nạn
D: bất hạnh
Dù nhiều đột phá khao học có được từ những tai nạn nhưng nó đã giúp các nhà tư tưởng xuất chúng nhận ra
tiềm năng của mình.
Mark the letter A, B. C, or D on your answer sheet to indicate the word(s) CLOSEST in meaning to the
underlined word(s) in each of the following
Question 23: An international medical conference initiated by Davison resulted in the birth of the League of
Red Cross Societies in 1991.
A. treated B. dedicated C. helped D. started
Initiated= started: bắt đầu/ khởi xướng
A. đối xử
B. cống hiến
C. giúp đỡ
D. bắt đầu
Câu này dịch như sau: 1 hội nghị y tế thế giới được khởi xướng bởi Davison đã dẫn đến sự hình thành hội chữ
thập đỏ năm 1991
Question 24: He drove so fast that I really felt my life was in danger.
A. at the target B. in accident C. at stake D. in comfort
A. ở mục tiêu
B. trong vụ tai nạn
C. trong nguy hiểm, bị đe dọa
D. thoải mái
Câu này dịch như sau: anh ấy lái xe quá nhanh đến nỗi tôi thực sự cảm thấy mạng sống của mình đang trong
tình thế nguy hiểm
Mark the letter A,B,C or D on your answer sheet to indicate the word(s) OPPOSITE in meaning to the
underlined word(s) in each of the following questions
Question 25: Be quick! We must speed up if we don’t want to miss the flight”

Hocmai– Ngôi trường chung của học trò Việt Tổng đài tư vấn: 1900 69 33 - Trang | 6-

263
Hocmai.vn – Website học trực tuyến số 1 tại Việt Nam
Khóa học Luyện thi THPT quốc gia PEN C–N3 Môn Tiếng Anh côHương Fiona

A. put forward B. look up C. slow down D. turn down


Speed up: nhanh lên >< C. slow down : chậm lại
A. put forward: đệ trình
B. look up: tìm kiếm, tra cứu
C. slow down : chậm lại
D. turn down: từ chối
Question 26: Although it’s a long day for us, we feel we are contented with what we do
A. interested B. dissatisfied C. excited D. shocked
Contented: thỏa mãn>< B. dissatisfied: thất vọng
A. interested : thích thú
B. dissatisfied: thất vong
C. excited: hào hứng
D. shocked: sốc
Mark the letter A, B, C, or D on your answer sheet to indicate the word(s) CLOSEST in meaning to the
underlined word (s) in each of the following questions
Question 27: Some studies have shown a strong association between pesticide and certain diseases.
A. cooperation B. collaboration C. consequence D. connection
Association: mối liên hệ/ sự liên quan
cooperation : sự hợp tác
collaboration: sự cộng tác
consequence: hậu quả
connection: sự kết nối
Question 28: Although they hold similar political views, their religious beliefs present a striking contrast.
A. minor comparison B. interesting resemblance
C. significant discrepancy D. complete coincidence
striking contrast: sự khác biệt đáng kể
A.sự so sánh nhỏ
B. sự giống nhau thú vị
C. sự khác biệt đáng kể
D. sự trùng hợp hoàn toàn
Câu này dịch như sau: Mặc dù họ có quan điểm chính trị giống nhau, nhưng tín ngưỡng tôn giáo của họ thể
hiện sự khác nhau đáng kể.
=>striking contrast = significant discrepancy

Hocmai– Ngôi trường chung của học trò Việt Tổng đài tư vấn: 1900 69 33 - Trang | 7-

264
Hocmai.vn – Website học trực tuyến số 1 tại Việt Nam
Khóa học Luyện thi THPT quốc gia PEN C–N3 Môn Tiếng Anh côHương Fiona

Câu này dịch như sau: Một số nghiên cứu cho thấy sự kiên kết mạnh mẽ giữa thuốc trừ sâu và một số căn
bệnh nhất định.=>Association = connection
Mark the letter A, B, C, or D on your answer sheet to indicate the word(s) OPPOSITE in meaning to the
underlined word (s) in each of the following questions
Question 29: The research, contrary to common belief, revealed some rather unexpected results.
A. impressive B. predictable C. surprising D. positive
unexpected : ngoài mong đợi
impressive: ấn tượng
predictable: có thể đoán trước được
surprising: ngạc nhiên
positive: tích cực
Câu này dịch như sau: Nghiên cứu, ngược lại với nhận định chung, tiết lộ một số kết quả ngoài mong đợi.
=>unexpected >< predictable
Question 30: I tried to patch things up after the argument, but they wouldn't speak to me.
A. fall out with B. make up with C. spice up D. straighten out
patch things up: làm lành/ làm hòa
A.cãi nhau
B. làm hòa
C. làm cho cái gì đó hấp dẫn hơn
D. khiến cho ai đó trở nên tốt đẹp hơn
Câu này dịch như sau: Tôi cố gắng làm hòa sau cãi nhau nhưng họ không nói chuyện với tôi.
=>patch things up >< fall out with
Mark the letter A, B, C or D on your answer sheet to indicate the word (s) OPPOSITE in meaning to
the underlined word (s) in each of the following questions
Question 31: If he gets to university, his parents will be walking on air
A. disgusted B. promising C. upset D. hopeful
Giải thích : to walk on air : vô cùng phấn khích (đi trên mây) >< upset = sad (adj) : buồn bã
Dịch nghĩa : Nếu anh ấy vào được trường ĐH, bố mẹ anh ấy sẽ vô cùng phấn khích.
A.kinh tởm B. hứa hẹn
C. buồn bã D. hy vọng
Question 32: She rarely smiles because she’s shy about exposing her crooked teeth.
A. pulling B. hiding C. showing D. brushing
Giải thích : to expose = to show : để lộ ra >< to hide = to conceal : che giấu

Hocmai– Ngôi trường chung của học trò Việt Tổng đài tư vấn: 1900 69 33 - Trang | 8-

265
Hocmai.vn – Website học trực tuyến số 1 tại Việt Nam
Khóa học Luyện thi THPT quốc gia PEN C–N3 Môn Tiếng Anh côHương Fiona

Dịch nghĩa : Cô ấy hiếm khi cười bởi vì cô ấy ngại phải để lộ hàm răng khấp khểnh của mình.
A. đẩy ra B. che giấu
C. để lộ D. đánh răng
Mark the letter A, B, C or D on your answer sheet to indicate the word (s) CLOSEST in meaning to the
underlined word (s) in each of the following questions.
Question 33: Did anyone acknowledge responsibility for the outbreak of the fire ?
A. inquire about B. accept C. report D. find out
Giải thích : to acknowledge = to accept : chấp nhận
Dịch nghĩa : Đã có ai nhận trách nhiệm về sự lan toả của ngọn lửa chưa ?
A. hỏi thăm B. chấp nhận
C. báo cáo D. khám phá
Question 34: There is a set of expensive spoons, forks and knives in the cupboard.
A. cutlery B. instrument C. equipment D. utensil
Giải thích : utensil [ju:'tensl] (n) : vật dụng nhà bếp
Dịch nghĩa : Có một bộ muỗng, dĩa và dao đắt tiền trong chạn bếp.
A.dao kéo B. công cụ C. thiết bị D. đồ làm bếp
Choose the letter A, B, C, or D on your answer sheet to indicate the word that is CLOSEST in meaning
to the underlined part in each of the following questions
Question 35: The newscaster have a concise account of tradegy.
A. complicated and intricate B. short and clear
C. long and detailed D. sad and depressing
Concise: ngắn gọn, súc tích
Đồng nghĩa là short and clear: ngắn và rõ ràng
Complicated and intricate: rắc rối phức tạp
Long and detailed: dài và chi tiết
Sad and depresssing: buồn và chán nản
Dịch: phát thanh viên có một báo cáo ngắn gọn về bi kịch
Question 36: At last, we succeeded in persuading those boys and girls to join our picnic.
A. At the end B. In the end C. Lastly D. Endlessly
At last: cuối cùng (kết thúc một sự kiện, 1 sự việc nào đó)
Phân biệt giữa in the end và at the end:
- at the end: cuối, phía cuối, phần cuối (của cái gì, ví dụ: cuối tháng at the end of the month, cuối sự kiện at
the end of the event,…)

Hocmai– Ngôi trường chung của học trò Việt Tổng đài tư vấn: 1900 69 33 - Trang | 9-

266
Hocmai.vn – Website học trực tuyến số 1 tại Việt Nam
Khóa học Luyện thi THPT quốc gia PEN C–N3 Môn Tiếng Anh côHương Fiona

- in the end = finally, eventually = cuối cùng


Dịch: Cuối cùng, chúng tôi thành công thuyết phục các cô cậu bé tham gia dã ngoại.
Choose the letter A, B, C, or D on your answer sheet to indicate the word that is OPPOSITE in meaning
to the underlined part in each of the following questions
Question 37: “That is a well-behaved boy whose behavior has nothing to complain about.”
A. behaving cleverly B. behaving nice
C. good behavior D. behaving improperly
Well-behaved: cư xử đúng mực
Trái nghĩa là behave improperly: cư xử không đúng
Dịch: Đó là cậu bé cư xử đúng mực, người mà hành vi không có gì phải phàn nàn cả
Question 38: He mentioned in particular electronics, his major at university.
A. one and all B. in general C. on whole D. in all
In particular: đặc biệt, cá biệt
Trái nghĩa là in all: chung, tất cả
Phần C phải dùng on the whole chứ không dùng on whole
In general: chung chung, nói chung
One and all (khẩu ngữ): mọi người (VD: happy new year to one and all: chúc mừng năm mới tất cả mọi người)
Mark the letter A, B, C, or D on your answer sheet to indicate the word or phrase CLOSEST in meaning
to the underlined word(s) in each of the two following questions.
Question 39. The shop assistant was totally bewildered by the customer’s behavior.
A. disgusted B. puzzled C. angry D. Upset
bewildered = puzzled: bị bối rối.
Các đáp án khác
A. disgusted : ghê tởm, khó chịu
C. absolutely (adv): hoàn toàn, chắc chắn
D. upset(adj): đau khổ
Question 40. The newspaper reporters bear out what the Minister told yesterday.
A. define B. confirm C. support D. Complain
Đáp án đúng B: bear out = confirm: xác nhận
Các đáp án khác
A. define: định nghĩa
C. support: ủng hộ, hỗ trợ
D. complain: phàn nàn

Hocmai– Ngôi trường chung của học trò Việt Tổng đài tư vấn: 1900 69 33 - Trang | 10-

267
Hocmai.vn – Website học trực tuyến số 1 tại Việt Nam
Khóa học Luyện thi THPT quốc gia PEN C–N3 Môn Tiếng Anh côHương Fiona

Mark the letter A, B, C, or D on your answer sheet to indicate the word or phrase OPPOSITE in
meaning to the underlined word(s) in each of the two following questions
Question 41. Unless you have been very lucky, you have undoubtedly experienced events in your life that
made you cry.
A. certainly B. questionably C. absolutely D. Definitely
Đáp án đúng: B
undoubtedly: không nghi ngờ gì >< trái nghĩa với questionably: đáng ngờ, đáng đặt câu hỏi
Các đáp án khác
A. certainly (adv): tất nhiên
C. absolutely (adv): tuyệt đối, hoàn toàn
D. definitely (adv): chắc chắn, không nghi ngờ, chính xác
Question 42. His boss has had enough of his impudence and doesn't want to hire him anymore.
A. respect B. rudeness C. obedience D. Agreement
Đáp án đúng: A
Impudence: sự hỗn láo, lảo xược >< trái nghĩa với respect: sự tôn trọng
Các đáp án khác
B. rudeness: sự thô lỗ
C. obedience: sự vâng lời
D. agreement: sự đồng ý

Mark the letter A, B, C, or D on your answer sheet to indicate the word or phrase CLOSEST in meaning
to the underlined word(s) in each of the two following questions.
Question 43. The student apologized to his teacher for submitting the essay late.
A. handing in B. dropping out of
C. carrying out D. bringing in
Đáp án đúng A: submit (v): giao nộp = hand in (v)
Các đáp án còn lại là:
B. dropping out of: bỏ học
C. carrying out: tiến hành, thực hiện
D. bringing in: giới thiệu, đưa ra, kiếm được, thu được
Question 44. I wonder when I’m finally going to receive news from Joe.
A. to hear of B. to hear from

Hocmai– Ngôi trường chung của học trò Việt Tổng đài tư vấn: 1900 69 33 - Trang | 11-

268
Hocmai.vn – Website học trực tuyến số 1 tại Việt Nam
Khóa học Luyện thi THPT quốc gia PEN C–N3 Môn Tiếng Anh côHương Fiona

C. to get in touch with D. turn away from


Đáp án đúng B: receive news from: nhận được tin từ… = to hear from Joe.
Các đáp án còn lại là:
A. to hear of: nghe tin, biết tin
C. to get in touch with: liên lạc với
D. turn away from: quay đi khỏi.

Mark the letter A, B, C, or D on your answer sheet to indicate the word or phrase OPPOSITE in
meaning to the underlined word(s) in each of the two following questions.
Question 45. We run a very tight ship here, and we expect all our employees to be at their desks by eight
o'clock and take good care of their own business.
A. manage an inflexible system B. have a good voyage
C. run faster than others D. organize things inefficiently
Đáp án đúng: D
run a very tight ship: quản lý chặt chẽ >< trái nghĩa với organize things inefficiently: sắp xếp mọi thứ không
hiệu quả.
Ngoài ra ta có manage an inflexible system: quản lý một hệ thống không linh hoạt
Question 46. He usually stays in peace in stressful situations but this time he really lost his head.
A. kept calm B. excited
C. took leave of his sense D. lost touch
Đáp án đúng: A lost his head: mất bình tĩnh >< trái nghĩa với kept calm: giữ bình tĩnh
Các đáp án còn lại là:
B. excited: phấn khích
C. took leave of his sense: điên dại
D. lost touch: mất liên lạc

Mark the letter A, B, C, or D on your answer sheet to indicate the word or phrase CLOSEST in meaning
to the underlined word(s) in each of the two following questions.
Question 47. We decided to pay for the furniture on the installment plan.
A. monthly payment B. cash and carry
C. credit card D. piece by piece
Đáp án đúng A:
installment (n): tiền thanh toán dần, tiền trả dần = monthly payment

Hocmai– Ngôi trường chung của học trò Việt Tổng đài tư vấn: 1900 69 33 - Trang | 12-

269
Hocmai.vn – Website học trực tuyến số 1 tại Việt Nam
Khóa học Luyện thi THPT quốc gia PEN C–N3 Môn Tiếng Anh côHương Fiona

Question 48. Teletext is continuously sent out at all times when regular television programs are broadcast.
A. transmitted B. electrified C. automated D. Aired
Đáp án đúng A:
Transmit (v) = send out: truyền thông tin
Mark the letter A, B, C, or D on your answer sheet to indicate the word or phrase OPPOSITE in
meaning to the underlined word(s) in each of the two following questions.
Question 49. Any student who neglects his or her homework is unlikely to do well at school.
A. puts off B. looks for C. attends to D. approves of
Đáp án đúng: C neglects: sao nhãng >< trái nghĩa là attends to: quan tâm tới
Các đáp án khác:
A. puts off: trì hoãn
B. looks for: tìm kiếm
D. approves of: tán thành, đồng ý
Question 50. Many political radicals advocated that women should not be discriminated on the basic of their
sex.
A. openly criticized B. rightly claimed
C. publicly said D. Protested
Đáp án đúng: D advocated: đồng tình, ủng hộ >< trái nghĩa là protested: phản đối
A: openly criticized: công khai chỉ trích
B: right claimed: tuyên bố chính xác

Nguồn : Hocmai.vn
Giáo viên : Hương Fiona

Hocmai– Ngôi trường chung của học trò Việt Tổng đài tư vấn: 1900 69 33 - Trang | 13-

270
Hocmai.vn – Website học trực tuyến số 1 tại Việt Nam
Khóa học Luyện thi THPT quốc gia PEN C –N3 Môn Tiếng Anh côHương Fiona

PHƯƠNG PHÁP LÀM BÀI CÂU ĐỒNG NGHĨA – NỐI CÂU


( ĐÁP ÁN BÀI TẬP TỰ LUYỆN )
Giáo viên : Nguyễn Thanh Hương
Các bài tập trong tài liệu này được biên soạn kèm theo bài giảng “Phương pháp làm bài câu đồng nghĩa – nối câu” thuộc Khóa học
Luyện thi THPT quốc gia PEN - C: Môn Tiếng Anh (cô Hương Fiona )” tại website Hocmai.vn để giúp các Bạn kiểm tra, củng cố lại
các kiến thức được giáo viên truyền đạt trong bài giảng tương ứng. Để sử dụng hiệu quả, Bạn cần học trước bài giảng, sau đó làm đầy
đủ các bài tập trong tài liệu này.

Mark the letter A, B, C, or D on your answer sheet to indicate the sentence that is CLOSEST in meaning
to each of the following sentences
Question 1: The scene is set in Normandy, but most of the characters in this novel are Londoners.
A. In the novel, the action moves backwards and forwards between Normandy and London.
B. The main characters in the novel are Londoners on a sightseeing holiday in Normandy.
C. In this novel, the story takes place in Normandy but the majority of the characters are from London.
D. The story is about Normandy, but the leading characters are all Londoners
→ Dịch đề: Khung cảnh diễn ra ở Normandy, nhưng hầu hết các nhân vật trong tiểu thuyết là người London.
Đáp án đúng: Trong tiểu thuyết này, câu chuyện được diễn ra ở Normandy nhưng hầu hết các nhân vật đều
đến từ London.
Question 2: The airline requested a confirmation call to ensure a seat on my flight back home.
A. The airline made sure I got a seat on my flight home
B. I secured a seat on my flight home.
C. The airline confirmed a seat on my flight home
D. It was necessary to confirm seat on my flight back home.
→ Dịch đề: Hãng hàng không yêu cầu một cuộc gọi xác nhận để đảm bảo một vé cho tôi trong chuyến bay về
nhà.
Đáp án đúng: Cần thiết phải xác nhận chỗ ngồi trên chuyến bay về nhà của tôi
Question 3. Mary should never have been allowed to try to swim in the sea alone.
A When Mary left to swim in the sea alone, she said she knew what she was doing.
B. It would probably be wrong to let Mary swim in the sea on her own.
C. No one could have stopped Mary from trying to swim in the sea by herself.
D. Someone ought to have stopped Mary from attempting to swim in the sea on her own
→ Đáp án đúng: D: Viết lại cấu trúc "S + should have P2" bằng cấu trúc "S + ought to have P2”
Question 4: The last time when I saw her was three years ago.
A. I have often seen her for the last three years.

Hocmai– Ngôi trường chung của học trò Việt Tổng đài tư vấn: 1900 69 33 - Trang | 1-

271
Hocmai.vn – Website học trực tuyến số 1 tại Việt Nam
Khóa học Luyện thi THPT quốc gia PEN C –N3 Môn Tiếng Anh côHương Fiona

B. About three years ago, I used to meet her.


C. I have not seen her for three years.
D. I saw her three years ago and will never meet her again
→ Dịch nghĩa: Lần cuối cùng tôi gặp cô ấy là 3 năm trước.
A. Tôi đã thường xuyên gặp cô ấy trong 3 năm gần đây.
B. Khoảng ba năm trước, tôi thường gặp cô ấy.
C. Tôi đã không gặp cô ấy trong vòng ba năm.
D. Tôi gặp cô ấy ba năm trước và sẽ không bao giờ gặp lại nữa
Question 5: Somebody cleans that room every day.
A. The room every day is cleaned.
B. The room is everyday cleaned.
C. The room is cleaned every day.
D. The room is cleaned by somebody everyday.
→ Dịch nghĩa: Ai đó đã lau phòng này mỗi ngày.
A và B đặt sai vị trí của trạng từ chỉ tần suất.
D thừa thông tin, trong câu bị động, khi người thực hiện hành động không rõ ràng như somebody, someone
thì sẽ bị lược bỏ.
Question 6: As we were heading out of the café, we bumped into our plumber, who we still owed money
to.
A. Our plumber, whom we hadn't paid back yet, was the person who we encountered by chance at the moment
we were exiting the café.
B. When we met our plumber in the café as we were about to leave, we realised that we hadn't yet paid him
all the money we owed him.
C. Our plumber, when we ran into him as he was entering the cafe that we were leaving, didn't bring up our
debt to him.
D. Not wanting to see the plumber to whom we still owed a debt, we quickly headed out of the café when we
saw him coming in.
→ Dịch nghĩa: Khi chúng tôi ra khỏi quán cà phê, chúng tôi tình cờ gặp người thợ sửa ống nước, người mà
chúng tôi vẫn còn nợ tiền.
A. Người thợ sửa ống nước, người mà chúng tôi chưa trả đủ tiền, là người mà chúng tôi tình cờ bắt gặp khi
chúng tôi đang rời quán khỏi cà phê.
B. Khi chúng tôi gặp người thợ sửa ống nước ở quán cà phê khi chúng tôi đang chuẩn bị rời đi, chúng tôi nhận
ra rằng chúng tôi chưa trả anh ấy hết tiền mà chúng tôi đã nợ.

Hocmai– Ngôi trường chung của học trò Việt Tổng đài tư vấn: 1900 69 33 - Trang | 2-

272
Hocmai.vn – Website học trực tuyến số 1 tại Việt Nam
Khóa học Luyện thi THPT quốc gia PEN C –N3 Môn Tiếng Anh côHương Fiona

C. Người thợ sửa ống nước của chúng tôi, khi mà chúng tôi tình cờ gặp anh ta khi anh ta đang bước vào quán
cà phê lúc chúng tôi rời đi, đã không đề cập đến việc nợ.
D. Không muốn thấy người người thợ sửa ống nước mà chúng tôi đang nợ tiền, chúng tôi nhanh chóng rời
khỏi quán cà phê khi chúng tôi thấy anh ta đến.
Question 7: They said that Paula had quit five jobs before working for us.
A. They said that five jobs had been quit by Paula before working for us.
B. Paula worked for us then she quit five other jobs.
C. It was said that Paula had quit five jobs before working for us
D. Paula said that she had quit five jobs before working for us.
→ Đề: “Họ nói rằng Paula đã bỏ 5 công việc trước khi làm việc cho chúng ta."
Câu A sai cấu trúc
Câu B sai vì ngược thời gian (Paula nghỉ việc trước rồi mới làm cho chúng ta)
Câu D sai vì người nói lên việc này không phải là Paula.
Thể bị động với động từ say
S + say that + S + V...
==> It + be + said+ that + S + V....
==> Đáp án C
Question 8: “You're always making terrible mistakes", said the teacher.
A. The teacher complained about his student making terrible mistakes.
B. The teacher made his students not always make terrible mistakes
C. The teacher asked his students why they always made terrible mistakes.
D. The teacher realized that his students always made terrible mistakes
→ Đề: Giáo viên nói: "Em luôn phạm những lỗi kinh khủng."
Câu B sai nghĩa (Giáo viên bắt học sinh không luôn phạm những lỗi kinh khủng.)
Câu C sai vì rõ ràng trong đề bài không hỏi tại sao học sinh mắc lỗi
Câu D sai nghĩa (Giáo viên nhận ra rằng những học sinh luôn phạm những lỗi kinh khủng.)
==> Đáp án A (Giáo viên phàn nàn về việc học sinh của mình phạm những lỗi kinh khủng.)
Question 9. Diana ran into her former teacher on the way to the stadium yesterday.
A. Diana caused an accident to her teacher while she was going to the stadium.
B. Diana's car ran over her teacher on the way to the stadium.
C. Diana happened to meet her teacher while she was going to the stadium.
D. Diana's teacher got run over whole she was going to the stadium.
→ "Diana đã tình cờ gặp giáo viên cũ của mình trên đường đến sân vận động hôm qua."

Hocmai– Ngôi trường chung của học trò Việt Tổng đài tư vấn: 1900 69 33 - Trang | 3-

273
Hocmai.vn – Website học trực tuyến số 1 tại Việt Nam
Khóa học Luyện thi THPT quốc gia PEN C –N3 Môn Tiếng Anh côHương Fiona

- happen to do sth: tình cờ, ngẫu nhiên làm gì


E.g: The door happened to be unlocked.
- run into = run across: tình cờ gặp ai
Đáp án C
A, B, D sai nghĩa của câu
Question 10: “Why I haven’t thought of this before?” Tony said to himself.
A. Tony advised himself not to have thought of that before.
B. Tony suggested himself not thinking of that before.
C. Tony said that why he hasn’t thought of that before.
D. Tony wondered why he hadn’t thought of that before.
→ Tony tự nói với mình: “ Sao mình không nghĩ ra điều này trước đó nhỉ?”
A. Tony tự khuyên mình không nghĩ ra điều đó trước đó.
B. Tony tự đề nghị mình không nghĩ ra điều này trước đó.
C. Tony nói rằng tại sao ấy anh không nghĩ ra điều này trước đó. => loại vì chưa lùi thì
D. Tony tự hỏi sao anh ấy không nghĩ ra điều này trước đó nhỉ.
Question 11: She is the most intelligent woman I have ever met.
A. I have never met a more intelligent woman than her.
B. She is not as intelligent as the women I have ever met.
C. I have ever met such an intelligent woman.
D. She is more intelligent than I am.
→ Cô ấy là người phụ nữ thông minh nhất mà tôi từng gặp.
A. Tôi chưa từng gặp người phụ nữ nào thông minh hơn cô ấy.
B. Cô ấy không thông minh như người phụ nữ mà tôi từng gặp.
C. Tôi đã từng gặp một người phụ nữ thông minh như vậy.
D. Cô ấy thông minh hơn tôi.
Question 12: She broke down the moment she heard the news.
A. She was broken for a moment when she heard the news.
B. She broke her leg when hearing the news.
C. On hearing the news, she broke down.
D. When she heard the news, she was sick.
→ Cô ấy mấy kiểm soát và bật khóc khi nghe tin đó.
A. Cô ấy đã bị vỡ vụn cho cái khoảnh khắc khi cô ấy nghe tin đó.
B. Cô ấy gãy chân khi nghe tin đó.

Hocmai– Ngôi trường chung của học trò Việt Tổng đài tư vấn: 1900 69 33 - Trang | 4-

274
Hocmai.vn – Website học trực tuyến số 1 tại Việt Nam
Khóa học Luyện thi THPT quốc gia PEN C –N3 Môn Tiếng Anh côHương Fiona

C. Khi nghe tin đó cô ấy mất kiểm soát và bật khóc.


D. Khi nghe tin đó cô ấy bị ốm.
Question 13: Although he was very tired, he agreed to help his child with his homework.
A. Despite of his tiredness, he was eager to help his child with his homework.
B. Tired as he was, he agreed to help his child with his homework.
C. Even if feeling very tired, he agreed to help his child with his homework.
D. He would have helped his child with his homework if he hadn’t been tired
→ Cấu trúc: “Although + clause, clause = Adj/Adv + as/though + S +be, clause “
Đề: Mặc dù anh ấy rất mệt, anh ấy vẫn đồng ý giúp bọn trẻ làm bài tập
A. Mặc dù anh ấy mệt (không có despite of), anh ấy háo hức giúp bọn trẻ bài tập
B. Dù rất mệt, anh ấy vẫn đồng ý giúp bọn trẻ bài tập
C. Ngay cả nếu như mệt, anh ấy vẫn đồng ý giúp bọn trẻ bài tập
D. Nếu anh ấy không mệt, anh ấy có thể giúp bọn trẻ bài tập.
Question 14: It’s Nick’s job to look after the tender plants in the garden.
A. Nick enjoys looking after the tender plants in the garden.
B. Nick is responsible for looking after the tender plants in the garden.
C. Looking after the tender plants in the garden is liable for Nick.
D. The tender plants in the garden give Nick opportunity to work.
→ Đề: Công việc của Nick là trông coi nhưng cây non trong khu vườn.
A. Nick thích việc trông coi nhưng cây non trong khu vườn.
B. Nick có trách nhiệm trong việc trông coi nhưng cây non trong khu vườn.
C. Trông coi nhưng cây non trong khu vườn thuộc trách nhiệm của Nick.
D. Những cây non trong vườn cho Nick cơ hội làm việc
Question 15: My suit needs to be cleaned before the interview but I’m too busy to do that.
A. I must have my suit cleaned before the interview.
B. I must have my suit to be cleaned before the interview.
C. I must clean my suit before the interview.
D. I must get my mother to clean my suit before the interview
→ Đề: Bộ comle của tôi phải được làm sạch trước buổi phỏng vấn nhưng tôi quá bận để làm điều đó.
A. Tôi đáng lẽ phải làm sạch bộ comle của tôi trước buổi phỏng vấn.
B. Không có cấu trúc: “ have sth to do sth”
C. Tôi phải làm sach bộ comle của tôi trước buổi phỏng vấn.
D. Tôi phải nhờ mẹ tôi làm sạch bộ comle trước buổi phỏng vấn

Hocmai– Ngôi trường chung của học trò Việt Tổng đài tư vấn: 1900 69 33 - Trang | 5-

275
Hocmai.vn – Website học trực tuyến số 1 tại Việt Nam
Khóa học Luyện thi THPT quốc gia PEN C –N3 Môn Tiếng Anh côHương Fiona

Question 16: He seemed very reluctant to take my advice.


A. He seemed quite willing to take my advice.
B. It seemed he was not ready to give me advice
C. It seemed he was not willing to take my advice.
D. He seemed very anxious to take my advice.
→ To be reluctant to do sth: rất miễn cưỡng khi làm gì → được viết lại là: to be not willing to do sth: không
tự nguyện làm gì.
Question 17: It was very impolite of him to leave without saying a word.
A.He was very impolite to leave without saying a word.
B.He didn’t say nothing when he left.
C. He didn’t say anything as he left, which was impolite.
D. Both A and c are correct.
→ To be impolite of sb to do sth: thật là bất lịch sự khi làm gì → A, C đều đúng vì có đầy đù ý nghĩa của đề
bài, cách đùng từ và ngữ pháp cũng đều đúng
Dịch A: Anh ta rất bất lịch sự khi rời khỏi mà không nới 1 tiếng.
C: Anh ta đã không nói gì cả khi rời khỏi, điều đó thật bất lịch sự
Question 18: She reminded her daughter of their table manners.
A. She wanted her daughter to be more polite while eating.
B. She wanted her daughter to leave the dinner table.
C. She wanted her daughter to eat a little more slowly.
D. She wanted her daughter to remember all meal time.
→ Remind sb of sth: nhắc nhở ai đó về việc gì đó → viết lại bằng: want sb to do sth. Cô ấy nhắc nhở con gái
về những lối ăn uống → Cô ấy muốn con gái mình trờ nên lịch sự hơn khi ăn uống — đáp án A
Question 19: He was driving so fast that he could have had an accident.
A. He wasn’t driving fast enough to avoid an accident.
B. He didn’t have an accident although he was driving very fast
C. If he had been driving very fast, he would have had an accident.
D. An accident happened, and it was caused by his very fast driving
→ Nghĩa câu gốc: Anh lái xe nhanh đến nỗi mà anh ta đã có thể gây tai nạn.
Trong 4 đáp án thì đáp án B là nghĩa tương tự nhất: May mà anh ta đã không gây tai nạn mặc dù anh ta lái xe
rât nhanh. Vậy chọn đáp án B
Question 20: Mrs. Jones told me that her neighbors were moving to Florida
A. Mrs. Jones and her neighbors live in Florida.

Hocmai– Ngôi trường chung của học trò Việt Tổng đài tư vấn: 1900 69 33 - Trang | 6-

276
Hocmai.vn – Website học trực tuyến số 1 tại Việt Nam
Khóa học Luyện thi THPT quốc gia PEN C –N3 Môn Tiếng Anh côHương Fiona

B. Mrs. Jones is planning to move to Florida with her neighbors.


C. I knew that Mrs. Jones had moved to Florida because her neighbors told me.
D. “My neighbors are moving to Florida,” said Mrs. Jones.
→ Câu đề bài là câu tường thuật: Cô Jone có bảo tôi rằng hàng xóm của cô ấy sắp chuyển qua Florida rồi. 3
đáp án đau ở khía cạnh ngữ nghĩa không phù họp (vì đối tượng chuyển nhà ở đày là hàng xóm của cô Jone,
chứ không phải là Jone)
Câu D là hình thức trực tiếp của câu gốc và phù hợp nghĩa nhất
Question 21: No matter how hard Fred tried to start the car, he didn’t succeed.
A. Fried tried very hard to start the car, and succeeded.
B. Fried tried hard to start the car, and with success,
C. However hard he fried, Fried couldn’t start the car.
D. It’s hard for Fried to start the car because he never succeeded
→ No matter how + adj + mệnh đề được viết lại bằng However + adj + mệnh đề.
Cấu trúc này tương đương với các cấu trúc although, in spite of, despite hay Adj/ADV + as though + S +V, +
S +V : mặc dù ...nhưng.
Question 22: When the unemployment rate is high, the crime rate is usually also high.
A.The unemployment rate and the crime rate are both higher.
B. The higher the unemployment rate is, the higher the crime rate is.
C. The unemployment rate is as high as the crime rate.
D. The high rate of unemployment depends on the high rate of crime
→ Câu đề bài chỉ nguyên nhân → hệ quả, do đó khi viết lại, sử dụng so sánh kép: Tỉ lệ thất nghiệp càng lớn,
tỉ lệ tội phạm càng cao
Question 23: I wish you hadn't said that
A. I wish you not to say that. B. If only you didn't say tot
C. I hope you will not say that. D. It would be nice if you hadn’t said that.
→ Dịch câu gốc: tôi mong là bạn đã không nói như thế (hành động nói đã xảy ra)
D: Thật tốt đẹp nểu bạn đã không nói như thể.
Loại A và C vì nói như 2 câu này thì có nghĩa là hành động nói chưa xảy ra.
Question 24: “You're always making terrible mistakes”, said the teacher.
A. The teacher asked his students why they always made terrible mistakes.
B. The teacher realized that his students always made terrible mistakes.
C. The teacher complained about his students making terrible mistakes.
D. The teacher made his students not always make terrible mistakes

Hocmai– Ngôi trường chung của học trò Việt Tổng đài tư vấn: 1900 69 33 - Trang | 7-

277
Hocmai.vn – Website học trực tuyến số 1 tại Việt Nam
Khóa học Luyện thi THPT quốc gia PEN C –N3 Môn Tiếng Anh côHương Fiona

→ Always + thì hiện tiếp diễn: thể hiện sự phàn nàn → chọn câu C. Complain about: phàn nàn về
Question 25: "It can't be Mike who leaked the document, it might be Tom.” said our manager.
A. Our manager suspected Tom of having leaked the document, not Mike.
B. Our manager blamed Tom for having leaked the document instead of Mike.
C. Our manager showed his uncertainty about who leaked the document: Mike or Tom
D. Our manager made it clear that Tom was the one who leaked the document, not Mike
→ Viết lại câu , đổi từ trực tiếp sang gián tiếp , sử dụng câu trúc “ suspect sb of doing / having done St”
Question 26: Their holiday plans fell through because there was a strike at the airport.
A. They couldn’t go on holiday as planned as a result of a strike at the airport
B. They failed to go on the holiday like they had planned because a strike took place at the airport
C. A strike at the airport almost put a stop to their holiday plans.
D. Disappointingly, a strike at the airport forced their holiday plans to nothing
→ Fall through: hỏng, thất bại → Họ không thể đi du lịch như đã định vì hậu quả của 1 vụ tấn công ờ sân bay
Question 27: He was driving so fast that he could have had an accident.
A. An accident happened, and it was caused by his very fast driving
B. He didn’t have an accident although he was driving very fast
C. If he had been driving very fast, he would have had an accident.
D. He wasn't driving slowly enough to avoid the accident
→ Anh ta lái xe nhanh đến mức mà có thể gây ra tai nạn ( nhưng thực tế chưa xảy ra tai nạn ) 4 Mặc dù anh
ta lái xe quả nhanh nhưng anh ta đã không gây ra tai nạn
Question 28: "Leave my house now or I'll call the police!" shouted the lady to the man.
A. The lady threatened to call the police if the man didn't leave her house.
B. The lady said that she would call the police if the man didn't leave her house.
C. The lady told the man that she would call the police if he didn't leave her house.
D. The lady informed the man that she would call the police if he didn't leave her house
→ Đề: “Ra khỏi nhà tôi ngay nếu không tôi sẽ báo cảnh sát!”, người phụ nữ hét lên với người đàn ông.
A. Người phụ nữ đe doạ (sẽ) báo cảnh sát nếu người đàn ông không rời khỏi nhà cô ấy.
B. Người phụ nữ nói rằng cố ấy sẽ gọi cảnh sát nếu người đàn ông không rời khỏi nhà cô ấy.
C. Người phụ nữ nói với người đàn ông rằng cố ấy sẽ gọi cảnh sát nếu người đàn ông không rời khỏi nhà cô
ấy.
D. Người phụ nữ thông báo với người đàn ông rằng cố ấy sẽ gọi cảnh sát nếu người đàn ông không rời khỏi
nhà cô ấy
Question 29: He last had his eyes tested ten months ago.

Hocmai– Ngôi trường chung của học trò Việt Tổng đài tư vấn: 1900 69 33 - Trang | 8-

278
Hocmai.vn – Website học trực tuyến số 1 tại Việt Nam
Khóa học Luyện thi THPT quốc gia PEN C –N3 Môn Tiếng Anh côHương Fiona

A. He had tested his eyes ten months before.


B. He had not tested his eyes for ten months then.
C. He hasn't had his eyes tested for ten months.
D. He didn't have any test on his eyes in ten months
→ Đề: Lần cuối anh ấy kiểm tra mắt là 10 tháng trước.
A. Anh ấy đã kiểm tra mắt 10 tháng trước đó. (“đó” là một thời điểm trong QK, không phải hiện tại)
B. Anh ấy đã không kiểm tra mắt được 10 tháng tính tới khi đó.
C. Anh ấy đã không kiểm tra mắt 10 tháng rồi.
D. Anh ấy không có bài kiểm tra nào về mắt trong 10 tháng qua
Question 30: The children couldn't go swimming because the sea was too rough.
A. The children were not calm enough to swim in the sea.
B. The sea was rough enough for the children to swim in.
C. The sea was too rough for the children to go swimming.
D. The sea was too rough to the children's swimming
→ Đề: Những đứa trẻ đã không thể đi bơi vì biển quá động.
A. Những đứa trẻ đã không đủ bình tĩnh để bơi ở biển.
B. Biển đã đủ động để lũ trẻ bơi ở đó.
C. Biển đã quá động để lũ trẻ (có thể) đi bơi.
D. Biển đã quá động đối với sự bơi lội của lũ trẻ
Question 31. Both English and Vietnamese use Roman scripts' but the latter is a tonal language.
A. Later both Vietnamese and English use Roman scripts.
B. English has tones but Vietnamese does not, although they both use Roman scripts.
C. Although English and Vietnamese are both Roman scripts, the former is not a tonal language while the
latter is.
D. Vietnamese and English which use Roman scripts later become tonal languages
→ Câu cho sẵn: có "but”
Câu viết lại dùng: "although” để kết nối 2 câu
Question 32. If I had known about their wedding plan earlier I would have been able to make time to
attend the reception party.
A. I knew their wedding would be planned earlier so I made some time to attend the morning reception.
B. I wish I had known their wedding plan sooner so that I could arrange time to attend the morning reception.
C. I don t know their wedding plan earlier so I can't make time to attend their morning reception.
D. When I knew their wedding party, it was too late to attend the morning reception.

Hocmai– Ngôi trường chung của học trò Việt Tổng đài tư vấn: 1900 69 33 - Trang | 9-

279
Hocmai.vn – Website học trực tuyến số 1 tại Việt Nam
Khóa học Luyện thi THPT quốc gia PEN C –N3 Môn Tiếng Anh côHương Fiona

→ Câu cho sẵn: điều kiện loại 3


Câu viết lại dùng: I wish + clause (quá khứ hoàn thành)
Question 33. "You did a great job! I'm proud of your achievement" said the woman to her grandchild.
A. The woman said that her grandchild's job was great and site was proud of his work achievement.
B. The woman told her grandchild that she was proud of his achievement at work.
C. The woman told her grandchild to do a great job so that she could be proud of his achievement.
D. The woman complimented her grandchild on his achievement
→ Câu cho sẵn: "You did a great job” là một lời khen!
Câu viết lại dùng "to compliment sb on sth/ Ving": khen ngợi
Mark the letter A, B, C, or D on your answer sheet to indicate the sentence that best combines each pair
of sentences in the following questions
Question 34. He cannot lend me the book now. He has not finished reading it yet.
A. Having finished reading the book, he cannot lend it to me.
B. He cannot lend me the book until he has finished reading it.
C. Not having finished reading the book, he will lend it to me.
D. As long as he cannot finish reading the book, he will lend it to me
→ Ghép 2 câu bằng cách sử dụng “until”
Question 35. I did not arrive in time. I was not able to see her off.
A. She had left because I was not on time.
B. I did not go there, so I could not see her off.
C. I was not early enough to see her off.
D. I arrived very late to say goodbye to her.
→ Tạm dịch: Tôi đã không đến đúng giờ. Tôi đã không thể tiễn cô ấy.
Đáp án đúng: C. Tôi đến quá muộn đế có thể nói lời tạm biệt với cô ấy
Question 36: I'm sorry I wasn’t in the office when you phoned. I know I promised to be.
A. I should be in the office when you phoned.
B. I should have been in the office when you phoned.
C. I must have been in the office when you phoned.
D. I might be in the office when you phoned.
→ Dịch nghĩa: Tôi xin lỗi tôi đã không có mặt ở văn phòng lúc bạn gọi. Tôi biết là tôi đã hứa với bạn như thế
rồi.
A. Tôi nên ở văn phòng lúc mà bạn gọi.
B. Lẽ ra tôi nên ở văn phòng lúc mà bạn gọi.

Hocmai– Ngôi trường chung của học trò Việt Tổng đài tư vấn: 1900 69 33 - Trang | 10-

280
Hocmai.vn – Website học trực tuyến số 1 tại Việt Nam
Khóa học Luyện thi THPT quốc gia PEN C –N3 Môn Tiếng Anh côHương Fiona

C. Tôi chắc chắn đã ở văn phòng lúc bạn gọi.


D. Tôi có thể đã ở văn phòng lúc bạn gọi.
Question 36: There was a terrible flood. All villagers, who had received a warning of the impending
flood, escaped to safety.
A. All of the villagers had been warned but only some escaped.
B. All of the villagers had been warned and all escaped.
C. Only some of the villagers had been warned and only some escaped.
D. Only some of the villagers had been warned and all escaped.
→ Dịch nghĩa: Có một cơn bão rất khủng khiếp. Tất cả dân làng, những người đã nhận được cảnh báo về lũ
lụt, đã di dời an toàn.
A. Tất cả những dân làng đã được cảnh báo nhưng chỉ một vài người di dời.
B. Tất cả những dân làng đã được cảnh báo và di dời.
C. Chỉ một số dân làng được cảnh báo và chỉ một số đã di dời.
D. Chỉ một số dân làng được cảnh báo và tất cả đã di dời.
Question 37. It isn't just that the level of education of this school is high. It's that it's also been consistent
for years.
A. The level of education in this school, which is usually quite high, shows only slight variations from year
to year.
B. The standard of education is not high in this school, but at least all the students are at the same level.
C. Not only are the standards of education good in this school, but it has maintained those standards over the
years.
D. It isn't fair to deny that this school is successful, as it has had the same high standards for many years now.
→ "Không chỉ mức độ giáo dục của trường này cao. Nó cũng bền vững nhiều năm rồi."
A. Mức độ giáo dục trong trường này thì thường xuyên cao chỉ cho thấy sự thay đổi nhẹ qua các năm.
B. Tiêu chuẩn giáo dục ở ngôi trường này không cao, nhưng ít nhất tất cả các học sinh ở mức độ giống nhau.
C. Không chỉ tiêu chuẩn giáo dục ở ngôi trường này tốt mà nó còn duy trì được những tiêu chuẩn đó qua các
năm.
D. Không công bằng để phủ nhận rằng ngôi trường này thành công, vì nó đã có tiêu chuẩn cao trong nhiều
năm nay.
A, B, D sai nghĩa của câu
Question 38. No one but the experts was able to realize that the painting was an imitation. It greatly
resembled the original.
A. It was obvious that only a person with great talent could fake a painting so successfully.

Hocmai– Ngôi trường chung của học trò Việt Tổng đài tư vấn: 1900 69 33 - Trang | 11-

281
Hocmai.vn – Website học trực tuyến số 1 tại Việt Nam
Khóa học Luyện thi THPT quốc gia PEN C –N3 Môn Tiếng Anh côHương Fiona

B. It was hard for ordinary people to judge between the fake painting and the real one, but not for the experts.
C. It was almost impossible for amateurs to realize that the painting was not authentic, though the experts
could judge it quite easily.
D. The painting looked so much like the authentic one that only the experts could tell it wasn't genuine.
→ Không ai ngoại trừ các chuyên gia có thể nhận ra rằng bửc tranh là sự bắt chước. Nó rất giống với cái gốc.
"The painting looked so much like the authentic one that only the experts could tell it wasn't genuine. = The
painting looked so much like the original one that only the experts could tell it wasn't the real one." (Bức tranh
trông rất giống với cái gốc đến nỗi mà chỉ những chuyên gia mới có thể nói đó không phải là cái thật."
Question 39: The basketball team knew they lost the match. They soon started to blame each other.
A. Hardly had the basketball team known they lost the match when they started to blame each other.
B. No sooner had the basketball team started to blame each other than they knew they lost the match.
C. As soon as they blamed each other, the basketball team knew they lost the match.
D. Not only did the basketball team lose the match but they blamed each other as well
→ Đội bóng chuyền biết rằng họ đã thua trận đấu. Họ bắt đầu đổ lỗi cho nhau ngay sau đó.
A. Ngay khi đội bóng rổ biết rằng họ thua họ bắt đầu đổ lỗi cho nhau.
B. Ngay khi đổi bóng rổ bắt đầu đổ lỗi cho nhau học biết rằng họ đã thua.
C. Ngay khi họ đổ lỗi cho nhau đội bóng rổ biết rằng họ đã thua.
D. Không chỉ đội bóng rổ thua trận đấu mà họ còn đổ lỗi cho nhau
Cấu trúc ngay khi…thì…:
Hardly had S Ved/ V3 when S+ quá khứ đơn
= No sooner had S + Ved/ V3 than S + quá khứ đơn
Question 40: We chose to find a place for the night. We found the bad weather very inconvenient.
A. Bad weather was approaching, so we started to look for a place to stay.
B. The bad weather prevented us from driving any further.
C. Seeing that the bad weather had set in, we decided to find somewhere to spend the night.
D. Because the climate was so severe, we were worried about what we'd do at night
→ Chúng tôi chọn một nơi để vui chơi cho buổi tối. Chúng tôi cảm thấy thời tiết xấu thật bất lợi.
A. Thời tiết xấu đang đến, vì vậy chúng tôi bắt đầu tìm một nơi để ở.
B. Thời tiết xấu ngăn cản chúng tôi lái xe đi xa hơn.
C. Thấy rằng thời tiết xấu , chúng tôi tìm chỗ nào đó để tiêu khiển cả đêm.
D. Vì khí hậu quá khắc nghiệt, chúng tôi lo lắng đến việc sẽ làm tối nay
Question 41: He had finished the report. He submitted it to the director.
A. Finishing the report, it was submitted to the director.

Hocmai– Ngôi trường chung của học trò Việt Tổng đài tư vấn: 1900 69 33 - Trang | 12-

282
Hocmai.vn – Website học trực tuyến số 1 tại Việt Nam
Khóa học Luyện thi THPT quốc gia PEN C –N3 Môn Tiếng Anh côHương Fiona

B. Having finished the report, it was submitted to the director.


C. Having finished the report, he submitted to the director.
D. Having finished the report, he submitted it to the director.
→ Đề: Anh ấy đã hoàn thành bản báo cáo. Anh ấy đã nộp cho giám đốc
Dịch: Sau khi hoàn thành bản báo cáo, anh ta nộp cho giám đốc
Question 42: The plan may be ingenious. It will never work in practice.
A. Ingenious as it may be, the plan will never work in practice.
B. Ingenious as may the plan, it will never work in practice.
C. The plan may be too ingenious to work in practice.
D. The plan is as impractical as it is ingenious.
→ Đề: Kế hoạch này có thể khá hay. Nó sẽ không bao giờ áp dụng trong thực tiễn
Dịch: Dù kế hoạch này khá hay,nó sẽ không bao giờ được áp dụng trong thực tiễn
Question 43: John was not here yesterday. Perhaps he was ill.
A. John needn't be here yesterday because he was ill.
B. Because of his illness, John shouldn't have been here yesterday.
C. John might have been ill yesterday, so he was not here.
D. John must have been ill yesterday, so he was not here.
→ Might have done sth: dự đoán việc gì đó xảy ra trong quá khứ (chưa biết chắc nó có như vậy hay không)
→ Đảp án C là chính xác
Cần phân biệt giữa must have done ST, might have done ST.
Must have done ST: suy đoán logic về 1 sự việc trong quá khứ thường có dấu hiệu đi kèm, mang hàm nghĩa
chắc chắn nhiều hơn may và might
Question 44: "Cigarette?", he said. "No, thanks.", I said.
A. He asked for a cigarette, and I immediately refitted.
B. He mentioned a cigarette, so I thanked him.
C. He offered me a cigarette, but I promptly declined.
D. He asked if I was smoking, and I denied at once.
→ Chuyển từ câu trực tiếp → câu gián tiếp, sử dụng câu trúc : offer sb sth/ decline sth
Question 45: It was an interesting novel. I stayed up all night to finish it.
A. I stayed up all night to finish the novel so it was interesting.
B. Unless it were an interesting novel, I would not stay up all night to finish it
C. Though it was an interesting novel, I stayed up all night to finish it.
D. So interesting was the novel that I stayed up all night to finish it

Hocmai– Ngôi trường chung của học trò Việt Tổng đài tư vấn: 1900 69 33 - Trang | 13-

283
Hocmai.vn – Website học trực tuyến số 1 tại Việt Nam
Khóa học Luyện thi THPT quốc gia PEN C –N3 Môn Tiếng Anh côHương Fiona

→ Tạm dịch: Quyển sách thú vị đến nỗi tôi phải thức khuya để đọc nó cho xong.
Đáp án A, B và C sai về ngữ nghĩa.
Cấu trúc đảo ngữ: So + (a) + to be + S that S + V
Question 46: We arrived at the airport. We realized our passports were still at home.
A. It was until we arrived at the airport that we realize our passports were still at home.
B. We arrived at the airport and realized that our passports are still at home.
C. Not until had we arrived at the airport, we realized our passports were still at home.
D. Not until we arrived at the airport, did we realize that our passports were still at home
→ Cấu trúc câu: Not until S + V + auxiliary + S + V (inf).
Đập án A sai cấu trúc: It was NOT until..that S + V
Đáp án B sai thì "...our passports are still at home"
Đáp án C sai cấu trúc.
Question 47: John is studying hard. He doesn’t want to fail the exam.
A. John is studying hard in Oder not to fail the next exam
B. John is studying hard in Oder that he not fail the next exam
C. John is studying hard so as to fail the next exam
D. John is studying hard in Oder to not to fail the next exam
→ Cấu trúc: In order (not) to + v= So as (not) to: để (không) làm gì
In order that+ Clause
Đáp án B “fail” chưa được chia ở ngôi thử 3 sổ ít
C sai nghĩa của câu
D sai cấu trúc “ In order not to”
Question 48: She gave in her notice. She planned to start her new job in January
A. She gave in her notice, plan to start her new job in January
B. She gave in her notice with a view to starting her new job in January
C. Her notice was given in with an aim to start her new job in January
D. Her notice was given in order for her to start her new job in January.
→ With a view to doing: với mục đích là
Question 49: Joe does a lot of exercise. He’s still very fat
A. Despite the fact that doing a lot of exercise, Joe’s still very fat
B. Joe does a lot of exercise, so he’s very fat
C. Even though joe does a lot of exercise, he’s very fat.
D. Joe’s very fat, but he does a lot of exercise.

Hocmai– Ngôi trường chung của học trò Việt Tổng đài tư vấn: 1900 69 33 - Trang | 14-

284
Hocmai.vn – Website học trực tuyến số 1 tại Việt Nam
Khóa học Luyện thi THPT quốc gia PEN C –N3 Môn Tiếng Anh côHương Fiona

→ Tạm dịch: Joe tập thể dục rất nhiều. Anh ấy vẫn béo.
A.Mặc dù sự thật là tập thể dục rất nhiều, Joe vẫn rất béo (sai vì the fact that + MĐ)
B.Joe tập thể dục rất nhiều, vì thế anh ý rất béo (sai nghĩa)
C. Mặc dù Joe tập thể dục rất nhiều, anh ẩy rất bẻo
D. Joe rất béo, nhưng anh ấy tập thể dục rất nhiều (sai nghĩa)
→ chọn đáp án C
Question 50: Canada does not require us citizens to obtain passports to enter the country. Mexico does
not require US citizens to do the same.
A. Canada does not require us citizens to obtain passports to enter the country, and Mexico does either
B. Canada does not require us citizens to obtain passports to enter the country, and Mexico does not, either
C. Canada does not require us citizens to obtain passports to enter the country, and neither Mexico does
D. Canada does not require us citizens to obtain passports to enter the country while Mexico does
→ Tạm dịch: Canada không yêu cầu công dân Mĩ phái có hộ chiếu khi nhập cảnh.Mexico cũng không yêu
cầu công dân Mĩ làm như thế.
Cấu trúc: S + trợ động từ có not, either = Neither + trợ động từ + S
Eg: I don’t, either = Neither do I → A, C sai Đáp án D sai vì nghĩa (Canada không yêu cầu công dân Mĩ phải
có hộ chiếu khi nhập cành trong đó Mexico thì có)
→ Chọn đáp án B.
Question 51. I do my homework and schoolwork in separate books. I don't get muddled up.
A. Having two separate books at home and at work helps me avoid getting muddled up.
B. I do my homework and schoolwork in separate hooks so that I don't get muddled up.
C. I do not get muddled up due to the separation between homework and schoolwork.
D. I would get muddled up if I did not separate homework from schoolwork.
→ Ghép hai câu bằng sử dụng "so that" để diễn đạt mục đích
Question 52. Some economists argue that new technology causes unemployment. Others feel that it
allows more jobs to be created.
A. Some economists argue that new technology causes unemployment, so others feel that it allows more jobs
to be created.
B. Arguing that new technology causes unemployment, other economists feel that it allows more jobs to be
created.
C. Besides the argument that new technology causes unemployment, some economists feel that it allows more
jobs to be created.

Hocmai– Ngôi trường chung của học trò Việt Tổng đài tư vấn: 1900 69 33 - Trang | 15-

285
Hocmai.vn – Website học trực tuyến số 1 tại Việt Nam
Khóa học Luyện thi THPT quốc gia PEN C –N3 Môn Tiếng Anh côHương Fiona

D. Some economists argue that new technology causes unemployment, whereas others feel that it allows more
jobs to be created.
→ Ghép hai câu bằng sử dụng "whereas" để chỉ sự ngược lại

Nguồn : Hocmai.vn
Giáo viên : Hương Fiona

Hocmai– Ngôi trường chung của học trò Việt Tổng đài tư vấn: 1900 69 33 - Trang | 16-

286
Hocmai.vn – Website học trực tuyến số 1 tại Việt Nam
Khóa học Luyện thi THPT quốc gia PEN C –N3 Môn Tiếng Anh côHương Fiona

PHƯƠNG PHÁP CHUNG LÀM BÀI HOÀN THÀNH CÂU


(ĐÁP ÁN BÀI TẬP TỰ LUYỆN )
Giáo viên : Nguyễn Thanh Hương
Các bài tập trong tài liệu này được biên soạn kèm theo bài giảng “Phương pháp chung làm bài hoàn thành câu” thuộc Khóa học Luyện
thi THPT quốc gia PEN - C: Môn Tiếng Anh (cô Hương Fiona )” tại website Hocmai.vn để giúp các Bạn kiểm tra, củng cố lại các kiến
thức được giáo viên truyền đạt trong bài giảng tương ứng. Để sử dụng hiệu quả, Bạn cần học trước bài giảng, sau đó làm đầy đủ các bài
tập trong tài liệu này.

Mark the letter A, B, C or D on your answer sheet to indicate the correct answer to each of the following
questions.
Question 1: Since he failed his exam, he had to_________for it again.
A. pass B. make C. take D. Sit
→ Sit và take đều có thể đi với exam nhưng điểm khác biệt là sit có thể dùng với for, trong khi take thì
không thể
Như vậy, mấu chốt của câu này nằm ở chỗ giới từ for, chính chữ for này đã loại take
Question 2: Is there _______ at all I can help?
A. everything B. anything C. something D. one thing
→ Anything (at all) được dùng trong câu hỏi
Trong câu hỏi đôi khi có thể dùng something, tuy nhiên phía sau có “at all” nên không thể dùng something
ở đây
Everything và one thing ở đây không phù hợp vê nghĩa
Question 3: Don’t worry. He’ll do the job as_________as possible.
A. economizing B. econimic C. economical D. Economically
→ Ở đây ta cần một trạng từ để bổ sung ý nghĩa cho động từ do
Chỉ có đáp án D là trạng từ
Economically: (một cách) kinh tế (hợp lý về kinh tế)
Question 4: Only when you grow up _________the truth.
A. you will know B. you know C. do you know D. will you know
→ Đây là câu đảo ngữ với only when
Cấu trúc: only when + mệnh đề bình thường + mệnh đề đảo ngữ
Ở đây, vế sau (vế đảo ngữ) rõ ràng là thì tương lai (vì vế trước nói: only when you grow up: chỉ khi bạn
lớn lên) => chúng ta phải sử dụng will you know (đảo will lên trước chủ ngữ)
Question 5: My brother left his job last week because he did not have any_________to travel.
A. position B. chance C. ability D. Location
→ Have (stand) a chance to do st: có cơ hội làm gì
Dịch câu: Anh trai tôi đã bỏ công việc của mình vào tuần trước vì anh không có bất kỳ cơ hội để đi du lịch.
Question 6: John paid $2 for his meal, _________he had thought it would cost.
A. not as much B. not so much as C. less as D. not so many as

Hocmai– Ngôi trường chung của học trò Việt Tổng đài tư vấn: 1900 69 33 - Trang | 1-

287
Hocmai.vn – Website học trực tuyến số 1 tại Việt Nam
Khóa học Luyện thi THPT quốc gia PEN C –N3 Môn Tiếng Anh côHương Fiona

→ $2 là danh từ không đếm được => không thể dùng many, D loại
C không phù hợp về nghĩa
A không chính xác bởi thiếu “as”, đáp án A sẽ đúng nếu như là “not as much as”
Đáp án là B: not so much as: không nhiều như…
Question 7: It is very important for a film or a company to keep_________the changes in the market.
A. pace of B. track about C. touch with D. up with
→ Keep up with st: bắt kịp, theo kịp với cái gì
Dịch: Nó là rất quan trọng đối với một bộ phim hay một công ty để theo kịp với những thay đổi trên thị
trường.
Question 8: I’m sure you’ll have no_________the exam.
A. difficulty passing B. difficulties to pass
C. difficulty to pass D. difficulties of passing
→ Cấu trúc have (no) difficulty (in) doing st: có (không có) khó khăn (trong việc) làm gì
Dịch câu: Tớ chắc rằng cậu sẽ không có khó khăn gì trong việc vượt qua kì thi
Question 9: I _________this letter around for days without looking at it.
A. carry B. must carry C. have been carrying D. am carrying
→ Ở đây có cụm trạng ngữ chỉ thời gian: for days => dùng thì hoàn thành hoặc thì hoàn thành tiếp diễn
Đáp án là C (thì hiện tại hoàn thành tiếp diễn)
Dịch: Tôi đã mang lá thư này nhiều ngày mà không nhìn tới nó
Question 10: Vietnam’s rice export this year will decrease_________about 10%, compared with that of last
year.
A. with B. at C. by D. On
→ Cụm decrease/increase by + …%: giảm/tăng …% (so với trước)
Dịch: xuất khẩu gạo của Việt Nam trong năm nay sẽ giảm khoảng 10%, so với cùng kỳ năm ngoái.
Question 11: I won’t change my mind_________what you say.
A. whether B. no matter C. because D. Although
→ No matter st/ who/what…: bất kỳ cái gì, ai, cái gì…
Dịch: tôi sẽ không thay đổi quyết định dù bạn nói gì đi nữa
Question 12: My car isn’t_________. It’s always letting me down.
A. believable B. reliable C. colorable D. Conceivable
→ Reliable : đáng tin cậy
Believable: có thể tin được
Conceivable: có thể quan niệm được, có thể tưởng tượng được
Không có từ colorable
Dịch: Xe của tôi không đáng tin cậy chút nào. Nó luôn luôn làm tôi thất vọng
Question 13: Many applicants find a job interview__________ if they are not well-prepared for it.
A. impressive B. stressful C. threatening D. time-consuming
→ Dịch câu: Nhiều ứng viên cảm thấy phỏng vấn rất áp lực nếu như họ chưa chuẩn bị tốt cho nó

Hocmai– Ngôi trường chung của học trò Việt Tổng đài tư vấn: 1900 69 33 - Trang | 2-

288
Hocmai.vn – Website học trực tuyến số 1 tại Việt Nam
Khóa học Luyện thi THPT quốc gia PEN C –N3 Môn Tiếng Anh côHương Fiona

Impressive: ấn tượng
Stressful: áp lực
Threatening: đe doạ
Time-consuming: tốn thời gian
Question 14: The manager__________ him for a minor mistake.
A. accused B. charged C. complained D. Blamed
→ Dịch câu: Người quản lý chê trách anh vì một sai lầm nhỏ
Accuse: buộc tội (đi với giới từ of)
Charge: buộc tội (nếu mang nghĩa là buộc tội ai thì dùng charge sb, không có giới từ)
Complain: phàn nàn
Blame: đổ tội, chê trách (đi với giới từ for)
Question 15: I __________ hurry. It’s nearly 8.00, and my first class starts at 8.15.
A. would prefer B. can’t help C. would rather D. had better
→ Had better = should: nên
Dịch câu: Tôi nên nhanh chóng thôi. Giờ gần 8h rồi, mà tiết học đầu tiên bắt đầu lúc 8h15
Question 16: He managed to keep his job__________ the manager had threatened to sack him.
A. although B. despite C. unless D. Therefore
→ Although (mặc dù) + một mệnh đề
Despite + danh từ/cụm danh từ
Unless và therefore không phù hợp về nghĩa của câu
Dịch câu: Anh đã giữ được công việc mặc dù quản lý đã doạ sa thải anh.
Question 17: Don’t touch that wire or you’ll get an electric__________.
A. shock B. fire C. charge D. Current
→ Cụm từ “electric shock”: sốc điện, điện giật
Dịch câu: Đừng chạm vào dây đó hoặc bạn sẽ bị điện giật.
Question 18: The car had a(n) __________ tire, so we had to change the wheel.
A. bent B. flat C. cracked D. Injured
→ “flat tire”: xịt lốp xe
Bent: bẻ cong, không thẳng
Cracked: bị vỡ, bị vụn
Injured: bị chấn thương
Dịch câu: Chiếc xe bị xịt lốp, vì vậy chúng tôi đã phải thay đổi bánh xe
Question 19: Does television adequately reflect the ethnic and cultural__________ of the country.
A. costom B. diversity C. alternations D. Article
→ Dịch câu: truyền hình có phản ánh đầy đủ sự đa dạng sắc tộc và văn hóa của đất nước?
Không có từ costom
Diversity: sự đa dạng
Alternations: sự xen kẽ

Hocmai– Ngôi trường chung của học trò Việt Tổng đài tư vấn: 1900 69 33 - Trang | 3-

289
Hocmai.vn – Website học trực tuyến số 1 tại Việt Nam
Khóa học Luyện thi THPT quốc gia PEN C –N3 Môn Tiếng Anh côHương Fiona

Article: đồ, vật phẩm, điều khoản, bài báo…


Question 20: You should make a(n) __________ to overcome this problem.
A. trial B. impression C. effort D. Apology
→ “make an effort to do st”: cố gắng, ráng sức làm gì đó
Dịch câu: Bạn nên nỗ lực để khắc phục vấn đề này
Trial: sự thử thách
Impression: sự ấn tượng
Apology: lời xin lỗi
Question 21: –“You look beautiful with your new hairstyle!”
– “____________”.
A. Not at all B. It’s kind of you to say so
C. Very kind of your part D. Willingly
→ Dịch:
- Bạn trông thật đẹp với kiểu tóc mới đó!
- Bạn thật tốt khi nói như vậy.
Not at all: không có gì
Very kind of your part: Bạn thật là tốt bụng. Dùng đáp lại khi nhận được sự giúp đỡ từ ai đó.
Willingly: Sẵn lòng, vui lòng. Dùng khi nhận được lời đề nghị giúp đỡ từ ai đó
Question 22: It was only__________ he told me his surname that I realized that we had been to the same
school.
A. then B. until C. as soon as D. When
→ Dịch câu: Chỉ khi anh nói với tôi họ của anh tôi mới nhận ra rằng chúng tôi đã học cùng trường.
Then: sau đó
Until: cho đến tận khi (Ở đây sẽ đúng nếu như sử dụng It was not until….)
As soon as: ngay khi
When: khi (only when: chỉ khi)
Question 23: The girl __________ design had been chosen stepped to the platform to receive the award.
A. whose B. whom C. that D. Which
→ Ở đây chủ ngữ của vị ngữ “had been chosen” là design
Mà chủ ngữ chính trong câu là the girl, do đó chúng ta phải sử dụng mệnh đề quan hệ sở hữu whose để nói
về mối quan hệ giữa the girl và design (whose design được hiểu là the girl‟s design)
Question 24: My responsibility is to wash dishes and__________ the garbage.
A. take care of B. take out C. take off D. take over
→ Take out được dùng ở đây với nghĩa tương tự như take away: mang đi
Dịch: trách nhiệm của tôi là phải rửa chén đĩa và mang rác đi
Question 25: Don’ t forget to ________ the alarm clock for 5 o‟clock tomorrow morning.
A. ring B. put C. wind D. set
→ Set: cài đặt

Hocmai– Ngôi trường chung của học trò Việt Tổng đài tư vấn: 1900 69 33 - Trang | 4-

290
Hocmai.vn – Website học trực tuyến số 1 tại Việt Nam
Khóa học Luyện thi THPT quốc gia PEN C –N3 Môn Tiếng Anh côHương Fiona

Câu này dịch như sau: Đừng quên cài chuông đồng hồ báo 5 giờ sáng mai
Question 26: Books are no longer the only _________ of stories and information.
A. basis B. site C. source D. style
→ Source of information: nguồn thông tin
Câu này dịch như sau: Sách không còn là nguồn thông tin và truyện duy nhất nữa
Question 27: ________ for our health.
A. One’s diet is helpful in extra fiber. B. Helpful one‟s diet is extra fiber
C. Extra fiber is one‟s helpful diet D. Extra fiber in one‟s diet is helpful
→ Câu này dịch như sau: ...................cho sức khỏe của chúng ta.
Đề bài đã cho cụm giới từ + danh từ => đáp án cần có chủ ngữ và động từ
Trong 4 đáp án cấu trúc phù hợp nhất là S+ be+ adj + giới từ + cụm danh từ vì chúng ta có cụm tính từ
helpful for [ có ích cho ai/ vật gì đó]
Xét về ngữ nghĩa:
A. Chế độ ăn uống thì có ích về chất xơ cho sức khỏe của chúng ta.
B. Chế độ ăn uống của một người hữu ích là chất xơ cho sức khỏe của chúng ta.
C. Chất xơ là chế độ ăn uống có ích của chúng ta cho sức khỏe của chúng ta.
D. Chất xơ trong chế độ ăn uống của chúng ta có ích cho sức khỏe của chúng ta.
Question 28: The growth of two-income families in the United States ______ of people moving to a new
social class.
A. has resulted in millions B. resulting in millions
C. results of millions D. millions of results
→ Result in: dẫn đến, kết quả là
Trong câu đã có chủ ngữ và tân ngữ -> động từ.
Câu này dịch như sau: Sự gia tăng số gia đình có 2 nguồn thu nhập ở nước mỹ đã dẫn đến hàng triệu người
chuyển sang một tầng lớp xã hội mới.
Question 29: Black, red, and even bright pink diamonds ______ .
A. occasionally found B. have occasionally been found
C. have occasionally found D. occasionally to find
→ “Black, red, bright pink diamonds” là các loại kim cương là chủ ngữ chỉ vật ->động từ chia dạng bị
động.
Câu này dịch như sau: Kim cương đen, đỏ và thậm chí hồng sáng được tìm thấy một cách ngẫu nhiên.
Question 30: An adviser to both Franklin Delano Roosevelt and Harry Truman, _____ of Bethune-Cook
man College.
A. the founder was Dr, Mary Mcleod Bethune
B. did the founder Dr, Mary Mcleod Bethune
C. Dr. Mary Mcleod Bethune, who was the founder
D. Dr. Mary Mcleod Bethune was the founder
→ Adviser: cố vấn viên -> người, trước “of” là danh từ (cụm danh từ).

Hocmai– Ngôi trường chung của học trò Việt Tổng đài tư vấn: 1900 69 33 - Trang | 5-

291
Hocmai.vn – Website học trực tuyến số 1 tại Việt Nam
Khóa học Luyện thi THPT quốc gia PEN C –N3 Môn Tiếng Anh côHương Fiona

Câu này dịch như sau: Một cố vấn viên của cả Franklin Delano Roosevelt và Harry Truman, Dr, Mary
Mcleod Bethune là nhà sáng lập của Bethune-Cook man College.
Question 31: Before _______ , they used horse drawn wooden carts
A. farmers had tractors B. farmers have had tractors
C. tractors owned by farmers D. having tractors farmers
→ Sau “before” dùng thì QKĐ
Câu trúc: Before S + QKĐ, S+ QKHT
Câu này dịch như sau: Trước khi nông dân có xe kéo, họ sử dụng xe ngựa kéo bằng gỗ
Question 32: The door is unlocked; _______ here last night.
A. Something strange was happened
B. Something strange should have happened
C. Something strange had happened
D. Something strange could have happened
→ Could have V(P2): dự đoán điều có thể xảy ra trong quá khứ.
Câu này dịch như sau: Cửa chính bị mở, điều gì đó kỳ lạ có thể đã xảy ra ở đây đêm qua.
Question 33: Although he supports the Council, he does not take an active _____ in politics.
A. affair B. play C. part D. charge
→ Take an active part/role: có, giữ vai trò tích cực
Câu này dịch như sau: Mặc dù anh ấy ủng hộ Hội đồng, nhưng anh ấy không có một vai trò quan trọng
trong chính trị.
Question 34: Why don‟t you wear that blue dress of yours? It _______ you.
A. agrees B. goes with C. suits D. watches
→ Suit: phù hợp với ( về quần áo)
Câu này dịch như sau: Tại sao bạn không mặc chiếc váy màu xanh của bạn nhỉ? Nó phù hợp với bạn đấy.
Question 35: Although he claims to have left his job voluntarily, he was actually______ for misconduct.
A. dismissed B. dispelled C. resigned D. released
→ Dismiss: sa thải
dispell: xua đuổi
resign: từ chức
release: thả, tha
Câu này dịch như sau: Mặc dù anh ta đã tuyên bố từ bỏ công việc của mình một cách tự nguyện, nhưng
anh ta đã thực sự bị sa thải vì hành vi sai trái.
Question 36: Because aluminum is lighter and cheaper_______, it is frequently used for high tension
power transmission.
A. as copper B. more copper C. for copper D. than copper
→ So sánh hơn với tính từ ngắn: tobe+ adj_er+than
Câu này dịch như sau: Bởi vì nhôm nhẹ và sáng hơn đồng nên nó thường được sử dụng cho
truyền tải điện thế cao.

Hocmai– Ngôi trường chung của học trò Việt Tổng đài tư vấn: 1900 69 33 - Trang | 6-

292
Hocmai.vn – Website học trực tuyến số 1 tại Việt Nam
Khóa học Luyện thi THPT quốc gia PEN C –N3 Môn Tiếng Anh côHương Fiona

Question 37: Since the flood the number of homeless people _______ dramatically.
A. are increasing B. had increased C. increase D. has increased
→ Trong câu có từ since: kể từ khi (dấu hiệu của hiện tại hoàn thành )
The number of + N => V chia ngôi 3 số ít
Câu này dịch là: kể từ sau trận lụt, số ng vô gia cư đã tăng đáng kể
Question 38: While everybody else in our class prefers working in groups, Mina likes working ______
A. on herself B. on her own C. of her ow D. in herself
→ Cấu trúc: on one’s own = by oneself : một mình làm gì đó câu C: cấu trúc: st of one’s own: cái gì của ai
đó
Câu này dịch là: trong khi mọi người trong lớp của chúng ta thích làm việc theo nhóm, Mina thích làm một
mình.
Question 39: ________, the young mother appeared visibly very happy after the birth of her child.
A. Tired as she was B. She was tired C. As tired D. Despite tired
→ A.Cấu trúc đảo tính từ trong câu có đại từ quan hệ: Though/ Even though + S + be + adj = Adj + as/though
+ S +be… : Mặc dù …..
B. 1 mệnh đề thông thường (không mang nghĩa tương phản)
C. không tồn tại
D. Despite + N/Ving: mặc dù…
=> chọn A vì phải mang nghĩa tương phản
Câu này dịch là: mặc dù mệt, bà mẹ trẻ vẫn tỏ ra rất vui sau khi hạ sinh đứa con của mình
Question 40: Could you please tell me __________ ?
A. where does my uncle's room B. where is my uncle’s room
C. where my uncle’s room is D. where my uncle's room
→ Cụm từ cần điền vào chỗ trống không phải là 1 câu hỏi mà là 1 cụm từ thông thường có nghĩa là “phòng
của bác tôi ở đâu”: không cần đảo tobe
Chọn C: where my uncle’s room is
Câu này dịch là: Bạn có thể chỉ cho tôi phòng của bác tôi ở đâu được không?

Nguồn : Hocmai.vn
Giáo viên : Hương Fiona

Hocmai– Ngôi trường chung của học trò Việt Tổng đài tư vấn: 1900 69 33 - Trang | 7-

293
Hocmai.vn – Website học trực tuyến số 1 tại Việt Nam
Khóa học Luyện thi THPT quốc gia PEN C –N3 Môn Tiếng Anh côHương Fiona

PHƯƠNG PHÁP CHUNG LÀM BÀI HOÀN THÀNH ĐOẠN VĂN


( ĐÁP ÁN BÀI TẬP TỰ LUYỆN )
Giáo viên : Nguyễn Thanh Hương
Các bài tập trong tài liệu này được biên soạn kèm theo bài giảng “Phương pháp chung làm bài hoàn thành đoạn văn” thuộc Khóa học
Luyện thi THPT quốc gia PEN - C: Môn Tiếng Anh (cô Hương Fiona )” tại website Hocmai.vn để giúp các Bạn kiểm tra, củng cố lại
các kiến thức được giáo viên truyền đạt trong bài giảng tương ứng. Để sử dụng hiệu quả, Bạn cần học trước bài giảng, sau đó làm đầy
đủ các bài tập trong tài liệu này.

I. Read the following passage and mark the letter A, B, C, or D on your answer sheet to indicate
the correct answer to each of the blanks.
When the weather is cold, it is not very much fun to (1)______ a bus. People have to
stand at a bus-stop for minutes or even hours. They talk about the bus and (2) _______
about the weather. Most of them feel cold. Some have to drink coffee to stay warm.
Traveling by bus (3) ___ pollution, but people often would rather drive their cars. Many
people are not used to the bus schedules, and they do not like to wait.
On the other hand, many people have been (4) _ the bus every day for many years. They
are used to it. They say the bus has been coming on time every day, and they have never
been late for work. In (5) ______, they do not need a parking place in all that time. Buses
are very convenient when you are used to them.
Question 1:A. search B. look for C. stand for D. wait for
Question 2:A. tell B. complain C. comment D. judge
Question 3:A. cuts B. reduces C. stops D. eliminates
Question 4:A. looking B. waiting C. getting D. taking
Question 5:A. end B. last C. addition D. conclusion
Hướng dẫn:
Question 1:
Đáp án đúng D: wait for a bus = chờ đợi xe buýt.
A. search (tìm kiếm ai/ cái gì) mà ở đây là chờ đợi xe buýt tại trạm xe buýt (a bus-stop).
B. look for = tìm kiếm ai/ cái gì (giống như phương án A).
C. stand for – đại diện cho, dùng để viết cho tên đầy đủ của một từ viết tắt.
Question 2: Đáp án là B: complain about something (than phiền cái gì)
Question 3:
B. Đáp án đúng. To reduce something, làm cái gì giảm về lượng, kích thước, mức độ, tầm
quan trọng.... → reduce pollution: làm giảm sự ô nhiễm (về mức độ)
A .cut something = làm cho cái gì ngắn hơn, thấp hơn, nhỏ hơn...
C .stop something = ngưng một hoạt động.
D .eliminate something = loại trừ đi cái gì

Hocmai– Ngôi trường chung của học trò Việt Tổng đài tư vấn: 1900 69 33 - Trang | 1-

294
Hocmai.vn – Website học trực tuyến số 1 tại Việt Nam
Khóa học Luyện thi THPT quốc gia PEN C –N3 Môn Tiếng Anh côHương Fiona

Question 4: take the bus/ the train... (đi đến đâu bằng một phương tiện giao thông nhất
định).
Đáp án đúng là D (taking).
Question 5: In addition (ngoài ra) được dùng để chỉ sự nói thêm cho ý trước — nêu những
lợi ích của việc dùng xe buýt (they have never been late for work...they do not need a parking
place...).
D. In conclusion dùng đế đưa ra kết luận.

II. Read the following passage and mark the letter A,B,C or D on your answer sheet to indicate
the correct word or phrase that best fits each of the numbered blanks.
A trend that has emerged recently is the sharing of childcare (1) __________ between husband and
wife. Young couples will try to arrange their work schedules so that they work opposite hours or
shifts in order that one parent is always home with the children. Since childcare is expensive, this
saves money for the young couple trying to establish themselves and provide a secure environment
for the family. Husband and wife may also share household chores. Some fathers are just as capable
as mothers at cooking dinner, changing and bathing the baby, and doing the laundry.
In some cases, the woman’s salary is for family (2) __________ and the father becomes the
“househusband." These cases are still fairly rare. One positive trend, however, is that fathers seem
to be spending more time with their children. In a recent survey, 41% of the children sampled said
they spend equal time with their mothers and fathers. “This is one of our most significant cultural
changes,” says Dr. Leon Hoffman, who co-directs the Parent Child Center at the New York
Psychoanalytic Society. In practice, for over 30 years, Hoffman has found "a very dramatic
difference in the involvement of the father in everything from care-taking to general decision (3)
__________ around kids' lives.”
Another factor has recently been added to the childcare formula. The number of people who work
from home nearly full-time rose 23% from the last decade. The (4) _________ of technology -
computers, faxes, teleconferencing - has made it easier for at-home workers to be constantly in
touch. Will this new flexibility in the workforce bring a positive change for the (5) __________ of
children? Only time will tell.
Question 1:A. abilities B.possibilities C.techniques D.responsibilities
Question 2:A. payment B. expenses C. fares D. fees
Question 3:A. making B. creating C. holding D. giving
Question 4:A. accessible B. accessibly C. access D. accessibility
Question 5:A.well being B. security C. comfort D. interests
HƯỚNG DẪN
Question 1: Đáp án D
Children responsibilities: trách nhiệm chăm sóc đối với con trẻ
Abilities: khả năng

Hocmai– Ngôi trường chung của học trò Việt Tổng đài tư vấn: 1900 69 33 - Trang | 2-

295
Hocmai.vn – Website học trực tuyến số 1 tại Việt Nam
Khóa học Luyện thi THPT quốc gia PEN C –N3 Môn Tiếng Anh côHương Fiona

Possibilities: khả năng (có thể xảy ra)


Techniques: kỹ thuật
Dịch: Một xu hướng đã nổi lên gần đây là sự chia sẻ trách nhiệm chăm sóc con trẻ giữa chồng và vợ
Question 2: Đáp án B
family expenses: phí tổn trong gia đình
fare: tiền vé
fee: tiền học, lệ phí
payment: sự chi trả
dịch: Trong một số trường hợp, tiền lương của người phụ nữ là dành cho chi phí gia đình và người cha
trở thành "househusband."
Question 3: Đáp án A
Make a decision: đưa ra quyết định
Decision making: (quá trình) đưa ra quyết định
Dịch: Trong thực tế, trong hơn 30 năm, Hoffman đã tìm thấy "một sự khác biệt rất lớn trong sự can
thiệp của người cha trong mọi thứ, từ chăm sóc đến đưa ra quyết định chung xung quanh cuộc sống
của trẻ."
Question 4: Đáp án D
Sau “the” là danh từ → Loại A và B.
Xét về nghĩa: cần tìm từ có nghia là “sự sẵn có” → CHỌN accessibility (access là sự tiếp cận → loại)
Dịch: Khả năng tiếp cận công nghệ - máy tính, fax, hội nghị qua điện thoại - đã làm cho nó dễ dàng
hơn cho công nhân tại nhà trong việc liên tục liên lạc.
Question 5: Đáp án A
Well-being: lợi ích
Security: an ninh, sự bảo đảm an toàn
Comfort: sự an nhàn dễ chịu, sự an ủi
Interests: sự quan tâm, sự chú ý, hấp dẫn
Dịch: Linh hoạt mới này trong lực lượng lao động sẽ mang lại một sự thay đổi tích cực cho lợi ích của
trẻ em?
Chỉ có thời gian sẽ trả lời.
III. Read the following passage and mark the letter A, B, C, or D on your answer sheet to indicate
the correct word or phases that best fits each of the numbered blanks from 1 to 5.
People have always dreamt of living forever. Although we all know this will never happen, we will
want to live as long as possible.
__(1)__, there are advantages and disadvantages of a long life.In the first place, people who live longer
can spend more time with their family and friends. Secondly, __(2)__have busy working lives look
forward to a long, relaxing life, when they can do the things they‘ve never had time for.__(3)__, there
are some serious disadvantages. Firstly, many people become ill and consequently have to spend time
in hospital or become burden __(4)__ the children and friends. Many of them find this dependence

Hocmai– Ngôi trường chung của học trò Việt Tổng đài tư vấn: 1900 69 33 - Trang | 3-

296
Hocmai.vn – Website học trực tuyến số 1 tại Việt Nam
Khóa học Luyện thi THPT quốc gia PEN C –N3 Môn Tiếng Anh côHương Fiona

annoying or embarrassing. In addition to this, __(5)__ the fewer friends they seem to have because old
friends die or become and it’s often difficult to make new friends.
To sum up, living to a very old age is worthwhile for those who stay healthy enough to remain
independent and enjoy life.
Question 1:A. Naturally B. Really C. Consequently D. Surprisingly
Question 2:A. people who B. people C. people D. everyone who
Question 3:A. In other words B. For example C. On the other hand D. Contrary to
Question 4:A. of B. for C. to D. with
Question 5:A. when people get older B. when the older people get
C. the old people get D. the older people get
HƯỚNG DẪN:
Question 1: Đáp án A
Natuarally: tự nhiên/ hiển nhiên
Really: thật sự
Consequently: vì vậy
Surprisingly: một cách ngạc nhiên
People have always dreamt of living forever. Although we all know this will never happen, we will want
to live as long as possible. (1______), there are advantages and disadvantages of a long life. In the first
place, people who live longer can spend more time with their family and friends.
[ Mọi người luôn mơ ước về việc sống mãi. Mặc dù chúng ta biết điều này sẽ không bao giờ xảy ra, chúng
ta muốn sống càng lâu càng tốt. Hiển nhiên, sống quá thọ cũng có những thuận lợi và bất lợi. Trước tiên,
những người sống thọ hơn có thể dành nhiều thời gian hơn bên ia đình và bạn bè. ]
Question 2: Đáp án A
Câu đã có động từ chính “ look forward to...” nên trước đó phải là chủ ngữ + mệnh đề quan hệ để bổ sung
cho chủ ngữ đó => chọn people who
Secondly, __(2)__have busy working lives look forward to a long, relaxing life, when they can do the
things they‘ve never had time for.
[ Thứ hai những người mà có đời sống công việc bận rộn hi vọng một cuộc sống thoải mái và dài lâu, khi
họ có thể làm những việc họ chưa bao giờ có thời gian để làm.]
Question 3: Đáp án C
In other words: nói cách khác
For example: Ví dụ như
On the other hand: mặt khác
Contrary to + danh từ : ngược lại
(3)_____, there are some serious disadvantages.
[ Mặt khác, có một số bất lợi nghiêm trọng.]
Question 4: Đáp án C
Cụm từ: burden to sb [ là gánh nặng đối với ai]

Hocmai– Ngôi trường chung của học trò Việt Tổng đài tư vấn: 1900 69 33 - Trang | 4-

297
Hocmai.vn – Website học trực tuyến số 1 tại Việt Nam
Khóa học Luyện thi THPT quốc gia PEN C –N3 Môn Tiếng Anh côHương Fiona

Firstly, many people become ill and consequently have to spend time in hospital or become burden (4)
the children and friends. Many of them find this dependence annoying or embarrassing.
[ Trước hết, nhiều người ôm đau và vì vậy phải dành thời gian ở bệnh viện hoặc trở thành gánh nặng đối
với con cái và bạn bè. Nhiều người trong số họ cảm thấy sự lệ thuộc này rất khó chịu hoặc ngượng ngùng.]
Question 5: Đáp án D
Mệnh đề đã có tân ngữ “ the fewer friends” nên cần chủ ngữ + động từ => Chọn D
Không chọn mệnh đề có when vì phía sau đã có mệnh đề lý do “ because”
In addition to this, (5) the fewer friends they seem to have because old friends die or become ill and it’s
often difficult to make new friends.
To sum up, living to a very old age is worthwhile for those who stay healthy enough to remain independent
and enjoy life.
[Thêm vào đó, người già trở nên có ít bạn bè hơn trước đây bởi vì bạn cũ của họ đã mất hoặc ốm đau và
thường khó kết giao bạn mới.Tóm lại, sống đến già rất đáng trân trọng đối với những người đủ khỏe mạnh
để tự lập và hưởng thụ cuộc sống.]
IV. Read the following passage and choose A, B, C, or D to indicate the correct answer to each of
the questions.
Tyler Perry
Perry had a rough childhood. He was physically and sexually abused growing up, got kicked out of
high school, and tried to commit suicide ___(1)___once as a preteen and again at 22. At 23 he move
to Atlanta and took ___(2)___ odd jobs as he started working on his stage career.
In 1992 he wrote, produced, and starred in his first theater ___(3)___, I Know I’ve Been Changed,
somewhat informed by his difficult upbringing. Perry put all his savings into the show and it failed
miserably; the run lasted just one weekend and only 30 people came to watch. He kept up with the
production, working more odd jobs and often slept in his car to get by. Six years later, Perry finally
___(4)___ through when, on its seventh run, the show became a success. He‘s since gone on to have
an extremely successful career ___(5)___ a director, writer, and actor. In fact, Perry was named Forbes’
highest paid man in the field.
Question 1: A. twice B. two C. second D. double
Question 2: A. up B. off C. in D. to
Question 3: A. producer B. productivity C. production D. productive
Question 4: A. went B. broke C. got D. put
Question 5: A. same B. as C. like D. as soon as
HƯỚNG DẪN:
Question 1. A
Twice :hai lần
Perry had a rough childhood. He was physically and sexually abused growing up, got kicked
out of high school, and tried to commit suicide ___(31)___—once as a preteen and again at 22.
=> Perry có thờ thơ ấu khó khăn. Ông ấy bị chế giễu về phát triền thể hình và giới tính, bị đuổi

Hocmai– Ngôi trường chung của học trò Việt Tổng đài tư vấn: 1900 69 33 - Trang | 5-

298
Hocmai.vn – Website học trực tuyến số 1 tại Việt Nam
Khóa học Luyện thi THPT quốc gia PEN C –N3 Môn Tiếng Anh côHương Fiona

học và cố gắng tự tử 2 lần, 1 lần trước tuổi 13 và một lần lúc 22 tuổi.
Question 2. A
Take up: bắt đầu( công việc/ sở thích)
At 23 he moved to Atlanta and took ___(32)___ odd jobs as he started working on his stage
career.=> Năm 23 tuổi ông ấy bắt đầu những công việc lạ khi bắt đầu giai đoạn sự nghiệp của
mình.
Question 3. C
Producer: nhà sản xuất
Productivity: năng suất
Production: việc sản xuất
Productive: năng suất cao
In 1992 he wrote, produced, and starred in his first theater ___(3)___, => Năm 1992 ôn ấy
viết, sản xuất và nổi tiếng với tác phẩm sản xuất phim đầu tiên.
Question 4. B
Go through: đi xuyên qua, trải qua
Break through: đột phá
Get through: làm xong
Put through: tiếp tục (hoàn thành công việc)
Six years later, Perry finally ___(34)___ through when, on its seventh run, the show became a
success. => 6 năm sau đó, Perry cuối cùng đã có bước đột phá khi trong lần sản xuất thứ 7, màn
trình diễn đã thành công.
Question 5. B
Same: giống như,
As: như là đi với nghề nghiệp, hay dùng “work as”
Like: giống, như là (ngoại hình)
As soon as: ngay khi
He‘s since gone on to have an extremely successful career ___(5)___ a director, writer, and
actor. => Kể từ đó sự nghiệp của ông thành công như là 1 đạo diễn, nhà văn và diễn viên.

V. Read the following passage and mark the letter A, B,C or D on your answer sheet to indicate the
correct word or phrase that best first each of the numbered blank from 1 to 5
American folk music originated with (1) _____ people at a time when the rural population was isolated
and music was not (2)______spread by radio, records, or music video. It was (3)_____by oral traditional
and is noted for its energy ,humor, and emotional impact. The major source of aerly American folk songs
was music from the Bristish Isles , but songs from Africa as songs of the American Indians have
significant part in its heritage. Later settler from other countries also contributed songs. In the nineteenth
century, composer Steven Foster wrote some of the most enduringly popular of all American songs
,(4)______soon became part of the folk tradition. Beginning in the 1930s , Woody Guthrie gained great

Hocmai– Ngôi trường chung của học trò Việt Tổng đài tư vấn: 1900 69 33 - Trang | 6-

299
Hocmai.vn – Website học trực tuyến số 1 tại Việt Nam
Khóa học Luyện thi THPT quốc gia PEN C –N3 Môn Tiếng Anh côHương Fiona

popularity by adapting melodies and lyrics and supplying new ones as well. In the 1950s and 1960s ,
singer – composers such as Peter Seeger , Bob Dylan , Joan Baez continued this tradition by „urban’
folk music. Many of these songs deal (5)_____important social issue, such as racial intergration and the
war in Vietnam.
Question 1:A. typical B. ordinary C. common D. popular
Question 2:A. yet B. still C. until D. even
Question 3:A. transferred B. transited C. transmitted D. transformed
Question 4:A. who B. which C. that D. this
Question 5:A. with B. in C. by D. at
HƯỚNG DẪN
Question 1: B
Typical: điển hình, đặc trưng
Ordinary: bình thường, thường
Common: thông thường, phổ biến
Popular: nổi tiếng, được ưa thích
American folk music originated with ordinary people at a time when the rural population was isolated:
âm nhạc dân gian Mỹ bắt nguồn từ những người bình thường tại một thời điểm khi dân số nông thôn
được phân lập
Question 2: A
Yet:chưa Not yet: chưa, còn chưa…
Still: vẫn, vẫn còn
Until: cho đến khi
Even: thậm chí (phó từ)
Music was not yet spread by radio, audios, compact disks, or music DVDs: âm nhạc vẫn chưa lan rộng
bởi đài phát thanh, âm thanh, đĩa CD, nhạc DVD.
Question 3: C
Transfer: dời, chuyển, chuyển giao
Transit: vận chuyển, quá cảnh
Transmit: truyền (truyền nhiệt, truyền âm thanh, ….)
Transform: biến đổi, biến hình
“it” ở đây là music => dùng động từ transmit (truyền âm thanh)
It was transmitted by oral tradition and is noted for its energy, humor, and emotional impact: Nó được
truyền miệng và được ghi nhận vì năng lượng của nó, hài hước, và tác động cảm xúc.
Question 4: B
Danh từ cần được thay thế ở đây là all American songs (vật) nên không thể sử dụng who => A loại
That không thể làm mệnh đề quan hệ khi đứng sau dấu phảy => C loại
This không có chức năng làm mệnh đề quan hệ trong câu
Đáp án là B: which được thay thế cho all American songs

Hocmai– Ngôi trường chung của học trò Việt Tổng đài tư vấn: 1900 69 33 - Trang | 7-

300
Hocmai.vn – Website học trực tuyến số 1 tại Việt Nam
Khóa học Luyện thi THPT quốc gia PEN C –N3 Môn Tiếng Anh côHương Fiona

In the nineteenth century, composer Stephen Foster wrote some of the most enduringly popular of all
American songs, which soon became part of the folk tradition.
Trong thế kỷ XIX, nhà soạn nhạc Stephen Foster đã viết một số trong tất cả các ca khúc Mỹ phổ biến
lâu dài nhất, cái mà đã sớm trở thành một phần của nhạc dân gian truyền thống.
Question 5: A
deal with something: giải quyết, xử lý(việc gì)
deal in something: buôn bán cái gì
Động từ deal không đi kèm với giới từ by và at something
Many of these songs dealt with important social issues, such as racial integration and the war in
Vietnam.
Nhiều trong số những bài hát này xử lý các vấn đề xã hội quan trọng, chẳng hạn như hội nhập chủng
tộc và cuộc chiến tranh ở Việt Nam.
VI. Read the following passage and circle the letter A, B, C or D on your answer sheet to indicate
the correct word for each of the blanks .
When you read something in a foreign language, you frequently come across words you do not fully
understand. Sometimes you (1) ______ the meaning in a dictionary and sometimes you guess. The
strategy you adopt depends very much upon the degree of accuracy you require and the time at your
disposal.
If you are the sort of person who tends to turn to the dictionary frequently, it is (2) ______ remembering
that every dictionary has its limitations. Each definition is only an approximation and one builds up an
accurate picture of the meaning of a word only after meeting it in a (3) ______ of contexts. It is also
important to recognize the special dangers of dictionaries that translate from English into your native
language and vice versa. If you must use a dictionary, it is usually far safer to consult an English-
English dictionary.
In most exams you are not permitted to use a dictionary. (4) ______ you are allowed to use one, it is
very time-consuming to look up words, and time in exams is usually limited. You are, therefore , forced
to guess the meaning of unfamiliar words.
When you come across unknown words in an exam text, it is very easy to panic. However, if you
develop efficient techniques for guessing the meaning, you will overcome a number of possible
problems and help yourself to understand far more of the text than you at first thought likely.
Two strategies which may help you guess the meaning of a word are: using contextual clues, both
within the sentence and outside, and making use of clues (5) ______ from the formation of the word

Question 1:A. control B. inspect C. check D. examine


Question 2:A. valuable B. worth C. essential D. vital
Question 3:A. variation B. multiple C. diversity D. variety
Question 4:A. Even if B. Provided C. Although D. In case
Question 5:A. originated B. extracted C. derived D. coming

Hocmai– Ngôi trường chung của học trò Việt Tổng đài tư vấn: 1900 69 33 - Trang | 8-

301
Hocmai.vn – Website học trực tuyến số 1 tại Việt Nam
Khóa học Luyện thi THPT quốc gia PEN C –N3 Môn Tiếng Anh côHương Fiona

HƯỚNG DẪN:
Question 1: C
Control: điều khiển, inspect: thanh tra , check: kiểm tra, examine: xem xét.
Sometimes you (1) ______ the meaning in a dictionary and sometimes you guess.
[thỉnh thoảng bạn tra nghĩa trong một từ điển, và đôi khi bạn đoán.]
Question 2: B
It is worth+ V_ing: đáng để làm gì
If you are the sort of person who tends to turn to the dictionary frequently, it is (2) ______
remembering that every dictionary has its limitations.
[Nếu bạn là kiểu người có xu hướng mở từ điển thường xuyên, nó rất quan trọng để nhớ rằng
mỗi từ điển có hạn chế riêng của nó.]
Question 3: D
Variety: khác nhau
Each definition is only an approximation and one builds up an accurate picture of the meaning of a word
only after meeting it in a (48) ______ of contexts.
[Mỗi định nghĩa là 1 sự gần đúng và một nghĩa để xây dựng hình ảnh chính xác nghĩa của một từ sau khi
gặp trong nhiều trường hợp khác nhau.]
Question 4: A
Even if: thậm chí
In most exams you are not permitted to use a dictionary. (4) ______ you are allowed to use one, it is very
time-consuming to look up words, and time in exams is usually limited.
[Trong hầu hết các kỳ thi bạn không được phép sử dụng từ điển. Ngay cả khi bạn được phép sử dụng, nó
rất tốn thời gian để tìm kiếm từ, và thời gian trong các kỳ thi thường là hạn chế.]
Question 5: C
Derive from: bắt nguồn từ
Two strategies which may help you guess the meaning of a word are: using contextual clues,
both within the sentence and outside, and making use of clues (5) ______ from the formation of the
word.
[Hai chiến lược có thể giúp bạn đoán ý nghĩa của một từ là: sử dụng theo ngữ cảnh các manh
mối, cả trong và bên ngoài câu, và làm cho việc sử dụng đầu mối bắt nguồn từ sự hình thành của từ.]

VII. Read the following passage and mark the letter A, B, C, or D on your answer sheet to indicate
the correct word or phrase that best fits each of the numbered blanks from 1 to 5.

Hocmai– Ngôi trường chung của học trò Việt Tổng đài tư vấn: 1900 69 33 - Trang | 9-

302
Hocmai.vn – Website học trực tuyến số 1 tại Việt Nam
Khóa học Luyện thi THPT quốc gia PEN C –N3 Môn Tiếng Anh côHương Fiona

We know what happens when we don't have water - we die. We use water, other animals use it, plants
use it... but the world never loses it. Basically, the (1) _______ of water on, above and within the planet
never changes.
Our use of it is part of an immense (2) _______, known simply enough as the water cycle. The cycle
can happen because water is one of the few substances that exists naturally, and at temperatures which
people can tolerate, as a liquid, a gas and a solid.
We all know something about the water cycle; it is part of our daily scene. Water (3) _______ from
lakes and oceans and from the leaves of plants. From there it forms clouds which come back to earth
(4) _______ rain, ice or snow. That water soaks into the ground or runs off. And the cycle continues
with rivers running into the sea, plants transpiring and so on. Animals like us use it on the way through
the cycle.
For millions of years, changes in the water cycle were slow, but now, with the (35) _______ increase
in the number of people on earth and the development of technology, we are having a dramatic impact
on the way the water cycle behaves and on the quality of the water itself.
Question 1. A. amount B. number C. sum D. figure
Question 2. A. method B. process C. procedure D. route
Question 3. A. steams B. evaporates C. dissolves D.disperses
Question 4. A. like B. similar C. for D. as
Question 5. A. quick B. abrupt C. rapid D. Hasty
HƯỚNG DẪN
Question 1. Đáp án đúng: A.
Amount đi với danh từ không dếm được là "water"
Question 2. Đáp án đúng: B.
process (n): quá trình (trong vòng tuần hoàn của nước) có thể nhầm lẫn với
C. procedure: tiến trình, thủ tục.
Question 3. Đáp án đúng: B. evaporate: bốc hơi, bay hơi
Các đáp án khác:
A. steam: hơi nước
C. dissolve: hòa tan, tan rã
D. disperse: gieo rắc, phân tán
Question 4. Đáp án đúng: D. As + N: với vai trò là
Question 5. Đáp án đúng: C. rapid (a): rất nhanh chóng
Các đáp án khác:
B. abrupt (a): đột ngột
D. hasty (a): vội vã, hấp tấp
VIII. Read the following passage and mark the letter A, B, C, or D on your answer sheet to indicate
the correct word or phrase that best fits each of the numbered blanks from 1 to 5.
Easter is a holiday in late March or early April, the first Sunday after the first full moon after 21 March.
Hocmai– Ngôi trường chung của học trò Việt Tổng đài tư vấn: 1900 69 33 - Trang | 10-

303
Hocmai.vn – Website học trực tuyến số 1 tại Việt Nam
Khóa học Luyện thi THPT quốc gia PEN C –N3 Môn Tiếng Anh côHương Fiona

Many people (1)_________it with their family or have a short holiday/ vacation. It is also an important
Christian festival. Easter Sunday, the day of the Resurrection, is the end of Lent (2) ________ the
most important date in the Christian year. Many people who do not go to church at other times go on
Easter Sunday. It was once (3)_________for people to wear new clothes to church on this day. Women
(4)_________new hats, called Easter bonnets. Today, people sometimes make elaborately decorated
Easter bonnets for fun. A few people send Easter cards with religious symbols on them or pictures of
small chickens, lambs, rabbits and spring flowers, all traditionally associated (5)_________Easter.
Question 1. A. take B. use C. spend D. expend
Question 2. A. for B. as C. and D. nor
Question 3. A. familiar B. ordinary C. common D. regular
Question 4. A. put on B. take on C. bore D. wore
Question 5. A. toward B. with C. to D. from
HƯỚNG DẪN
Question 1. Đáp án C
- use /ju:z/ (v): dùng, sử dụng
- spend /spend/ (v): dành, tiêu tốn (thời gian,...)
spend....with sb
- expend /ik'spend/ (v): tiêu dùng, dùng hết
E.g: She expended all her efforts on the care of home and children.
"Easter is a holiday in late March or early April, the first Sunday after the first full moon after 21 March.
Many people spend it with their family or have a short holiday/ vacation." (Lễ phục sinh là một ngày lễ
vào cuối tháng 3 hoặc đầu tháng tư, ngày chủ nhật đầu tiên sau lần trăng tròn đầu tiên vào sau ngày 21
tháng 3. Nhiều người dành thời gian đó cùng với gia đinh hoặc đi nghỉ.)
Question 2. Đáp án C
Easter Sunday, the day of the Resurrection, is the end of Lent and the most important date in the
Christian year. (Ngày Chúa Nhật Phục Sinh, ngày của sự hồi sinh, là kết thúc của Mùa Chay và là ngày
quan trọng nhất trong năm của ki tô giáo.)
Question 3. Đáp án C
- familiar /fəˈmɪliə(r)/ (adj): quen thuộc, thân thuộc
- ordinary /ˈɔːdnri/ (adj): not unusual or different in any way: bình thường
E.g: ordinary people like you and me
- common /ˈkɒmən/ (adj): chung, phổ biến
- regular /ˈreɡjələ(r)/ (adj): thường xuyên
"It was once common for people to wear new clothes to church on this day." (Mặc quần áo mới tới nhà
thờ vào ngày đó đã từng rất phổ biến với mọi người.)
Question 4. Đáp án D
- put on: mặc, đội
- take on: đảm nhận, gánh vác

Hocmai– Ngôi trường chung của học trò Việt Tổng đài tư vấn: 1900 69 33 - Trang | 11-

304
Hocmai.vn – Website học trực tuyến số 1 tại Việt Nam
Khóa học Luyện thi THPT quốc gia PEN C –N3 Môn Tiếng Anh côHương Fiona

- bore (v): làm buồn


- wear => wore (v): mặc, đội, đeo
Câu này chúng ta cần chia động từ ở Quá khứ, và D. wore là thích hợp nhất
"Hai động từ to put on và to wear đều có nghĩa là "mặc", nhưng "to put on" chỉ một hành động; còn "to
wear" chỉ một tình trạng.
E.g: I put on my clothes before going out.
The girl who wears a purple robe, is my sister.
Question 5. Đáp án B
- toward: hướng về, về phía
- associate with: liên hợp, kết hợp
"...pictures of small chickens, lambs, rabbits and spring flowers, all traditionally associated with Easter"
(...những bức tranh về những con gà con, cừu non, thỏ và hoa mùa xuân, tất cả đều kết hợp về mặt truyền
thống với lễ Phục Sinh)

IX. Read the following passage and mark the letter A, B, C, or D on your answer sheet to indicate
the correct word or phrase that best fits each of the numbered blanks from 1 to 5.
Early writing and Alphabets
When people first began to write, they did not use an alphabet. Instead, they drew small pictures to (1)
_______ the objects they were writing about. This was very slow because there was a different picture
for any word.
The Ancient Egyptians had a system of picture writing that was described hieroglyphics. The meaning
of this writing was forgotten for a very long time but in 1799 some scientists (2) _______ a stone near
Alexandria, in Egypt. The stone had been there for (3) _______ a thousand years. It had both Greek
and hieroglyphics on it and researchers were finally able to understand what the hieroglyphics meant.
An alphabet is quite different from picture writing. It (4) _______ of letters or symbols that represent
a sound and each sound is just part of one word. The Phoenicians, who lived about 3,000 years ago,
developed the modern alphabets. It was later improved by the Roman’s and this alphabet is now used
(5) _______ throughout the world.
Question 1:A. notice B. show C. appear D. mark
Question 2:A. discovered B. realized C. delivered D. invented
Question 3:A. quite B. more C. over D. already
Question 4:A. consists of B. includes C. contains D. involves
Question 5:A. broadly B. widely C. deeply D. hugely
HƯỚNG DẪN
Question 1: Đáp án B
Notice (v): chú ý
Show (v): cho biết, chỉ ra
Appear (v): có vẻ, giống như

Hocmai– Ngôi trường chung của học trò Việt Tổng đài tư vấn: 1900 69 33 - Trang | 12-

305
Hocmai.vn – Website học trực tuyến số 1 tại Việt Nam
Khóa học Luyện thi THPT quốc gia PEN C –N3 Môn Tiếng Anh côHương Fiona

Mark (v): đánh dấu


Dịch: “… Thay vào đó, họ vẽ những bức vẽ nhỏ để chỉ ra những vật thể mà họ đang viết…”
Question 2: Đáp án A
Discover (v): khám phá, phát hiện
Realize (v): nhận ra
Deliver (v): phân phát
Invent (v): phát minh, sáng chế
Dịch: “… Ý nghĩa của bài viết này đã bị lãng quên suốt một thời gian dài nhưng vào năm 1799 một số
nhà khoa học đã phát hiện ra một tảng đá gần Alexandria, ở Ai Cập…”
Question 3: Đáp án C
Over + a period of time: hơn …
Dịch: “… Tảng đá đã ở đó khoảng hơn 6 nghìn năm…”
Question 4: Đáp án A
Consist of: bao gồm
Dịch: “… Nó bao gồm nhiều ký tự hoặc biểu tượng mà thể hiện một âm thanh và mỗi âm thanh chỉ là
một phần của một từ…”
Question 5: Đáp án B
Broadly (adv): chung, đại khái, đại thể
Widely (adv): một cách rộng rãi
Deeply (adv): một cách sâu sắc
Hugely (adv): rất đỗi, hết sức
Dịch: “… bảng chữ cái này hiện đang được sử dụng một cách rộng rãi khắp thế giới.”

Nguồn : Hocmai.vn
Giáo viên : Hương Fiona

Hocmai– Ngôi trường chung của học trò Việt Tổng đài tư vấn: 1900 69 33 - Trang | 13-

306
Hocmai.vn – Website học trực tuyến số 1 tại Việt Nam
Khóa học Luyện thi THPT quốc gia PEN C –N3 Môn Tiếng Anh côHương Fiona

PHƯƠNG PHÁP LÀM BÀI ĐỌC HIỂU


( ĐÁP ÁN BÀI TẬP TỰ LUYỆN )
Giáo viên : Nguyễn Thanh Hương
Các bài tập trong tài liệu này được biên soạn kèm theo bài giảng “Phương pháp làm bài đọc hiểu” thuộc Khóa học Luyện thi THPT quốc
gia PEN - C: Môn Tiếng Anh (cô Hương Fiona )” tại website Hocmai.vn để giúp các Bạn kiểm tra, củng cố lại các kiến thức được giáo
viên truyền đạt trong bài giảng tương ứng. Để sử dụng hiệu quả, Bạn cần học trước bài giảng, sau đó làm đầy đủ các bài tập trong tài
liệu này.

Reading 1.
Question 1: Đáp án A
Theo đoạn văn, một trong những vấn đề của học sinh vào học trung học là ...........
A. chúng được dạy bởi nhiều giáo viên khác nhau.
B. chúng không được học trong mọi môn học.
C. các giáo viên không muốn thân thiện.
D. các giáo viên chú ý nhiều nhất đến học sinh nhiều hơn.
Dẫn chứng: Teachers with specialist roles may see hundreds of children in a week, and a pupil may be able
to form relationships with very few of the staff.
Question 2: Đáp án B
Các giáo viên ở trường trung học không nói nhiều với học sinh vì ............
A. họ muốn giữ một khoảng cách nhất định với các học sinh.
B. họ đang quá bận rộn với công việc học thuật của họ.
C. các em học sinh sợ họ.
D. đó là quy định của nhà trường.
Dẫn chứng: Secondary staff and pupils suffer from the pressures of academic work and seem to have less
time to stop and talk.
Question 3: Đáp án A
Ở các trường trung học, học sinh có vấn đề nên ................
A. biết làm thế nào để yêu cầu giúp đỡ.
B. được giải thoát khỏi bất kỳ áp lực của công việc học tập.
C. Có thể thảo luận các vấn đề của mình trong lớp.
D. có thể thảo luận các vấn đề của mình với giáo viên nào.
Dẫn chứng: the reality of life in the institution actually encourages requests for help is another matter.
Question 4: Đáp án A
Trường học mong đợi học sinh làm gì?
A. trước tiên họ nhận được sự giúp đỡ. B. chúng thinh cách cẩn thận.
C. chúng hiểu rõ. D. chúng ta có thể làm được.
Dẫn chứng: The school will rightly expect the pupil to take the first steps to obtain the help he needs
Question 5: Đáp án C
Từ "người lớn" ở đoạn thứ hai có ý nghĩa gần nhất với ...................

Hocmai– Ngôi trường chung của học trò Việt Tổng đài tư vấn: 1900 69 33 - Trang | 1-

307
Hocmai.vn – Website học trực tuyến số 1 tại Việt Nam
Khóa học Luyện thi THPT quốc gia PEN C –N3 Môn Tiếng Anh côHương Fiona

A. Các học sinh trung học. C. các nhân viên ở trường trung học.
B. các bậc cha mẹ học sinh trung học. D. các giáo viên và học sinh cấp hai.
Dẫn chứng: He has to decide which adults are approachable→ quyết định tạo mối quan hẹ với giáo viên
nào.
Question 6: Đáp án C
Bao nhiêu học sinh mỗi giáo viên nhìn thấy trong một tuần?
A. Một học sinh B. Một vài học sinh C. Hàng trăm học sinh D. Nhiều học sinh
Dẫn chứng: Teachers with specialist roles may see hundreds of children in a week
Question 7: Đáp án A
Những học sinh nào làm quen với?
A. Một số nhân viên B. Cha mẹ khác C. Các học sinh khác D. Một vài nhân viên

Dẫn chứng: and a pupil may be able to form relationships with very few of the staff.
Bao nhiêu học sinh mỗi giáo viên nhìn thấy trong một tuần?
A. Một học sinh B. Một vài học sinh C. Hàng trăm học sinh D. Nhiều học sinh
Dẫn chứng: Teachers with specialist roles may see hundreds of children in a week
Question 8: Đáp án D
Trong đoạn văn này về các trường trung học, tác giả chủ yếu quan tâm đến ................
A. Các tiêu chuẩn giáo dục C. Đào tạo của các giáo viên cá nhân.
B. vai trò của giáo viên chuyên môn. D. sự phát triển cá nhân của học sinh.
Dẫn chứng: On entering secondary school, a new world opens up and frequently it is much more difficult
world.

Reading 2.

Question 1: Đáp án B
Theo đoạn văn, tia cực tím từ mặt trời ..............
A. gây ra thiệt hại nghiêm trọng đối với tầng ôzôn của trái đất
B. chỉ là một phần nhỏ của bức xạ điện từ của mặt trời
C. tạo ra bức xạ điện từ
D. luôn luôn đến Trái đất
Dẫn chứng: Although only a small percentage of the electromagnetic radiation that is emitted by the Sun is
ultraviolet (UV) radiation
Question 2: Đáp án D
Từ "bao vây" trong đoạn 1 có ý nghĩa gần nhất với ................
A. attacking: tấn công B. raising: tăng lên
C. rotating: xoay quanh D. surrounding: xung quanh
Question 3: Đáp án C
Trong đoạn văn nói rằng tầng ôzôn ...............

Hocmai– Ngôi trường chung của học trò Việt Tổng đài tư vấn: 1900 69 33 - Trang | 2-

308
Hocmai.vn – Website học trực tuyến số 1 tại Việt Nam
Khóa học Luyện thi THPT quốc gia PEN C –N3 Môn Tiếng Anh côHương Fiona

A. Cho phép bức xạ cực tím đến Trái đất B. phản xạ tia cực tím
C. che chắn cho trái đất khỏi rất nhiều tia cực tím D. bức xạ lên xuống Trái Đất
Dẫn chứng: Fortunately, all of the Sun’s ultraviolet radiation does not reach the Earth because of a layer of
oxygen, called the ozone layer encircling the Earth in the stratosphere at an altitude of about 15 miles
above the Earth.
Question 4: Đáp án A
Đại từ "nó" trong đoạn 2 nói đến ...................
A. Bức xạ B. Tiến trình C. Sự hình thành D. tổn hại
Dẫn chứng: When UV radiation is absorbed during the process of ozone formation and reformation, it is
unable to reach Earth and cause damage there.
Question 5: Đáp án A
Từ "culprits" trong đoạn 3 có ý nghĩa gần nhất với câu nào dưới đây?
A. Các bên phạm tội B. Các nhà thám tử
C. Các thành viên của nhóm D. Các nhà lãnh đạo
Culprit: tội phạm
Question 6: Đáp án B
Theo đoạn văn, những gì xảy ra sau khi một phân tử clo phản ứng với một phân tử ozon?
A. ozon phân hủy thành ba nguyên tử oxy.
B. Hai phân tử khác nhau được tạo ra.
C. Hai phân tử này kết hợp thành một phân tử.
D. Ba kết quả phân tử khác biệt.
Dẫn chứng: The released chlorine reacts with ozone in the stratosphere to form chlorine monoxide (CIO)
and oxygen (O2).
Question 7: Đáp án A
Đoạn văn sau đoạn văn có nhiều khả năng thảo luận ..................
A. các kết quả âm của chu trình phá hủy ozone
B. nơi chlorofluorocarbons (CFCs) đến từ
C. các nguyên nhân phá hủy các phân tử ozon
D. làm thế nào bức xạ điện từ được tạo ra
Dẫn chứng: the effects of this destructive process are now becoming evident.

Reading 3
Question 1: Đáp án D
Dựa theo bài viết, câu nào sau đây là đúng?A. Phụ nữ có thể lái xe ở Ả-rập Xê-útB. Người dân Ấn Độ lái
xe ở bên phải đường.C. Bạn có thể đi nhanh như thế nào cũng được nếu bạn muốn ở mọi nơi trên con
đường Autobahn của Đức.D. Bạn không được vừa cầm điện thoại vừa nói chuyện khi đang lái xe nếu bạn
đang ở Úc.
Câu cuối đoạn 4: However, in Argentina and Australia, drivers are allowed to talk on their mobile phones
as long as they do not use their hands. Câu này hiểu là: [...] ở Úc, lái xe có thể nói chuyện điện thoai, chỉ
cần họ không cầm nó ở trên tay.

Hocmai– Ngôi trường chung của học trò Việt Tổng đài tư vấn: 1900 69 33 - Trang | 3-

309
Hocmai.vn – Website học trực tuyến số 1 tại Việt Nam
Khóa học Luyện thi THPT quốc gia PEN C –N3 Môn Tiếng Anh côHương Fiona

Question 2: Đáp án A
Điều nào không được đề cập đến như là một sự khác biệt trong luật lệ giao thông?
A. Giới hạn sự lái xe khi có dùng đồ uống
B. Tuổi hợp pháp để lái xe
C. Lái xe ở bên trái hay bên phải đường
D. Những cuộc gọi an toàn
Question 3: Đáp án A
Theo bài viết, ăn uống khi đang lái xe là phạm pháp khi bạn đang ở?
A. Cộng hòa Síp B. Anh C. Trung Quốc D. Úc
Câu 2 đoạn cuối: For example, in the country of Cyprus, it is against the law to eat or drink anything while
driving
Question 4: Đáp án B
Việc tìm hiểu về pháp luật trước khi lái xe ở nước ngoài là vô cùng quan trọng bởi...
A. Thật thú vị khi biết thêm về luật giao thông.
B. Bạn có thể sẽ gặp rắc rồi nếu không hiểu rõ về quy định luật pháp của một nước nào đó.
C. Bạn sẽ có thể gặp tai nạn nếu không biết luật.
D. Nó sẽ giúp bạn xác định được phần đường đúng để bạn có thể đi vào.
Question 5: Đáp án D
Từ ''inquisitive" ở đoạn 2 có thể được thay thế bởi từ nào?
inquisitive = curious: tò mò
Question 6: Đáp án C
Từ "those" ở đoạn 4 đề cập đến cái gì?
Đáp án: driving laws
Question 7: Đáp án B
Ý chính của bài văn này là gì?
A. Việc lái xe ở tất cả các quốc gia đều có một mục đích chung.
B. Có những sự khác nhau trong luật giao thông giữa các quốc gia với nhau.
C. Các quốc gia có những cách khác nhau để ban hành luật pháp liên quan đến việc lái xe.
D. Người dân ở những nơi khác nhau thì lái xe ở những bên khác nhau của con đường.
Chúng ta có thể nhận thấy, B là đáp án phù hợp nhất cho câu hỏi này.

Reading 4
Question 1: Đáp án B
Từ “They” được in đậm trong đoạn 4 được thay thế cho từ nào? A. Học sinh Anh Quốc B. Học sinh châu
Á C. Học sinh Hàn Quốc D. Học sinh Nhật Bản
Asian students tend to put their education before literally everything else. They do very few extracurricular
activities and devote far more time to their studies than their British peers. => Học sinh châu Á thường đặt
việc học lên hàng đầu. Họ ít khi tham gia các hoạt động ngoại khóa và dành thời gian cho học tập nhiều
hơn các học sinh ở Anh

Hocmai– Ngôi trường chung của học trò Việt Tổng đài tư vấn: 1900 69 33 - Trang | 4-

310
Hocmai.vn – Website học trực tuyến số 1 tại Việt Nam
Khóa học Luyện thi THPT quốc gia PEN C –N3 Môn Tiếng Anh côHương Fiona

Question 2: Đáp án B
Câu này nói về học sinh cấp 2 ở Anh quốc.
Câu cuối đoạn 1 có nói: Meanwhile, British secondary school students fail to shine in conditions most
educational researchers would say are far more likely to help them succeed.
Nghĩa là, học sinh cấp 2 ở Anh thất bại trong những điều kiện mà các nhà nghiên cứu giáo dục cho rằng
đáng nhé phải giúp các em ấy thành công.
Question 3: Đáp án C
Có thể suy ra được điều gì từ quan điểm của tác giả về hai hệ thống giáo dục được nhắc tới?
A. Hệ thống của châu Á rõ rang là tốt hơn
B. Hệ thống của người Anh quá nghiêm khắc
C. Không có hệ thống nào là hoàn hảo
D. Cả hai hệ thống giáo dục đều thỏa mãn những mục đích khác nhau
Question 4: Đáp án A
Hệ thống gia đình truyền thống...
A. phổ biến ở Hàn Quốc hơn là ở Anh
B. đang mất dần ở Hàn Quốc bởi tỷ lệ ly hôn cao
C. không tốt cho những đứa trẻ xuất thân từ gia đình đổ vỡ
D. là không ổn định ở Hàn Quốc do điều kiện của mỗi gia đình
Đoạn 3 thể hiện rõ nhất điều này: hệ thống gia đình truyền thống vẫn còn nguyên vẹn ở Hàn, trong khi tỷ
lệ ly hôn ở Anh lại ở mức cao.
Question 5: Đáp án B
Tác giả có ý gì khi dùng từ "paradox" để đề cập đến mô hình giáo dục ở châu Á?
A. Có rất nhiều học sinh trong 1 lớp học.
B. Bạn sẽ nghĩ lớp học càng đông thì kết quả càng kém nhưng thật ra không phải vậy.
C. Sỹ số lớp học rất bé so với các nơi khác trên thế giới
D. Học sinh châu Á nổi trội hơn so với các bạn đồng trang lứa ở những nơi khác.
"paradox" có nghĩa là nghịch lý. Vậy đáp án ở đây là B
Question 6: Đáp án D
Người viết cho rằng điều gì đã khiến cho các bài học ở Hàn Quốc hiệu quả hơn ở Anh Quốc?
Đoạn 2: Sự quản lý lớp học ở Hàn Quốc khá dễ dàng. Nhưng giáo viên ở Anh Quốc thì bất lực với học
sinh của mình. Suy ra đáp án D (Kỷ luật tốt hơn)
Question 7: Đáp án A
Từ "unstable" ở đoạn 3 có thể thay thế bởi từ nào?
unstable = unsteady: không ổn định, không vững vàng
unchangeable : không thể thay đổi đượcunpredictable: không thể dự đoán đượcunimportant: không quan
trọng
Question 8: Đáp án C
Theo tác giả, học sinh châu Á thì...
A. tập trung quá nhiều vào các hoạt động vui chơi
B. không có đạo đức nghề nghiệp tốt bằng các bạn nước Anh

Hocmai– Ngôi trường chung của học trò Việt Tổng đài tư vấn: 1900 69 33 - Trang | 5-

311
Hocmai.vn – Website học trực tuyến số 1 tại Việt Nam
Khóa học Luyện thi THPT quốc gia PEN C –N3 Môn Tiếng Anh côHương Fiona

C. không cho phép bản thân mình dành nhiều thời gian vui chởi giải trí
D. tỏ vẻ tự hào lắm về kết quả tốt của mình.
Câu cuối của đoạn 4: They do very few extracurricular activities and devote far more time to their studies
than their British peers.Nghĩa là họ dành rất ít thời gian cho hoạt động ngoại khóa và tập trung vào việc
học nhiều hơn học sinh ở Anh

Reading 5.
Question 1: Đáp án D
Những thợ lặn tự do là những người_______
A. Có thể đi xuống đến độ sâu cực đại B. sử dụng các phương tiện lặn
C. không sử dụng thiết bị D. tiếp xúc với nước xung quanh
Dẫn chứng: Ambient divers are, unlike divers who go underwater in submersible vehicles of pressure
resistant suits, exposed to the pressure and temperature of the surrounding ambient water. Thợ lặn tự do,
không giống như các thợ lặn sử dụng phương tiện có thể lặn có khả năng chịu áp lực phù hợp, họ tiếp xúc
với áp lực và nhiệt độ xung quanh môi trường nước
Question 2: Đáp án A
Theo đoạn văn này, một thợ lặn tự do có thể sử dụng bất kỳ một trong các thứ sau đây TRỪ _______
A. Thiết bị lặn B. một ống thở C. chân vây D. một mặt nạ
Dẫn chứng: Free divers may use no equipment at all, but most use a face mask, foot fins, and a snorkel. ->
không có đáp án A.
Question 3: Đáp án B
Theo đoạn văn, độ sâu tối đa của thợ lặn tự do khoảng _______
A. 40 feet B. 100 feet C. 200 feet D. 1,000 feet
Dẫn chứng: Most free divers can only descend 30 to 40 feet, but some skilled divers can go as deep as 100
feet.
Question 4: Đáp án C
Khi sử dụng các thiết bị mạch kín, các chất lỏng _______
A. thở ra không khí vào trong nước B. giữ hơi thở của chúng
C. hít thở lại cùng một luồng khí D. nhận không khí từ bề mặt
Dẫn chứng: A closed-circuit breathing device, also called a rebreather, filters out carbon dioxide and other
harmful gases and automatically adds oxygen. This enables the diver to breathe the same air over and over.
Question 5: Đáp án A
Từ "hạ xuống" trong đoạn văn có nghĩa là gì?
A. di chuyển đến một mức thấp hơn B.leo đến một nơi cao hơn
C. bơi dưới đáy sông D. lên và xuống từ từ
Question 6: Đáp án D
Từ "dụng cụ" trong đoạn văn là tủ quần áo có nghĩa là _______
A. Quần áo B. Dưỡng khí C. Mặt nạ D. Thiết bị
Apparatus: dụng cụ= equipment
Question 7: Đáp án C

Hocmai– Ngôi trường chung của học trò Việt Tổng đài tư vấn: 1900 69 33 - Trang | 6-

312
Hocmai.vn – Website học trực tuyến số 1 tại Việt Nam
Khóa học Luyện thi THPT quốc gia PEN C –N3 Môn Tiếng Anh côHương Fiona

Mũ bảo hiểm mà các thợ lặn dùng bình dưỡng khí sử dụng ngày nay được làm từ vật liệu gì, theo đoạn
văn?
A.glass: thủy tinh B.copper: đồng
C.plastic: chất dẻo D.canvas: vải bạt
Dẫn chứng: Today, sophiticated plastic helmets have replaced the heavy copper helmets used in the past.

Reading 6.
Question 1: Đáp án A
Câu nào dưới đây giải thích cho tiêu đề đoạn 1?
A. Sóng thần xảy ra cùng với trận động đất.
B. Loại thiên tai nghiêm trọng nhất là động đất.
C. Động đất gây ra nhiều tàn phá hơn sóng thần.
D. Động đất thường xảy ra sau khi sóng thần xảy ra.
Dẫn chứng: Two that are frequently linked to one another are earthquakes and tsunamis... However,
tsunamis are the direct result of earthquakes and cannot happen without them.
Question 2: Đáp án B
Từ "nó" đậm trong đoạn 2 đề cập đến _________.
Lớp lõi B. lớp vỏ trái đất C. Trái Đất D. Lớp trung gian
Dẫn chứng: The crust is the outer layer of the Earth. It is not a single piece of land.-> the crust.
Question 3: Đáp án C
Phần này chủ yếu là về vấn đề gì?
A. Khi nào động đất hay xảy ra nhất.
B. Những loại phá hủy nào mà thiên tai có thể gây ra.
C. Động đất và sóng thần xảy ra như thế nào.
D. Tại sao sóng thần lại nguy hiểm hơn trận động đất.
Dẫn chứng: Over a long time, this tension may build up. When it is released, an earthquake happens.
Question 4: Đáp án C
Từ "gần kề" in đậm ở đoạn 2 có ý nghĩa gần nhất với ________.
A. residing: cư trú B. approaching: đến gần
C. bordering: tiếp giáp D. appearing: xuất hiện
Adjoining= bordering
Question 5: Đáp án A
Từ "cảm nhận" in đậm ở đoạn 3 có ý nghĩa gần nhất với ________.
A. nhận thấy B. ngăn ngừa C. hiểu rõ D. xác định vị trí
Perceive= detect
Question 6: Đáp án C
Câu nào dưới đây đúng với lớp vỏ?
A. Đó là lớp nhỏ nhất trong ba lớp
B. Nó dày trên đất hơn là dưới mặt nước.
C. Có nhiều miếng riêng biệt tạo nên nó
D. Lớp trung gian bên dưới giữ nó không di chuyển quá nhiều.

Hocmai– Ngôi trường chung của học trò Việt Tổng đài tư vấn: 1900 69 33 - Trang | 7-

313
Hocmai.vn – Website học trực tuyến số 1 tại Việt Nam
Khóa học Luyện thi THPT quốc gia PEN C –N3 Môn Tiếng Anh côHương Fiona

Dẫn chứng: Instead, it is comprised of a number of plates. There are a few enormous plates and many
smaller ones.
Question 7: Đáp án D
Dựa vào đoạn văn, điều gì có lẽ đúng với sóng thần?
A. Nó giết nhiều người hơn so với động đất mỗi năm.
B. Chúng có thể di chuyển nhanh như tốc độ của âm thanh.
C. Không thể làm hỏng tàu thuyền trên biển.
D. Chúng có thể nguy hiểm tính mạng cho những người đứng gần bờ
Dẫn chứng: As it does that, it can flood the land, destroy human settlements, and kill large numbers of
people.
Question 8: Đáp án C
Điều nào dưới đây không được đề cập đến trong đoạn 3 về động đất?
A. Mức độ nguy hiểm của phần lớn trong số chúng
B. Những loại thiệt hại mà chúng có thể gây ra
C. Tần suất xảy ra của những trận động đất lớn
D. Con người thường bị tiêu diệt bao nhiêu
Dẫn chứng: Since most of the Earth’s surface is water, numerous earthquakes happen beneath the planet’s
oceans.-> đề cập đến tần suất phần lớn những trận động, không phải tần suất xảy ra của những trận động
đất lớn.

Reading 7.
Câu 1. Câu hỏi tổng quát
Đáp áp đúng: C. Halley's invention of a contraption for diving
Câu 2. Câu hỏi thông tin thực
Đáp áp đúng: B. provided more air
Thông tin ở câu: "Halley's bell was an improvement in that its design allowed for an additional supply
of fresh air that enabled a crew of divers to remain underwater for several hours."
Câu 3. Câu từ vựng theo ngữ cảnh
Đáp áp đúng: D. was almost exhausted= 'ran low’: cạn kiệt, hết
Câu 4. Câu hỏi thông tin thực
Đáp áp đúng: C. For hours at a time
Thông tin ở câu: "Halley's bell was an improvement in that its design allowed for an additional supply
of fresh air that enabled a crew of divers to remain underwater for several hours."
Câu 5. Câu hỏi loại trừ: 3 câu đúng 1 câu sai và đáp án là câu có thông tin sai.
Đáp áp đúng: A. was wider at the top than at the bottom
Thông tin ở câu: "The diving contraption that Halley designed was in the shape of a bell that measured
three feet across the top and five feet across the bottom and could hold several divers comfortably; it was
open at the bottom so that divers could swim in and out at will."
Câu 6. Câu từ hỏi vựng theo ngữ cảnh
Đáp áp đúng: B. as they wanted = “at will” tự do, tùy ý

Hocmai– Ngôi trường chung của học trò Việt Tổng đài tư vấn: 1900 69 33 - Trang | 8-

314
Hocmai.vn – Website học trực tuyến số 1 tại Việt Nam
Khóa học Luyện thi THPT quốc gia PEN C –N3 Môn Tiếng Anh côHương Fiona

Câu 7. Câu hỏi suy luận:


Đáp áp đúng: A. float
Thông tin ở câu: "In addition to the air already in the bell, air was also supplied to the divers from a
lead barrel that was lowered to the ocean floor close to the bell itself."

Reading 8.
Câu 1. Câu hỏi tổng quát
Đáp áp đúng :D. Why Timber rattlesnakes are endangered
Câu 2. Câu từ hỏi vựng theo ngữ cảnh
Đáp áp đúng: A. Unprotected= "vulnerable” dễ bị tổn thương
Câu 3. Câu hỏi thông tin thực
Đáp áp đúng: C. They migrated to escape a cold climate
Thông tin nằm ở câu: "Rattlesnakes are generally found in warm climates because, like all reptiles,
they cannot generate or regulate their own body temperature internally and must rely on the sun’s warmth
for heat"
Câu 4. Câu từ hỏi vựng theo ngữ cảnh
Đáp áp đúng:C. Get up="emerge": thứ dậy, tỉnh dậy (sau khi ngủ đông những con rắn thức dậy)
Câu 5. Câu hỏi loại trừ
Đáp áp đúng: D. In canyons
Câu 6. Câu từ hỏi vựng theo ngữ cảnh
Đáp áp đúng: C. Victims = "easy prey”: con mồi
Câu 7. Câu từ hỏi vựng theo ngữ cảnh
Đáp áp đúng: D. Lying = "basking"
Câu 8. Câu hỏi loại trừ
Đáp áp đúng: A. Hibernating for eight months

Reading 9.
Câu 1. Câu hỏi tổng quát
Đáp áp đúng: B. life as a househusband
Câu 2. Câu hỏi thông tin thực
Đáp áp đúng: A. have to hurry
Thông tin ở câu: “Everyone was always in a hurry ”
Câu 3. Câu hỏi từ vựng theo ngữ cảnh
Đáp áp đúng: B. move about hurriedly = rush
Câu 4. Câu hỏi thông tin thực
Đáp áp đúng: A. to do things more slowly.
Thông tin ở câu: "So you have to learn to slow down”
Câu 5. According to the passage, children _______.
Đáp áp đúng: C
Câu 6. Câu hỏi quy chiếu
Đáp áp đúng: B. men who talk about feelings
Câu 7. Câu hỏi suy luận
Hocmai– Ngôi trường chung của học trò Việt Tổng đài tư vấn: 1900 69 33 - Trang | 9-

315
Hocmai.vn – Website học trực tuyến số 1 tại Việt Nam
Khóa học Luyện thi THPT quốc gia PEN C –N3 Môn Tiếng Anh côHương Fiona

Đáp áp đúng:A. learning about housework is easy.

Reading 10.
Câu 1. Câu hỏi suy luận
Đáp áp đúng: D. it was invented by accident.
Thông tin ở câu: "According to some accounts, the first optical telescope was accidentally invented in
the 1600s by children who put two glass lenses together while playing with them in a Dutch optical shop."
Câu 2. Câu hỏi loại trừ
Đáp áp đúng: C. He sold his invention in 1608i
Thông tin ở câu: “In 1608 he tried to sell his invention unsuccessfully.”
Câu 3. Câu hỏi thông tin thực
Đáp áp đúng: A. in 1611
Thông tin ở câu: "In April of 1611, Galileo showed his device to guests at a banquet in his honor. One
of the guests suggested a name for the device: telescope.”
Câu 4. Câu hỏi thông tin thực
Đáp áp đúng: B. It refracted the light
Thông tin ở câu: "The type of telescope that Galileo designed is called a refractor because the front
lens bends, or refracts the light."
Câu 5. Câu hỏi thông tin thực
Đáp áp đúng: C. He improved it.
Thông tin ở câu: "He solved this problem by designing a new type of telescope that used a curved
mirror"
Câu 6. Câu hỏi thông tin thực
Đáp áp đúng: B. in the 18th century
Câu 7. Câu hỏi thông tin thực
Đáp áp đúng: D. in Caucasus Mountains
Thông tin ở câu: “This huge reflecting telescope was for many years the largest reflecting telescope in
the world until an even larger reflecting telescope was built in the Caucasus Mountains."
Câu 8. Câu hỏi thông tin thực
Đáp áp đúng: B. Allowing a collection of data from outer space.
Thông tin ở câu: “Radio telescopes, like optical telescopes, allow astronomers to collect data from
outer space"hut they are different in important ways."

Nguồn : Hocmai.vn
Giáo viên : Hương Fiona

Hocmai– Ngôi trường chung của học trò Việt Tổng đài tư vấn: 1900 69 33 - Trang | 10-

316
Hocmai.vn – Website học trực tuyến số 1 tại Việt Nam
Khóa học Luyện thi THPT quốc gia PEN C –N3 Môn Tiếng Anh cô Hương Fiona

VOCABULARY – CONSERVATION
( BÀI TẬP TỰ LUYỆN)
Giáo viên : Nguyễn Thanh Hương
Các bài tập trong tài liệu này được biên soạn kèm theo bài giảng “Conservation” thuộc Khóa học Luyện thi THPT quốc gia PEN -
C: Môn Tiếng Anh (cô Hương Fiona )” tại website Hocmai.vn để giúp các Bạn kiểm tra, củng cố lại các kiến thức được giáo viên
truyền đạt trong bài giảng tương ứng. Để sử dụng hiệu quả, Bạn cần học trước bài giảng, sau đó làm đầy đủ các bài tập trong tài liệu
này.

EXERCISE 1

Read the following passage and mark the letter A, B, C, or D on your answer sheet to indicate the
correct answer to each of the questions from 1 to 8.
The Timber rattlesnakes, once widespread throughout the eastern United States, is now on the
endangered species list and is extinct in two eastern states in which it once thrived. Compared to its
Western cousins the Timber rattlesnakes may be especially vulnerable because of certain behaviors
adapted for coping with the cold climate in which it lives.
Rattlesnakes are generally found in warm climates because, like all reptiles, they cannot generate or
regulate their own body temperature internally and must rely on the sun’s warmth for heat. But Timber
rattlesnakes migrated into colder northern areas about 8,000 years ago when glaciers retreated. In these
northern regions, the snakes developed a number of adaptive strategies to survive, but ultimately these
behaviors make them more vulnerable to human predation, their main threat.
One survival strategy the snakes have developed is hibernation. For approximately eight months of the
year, the rattlers remain motionless in deep frost-free crevices, with their body temperature dropping as
low as 40 degrees. In the spring, when they emerge, they must warm their chilled bodies by sunning for
three or four days on rocks in the open. This behavior coupled with the fact that Timber rattlesnakes tend
to concentrate in large numbers at their wintering sites, make them easy prey. Gestating females are
particularly vulnerable because they spend much of their time basking in the sun in order to produce live
young from eggs. In addition, females have very long interbirth intervals, producing live young only every
three to five years. If a frost or cold spell comes late in the year, the entire litter of 6 to 12 young may die.
Efforts are underway to protect the Timber rattlesnakes and its habitats from further human
depredation, but in many states it is already too late.
Question 1. What is the main topic of the passage?
A. Why Timber rattlesnakes hibernate
B. How Timber rattlesnakes are surviving
C. How Timber rattlesnakes adapted to northern climates
D. Why Timber rattlesnakes are endangered
Question 2. Which of the following is closest in meaning to the word “vulnerable” in the first paragraph?
A. Unprotected B. Impervious C. Insensitive D. Deprived
Question 3. Which of the following in the true about Timber rattlesnakes?

Hocmai – Ngôi trường chung của học trò Việt Tổng đài tư vấn: 1900 69 33 - Trang | 1 -

317
Hocmai.vn – Website học trực tuyến số 1 tại Việt Nam
Khóa học Luyện thi THPT quốc gia PEN C –N3 Môn Tiếng Anh cô Hương Fiona

A. They migrated to eastern states


B. They migrated northward during a warming climate
C. They migrated to escape a cold climate
D. They migrated to the South to seek a warmer climate
Question 4. Which of the following could best replace the word "emerge" in the third paragraph?
A. Come out B. Set off C. Get up D. See through
Question 5. In which of the following places might a person be most likely to find Timber rattlesnakes in
the spring?
A. in the woods B. In meadows C. In bushy areas D. In canyons
Question 6. The phrase “easy prey” in the third paragraph could best be replaced by which of the
following?
A. Relaxed B. Protective C. Victims D. Sociable
Question 7. Which of the following word can best replace the word "basking" in the third paragraph?
A. Washing B. Eating C. Sleeping D. Lying
Question 8. Which of the following does NOT contribute to the rattlesnake being an endangered animal?
A. Hibernating for eight months B. Basking in the sun
C. Congregating together D. Having long intervals between births

Exercise 2.
Read the following passage and mark the letter A, B, C or D on your answer sheet to indicate the
correct answer to each of the questions.
Endangered species
Aesthetic justification contends that biodiversity contributes to the quality of life because many of the
endangered plants and animals are particularly appreciated for their unique physical beauty. The aesthetic
role of nature in all its diverse forms is reflected in the art and literature of every culture attaining symbolic
status in the spiritual life of many groups. According to the proponents of the aesthetic argument, people
need nature in all its diverse and beautiful forms as part of the experience of the world.
Another argument that has been put forward, especially by groups in the medical and pharmacological
fields, is that of ecological self-interest. By preserving all species, we retain a balance of nature that is
ultimately beneficial to humankind. Recent research on global ecosystems has been cited as evidence that
every species contributes important or even essential functions that may be necessary to the survival of our
own species. Some advocates of the ecological argument contend that important chemical compounds
derived from rare plants may contain the key to a cure for one of the diseases currently threatening human
beings. If we do not protect other species, then they cannot protect us.
Apart from human advantage in both the aesthetic and ecological arguments, the proponents of a moral
justification contend that all species have the right to exist, a viewpoint stated in the United Nations World

Hocmai – Ngôi trường chung của học trò Việt Tổng đài tư vấn: 1900 69 33 - Trang | 2 -

318
Hocmai.vn – Website học trực tuyến số 1 tại Việt Nam
Khóa học Luyện thi THPT quốc gia PEN C –N3 Môn Tiếng Anh cô Hương Fiona

Charter for Nature, created in 1982. Furthermore, if humankind views itself as the stewards of all the
creatures on Earth, then it is incumbent upon human beings to protect them, and to ensure the continued
existence of all species. Moral justification has been extended by a movement called "deep ecology," the
members of which rank the biosphere higher than people because the continuation of life depends on this
larger perspective. To carry their argument to its logical conclusion, all choices must be made for the
biosphere, not for people.
Question 1: Which of the following is the main topic of the passage?
A. The beauty of the world B. The quality of life
C. The preservation of species D. The balance of nature
Question 2: Which of the arguments supports animal rights?
A. Aesthetic justification B. Ecological argument
C. Self-interest argument D. Moral justification
Question 3: The word "perspective" in the 3rd paragraph could best be replaced by
A. idea B. event C. truth D. view
Question 4: The word "them" in the 3rd paragraph refers to
A. humankind B. stewards C. creatures D. human beings
Question 5: Where in the passage does the author explain how rare species contribute to the health of the
human species?
A. in the underlined sentence B. in the second paragraph
C. in the first paragraph D. in the last paragraph
Question 6: According to the passage, what do we know from research on global ecosystems?
A. Nature is very diverse. B. A balance of nature is important
C. Humans have a responsibility to nature. D. Nature represents spiritual values
Question 7: The author mentions all of the following as justifications for the protection of endangered
species EXCEPT
A. the natural compounds needed for medicines. B. the intrinsic value of the beauty of nature,
C. the control of pollution in the biosphere. D. the right to life implied by their existence.
Question 8: It can be inferred from the passage that the author
A. is a member of the "deep ecology" movement
B. does not agree with ecological self-interest,
C. supports all of the arguments to protect species.
D. participated in drafting the Charter for Nature.

Hocmai – Ngôi trường chung của học trò Việt Tổng đài tư vấn: 1900 69 33 - Trang | 3 -

319
Hocmai.vn – Website học trực tuyến số 1 tại Việt Nam
Khóa học Luyện thi THPT quốc gia PEN C –N3 Môn Tiếng Anh cô Hương Fiona

Exercise 3.
Circle the word whose underlined part is pronounced differently from that of the others
1.A. bamboo B. good C. foot D. cook
2. A. cake B. panda C. face D. late
3. A. social B. science C. sour D. sure
Circle the word whose stress part is placed differently from that of the others
4. A. reserve B. schedule C. wildlife D. beauty
5. A. derive B. contain C. leopard D. prevent
Exercise 4.
Make the correct choice:
6. Many plants and endangered species are now endangering of .
A. expression B. expulsion C. extinction D. extension
7. is destroying larger areas of tropical rain forests.
A. Disforestation B. Deforestation C. Anti-forestation D. forests
8. A lot of different conservation efforts have been made to endangered species.
A. save B. kills C. make D. do
9. The raw sewage needs to be treated.
A. chemically B. chemical C. chemist C. chemistry
10. There are more than 20 working on the water treatment project.
A. researches B. researcher C. researchers D. research
11. More and more people of food poisoning nowadays.
A. exits B. survive C. die D. starve
12. The government the flood victims with food, clothers and money.
A. gave B. provided C. offered D. presented
13. Many people that natural resources will never be used up.
A. view B. consider C. believe D. regard
14. Conservation is the protection of the environment.
A. nature B. natural C. naturally D. naturalize
15. There are many of pollution in our modern world
A. resources B. sources C. foundations D. bases
Nguồn : Hocmai.vn

Giáo viên : Hương Fiona

Hocmai – Ngôi trường chung của học trò Việt Tổng đài tư vấn: 1900 69 33 - Trang | 4 -

320
Hocmai.vn – Website học trực tuyến số 1 tại Việt Nam
Khóa học Luyện thi THPT quốc gia PEN C –N3 Môn Tiếng Anh cô Hương Fiona

Hocmai – Ngôi trường chung của học trò Việt Tổng đài tư vấn: 1900 69 33 - Trang | 5 -

321
Hocmai.vn – Website học trực tuyến số 1 tại Việt Nam
Khóa học PEN C – N3 (Cô Nguyễn Thanh Hương)

FRIENDSHIP
Bài tập tự luyện
Giáo viên: Nguyễn Thanh Hương

I. PRONUNCIATION

1. A. machine B. change C. teacher D. choose


2. A. condition B. option C. suggestion D. relation
3. A. believe B. readily C. friend D. pleasure
4. A. good B. gossip C. game D. geometry
5. A. trust B. mutual C. number D. uncertain

II. Complete these sentences with the correct form of the word in the box.

constant capable selfish loyal suspicion


acquaint enthusiast sympathize hospitable friend

1. His ________ for music has stayed strong, throughout his 23 years in radio.
2. He showed unswerving ___________ to his friends.
3. We all have great __________ for the victims of the flood.
4. __________ is very important because uncertain people cannot have a lifelong friendship.
5. It was very ___________ of him to offer us his room.
6. We were delighted by the wonderful _________ of the local people.
7. The wine had made him ________ of thinking clearly.
8. Playing a game with the children is a good way of getting them _________.
9. Inform the police immediately if you see anything ___________.
10. ________ need time to develop.
III. Complete each of the sentences with an appropriate preposition.
1. A selfish person is incapable __________ true friendship.
2. He was accused _________ being disloyal _________ the government.
3. How much time do you spend ________ homework?
4. Good friendship should be based ______ mutual understanding.

Hocmai – Ngôi trường chung của học trò Việt Tổng đài tư vấn: 1900 6933 - Trang | 1 -
322
Hocmai.vn – Website học trực tuyến số 1 tại Việt Nam
Khóa học PEN C – N3 (Cô Nguyễn Thanh Hương)

5. Do you keep in touch _________ any school friends?


6. Some adults have a total lack ______ sympathy _______ young people.
7. He's more concerned ______ what people think about him than anything else.
8. She had been briefly acquainted ______ him more than 20 years earlier.
9. She thought _________ him very often.
10. Some people take __________ interest with enthusiasm, but they are soon tired ________it.

IV. Choose the correct words or phrases that best complete the passage.
Dear Jane,

I'm sorry for not (1) ______ to you for two months. In my postcard sent to you when I was on vacation
in July, I told you about a new friend of (2) ____, Lisa Moran, who lives near my uncle's farm in Napan
Valley, California. You may be curious to know what she is (3) ______.

Well, Lisa is of our age - 17 - and not very tall or slim. In fact, she has (4) ______ rounded shoulders
and a plump face. The best (5) ______ on Lisa's face are her green eyes and the two dimples on her
checks. She (6) ______ lively with her hair cut short and dressed in (7) ______ clothes - jeans and a
T-shirt. When I first met her at my cousin's birthday party, I was attracted by her (8) ______ and
pleasingly direct manners. Curiously enough, Lisa has the same (9) ______ in music as you and I.

That's (10) ______ about Lisa for the moment. How have you been doing with your school work?
Please write soon.
Best wishes
Ann Maxfield
1. A. write B. to write C. writing D. wrote
2. A. I B. me C. my D. mine
3. A. looking B. like C. seen D. similar
4. A. slightly B. hardly C. scarcely D. reasonably
5. A. aspects B. points C. features D. marks
6. A. feels B. seems C. appears D. looks
7. A. casual B. formal C. sports D. trendy
8. A. light-hearted B. high-handed C. self-centered D. easy-going
9. A. taste B. hearing C. sense D. way
10. A. all B. enough C. end D. most

Hocmai – Ngôi trường chung của học trò Việt Tổng đài tư vấn: 1900 6933 - Trang | 2 -
323
Hocmai.vn – Website học trực tuyến số 1 tại Việt Nam
Khóa học PEN C – N3 (Cô Nguyễn Thanh Hương)

V. Read the passage carefully, then choose the correct answers.


Having a best friend to confide in can bring a positive effect on our emotional health. An evening out
with the closest friend may be the best guarantee of a good time. In fact, our best friend can prevent us
from developing serious psychological problems such as depression and anxiety.
Best friendship evolves with time - we cannot go out and pick our best friend. We become friends with
people who share common interests – at school or through hobbies, for example.
Best friends have usually known each other for years and stuck together through good and bad times.
If you haven't got one, perhaps you are being too distant from people, or focusing too much on your
work.
11. A best friend can ________.
A. give us a healthy life B. go out with us in the evening
C. spend much time finding D. share joy and sadness with us
12. Close friends need to ________.
A. study at the same school B. have the same interests
C. pursue the same hobbies D. spend time together
13. According to the passage, ________.
A. it takes a lot of time to make close friendships
B. we can go out and choose a good friend easily
C. best friends have good and bad times
D. It’s very difficult to make lasting friendships
14. The word 'one' in the last paragraph refers to ________.
A. good time B. bad time C. a friendship D. a close friend
15. Which of the following sentences is not mentioned?
A. We often have satisfaction being with a close friend.
B. the ties of close friendship will never be broken.
C. An unsociable person may not have a close friend.
D. Friendships need time to develop.

Giáo viên: Nguyễn Thanh Hương


Nguồn : Hocmai

Hocmai – Ngôi trường chung của học trò Việt Tổng đài tư vấn: 1900 6933 - Trang | 3 -
324

You might also like